You are on page 1of 357

JC2 Examination Papers

2015
General Paper
H1
1

ANGLO CHINESE JUNIOR COLLEGE

P2

ANDERSON JUNIOR COLLEGE

P2

CATHOLIC JUNIOR COLLEGE

P2

DUNMAN HIGH SCHOOL

P2

HWA CHONG INSTITUTION

P2

INNOVA JUNIOR COLLEGE

P2

JURONG JUNIOR COLLEGE

P2

NANYANG JUNIOR COLLEGE

P2

PIONEER JUNIOR COLLEGE

P2

10

RAFFLES INSTITUTION

P2

11

RIVER VALLEY HIGH SCHOOL

P2

12

ST. ANDREWS JUNIOR COLLEGE

P2

13

SERANGOON JUNIOR COLLEGE

P2

14

TEMASEK JUNIOR COLLEGE

P2

15

TAMPINES JUNIOR COLLEGE

P2

16

VICTORIA JUNIOR COLLEGE

P2

17

YISHUN JUNIOR COLLEGE

P2

Click on the Exam Paper

trendyline

All Colleges 2015 Prelims P1 Questions


ACJC Prelims 2015
1. A career as an artist is more attractive now than ever before. Comment.
2. How important is it for people in your society to embrace uncertainty?
3. Consider the view that affluence poses more problems than it solves.
4. Is a world without borders a cause for celebration?
5. Environmental conservation should be made compulsory. Do you agree?
6. To what extent can we rely on newspapers to keep us well-informed?
7. Is increasing the strength of the police force the best way to combat crime?
8. Scientific advancement makes us less human. Do you agree?
9. Discuss the claim that education guarantees progress.
10. How far should the government interfere with personal decisions?
11. Out of sight, out of mind. Is this an accurate description of your societys attitude towards the less fortunate
today?
12. There is no room for the weak in competitive sport. Comment.
AJC Prelims 2015
1. Consider the view that more could and should be done to ease the problems resulting from the relocation of
people.
2. Skills are more important than academic qualifications in todays world. Discuss.
3. To what extent does the law ensure justice is served?
4. Not enough has been done to address the global outbreak of diseases. Is this a fair assessment?
5. Does religion do more harm than good in todays world?
6. How far does technology have an impact on the popularity of the arts?
7. The ability to make difficult decisions is the key to success. How true is this?
8. Examine the extent to which alternative lifestyles should be encouraged in your society.
9. People should help the poor in their own country rather than those elsewhere. How far do you agree?
10. Discuss the claim that too much faith is placed in democracy.
11. Is there any value in suffering for a cause?
12. Should we be concerned about the increasing use of artificial intelligence?
CJC Prelims 2015
1. How far are we justified in making decisions for those who cannot?
2. The more attention men give to their roles at home, the less problems there would be in the family.
Discuss.
3. Discuss the view that it is increasingly difficult to engage in ethical business practices.
4. Is green technology the most effective solution to the environmental problems in cities?
5. Consider the view that it is more important for a leader to be a visionary than a realist.
6. Money ultimately determines every success in sport. Do you agree?
7. Real power lies with the one who controls the media. Comment.
8. How important is it for people in your society to possess a sense of belonging?
9. Do museums have a future?
10. Society is obsessed with being the best. To what extent is this true of your society?
11. How far should Science be held accountable for perpetuating discrimination?
12. A long life does not necessarily mean a good life. Do you agree?
DHS Prelims 2015
1. The roles of men and women should be complementary rather than equal. Discuss.
2. To what extent is cultural heritage really important in your society?
3. The world would be a better place if we had a universal language. Comment.
4. Technology has produced a generation unable to distinguish what is worth reading. Is this true?
5. We cannot expect individuals to save the environment. Do you agree?
6. Consumers are at the mercy of corporations. Discuss.
7. Does tragedy always unite a nation?
8. Politicians today are too obsessed with the art of persuasion. Discuss.
9. A little rebellion now and then is a good thing. Comment on the validity of this proposition.
10. The truth should never be hidden from the public. Discuss.
11. The harsher the punishment, the more effective it is. Discuss.
12. Modern art is merely the expression of ones imagination, and is of little significance. Do you agree?
trendyline

HCI Prelims 2015


1. What valuable lessons do you think your country could learn from some of its neighbours?
2. Do your duty! Explore both the possible merits and potential drawbacks of always obeying this directive.
3. Should we be extremely excited or seriously alarmed by the rapid progress being made in the fields of robotics
and artificial intelligence?
4. Rather than broadening our minds, most foreign travel only serves to reinforce our prejudices! How far do you
agree?
5. The greatest happiness for the greatest number. Is this the best principle to adopt as the basis for good
government?
6. Do you agree with the criticism that charities are ineffective organisations that usually do more harm than
good?
7. Mother Nature knows best. To what extent do you agree with this piece of traditional wisdom?
8. Most non-conformists are merely attention-seekers. How true is this in your society?
9. Nothing happens by chance there is no such thing as luck. Discuss.
10. Parents should decide for themselves how many children they want to have governments have no business
trying to influence their decision. What are your views?
11. There is nothing special about our species mankind is merely a slightly more intelligent chimpanzee.
Discuss.
12. Do you agree with the view that art and politics should never mix?
IJC Prelims 2015
1. It is better to be a man than a woman. How far is this true in today's world?
2. Do written examinations serve any useful purpose?
3. To what extent should genetic research be supported?
4. People are too pragmatic. How true is this of your society?
5. 'The mass media are to be blamed for perpetuating stereotypes.' Do you agree?
6. 'When it comes to the environment, we have failed to walk the talk.' How far is this true?
7. The performing arts have little value beyond entertainment. Is this a fair comment?
8. Can freedom of expression be too much of a good thing?
9. Parents know best. To what extent is this true in a world of rapid change?
10. Should developing the young be the main focus of your country?
11. Medical science has extended life and prolonged suffering. Do you agree?
12. Regress to progress. How far is this true?
JJC Prelims 2015
1. Are certain languages superior to others?
2. Population pressure is the root cause of conflict. How far do you agree?
3. How far should governments exercise control over scientific development?
4. The Arts should play a significant role in your society. How far do you agree?
5. Discuss the view that, in an increasingly connected world, international issues must take precedence over
national issues.
6. Women will always be losers in the fight for equality. Comment.
7. Consider the view that social media undermines the truth and should be regulated.
8. Politicians merely react to events. Is this a fair comment?
9. Countries have far more urgent priorities than protecting the environment. Do you agree?
10. To what extent is the value of sport determined by its profitability?
11. How far is your country prepared for a serious outbreak of disease?
12. Discuss the value of music.
MJC 2015 Prelims
1. A good government is measured solely by the freedom of its people. Discuss.
2. Do you agree that we have less control over our health today?
3. How justifiable is it for poor countries to be concerned with the preservation of their historical monuments?
4. People, not technology, must solve mankinds problems. Is this a fair comment?
5. How far should migrants conform to the culture of their host country?
6. Being a small country is as much a good thing as a bad thing. Discuss.
7. Is it ever justifiable to break the law?
8. Should an education in the arts be encouraged as much as the sciences in your society?
9. Is there any point in caring about environmental problems in countries other than your own?
10. Research in the life sciences presents more dilemmas than solutions. Do you agree?
11. Consider the view that humility is unnecessary for one to succeed in life.
12. To what extent is the welfare of the elderly in your society the responsibility of the state?

trendyline

NJC 2015 Prelims


1. More than anything else, history instills a sense of belonging. Is this true?
2. Parents are mainly responsible for their childrens success and failure in life. Comment.
3. The most valuable commodity in modern life is time. Do you agree?
4. Religion divides rather than unites in todays world. Discuss.
5. Discuss the importance of understanding and appreciating the power of the written word.
6. How far is healthcare a matter of personal responsibility rather than the business of the government?
7. To what extent does your society celebrate diversity?
8. Despite all the technological or medical advancements that we have made, we have not create a better world
for mankind. Do you agree?
9. Consider the view that there is too much importance placed on freedom of expression.
10. To what extent do women in your society face discrimination?
11. Experimentation on animals is necessary for scientific research. How far do you agree?
12. Uncertainty may not necessarily be a bad thing. Discuss.
NYJC Prelims 2015
1. Should rich countries help poor countries with scientific and technological advancement?
2. Migration should be discouraged, not embraced. Discuss.
3. Assess the impact of the digital age on the way business is done today.
4. Religion has no place in politics. Do you agree?
5. Should more emphasis be placed on traditions in your society?
6. Let bygones be bygones. Is this good advice?
7. There is nothing to fear about growing old when the government is paying increased attention to the needs of
the elderly. Is this true in your society?
8. People no longer read for pleasure. Discuss.
9. Consider the view that security is more important than democracy.
10. Assess the effectiveness of international efforts at addressing crises today.
11. Photography has little artistic value today. Is this a fair comment?
12. To what extent is it acceptable for science to be used in the pursuit of human perfection?
PJC 2015 Prelims
1. Has technology brought about a highly disconnected world?
2. Learning beyond grades. To what extent is this a realistic goal in your society?
3. Kindness cannot be legislated. What is your view?
4. Stand up for your rights. Is this sound advice?
5. Is singlehood an attractive option in todays world?
6. Do you agree that scientific research should not be limited by ethical considerations?
7. Language is the key to cultural identity. To what extent is this true in your society?
8. The artist, not the government, should be regulating what is appropriate for the audience. Do you
agree?
9. The integrity of the game is everything. Is this true of sports today?
10. How important is it for people in your society to retain a sense of nostalgia?
11. In todays world, has ethics taken a backseat where food is concerned?
12. Security is far more important than liberty. Discuss.
RI Prelims 2015
1. Disease is the greatest threat facing mankind today. To what extent do you agree with this statement?
2. In the digital age do newspapers still have a role in society?
3. To what extent can Mathematics be considered a form of art?
4. Is the study of History increasingly losing its relevance in todays world?
5. Unlimited scientific research is the only way to make real scientific progress. Do you agree?
6. The State has no place in the private lives of its citizens. Do you agree?
7. The developed world should bear the burden of climate change. Discuss.
8. How far is your society prepared for the challenges that diversity brings?
9. Consider the view that science serves mankind better than religion.
10. There are enough opportunities in your society to improve ones life. Do you agree?
11. Tradition does more harm than good. To what extent do you agree with this statement?
12. Should international aid only be given to others during times of economic prosperity?

trendyline

RVHS Prelims 2015


1. Love for ones country is always a good thing. Comment.
2. Education produces a population able to read but unable to think. Discuss.
3. One cannot have ones cake and eat it too. Consider this statement in view of the issues faced by women
today.
4. Happiness is in your own hands. How far do you agree with this claim?
5. To what extent should the success of your society be measured by its economic growth?
6. Do you think the world will be a better place if everyone has a religion?
7. The traditional roles of a father are no longer relevant today. Do you agree?
8. To what extent has the media advanced the cause of conservation?
9. Can the car as a form of private transport ever be eliminated?
10. Science requires as much faith as it requires logic. Comment.
11. There is no greater threat to countries today than terrorism. Do you agree?
12. How far is elitism a problem in your society?
SAJC Prelims 2015
1. Good character hinders success. Discuss.
2. Consider the importance of architecture and city planning.
3. Should the pursuit of environmental causes be embraced at all costs?
4. Money is the only motivator for people today. Comment.
5. Compassion has no place in a competitive society. Is this true?
6. Is it true that the more interconnected the world becomes, the more danger there is?
7. Does tourism impoverish us?
8. A world without religion will be a better one. Do you agree?
9. Should we always accept opportunities that come our way?
10. Parenting today is an impossible task. Do you agree?
11. Is it possible to forgive and forget?
12. Can your society do without the Arts?
SRJC Prelims 2015
1. To what extent is it mankind's duty to save endangered species?
2. 'Democracy is not for every country.' Discuss.
3. There has never been a better time than now to be a woman. Comment.
4. We cannot eradicate poverty without first eliminating discrimination. Do you agree?
5. 'No one dares to be different.' Is this true of your society?
6. Should the press be completely free?
7. There is no reason for society to spend on the Arts when people are still suffering in poverty. Do you agree?
8. To what extent is the development of a country affected by geographical factors?
9. Technology has made learning easier, not harder. To what extent do you agree?
10. 'There is too much emphasis on economic development.' Is this true in your society?
11. 'Competitive sports bring out the best not worst in people.' Do you agree?
12. Should countries that are suffering from economic crises still try to save the environment?
TJC 2015 Prelims
1. Do you agree that ethical considerations only get in the way of medical progress?
2. In your society, is there any point in learning a foreign language?
3. Examine the view that the Arts are only for the wealthy.
4. Do superheroes serve any useful purpose?
5. Celebrities should stay out of politics. Comment.
6. Does the notion that big businesses do more harm than good still hold true today?
7. Do you agree that social media has given rise to a more divided world?
8. Is genetically modified food the answer to world hunger?
9. Nothing ventured, nothing gained. How well does this describe the attitude of young people in your society
today?
10. How tolerant is your society of alternative views and lifestyles?
11. Is lack of education the root cause of crime today?
12. Age is no barrier to success. How far do you agree?

trendyline

TPJC 2015 Prelims


1. Should wild animals be kept in captivity?
2. Leaders must keep their eyes more on the present than the future. Comment.
3. Should fathers today stay at home and look after their children?
4. Aging is not lost youth, but a new stage of opportunity. How far is this true of the
elderly in your society?
5. It is impossible to contain conflict within national borders in todays world. Discuss.
6. Is it true that humour makes the world a better place?
7. Examine the claim that people in your society are too dependent on their
government.
8. Having designer babies is our right. Do you agree?
9. Consider the view that the arts will play a more significant role in your society.
10. Live as if you were to die tomorrow. Is this good advice?
11. Healthcare is the responsibility of the individual, not the government. Do you
agree?
12. Is international cooperation the most effective way of reducing poverty?
VJC Prelims 2015
1. Political leaders should ignore public opinion. How far do you agree with this statement?
2. The preservation of culture has been overlooked in the arts. Discuss.
3. Is the study of History increasingly irrelevant in todays world?
4. Should science be the primary focus of education in your country?
5. Have children today lost their childhood?
6. Should developed countries be expected to do more to solve the water crisis?
7. Are we taking sport too seriously?
8. With the emphasis on technology, will the elderly in your country be left behind?
9. We lack role models who can inspire our young to make change. Comment.
10. Do choices actually make our lives better?
11. How far is it acceptable for the public to mete out justice for wrongdoing?
12. Do museums still have a role to play in society?
YJC Prelims 2015
1. Consider the view that the good of the society should not be sacrificed for that of the individual.
2. Animals should have the same right to life as humans. Discuss.
3. Do you agree that only the rich can appreciate the Arts?
4. Examine the extent to which terrorism is a threat to global security today.
5. How far should individuals be blamed for global environmental problems?
6. Free speech entails the right to offend or provoke. Is this a fair comment?
7. The pursuit of money necessarily results in an ungracious society. Discuss.
8. Examine the extent to which the young in your society should be allowed to make their own decisions.
9. Horrific events are the only catalysts for social or political change. Discuss.
10. Is there any point in trying to regulate social media?
11. Historical records are questionable, being based as much on perception as on fact. Is this a fair comment?
12. To what extent do new technologies enhance the quality of education in your society?

trendyline

ANGLO-CHINESE JUNIOR COLLEGE


JC2 PRELIMINARY EXAMINATION 2015
CANDIDATE
NAME
INDEX
NUMBER

GENERAL PAPER

8807/02

Paper 2

1 hour 30 minutes

Candidates answer on the Question Paper.


Additional Materials: 1 insert
READ THESE INSTRUCTIONS FIRST
Write your index number and name on all the work you hand in.
Write in dark blue or black pen in the spaces provided on the Question Paper.
Do not use staples, paper clips, highlighters, glue or correction fluid.
Answer all questions.
The insert contains the passage for comprehension.
Note that up to 15 marks out of 50 will be awarded for your use of language.
At the end of the examination, fasten all your work securely together.
The number of marks is given in brackets [ ] at the end of each question or part question.

For Examiners Use


Content

/35

Language

/15

Total

/50

This document consists of 7 printed pages.


GP 8807_2
ACJC 2015

Anglo-Chinese Junior College


General Paper Department

[Turn over]

trendyline

For
Examiners
Use

Read the passage in the insert and then answer all the questions which follow below. Note
that up to fifteen marks will be given for the quality and accuracy of your use of English
throughout this Paper.
Note: When a question asks for an answer IN YOUR OWN WORDS AS FAR AS POSSIBLE
and you select the appropriate material from the passage for your answer, you must still use
your own words to express it. Little credit can be given to answers which only copy words or
phrases from the passage.
1

What is the author implying by describing nostalgia as at best a trait to avoid and at worst
a root cause of depressive illness in lines 3-4?

[1]
2

Explain the authors use of the phrase rose-tinted to describe peace in line 6.

[1]
3

Explain what the author means by the universality (line 13) of nostalgia. Use your own
words as far as possible.

[2]
4

Explain the two positive effects of nostalgia highlighted in paragraph 2. Use your own
words as far as possible.

[2]

trendyline

For
Examiners
Use

Why has the author placed brackets around the comment in lines 16-18?

[2]
6

Using material from paragraphs 3-5 only (lines 19-53), summarise how nostalgia benefits
us. Write your summary in no more than 120 words, not counting the opening words
which are printed below. Use your own words as far as possible.
Nostalgia benefits us because

[8]
[Number of words: .. ]

trendyline

For
Examiners
Use

What is the authors purpose in describing nostalgia as being like choosing neural
pathways and an inexhaustible bank account (lines 64-65)? Use your own words as
far as possible.

[3]
8

Why does the author describe storytellers as professional nostalgics (line 78)?

[2]
9

What does the author mean when he describes nostalgic memory as bittersweet in line
79? Use your own words as far as possible.

[3]
10

What danger is the author referring to in line 90?


possible.

Use your own words as far as

[1]

trendyline

11

In this article, Tim Adams describes the functions of nostalgia. How far would you agree
with Tim Adams observations? Relate your opinions to your own society.
Justify your answer with ideas from the passage and your knowledge of your own society.

trendyline

For
Examiners
Use

For
Examiners
Use

trendyline

For
Examiners
Use

[10]

REEC
Band
Mark

End of Paper

trendyline

ACJC General Paper Department


JC2 Preliminary Exam 2015 P2 Answer Scheme (with Examiners Notes)
From Paragraph 1
1. What is the author implying by describing nostalgia as at best a trait to avoid and at worst a root
cause of depressive illness in lines 3-4?
[1m]
From Passage
The habit of living in memory rather than the
present, of comparing how things once were with
how things are now, was for several centuries
thought at best a trait to avoid and at worst a root
cause of depressive illness. (lines 2-4)

Inferred
Nostalgia was deemed as completely negative,
with no redeeming features at all.
NB: must emphasise that there is nothing good
about it

Question type: Inferential


Examiners notes:

Many students merely wrote negative, which did not show that nostalgia was completely negative.

It is not sufficient to merely paraphrase the quote.

2. Explain the authors use of the phrase rose-tinted to describe peace in line 6.
From Passage
Nostalgia was the soldiers malady a state of
mind that made life in the here and now a
debilitating process of yearning for that which had
been lost: rose-tinted peace, happiness, loved
ones. (lines 4-6)

[1m]

Inferred
It suggests that the soldiers perceptions of peace
are idealised/ overly positive.
OR
The soldiers only see the positive aspects of peace.

Question type: Vocab


Examiners notes:

Many students wrongly interpreted rose-tinted as a metaphor for beauty or romance and therefore provided wrong
contextualisation of the phrase.

Many students did not capture the extent of the positivity.

From Paragraph 2
3. Explain what the author means by the universality (line 13) of nostalgia. Use your own words as far
as possible.
[2m]
From Passage
Some of the research has proved the universality
of the feeling itself - new study shows the
commonality of nostalgia effects (lines 12-14)

Paraphrased
The impact of nostalgia/ feeling nostalgic is shared/
similar/ experienced by many
Inferred

in 18 countries in five continents. (line 14)

all around the world/ globally.

Question type: Paraphrase and inference


Examiners notes:
This question was successfully attempted by most students.

Get PSJ Private Tutor to Guide you through Exams Now!


Contact www.privatetutor.com.sg
trendyline

4. Explain the two positive effects of nostalgia highlighted in paragraph 2. Use your own words as far
as possible.
[2m]
From Passage
nostalgia is shown to be both a driver of
empathy and social connectedness, (line 15)

and a potent internal antidote for loneliness and


alienation. (lines 15-16)

Paraphrased
Nostalgia motivates/ encourages/ spurs/ inspires
people to understand/ relate to/ identify with each
other at a deeper level and promote community
cohesion/ bonds/ ties/ relationships
NB: driver is the main idea and MUST be paired
with either empathy or social connectedness (or
both) for 1m.
as well as function as a solution/ answer/ cure for
solitude/ isolation and marginalisation/ detachment.
NB: antidote is the main idea and MUST be
paired with either loneliness or alienation (or
both) for 1m.

Question type: Paraphrase


Examiners notes:

Many students either missed out on the main idea of driver or explained it inaccurately. Words like enables and allows
do not capture the meaning of pushing for something to happen.

5. Why has the author placed brackets around the comment in lines 16-18?
From Passage
Among the measurable effects, nostalgia is shown
to be both a driver of empathy and social
connectedness, and a potent internal antidote for
loneliness and alienation (a fact which has led to
the beginnings of nostalgia-based therapies for
illnesses that include clinical depression and
perhaps even Alzheimers). (lines 14-18)

[2m]

Inferred
[Function] The comment in brackets is additional
information [1m]
[Context]
to illustrate how nostalgia can be used for medical
treatment/ to show that the benefits of nostalgia
have been extended to medical treatments [1m]

Question type: Inference/punctuation


Examiners notes:

Most students handled this question well, but some simply regurgitated the function of brackets from the skills package/past
year A Level exam answer schemes without explaining the context of nostalgia being used for medical treatment.

[SUMMARY ANSWER SCHEME AFTER THE SHORT ANSWER QUESTIONS]

trendyline

From Paragraph 6
7. What is the authors purpose in describing nostalgia as being like choosing neural pathways and an
inexhaustible bank account (lines 64-65)? Use your own words as far as possible.
[3m]
From the passage
Nostalgia in this sense is like

Inferred
Purpose:
The author wants to create an analogy/a comparison
for the reader to recognise/understand that nostalgia
functions like choosing a pathway and creating a bank
account,
NB: We accept answers that say the author wants the
reader to better understand the concept of nostalgia.

Reinforcing some formative positive


experiences over and over could be one
way of attempting to manipulate that
subconscious selection process. (lines
56-58)

Paraphrased
where nostalgia is a process of picking/deciding on
specific memories that we want to revisit,

choosing the neural pathways you


want to tread most often (lines 64-65)
most people have at least one nostalgic
memory that they cherish and that they
can use repeatedly (lines 62-63)

and also a process of building a collection of memories


that can be used endlessly/without limit/ is always
accessible.

we can dwell on them and return to


them forever. (lines 63-64)
.creating an inexhaustible bank
account which is there for you if you want
to withdraw from it. (lines 65-66)
Reinforcing some formative positive
experiences over and over could be one
way of attempting to manipulate that
subconscious selection process. (lines 5658)
choosing the neural pathways you want
to tread most often (lines 64-65)
Question type: Inference/ paraphrase
Examiners notes:

Most students did not explain the authors purpose of using the metaphor.

Selection and Choosing were common lifts.

trendyline

From Paragraph 7
8. Why does the author describe storytellers as professional nostalgics (line 78)?
From Passage
The essays written in a nostalgic state were judged
more imaginative and creative (storytellers,
professional nostalgics, have long intuited this, not
to mention poets). (lines 77-78)

[2m]

Inferred
As the livelihood/ work/ job of storytellers depends
on their imagination and creativity,
they are proficient/ skilled/ experts at using feelings
of nostalgia.

Question type: Inference/ vocab


Examiners notes:

Many students did not capture both characteristics of what it means to be professional.

From Paragraph 8
9. What does the author mean when he describes nostalgic memory as bittersweet in line 79? Use
your own words as far as possible.
[3m]
From Passage
Nostalgic memory is a bittersweet combination of
rumination, counterfactual thinking and nostalgia.
(lines 79-80)
Rumination and counterfactual thinking are related
to despair and perhaps to depression (lines
80-81)
People want to remind themselves of the people
who are no longer here and what they meant.
(lines 84-85)
OR
It serves to remind them of what intimacy they
have achieved and therefore what they are
capable of. (lines 85-86)

Paraphrased
He describes nostalgic memory as both painful and
pleasurable.
While nostalgic memory is associated with grief/
affliction/ dejection/ melancholy,
it also reminds us of the significance/ importance/
value of loved ones whom we have lost.
OR
the affection/ close relationships/ love they once
had/enjoyed (and may enjoy again).

Question type: Paraphrase


Examiners notes:

Many students did not explain the term bittersweet fully and accurately, and gave a vague explanation of the term
bittersweet (e.g. good and bad, positive and negative).

From Paragraph 9
10. What danger is the author referring to in line 90? Use your own words as far as possible. [1m]
From Passage
Of course advertisers and political speechwriters
have long understood the power of collective
nostalgia. Is it not the fact that such feelings can
also manipulate us into doing things, buying
things, voting for things that in more coldly
rational states we might resist? (lines 87-89)

Paraphrased
Collective nostalgia can be exploited/ made used
of by others to make people act in a way that they
would usually/ normally not.
NB: Holistic understanding must be shown.

Question type: Paraphrase


Examiners notes:

Some students did not capture the irrationality of the behavior as a consequence of exploitation by advertisers and
politicians.

trendyline

SUMMARY
6. Using material from paragraphs 3-5 only (lines 19-53), summarise how nostalgia benefits us. Write your summary in no more
than 120 words, not counting the opening words which are printed below. Use your own words as far as possible. [8m]
No.
1

No.
1

Possible Paraphrase
Nostalgia benefits us because it is an innate/ inherent/
inborn/ natural/ instinctive response/ tool that

helps us guard against/ shields us/ fortifies us from

3
4

anxiety/ worry/ despair/ pessimism.


Nostalgia neutralises/ lessens the impact/ harm

posed by threatening/ unfavourable circumstances,


OR
during harsh/ painful/ intolerable/ unbearable times,

momentarily change our perspectives of the current


situation

and allows us to hold out further during/ endure through/


stressful times.

Nostalgia builds positivity/ helps (us) expect better


prospects/ things to come,

9
10
11

Possible Points
Nostalgia seems to be a kind of inbuilt (line 19)
OR
Nostalgia spontaneously rushes in (and counteracts
those things) (lines 22 - 23)
neurological defence mechanism (line 19)
OR
which can be marshalled to protect us (lines 19-20)
OR
as a defence against the present (for children) (line
46)
against negative thoughts (line 21)
nostalgia compensates for (line 21)
OR
counteracts those things (line 23)
OR
posed in a redemption sequence (line 42)
(negative) situations. (line 20)
OR
Particularly in times of hardship and difficulty (lines
20-21)
OR
uncomfortable states (line 21)
OR
(helps children) through difficult periods (line 45)
can temporarily affect your perceived body state
(lines 26 - 27)
OR
to temporarily alter their perception of the state they
were in (line 30)
OR
the temporary change in perception (line 31)
allowed them to crucially persevere just a bit
longer. (lines 31-32)
OR
which are correlated with mental fortitude (line 34)
OR
and in doing so it perhaps builds resilience (line 36)
nostalgia helps build resources like optimism (line
34)
OR
and a hope for the future. (line 46)
or inspiration (line 34)
or creativity (line 34)
nostalgia grounds us

9
10
11

12

and gives us a base (line 35)

12

13

on which to evaluate the present as a temporary


state (line 36)
reminding them (children) to think of happier
moments (lines 45-46)

13

15

16

The ability and encouragement to access nostalgia


also builds gratitude (line 52)
connectedness towards others (line 52)

motivates / spurs us on,


promotes resourcefulness/ inventiveness/ imagination.
Nostalgia anchors us/ gives us a realistic perspective /
sense of reality,
gives us a yardstick / standard against which we
compare our current situation,
and helps us to realise/ consider that the present is
short-lived/ will not last/ will be over soon.
Nostalgia allows children to remember more joyful/
blissful times.
NB: Comparative MUST be present.
Nostalgia develops in children an appreciation for others
/ thankfulness,
helps children to relate to others/ grow closer to others/
empathise with others,

17

and tends to make children less selfish. (line 53)

17

3
4

14
15

No. of
Points
Marks

14

16

and makes them less self-centred/ inconsiderate.

1-2

3-4

5-6

8-9

10-11

12-13

14 & more

trendyline

11. In this article, Tim Adams describes the functions of nostalgia. How far would you agree with Tim Adams
observations? Relate your opinions to your own society.
Requirement:
Students should a) explain the functions nostalgia plays in their own society
b) describe the benefits and problems of being nostalgic
c) show understanding and engage with the ideas and views raised in the passage
d) support their views with relevant examples from their own society
Explanation:
Discuss some of the following in relation to their own society
a) nostalgia and its ability to forge social cohesion
b) nostalgia and its ability to forge optimism
c) nostalgia and its ability to encourage resilience and perseverance
d) the changing perceptions of optimism and pessimism
e) how people use nostalgia in different situations
f) how nostalgia encourages creativity and imagination
g) how nostalgia can be manipulated
Evaluation:
a) question/show reasons for subscribing/not subscribing to the authors ideas
b) provide insightful analysis of the changing perceptions towards and the reasons why people would
want to adopt these mindsets
c) critically evaluate the functions and benefits of nostalgia
d) provide cogent development of arguments
e) give examples from their own society to support their views
Coherence:
a) adopt a consistent viewpoint
b) argue logically
c) organise answers into cohesive, themed paragraphs
d) link paragraphs to show continuity and direction of argument
e) maintain relevance to the task in everything they write
f) end with a summative or concluding paragraph/ sentence
Examinersnotes:
Some students misinterpreted nostalgia as a bad memory, when it should refer to memories of a more
pleasant past (e.g. the brutality of WWII is not an example of a nostalgic memory).
Some students launched into long descriptions of examples without sufficient analysis of how nostalgia is
used in the context of the students society.
Several AQs were incomplete, lacked an introduction or had several lines left blank in the beginning.
Many students discuss nostalgias effect on Singaporean society in the past, without drawing a link to
current times.
Many students hastily launch into describing the Singaporean context without making a link back to the
reference/ main topic of nostalgia.
Students frequently misunderstood how nostalgia was a nationalistic, chauvinistic tool that could promote
national identity and patriotism, whereas the author intended to show the exploitative effects of nostalgia.
Some students hijacked the AQ, for instance, instead of discussing how nostalgia can or cannot inspire
altruism, they went on to describe how other factors inspired people to be more altruistic instead.
Some students selected quotes/ references that are not related to the functions of nostalgia.
Some students rephrased the authors points too loosely and hence distorted the authors intended
meaning.
Many students provided examples that are trivial, generic, or/and insignificant.

trendyline

11. In this article, Tim Adams describes the functions of nostalgia. How far would you agree with Tim Adams observations? Relate your
opinions to your own society.
From the Passage

1. Nostalgia fosters
social cohesion
Claims:
nostalgia is known to
be both a driver of
empathy and social
connectedness, and a
potent internal antidote
for loneliness and
alienation (lines 1416)
In group situations those
with induced nostalgia
not only tend to feel
more closely bonded
with the group but also
more willing to form
intimate associations
with strangers and to be
freer in their thinking.
(lines 66-67)
The ability and
encouragement to
access nostalgia also
builds gratitude and
connectedness towards
others. (lines 48-50)

Guiding
Questions /
Discussion
Issues
Does nostalgia
create closer
bonds amongst
Singaporeans?
Does nostalgia
build gratitude and
connectedness
towards others?

Agree with author

Reminiscing about past experiences


does create closer bonds, especially if
the past experience is meaningful. For
example, national service is a common
experience for Singaporean males and
many NS men recall their past
experiences fondly and form lasting
friendships with their ex-army mates.

Reminiscing about a past common


experience that Singaporeans identify
with also unites and bonds them. E.g.
Most Singaporeans felt united in their
sense of loss during the passing of Mr
Lee Kuan Yew.

Singaporeans also feel a sense of


identity and pride whenever
Singapores remarkable development
the past 50 years is mentioned. The
sense of achievement is palpable and
this common pride can be used to
drive them to be more compassionate
towards fellow Singaporeans. For
example, as part of the SG50
celebrations, numerous clips of
Singapores past were aired to
highlight Singapores past to the effect
of creating a common, shared history
and identity.

Disagree with author

Nostalgia may lead to the exclusivity of a


group of people. Singaporeans may
become all the more xenophobic since its
history drives social connectedness that
may leave foreigners out of the picture. It
might be dangerous to falsely appropriate
nostalgia for nationalistic agenda (E.g. A
retelling of Singapores history may
portray the British and the Japanese
unfavourably). Taken out of context, this
may lead to xenophobic, anti-foreigner
sentiments. It may also lead to minority
groups being marginalised and viewed
negatively, being seen as riding on the
coattails of their predecessors and
enjoying the fruits of the labour of others.

trendyline

2. Nostalgia provides
us with a sense of
optimism and hope for
the future.
Claims:
Nostalgia seems to be a
kind of in built
neurological defence
mechanism, which can
be marshalled to protect
us against negative
thoughts and situations.
(lines 18-20)
nostalgia
compensates for
uncomfortable states
such as feelings of
meaninglessness or a
discontinuity between
past and present. (lines
20-21)
nostalgia helps build
resources like
optimism (lines 31-32)
reminding them to
think of happier moments
as a defence against the
present and a hope for
the future.(lines 42-43)

Do Singaporeans
use nostalgic
thoughts as a form
of defence against
pessimism?
Does nostalgia
provide
Singaporeans with
a sense of hope for
the future?

Thinking about the past glory days


calms any anxiety we may have about
the uncertain and volatile future as we
feel that if we could cope with more
tumultuous times in the past, when
Singapore was in a much less
prepared state, then what the future
holds cannot be that bad and we can
handle and cope with whatever new
challenges come our way.
Common unpleasant experiences such
the Hotel New World disaster, WWII,
the split with the Malayan Federation,
the early struggle post-independence
and SARS remind Singaporeans of our
tenacity and fortitude, inspiring us to be
stronger and to ride out whatever
current crisis we may face. Negative
past experiences and the positive
lessons learnt from tackling them teach
us to build resources to combat
potential crises.

The more people dwell on the past, the


more dissatisfied or uncomfortable they
may become. For example, in sport,
comparisons with the victorious Malaysia
Cup teams of the past cast the current
football teams in Singapore in a bad light
where the team used to be regional
powerhouses, one cannot help but feel
that the national team has somehow lost
its way. For older Singaporeans, it could
be a longing for the days when life in
Singapore was simpler, less competitive
and less crowded, leading to more
bitterness directed towards the
government today.
Nostalgia can evoke a sense of
pessimism it is easy to see the past as
the good old days while the present as a
pale imitation of the past and for us to
worry about our society in the future. For
example, in the past, education was
certainly a lot less competitive and
stressful for children. The frequent
complaints and pessimism about our
hyper-competitive education system and
its bleak future have prompted
government leaders to take action to
reduce the stress placed on our students
(E.g. Minister of Education Heng Swee
Keats repeated calls for our education to
move away from the emphasis on grades,
PM Lee Hsien Loongs exhortation to
parents to let children play and learn
through play).

trendyline

We may get trapped in a never-ending


loop wherein we dwell upon these
idealised forms of the past so much so
that we feel pessimistic about the future.
Like a broken record, nostalgia may
instead remind one of the past, that we
cannot help but feel apprehensive about
the future. For example, the sense of a
kampung spirit has been replaced by
unease over immigration and uncertainty
over rising costs of living.
3. Nostalgia helps us
build perseverance and
resilience.
Claims:
an as if loop a
mechanism by which
your mind can
temporarily affect your
perceived body state.
(lines 25-26)
It was not a solution, but
the temporary change in
perception allowed them
to crucially persevere
just a bit longer. (lines
29-30)
nostalgia grounds us
and gives us a base on
which to evaluate the
present as a temporary
state, and in doing so it
perhaps builds
resilience. (lines 33-34)

Does thinking
about the past help
Singaporeans build
resilience?
Does nostalgia
help Singaporeans
ride out tough
times?

Thinking about the past hardships


Singapore endured as a nation as
well as learning from them (e.g. SARS,
2008 economic crisis, race riots, haze)
- has indeed driven Singaporeans to
be more resilient in the face of
hardship. For example, plans for
recurrent problems such as the haze
and MERS have been put in place by
the government to ensure Singapore
continues to run smoothly in case we
face the same problems again.
On a personal level, there are many
stories of successful people who use
their past experiences to fuel their
resilience and determination to
succeed that have inspired
Singaporeans to endure hardship and
to thrive in difficult circumstances. E.g.
Lim Hock Chee experienced being a
pig farmer and a failed business to
become the owner of the Sheng Siong
group of supermarkets. Navy regular
Jason Chee, who lost three limbs in an

In Singapore, the past was definitely a


much tougher and tumultuous era as
compared with the present as such, we
do not think about the good moments we
had in the past as much as we think about
the bad ones we constantly draw on
instances like the racial riots we
experienced in the 1960s and the preindependence communist insurgency as
warnings that our current situation is one
that is fraught with potential missteps. We
are constantly worried that issues that
plagued us in the past will crop up again
and we compensate for this pessimism by
passing heavy-handed laws and policies
(for example our libel and sedition laws).
We are not necessarily more resilient
when we think about happier times in the
past instead, Singapores economic
success through the decades and
relatively good governance have bred a
generation of complainers whose sense of
entitlement is a worrisome trait. In fact,
some worry that Singaporeans are more

trendyline

accident in 2012, is back with the Navy


because of his love for his past career.

4. Nostalgia drives
creativity and
imagination.
Claim:
inspiration or
creativity, which are
correlated with mental
fortitude. (line 32)

Does nostalgia
drive creativity and
innovation in
Singapore?

content to take the foot off the pedal and


demand a less competitive society, with
more social welfare benefits and safety
nets provided by the government.

Nostalgia can be marketed to the


younger generation in novel ways
such as retro-themed cafes like The
Coastal Settlement, Carpenter & Cook
and Tian Lee & Co. Nostalgia can be
repackaged for it to be relevant to the
younger generation.

Nostalgia may only fuel fleeting frivolous


fads. What the youth in particular are
experiencing is a hipster heritage
impulse, a form of nostalgia that appeals
specifically to young middle-class, urban
professionals who pick certain aspects to
reminisce about.

Nostalgia can be an inspiration for


creative works of art. For example, 50
Years of Theatre Memories presents
the memories of 50 significant theatre
practitioners in an exhibition that not
only shares their diverse, enlightening
and personal moments with the
audience but also creates an
informative, stimulating and playful
experience to enrich a wider
appreciation and understanding of the
theatre scene in Singapore. Films like
1965, Old Romances, and most
recently, Seven Letters, are also made
with nostalgia as the main backdrop.

Nostalgia may encourage people to just


do what worked in the past and not take
risks, resulting in an unwillingness to
adapt to the changing times.

Nostalgia can inspire authentic artistic


expression. The book, Balik
Kampung, is a collection of eight new
tales written by Singaporean authors
who have lived in their neighbourhoods
for 10 years or more.

The essays written in a


nostalgic state were
judged more imaginative
and creative (lines 7274)

trendyline

5. Nostalgia has
longevity in its use and
can be easily adapted
across various forms
and over time.

Is it always true
that nostalgia
entails positive
memories or
experiences?

Claim:
most people have at
least one nostalgic
memory that they cherish
and that they can use
repeatedly. (lines 57-58)

How might a
persons inherent
temperament
colour or
undermine the use
of nostalgia for
more beneficial
purposes?

Nostalgia in this sense is


like choosing the neural
pathways you want to
tread most often, like
creating an inexhaustible
bank account (lines
60-62)
nostalgia stands out
as adaptive - it is distinct
from these negative
memories in that it is
always related to
intimacy maintenance.
(lines 79-81)

Can everyone tap


into or meaningfully
harness nostalgia
to seek hope and
purpose in their
lives? Are there
exceptional
circumstances?

Nostalgia has been very often applied


in national celebrations, such as the
short clip of how Singapores first
Prime Minister teared in his
announcement of Singapores
separation from Malaysia in 1965,
describing it as a moment of anguish.
That moment also marked Singapores
independence and the revisiting of this
memory has been used in many
circumstances in schools, as part of
the National Education Curriculum,
during National Day celebrations and a
key video clip in MM Lees
remembrance documentaries.

Nostalgia would not be relevant or as


relevant to individuals who are suffering
from any form of mental incapacities e.g.
children born with severe autism, geriatric
patients with degenerative Alzheimers
disease etc. In the case of the latter,
memories and nostalgia of the elderlys
better, healthier days may actually bring
more hurt and grief to the immediate
family or caregivers as they witness the
withering of their loved ones former
selves.

Nostalgia can be encapsulated in


various art forms such as movies and
songs. National Day songs such as
We are Singapore allow us to revisit
the days where Singapore was
struggling to build its economy and
infrastructure, and striving to maintain
peace and stability within its
community. It reminds us of the past
with the lyrics, There was a time when
people said that Singapore wont make
it, but we did. There was a time when
troubles seem too much for us to take,
but we did. This popular National Day
song takes the community through
Singapores history and though some
might view it as propaganda material,
most Singaporeans young and old
resonate with the ideas presented as it
revisits this nostalgia of Singapores
growth and development.

trendyline

6. Nostalgia can inspire


acts of charity.
Claims:
nostalgia is known to
be both a driver of
empathy (lines 14-15)

Is nostalgia a
compelling reason
or motivation for
acts of grace,
charity or
compassion?

The ability and


encouragement to
access nostalgia also
builds gratitude tends
to make children less
selfish. (lines 48-50)

7. Collective nostalgia
can be manipulated.
Claim:
...the power of collective
nostalgia. Is it not the
fact that such feelings

Is collective
nostalgia similarly
manipulated in our
context?

Nostalgia can inspire acts of altruism.


As part of the campaign of 50 ACtS
in celebration of SG50, the ACS
schools participated in an event to give
mandarin oranges as an act of
gratitude and thanks to those who
have contributed to Singapore society
(they include healthcare workers,
lawyers, the Singapore Civil Defence
Force and cleaning staff at various
locations) during Chinese New Year
Eve earlier this year.

Singaporeans also feel a sense of


identity and pride whenever
Singapores remarkable development
the past 50 years is mentioned. The
sense of achievement is palpable and
this common pride can be used to
drive them to be more compassionate
towards fellow Singaporeans. For
example, as part of the SG50
celebrations, ComfortDelgro sold little
red dots, where the profits went to the
Lions Befrienders Service Association
(Singapore), the Handicaps Welfare
Association, the Cerebral Palsy
Alliance Singapore, and the Singapore
Association of the Visually
Handicapped.

The ruling PAP may be leveraging on


Singapores nostalgic jubilee
celebrations to create a sense of
goodwill and connectedness with
Singaporeans, in order to gain the
upper hand in the General Elections to

Ones sense of compassion or altruism


and less due to nostalgia and memories
nor personal experience that inspire acts
of charity. For example, when
Singaporeans donated to causes
supporting disasters like the recent Nepal
earthquake, many did it for the sheer
empathy for the plight of the Nepalese
and less due to having experienced the
devastation of a disaster.

Collective nostalgia may be challenging to


muster/rally, and may also encounter
resistance especially in a multi-ethnic
society like Singapore with a people of
diverse aspirations and goals. As the
income gap widens with economic

trendyline

can also manipulate us


into doing things, buying
things, voting for things,
that in more coldly
rational states we might
resist? (lines 84-86)
Nostalgia can certainly
be a nationalistic,
chauvinistic tool. (line
87)

be held by the end of the year.


Advertisers/Businesses make use of
nostalgia as a marketing strategy to
encourage mindless consumption
during festive seasons like Christmas
and Chinese New Year, and major
sale seasons like the Great Singapore
Sale.

development, the Singapore government


has been finding it an uphill task to appeal
to Singaporeans shared sense of national
identity, despite the inclusion of National
Education into the school curriculum and
the commemoration of various ethnic
festivities and national celebrations. It can
thus be said that even as a nationalistic
tool, nostalgia merely enjoys fleeting
appeal or short-term indulgence as more
practical bread-and-butter issues take
precedence for most Singaporeans.
Over-emphasis on nostalgia may lead to
the museumification of heritage and
history, which is reflective of a country
with token heritages. In Singapore, the
new wave of nostalgia has resulted in the
setting up of numerous retro-themed
cafes, such as Sinpopo, Chye Seng Huat
and Dong Po Colonial caf. However,
these places are products of
commercialised nostalgia and may not be
true representations of the past, providing
an ostentatious as opposed to truly
meaningful - display of heritage instead.
Similarly, the miserable attempts by
Breadtalk to market a bun based on Mr
Lee Kuan Yew is similarly thoughtless
and predatory.

trendyline

ANGLO-CHINESE JUNIOR COLLEGE


JC2 PRELIMINARY EXAMINATION 2015

GENERAL PAPER

8807/02

Paper 2
1 hour 30 minutes

INSERT

READ THESE INSTRUCTIONS FIRST


This insert contains the passage for Paper 2.

This document consists of 3 printed pages.


GP 8807_2
ACJC 2015

Anglo-Chinese Junior College


General Paper Department

[Turn over]

trendyline

2
Tim Adams writes about nostalgia.
1

Is it healthy to dwell in the past? Up until about 15 years ago most psychologists would have
suggested probably not. The habit of living in memory rather than the present, of comparing
how things once were with how things are now, was for several centuries thought at best a
trait to avoid and at worst a root cause of depressive illness. Nostalgia was the soldiers
malady a state of mind that made life in the here and now a debilitating process of 5
yearning for that which had been lost: rose-tinted peace, happiness, loved ones. It had been
considered a psychological disorder ever since the term was coined by a 17th-century Swiss
army physician who attributed the fragile mental and physical health of some troops to their
longing to return home nostos in Greek, and algos, the pain that attended thoughts of it.

Since the turn of this century, however, things have been looking up for nostalgia. It has 10
become a focus of enquiry in university departments across the globe, a whole new field of
academic study that takes in sociology and political science as well as psychology. Some of
the research has proved the universality of the feeling itself a new study shows the
commonality of nostalgia effects in 18 countries in five continents. Among the measurable
effects, nostalgia is shown to be both a driver of empathy and social connectedness, and a 15
potent internal antidote for loneliness and alienation (a fact which has led to the beginnings
of nostalgia-based therapies for illnesses that include clinical depression and perhaps even
Alzheimers).

Nostalgia seems to be a kind of inbuilt neurological defence mechanism, which can be


marshalled to protect us against negative thoughts and situations. Particularly in times of 20
hardship and difficulty, nostalgia compensates for uncomfortable states such as feelings of
meaninglessness or a discontinuity between past and present. Nostalgia spontaneously
rushes in and counteracts those things. Strong anecdotal evidence exists of women in
concentration camps during the Holocaust who responded to starvation by waxing nostalgic
about shared meals with their families and arguing about recipes. Scientists describe this 25
effect as being an as if loop a mechanism by which your mind can temporarily affect your
perceived body state linking it to research which showed that people were significantly
more likely to generate nostalgic emotions in a cold room than a warm one and that those
emotions had the effect of making the room seem warmer. Concentration camp survivors
describe using their memories to temporarily alter their perception of the state they were in. 30
It was not a solution, but the temporary change in perception allowed them to crucially
persevere just a bit longer.

In community experiments, research suggests that nostalgia helps build resources like
optimism or inspiration or creativity, which are correlated with mental fortitude. In difficult
situations and at lifes major transitions, nostalgia grounds us and gives us a base on which
to evaluate the present as a temporary state, and in doing so it perhaps builds resilience.
Leaving home for the first time, increasingly to study abroad, is among the most powerful of
these types of situations. By getting homesick students to describe one particularly
meaningful or positive memory, scientists have found that these memories are mostly
nostalgic narratives whether collective or personal and are predominantly positive
experiences. They do have elements of loss, maybe even trauma and sadness. But that is
posed in a redemption sequence: for example, I lost my grandmother, but we went to the
funeral and realised how close we are as a family.

It seems that as parents, people habitually and subconsciously invoke nostalgia as one
technique of helping children through difficult periods reminding them to think of happier
moments as a defence against the present and a hope for the future. Does part of
successful parenting lie in trying to lay down experiences that children can refer back to and
use in this way is that the impulse behind memorable birthdays and holidays? One of the
strongest predictors is the parents use of mental time travel. Parents who had encouraged
their children to think about past things that had been fun (and also future things that their

trendyline

35

40

45

50

3
children would do) had children who were always the highest nostalgia-prones. The ability
and encouragement to access nostalgia also builds gratitude and connectedness towards
others and tends to make children less selfish.
6

One problem parents face is that however hard parents might try to engender such feelings,
we have little control over what childhood experiences children will actually return to and
what memories they will use to create their sense of identity. Reinforcing some formative
positive experiences over and over could be one way of attempting to manipulate that
subconscious selection process. If there were to be therapeutic uses of nostalgia, they would
have to include methods to direct victims of one kind of trauma or another to positive
memories. One of the strengths of nostalgia is that even if they have not had a good
childhood, most people have at least one nostalgic memory that they cherish and that they
can use repeatedly. Once positive memories are instantiated they might have only
represented half an hour of ones entire childhood, but we can dwell on them and return to
them forever. Nostalgia in this sense is like choosing the neural pathways you want to tread
most often, like creating an inexhaustible bank account which is there for you if you want to
withdraw from it.

55

60

65

In strongly nostalgic states, individuals are shown to be more likely to commit to volunteering
or other expressions of altruism. Their sense of the value of money is weakened, leading
them to make wilful purchases. Couples use shared nostalgia narratives to create and
strengthen bonds between them. In group situations those with induced nostalgia not only 70
tend to feel more closely bonded with the group but also more willing to form intimate
associations with strangers and to be freer in their thinking. In one experiment, subjects in
whom nostalgia had been induced were asked to set up a room for a meeting those in a
nostalgic frame of mind consistently set up the chairs closer than those in the control. In
another experiment, those in nostalgic moods were asked to write essays, which were 75
compared in a blind judging process with those of peers, who had no induced feelings of
nostalgia. The essays written in a nostalgic state were judged more imaginative and creative
(storytellers, professional nostalgics, have long intuited this, not to mention poets).

Nostalgic memory is a bittersweet combination of rumination, counterfactual thinking and


nostalgia. Rumination and counterfactual thinking are related to despair and perhaps to 80
depression for instance, using memories to remind oneself of how poorly one has been
treated or to reinforce regret. In the grouping of past-oriented thought, nostalgia stands out
as adaptive it is distinct from these negative memories in that it is always related to
intimacy maintenance. People want to remind themselves of the people who are no longer
here and what they meant. It serves to remind them of what intimacy they have achieved 85
and therefore what they are capable of.

Of course advertisers and political speechwriters have long understood the power of
collective nostalgia. Is it not the fact that such feelings can also manipulate us into doing
things, buying things, voting for things, that in more coldly rational states we might resist?
The nostalgists concede this danger certainly exists, particularly in group situations. 90
Nostalgia can certainly be a nationalistic, chauvinistic tool. We have to tread carefully if we
use nostalgia as a group therapy. Anything that increases the bonds within the group also
has the power to increase the negativity towards other groups.

Adapted from Look back in joy by Tim Adams

trendyline

Candidates Name

PDG

GP Tutors Name

/14

ANDERSON JUNIOR COLLEGE


JC2 Preliminary Examination 2015

GENERAL PAPER

8807/02

Paper 2

31 August 2015
1 hour 30 minutes

Candidates answer on the Question Paper.


Additional Materials:

Insert

READ THESE INSTRUCTIONS FIRST


Write your name, PDG and GP tutors name on all the work you hand in.
Write in dark blue or black pen on both sides of the paper.
Do not use staples, paper clips, highlighters, glue or correction fluid.
Answer all questions.
The Insert contains the passage for comprehension.
Note that up to 15 marks out of 50 will be awarded for your use of language.
At the end of the examination, fasten all your work securely together.
The number of marks is given in brackets [ ] at the end of each question or part question.

For Examiners Use

Content

/35

Language

/15

Total

/50

This document consists of 6 printed pages and 1 Insert.


[Turn over

trendyline

Read the passage in the Insert and then answer all the questions. Note that up to fifteen
marks will be given for the quality and accuracy of your use of English throughout this Paper.
NOTE: When a question asks for an answer IN YOUR OWN WORDS AS FAR AS
POSSIBLE and you select the appropriate material from the passage for your answer, you
must still use your own words to express it. Little credit can be given to answers which only
copy words and phrases from the passage.

For
Examiners
Use

What does the title First Lady of Food (lines 8-9) suggest about the perception
Americans had of Betty Crocker?

...[1]

Explain what the author means by using seduce (line 20) to describe the efforts of the
companies. Use your own words as far as possible.

[2]

Explain the paradox in this illusion becomes their reality (lines 27-28).

[2]

Why does the author describe peoples attempts to take an immaculate Instagramworthy selfie as hopeless (line 42)? Use your own words as far as possible.

[2]

What do you understand by won the gene pool lottery in line 45?

[1]

Why is the word lesser (line 46) in inverted commas?

[1]

trendyline

Explain how the author illustrates his view that image has a stranglehold on society
(line 56) in paragraph 5. Use your own words as far as possible.

For
Examiners
Use

[3]
8

In paragraph 6, what explanations does the author offer for some companies profits
remaining unscathed despite their less than scrupulous practices (line 73)? Use your
own words as far as possible.

[2]

Using your own words as far as possible, explain the irony in lines 77 to 79.

[2]

10 Suggest why the evolution of Betty Crockers image over the years has helped the brand
to stay relevant (line 88).

.......[1]

trendyline

11 Using material from paragraphs 24 only (lines 1253), summarise what the author has
to say about the benefits that branding and image have for companies, and the harms
they may cause to individuals.

For
Examiners
Use

Write your summary in no more than 120 words, not counting the opening words which
are printed below. Use your own words as far as possible.
Branding and image are of benefit to companies because ...............................................

[8]

trendyline

12 Lee Anderson writes about the power of image and branding, and highlights some
possible effects on individuals and society. How far would you agree with his
observations? Relate your opinions to your own society.

For
Examiners
Use

trendyline

For
Examiners
Use

.[10]

trendyline

trendyline

ANDERSON JUNIOR COLLEGE


JC2 Preliminary Examination 2015
____________________________________________________________________________

GENERAL PAPER
PAPER 2

8807/02
31 August 2015

INSERT

1 hour 30 minutes

____________________________________________________________________________

READ THESE INSTRUCTIONS FIRST


This Insert contains the passage for Paper 2.

___________________________________________________________________________
This document consists of 3 printed pages and 1 blank page.
[Turn over

trendyline

INSERT
Lee Anderson considers the effects of branding and image on society.
1

Born in 1921 to proud corporate parents, the endearing Betty Crocker, synonymous
with boxed cake mixes and canned frosting, has been carefully nurtured into one of
the most successful branding campaigns the world has ever known. She was
originally created to provide housewives with cooking, baking, and domestic advice,
and, as her popularity burgeoned, she was given a face, a voice, and her very own
signature. Her appeal to housewives was so great that she had her own cooking show
and radio programme. She even became the second best-known woman in America
in 1945, following First Lady Eleanor Roosevelt. Previously known as the First Lady of
Food, the name of Betty Crocker remains current today, sharing advice and recipes
with loyal followers on a contemporary and well-maintained website. All this is only
made possible with clever marketing and effective branding.
Branding has never been more important than it is today, as it helps a company to
maintain its prominence in the market. Apt brand promotion is necessary especially
with the increasing levels of global competition that businesses face in most
industries. To capture new markets, companies need effective branding to penetrate
peoples wall of indifference towards their products or services and create brand heat
and product lust. In the face of current economic challenges that have been gripping
much of the modern world, it is worth noting that brands do better in tough times
compared to unbranded products. Indeed, companies that can withstand economic
recessions are those that manage to seduce the growing middle class in countries like
India, China and Brazil into buying premium brands at premium prices, arguably to a
point of obsession.
Are brands really worth the price consumers pay for them? Nobody can deny that
some brands can remain appealing for long periods because of the tried and tested
quality of their products. Still, not all brands necessarily have products that truly match
up to their name. The appeal of brands is often built around peoples perception.
People see what they want regardless of the reality that surrounds them. Indeed, this
illusion becomes their reality. If facts were to rule the day in marketing, many of the
top consumer brands would be relegated to the bottom shelf. If not for the power of
branding in manipulating perception, many new brands would not evolve beyond
mere business plans, as some recognised brands have already established a
perception of strength, quality, or leadership in the minds of consumers. It seems that
all that matters in the world of business is that consumers believe in the image that
brands strive to portray. Hence, it is not without reason that some marketing
strategists believe that having a better brand is better than having a better product.
Brand is perception and perception is reality.
Indeed, so powerful is perception that it can even devalue how we view others and
even ourselves. Many, including celebrities, go online to build a particular image
one that they may not morally agree with, and may even resort to lying and
deliberately engaging in destructive behaviour just to get more attention from others
and increase their self-worth. Such obsession has morphed conventional branding
into a highly narcissistic exercise one selfie at a time. People may make hopeless
attempts to take that immaculate Instagram-worthy selfie, only to be obsessively
analysed by others and even by themselves. On one end of the spectrum will be
those believed to have won the gene pool lottery and are deemed to be the epitome

10

15

20

25

30

35

40

45

trendyline

of beauty, and on the other, the lesser mortals. Such obsession with image can also
cause great harms to the body and bank account. A simple and mindless comment
can drive some to break the bank to go under the knife to achieve the perfect look,
only to end up as victims of botched surgeries. Others seek to complete presumably
worthy challenges planking 1, batmanning 2, and fire challenge 3 that get them burnt,
bruised, battered and bloodied. When peoples attempts to build a perfect image fail,
they often get ridiculed, which deals a further blow to their already low esteem, and
can cause them to isolate themselves from their peers and the world.
5

Despite knowing that image is superficial and that it is rarely real, people remain
fixated on their image and that of others. Seductive or repulsive, dazzling or ordinary,
chic or dated image has a stranglehold on society. No one understands this better
than people with political ambitions, who often engage a team of professional image
consultants to coach them on their physical appearance, body language, and even
the way they speak to convey an image of a strong, approachable leader. Voters are
so attuned to embracing images that they are unknowingly swayed by such subliminal
messages embedded in campaigns. At the ballot box, it is the overall package they
vote for. Candidates with sound policies but with less adept political campaigners
often end up trailing behind in the polls and may not get elected.
A well-crafted positive image weathers negative comments on controversial business
practices. Nikes Swoosh remains popular even if it has been linked to sweatshops.
Clever advertising allows companies to distract consumers from their immoral deeds.
Unilever, which owns Dove, has been accused of deforestation of the Sumatran
Rainforest to grow oil palms used in its products. Yet, Dove has grown from a $200
million soap brand in the early 1990s into a brand that is estimated to be worth nearly
$4 billion today, all thanks to the impressive cultivation of a socially responsible
image. In addition, some of these companies success can also be attributed to them
consciously associating the brand with a higher purpose and making consumers
identify with them. Hence, despite some companies less than scrupulous practices,
their profits are unscathed.
Branding can have unintended effects too. Companies can be so fixated on projecting
an impeccable image of their brand that they may not be mindful of how a diverse
audience can interpret their intended advertised messages. Sometimes, the image
which was carefully cultivated can be perceived by consumers in markedly different
ways and ironically end up reducing sales. Victorias Secrets tagline The Perfect
Body and its stereotyping of feminine beauty unwittingly communicated an offensive
message to an increasingly informed and vocal consumer base. Consumers today are
more than able to see beyond the guise of sophisticated advertising, and have no
qualms about castigating companies which they believe are promoting socially
damaging values. Companies that fail to understand evolving market sentiments can
unwittingly hurt their own image and brand beyond measure.
It is nonetheless unlikely that we are able to shake off the influence that brands have
on us completely. Brands seek to become symbols of quality and reliability in the
marketplace. Reinventing an image can help a brand stay relevant in the long haul.
Just look at how Bettys image has evolved over the years from a typical housewife
in the 1920s to a strong, empowered career woman in the 1990s, to a faceless brand
today. A powerful brand dwells and grows in the mind of the consumer.

50

55

60

65

70

75

80

85

90

planking: also known as the lying down game where the participant lies face down in unusual locations, keeping the hands along the body and the feet outstretched. When this
plank-like position is achieved, a photo is taken and then posted to the Internet.
2
batmanning: hanging upside down from something by the strength of your feet and ankles
3
fire challenge: the application of flammable liquids to one's body and then setting the liquids aflame, while being recorded on video

trendyline

trendyline

Anderson Junior College


JC2 Preliminary Examinations 2015
General Paper Paper 2 Answer Key
1. What does the title First Lady of Food (lines 8-9) suggest about the perception Americans had of Betty
Crocker? [1]
Answer

Text
She even became the second best-known
woman in America in 1945, following First
Lady Eleanor Roosevelt. Previously known
as the First Lady of Food, the name of Betty
Crocker remains current today, sharing
advice and recipes with loyal followers on a
contemporary and well-maintained website.

Americans perceived/saw Betty Crocker as an/a


1. Esteemed person (the degree of the intensity must be
suggested in the students answer)
2. in the field of food/cooking/culinary arts.

2. Explain what the author means by using seduce (line 20) to describe the efforts of the companies. Use
your own words as far as possible. [2]
Answer

Text
Indeed, companies that
can withstand economic
recessions are those that
manage to seduce the
growing middle class in
countries like India,
China and Brazil, into
buying premium brands
at premium prices,
arguably to a point of
obsession.

Companies
Idea of companies drawing consumers (force of attraction):
1. lure consumers into [1m]
To purchase products against their logic (Negative connotation):
2. making ill-advised purchases. [1m]

Note: Words in bold


cannot be lifted (as they
answer the question)
3. Explain the paradox in this illusion becomes their reality (lines 27-28). [2]
Text

Answer

Are brands really worth the price


1. Stating of the apparent contradiction in the paradox:
consumers pay for them? Nobody can
Products may not actually possess the same level of
quality in reality as what consumers think
deny that some brands can remain appealing
for long periods because of the tried and
tested quality of their products. Still, not all 2. Reason why the paradox exists:
brands necessarily have products that
However, consumers can choose to believe their untrue
perception of these brands (so strongly) that
truly match up to their name. The appeal
of brands is often built around peoples
3. Possible resolution of the contradiction in the paradox:
perception. People see what they want
regardless of the reality that surrounds them.
they think what is unreal about the brands is actually
real.
Indeed, this illusion becomes their reality. If

Get PSJ Private Tutor to Guide you through Exams Now!


Contact www.privatetutor.com.sg

trendyline

facts were to rule the day in marketing, many


of the top consumer brands would be
relegated to the bottom shelf.
4. Why does the author describe peoples attempts to take an immaculate Instagram-worthy selfie as
hopeless (line 42)? Use your own words as far as possible. [2]
Text

Answer

People may make hopeless attempts to take


that immaculate Instagram-worthy selfie,
only to be obsessively analysed by others
and even by themselves.
Words in bold cannot be lifted

The attempt to take an immaculate Instagram-worthy selfie


is a futile endeavour
1. because there will always be criticisms by the self or
their peers [1m]
2. and a picture can therefore never appear perfect. [1m]

5. What do you understand by won the gene pool lottery in line 45? [1]
Text

Answer

On one end of the spectrum will be those Some people are lucky to be born with exceptionally good looks.
believed to have won the gene pool
and whether one is born with the perfect look or otherwise is
entirely random.
lottery and are deemed to be the
epitome of beauty, and on the other, the
lesser mortals.
6. Why is the word lesser (line 46) in inverted commas? [1]
Text
On one end of the spectrum are those
believed to have won the gene pool
lottery and are deemed to be the
epitome of beauty, and on the other,
the lesser mortals.

Answer
Those who are more plain-looking may be perceived by some to
have less worth than those who are blessed with good looks but
do not truly have less worth.

7. Explain how the author illustrates his view that image has a stranglehold on society in paragraph 5.
Use your own words as far as possible. [3]
Text
Despite knowing that image is
superficial and that it is rarely real,
people remain fixated on their image
and that of others. Seductive or
repulsive, dazzling or ordinary, chic or
dated image has a stranglehold on
society. No one understands this

Answer
The author gives the example of:
1. Politicians who are able to package themselves very well (to
appear confident),
2. and the voters/people are subconsciously impressed by their
image
trendyline

better than people with political


ambitions, who often engage a team
of professional image consultants to
coach them on their physical
appearance, body language, and
even the way they speak to convey
an image of a strong, approachable
leader. Voters are so attuned to
embracing images that they
are unknowingly swayed by
such subliminal messages
embedded in campaigns. At the ballot
box, it is the overall package they
vote for. Candidates with sound
policies but with less adept political
campaigners often end up trailing
behind in the polls and may not
get elected.

3. These politicians who may not have very strong policies often
win the vote

8. In paragraph 6, what explanations does the author offer for some companies profits remaining unscathed despite their less than
scrupulous practices (line 73)? Use your own words as far as possible. [2]
Text
A well-crafted positive image weathers negative
comments on controversial business practices. Nikes
Swoosh remains popular even if it has been linked to
sweatshops. Clever advertising allows companies
to distract consumers from their immoral deeds.
Unilever, which owns Dove, has been accused of
deforestation of the Sumatran Rainforest to grow oil
palms used in its products. Yet, Dove has grown from
a $200 million soap brand in the early 1990s into a
brand that is estimated to be worth nearly $4 billion
today, all thanks to the impressive cultivation of a
socially responsible image. In addition, some of these
companies success can also be attributed to them
consciously associating the brand with a higher
purpose and making consumers identify with them.
Hence, despite some companies less than scrupulous
practices, their profits are unscathed.

Answer
1.

(Skilful) portrayal of a product turns consumers attention away from


their wrongdoings

2.

Through crafting a perception of an ethical company

Associating the brand with a higher purpose:


3. They also relate their products to more lofty ideals
Consumers therefore identify with these companies:
4. thus being able to connect with consumers.

9. Using your own words as far as possible, explain the irony in lines 77 to 79. [2]
Text
Sometimes, the image which was carefully
cultivated can be perceived by consumers
in markedly different ways and ironically
end up reducing sales.

Answer
1. One would expect that the image that is nurtured would
help appeal to customers
2. yet the image backfired and resulted in decreased profits

10. Suggest why the evolution of Betty Crockers image over the years has helped the brand to stay
relevant (line 88). [1]
Text

Answer

trendyline

Reinventing an image can help a brand


stay relevant in the long haul. Just look at
how Bettys image has evolved over the
years from a typical housewife in the
1920s to a strong, empowered career
woman in the 1990s, to a faceless brand
today. A powerful brand dwells and grows in
the mind of the consumer.

The evolution of her image manages to adapt to changing


preferences/circumstances in the world (in order to continue
to be appealing to consumers). [1m]
OR
The reason it is able to stay relevant is that it constantly
evolves to be in line with societal changes over time. [1m]
OR
The reason it is able to stay relevant is that it adapts to
changing world views about women aligned with current
sentiments. [1m]

trendyline

11. Using material from paragraphs 24 only, summarise what the author has to say about the benefits that
branding and image have for companies, and the harms they cause to individuals.
Write your summary in no more than 120 words, not counting the opening words which are printed below.
Use your own words as far as possible.
Branding and image are of benefit to companies because [8]
No.

Lifted

Banned Words

Paraphrased

From Paragraph 2 (Benefits to Companies)


1

helps a company to maintain


its prominence

Prominence

Helps a company to (continue to) stand


out / to distinguish itself (from others / in
the market)

Apt brand promotion is necessary


especially with the increasing
levels of global competition that
businesses face.
(lines 12-14)
2

To capture new markets (line 15)

Capture
Market(s)

To attract new/additional consumer pool


/ consumers / expanding economies
(economies include people and markets)

to penetrate peoples wall of


indifference towards their products
or services (lines 15-16)

Penetrate
Wall of indifference

To make them interested in the product /


attract consumers attention

Create brand heat (line 16)

Brand heat

Create hype
*Do not accept trendy

Create product lust (line 17)

Product lust

Yearn / desire / covet / crave / long for


the product

brands do better in tough times


compared to unbranded products.
Indeed, companies that can
withstand economic recessions
(lines 19-20)

Do better
Tough times
Withstand
Economic
recessions

Strong brands / companies with good


branding are able to cope with
difficult economic crises

seduce the growing middle class in


countries like India, China and
Brazil, into buying premium brands
at premium prices (line 21)

Seduce
premium

make their consumers to exorbitant


prices (for their products)

To a point of obsession (line 22)

Obsession

Where consumers keep paying high


prices in an addicted manner

Worth
Match up

Good branding allows companies to get


away with selling inferior goods

Note: Context of economics must be


present

From Paragraph 3 (Benefits to Companies)


9

Are brands really worth the price


consumers pay for them? not all
brands necessarily have products
that truly match up to their name
(lines 25-26)

trendyline

10

new brands would not evolve


beyond mere business plans (lines
30-31)

Evolve
Plans

New companies can enter the market


with branding

11

some recognised brands have


already established a perception
of strength, quality, or leadership in
the minds of consumers (lines 3132)

Established
Branding allows companies to build a
Perception
lasting reputation
of strength, quality,
or leadership

From Paragraph 4 (Harms to Individuals)


12

devalue how we view others (line


37)

Devalue

However, branding can lead to


degrading / deprecating views of others

13

(devalue how we view) even


ourselves (line 38)

Devalue

(branding can lead to degrading views)


and ourselves

14

build a particular image one that


they may not morally agree with
(line 38)

15

resort to lying (line 39)

Resort

(turn to) lying,

16

engaging in destructive behaviour


(line 40)

Destructive
behaviour

And damaging actions


*Do not accept negative behaviour (too
vague)

17

highly narcissistic exercise (line 42) Narcissistic

overly focusing on themselves

18

harms to the body (line 47)

Harms their body


Do not accept waste away (wrong
meaning)

19

(harms to the) bank account (line


47)

It can lead to people engaging in


unethical /

Bank account

Harms wealth
OR
wastes their money
(negative shade must be in the answer)

20

A simple and mindless comment


can drive some to [Inferred
impact on the people] (lines 47-48)

Needs inference,
plain lifting or literal
paraphrasing will
not get the mark

People become overly sensitive to the


perception of others (towards their
image / appearance)

21

When peoples attempts to build a


perfect image fail, they often get
ridiculed, which deals a further
blow to their already low esteem
(lines 51-52)

Blow
Esteem

Failure to achieve their desired image


can further damage their self-worth /
confidence
OR
their low self-worth / confidence is hurt
even more

22

isolate themselves from their peers


(line 53)

Isolate
Peers

Segregate themselves from their


*Do not accept general terms like
people

23

(isolate themselves from) the world


(line 53)

Isolate
World

(Segregate themselves from) and the


society
engage in anti-social behaviour
Branding separates people from the
trendyline

world
*Do not accept general terms like the
earth, the universe, the planet

No of Pts correctly
identified
15 +
13-14
11-12
10
8-9
6-7
4-5
1-3

Marks
8
7
6
5
4
3
2
1

12. Lee Anderson writes about the power of image and branding, and highlights some possible effects on
individuals and society. How far would you agree with his observations? Relate your opinions to your own
society. [10]

trendyline

CATHOLIC JUNIOR COLLEGE


JC2 Preliminary Examination 2015
General Certificate of Education Advanced Level
Higher 1
CANDIDATE
NAME
CLASS

GENERAL PAPER

8807/02

Paper 2

21 August 2015
1 hour 30 minutes

Candidates answer on the Question Paper.

READ THESE INSTRUCTIONS FIRST


Write your name and class on all the work you hand in.
Write in dark blue or black pen on both sides of the paper.
Do not use paper clips, highlighters, glue or correction fluid on your work.
Answer all questions.
Note that up to 15 marks out of 50 will be awarded for your use of language.
At the end of the examination, fasten all your work securely together.
The number of marks is given in brackets [ ] at the end of each question or part question.

For Examiners Use


Content

/35

Language

/15

Total

/50

This document consists of 8 printed pages and 1 2 blank pages.

trendyline

Read the passage and then answer all the questions. Note that up to fifteen marks will be given for
the quality and accuracy of your use of English throughout this Paper.

For
Examiners
Use

NOTE: When a question asks for an answer IN YOUR OWN WORDS AS FAR AS POSSIBLE and
you select the appropriate material from the passage for your answer, you must still use your own
words to express it. Little credit can be given to answers which only copy words and phrases from the
passage.

Questions from Passage 1


1

From paragraph 1, explainExplain why the author compares the mass tourism industry to a high
speed train racing toward a cliff edge (line 3)..? Use your own words as far as possible.

[2]

From paragraph 1, Wwhat is the author implying when he uses the word illusive (line 9)? Use
your own words as far as possible.

.......
[2]

From paragraph 2, Wwhat does the word infected (line 16) suggest about the authors attitude
towards budget travel (line 16)? Using your own words as far as possible.

.
[21]

trendyline

According to the author in paragraph 2, how are travellers different from tourists in paragraph 2?
Use your own words as far as possible.

For
Examiners
Use

.
.
.
.
[2]

In paragraph 6, explainExplain what the author means by cheap travel being a right, not a
privilege (line 567).? Use your own words as far as possible.
..
..
..
.... [2]

From paragraph 6, explainExplain what the author means by there is no magic wand or silver
bullet as used in line 589.? Use your own words as far as possible.

Formatted: No Spacing

......
..
.... [1]

trendyline

Using material from paragraphs 3 to 5, summarize what the author has to say about the current
harms posed by mass tourism.

For
Examiners
Use

Formatted: Font: (Default) Arial, Italic

Write your summary in no more than 120 words, not counting the opening words which are
printed below. Use your own words as far as possible.
The current harms of tourism are
..........
..........
..........
..........
..........
..........
..........
..
..
..
..
..
.. [8]

Formatted: Font: (Default) Arial


Formatted: Font: (Default) Arial
Formatted: Font: (Default) Calibri, Bold

trendyline

Questions from Passage 2


87

From paragraph 1, Wwhy does the author insert the phrase still exclusive in parentheses (line
6)? Use your own words as far as possible.

For
Examiners
Use

..
..
.... [1]

98

From Wparagraph 2, why is ecotourisms claim to be sustainable considered circular logic (line
11)? Use your own words as far as possible.

For
Examiners
Use

Formatted: Tab stops: 0.01", Left


Formatted: Space After: 0 pt

.
.....
.
.
.... [2]

10
9a

a.

In paragraph 2, Wwhy is the word ethical in inverted commas (line 8)?

..........
..

Formatted: Font: Arial, 11 pt


Formatted: List Paragraph, Indent: Left: 0.01", Hanging:
0.3", Numbered + Level: 1 + Numbering Style: a, b, c, +
Start at: 1 + Alignment: Left + Aligned at: 0.25" + Indent at:
0.5", Tab stops: 0.01", Left
Formatted: Tab stops: 0.01", Left

.... [1]
10
9b

b.

Explain why the author uses the word nevertheless (line 13).

Formatted: Font: (Default) Arial, 11 pt, Font color: Black

...
. [1]

11

Formatted: Font: Arial, 11 pt


Formatted: Font: Arial, 11 pt
Formatted: List Paragraph, Indent: Left: 0.01", Hanging:
0.3", Numbered + Level: 1 + Numbering Style: a, b, c, +
Start at: 1 + Alignment: Left + Aligned at: 0.25" + Indent at:
0.5"

What explanations does the author offer for the growth of dark tourism in paragraph 3? Use
your own words as far as possible.

trendyline

...
...
..... [2]

trendyline

12

Anna Pollock argues that mass tourism has reduced the value of travel, while Jim Butcher
argues that some types of mass tourism are beneficial. Which of the two authors are you more
inclined to agree with? Explain the reasons for your choice with reference to you, and your
society.

For
Examiners
Use

..........
.........
.........
.........
.........
.........
.........
.........
.........
.........
.........
.........
.........
.........
.........
.........
.........
.........
.........
.........
.........
.........
.........
.........
.........
.........
.........

trendyline

.........

For
Examiners
Use

.........
.........
.........
.........
.........
.........
.........
.........
.........
.........
.........
.........
.........
.........
.........
.........
.........
.........
.........
.........
.........
.........
.........
.........
.........
.........
.........
.........

trendyline

.........
.........

For
Examiners
Use

.........
.........
.........
.........
.........
.........
.........
.........
.........
.........
.........
.........
.........
.........
.........
.........
.........
.........
.........
..........
..........
..........
..........
..........
.........
[10]
END OF PAPER

trendyline

Formatted: Tab stops: 5.91", Left

trendyline

11

Using material from Passage 1, Paragraphs 3 to 5, summarize what the author has to say about
the current harms and future challenges posed by mass tourism.

For
Examiners
Use

Formatted: Space After: 10 pt, Tab stops: 5.91", Left


Formatted: Space After: 10 pt, Tab stops: 5.91", Left
Formatted: Tab stops: 5.91", Left

Write your summary in no more than 120 words, not counting the opening words which are
printed below. Use your own words as far as possible.

The current harms of tourism are


Formatted: Normal, Tab stops: 5.91", Left

..........

Formatted: Normal, Line spacing: single, Tab stops: 5.91",


Left

..........
............
........
..........
..........
..........
..
..
....
..
..
[8]

trendyline

Formatted: Tab stops: 5.91", Left

12

Anna Pollock argues that mass tourism has reduced the value of travel, while Jim Butcher argues that
some types of mass tourism are beneficial. Which of the two authors are you most inclined to agree
with? Explain the reasons for your choice with reference to you, and your society.

..........

For
Examiners
Use

Formatted: Space After: 10 pt, Tab stops: 5.91", Left

Formatted: Tab stops: 5.91", Left

Formatted: Normal, Line spacing: single, Tab stops: 5.91",


Left

.........
.........
.........
.........
.........
.........
.........
.........
.........
.........
.........
.........
.........
.........
.........
.........
.........
.........
.........
.........
.........
.........
.........
.........
.........

Formatted: Normal, Tab stops: 5.91", Left


Formatted: Space After: 10 pt, Tab stops: 5.91", Left

trendyline

.........

.........
.........

Formatted: Normal, Line spacing: single, Tab stops: 5.91",


Left
For
Examiners
Use

.........
.........
.........
.........
.........
.........
.........
.........
.........
.........
.........
.........
.........
.........
.........
.........
.........
.........
.........
.........
.........
.........
.........
.........
.........
.........

Formatted: Normal, Tab stops: 5.91", Left

trendyline

.........

.........

Formatted: Normal, Line spacing: single, Tab stops: 5.91",


Left

.........
.........
.........
.........
.........
.........
.........
.........
.........
.........
.........
.........
.........
.........
.........
.........
.........
.........
.........
.........
.........
.........
[10]
END OF PAPER

Formatted: Normal, Left, Line spacing: single, Tab stops:


5.91", Left

trendyline

For
Examiners
Use

Formatted: Tab stops: 5.91", Left

Formatted: Left, Space After: 10 pt, Tab stops: 5.91", Left

trendyline

BLANK PAGE

Formatted: Centered, Space After: 0 pt

trendyline

BLANK PAGE
Formatted: Centered, Space After: 0 pt

trendyline

Passage 1: Anna Pollock writes about the drawbacks of mass tourism


1

Despite the slow but steady increase in the number of enterprises claiming to be responsible
or green, the fact remains that the current system of mass international tourism is utterly
unsustainable. The industry resembles a high-speed train racing toward a cliff edge, crammed
with passengers with cheap tickets. International tourism has exploded in size since the 1950s
and swept into virtually every nook and cranny of the planet, washing up cash, jobs, golf
courses, airports and enormous amounts of real estate. Undoubtedly, tourism continues to be
a stimulant of economic recovery, generating more than $2.1 trillion in annual revenues. In
many countries and regions of the world, tourism is now the primary source of foreign
exchange, employment and cash. This illusive statistic masks the quality of jobs generated
through tourism for many developing countries such as Kenya in East Africa, which are often
low-paid, seasonal shift work.
Gleefully, the media reaps a lot of money from advertising holiday locations around the world,
but on a subliminal level, perhaps consumers have each become so addicted to the promise
and pleasures of cheap and frequent travel that the prospect of going without is simply too
much to contemplate. Perhaps, with the democratisation through budget travel, a form of
myopia has infected us all and we take for granted the chance to travel. Travel is reduced to a
superficial sojourn and we fail to see the bigger purpose of travel as a privilege to open
ourselves to new experiences. For one thing, travellers stay longer, prepare better, are more
discreet when there, and make an effort to know something of the language. Tourists move in
crowds from one must-see to another, typically looking but not always seeing, often
photographing instead of thinking, invariably devoting more energy to worries about finding a
lavatory and somewhere to eat rather than being open to new experience.
With more than 1.8 billion travellers within the next 17 years, the challenge turns out to be far
more complex than just the prospect of run-away volume growth on a finite planet. Even with
the worldwide growth of the travel industry, there remains uncertainties in visitor arrivals. With
the misguided focus on growing the tourism industry, many sectors are running themselves
into the ground financially as margins dwindle to razor thin. Meanwhile, thanks to congestion or
overuse of scarce water and land resources, many destinations are destroying the natural
landscapes and cultural attractions on which they depend. In Bali, each day, 13,000 cubic
metre of trash are thrown into the public dumps, only half of which is recycled. Colossal traffic
jams created by unchecked car growth congests many arteries: there are 13 % more cars
every year, for a mere 2.28 % increase of roads suitable for motor vehicles. The irony is stark
when the locally elected prefects in Bali are fixated on speeding up development to attract
more tourists to the extent that it is unsustainable. The environmental damage distances
tourists from the romanticised idyllic and pristine environment.
Apart from the ecological catastrophe, it is clear that mass industrial tourism would inevitably
dilute the wondrous experience of travel through sterile commodification. Travel has been
reduced to a sleekly packaged product that is based on mass-market assembly, distribution
and consumption and, as a consequence, one product is substitutable for another. The
commodification of what should be revered as unique is further aggravated by the application
of industrial cost cutting strategies of homogenisation, standardisation and automation that
further strip out any remaining vestiges of difference, let alone mystique. Tourists do places
and rarely get the chance to stand in awe and wonder at unique attractions.
The unruly behaviour of tourists also poses concerns for the host country. Some tourists
imagine that because they are abroad, they are no longer subject to the restraint of home. The
clich of the drunk English youth burned lobster-pink by the sun, yelling and brawling, dressed
in nothing but Union Jack shorts and a can of lager is all too true (though admittedly his degree
of restraint at home is not much greater). Their inconsiderate acts have caused disruption to
the lives of the people in the host country. Even the sacred historic sites are not spared from

trendyline

10

15

20

25

30

35

40

45

the mischief of tourists. A Chinese student sparked an outcry in Egypt after scratching his
name on the wall of an ancient temple in Luxor. Thai authorities issued thousands of Chineselanguage etiquette manuals after Chinese tourists were caught drying underwear at a temple,
kicking a bell at a sacred shrine and washing their feet in a public restroom. Such culturally
insensitive acts have earned the ire and frustration of local residents.

50

Regrettably, more tourism often means less benefit to the host communities. Technological
connectivity and price comparison engines have shifted purchasing power to consumers, who
have been convinced, by repeated discounting, that travel is now a right, not a privilege. We
need to develop the idea of conscious travel and start to imagine a better alternative.
Unfortunately, there is no magic wand or silver bullet - change will need to occur at the
grassroots level, one destination at a time. It will require hosts to wake up and see their world
differently - not as a resource to be exploited, but as a sacred place to be protected and
celebrated for its uniqueness.

55

Adapted from Six Reasons Why Mass Tourism in Unsustainable by Anna Pollock

trendyline

60

Passage 2: Jim Butcher claims that some forms of mass tourism have value
1

Does mass tourism deserve this level of contempt? Is it really as bad as it is painted by the
champions of ethical holidays? One place I have never heard associated with ethical tourism
is the Spanish town of Torremolinos, 50 years ago, Torremolinos was a poor fishing
community, with high rates of infant mortality and low levels of literacy. Partly due to the
development of mass package tourism, it now enjoys levels of wealth and education that
enable many Spanish people to join the (still exclusive) club of leisure travellers.
But can ecotourism sustain the high moral claims that have been made for it? As a model of
development, ecotourism's claim to be sustainable and ethical resides in its ability to link
conservation and development. It is part of the growth of integrated conservation and
development projects that attempt to bring together these two apparently competing aims
within biodiversity-rich areas. The argument is carried by its own circular logic: revenue
through ecotourism means that conservation is incentivised, and conservation ensures that
the ecotourist revenue will keep on coming. Nevertheless, ecotourism has proven itself to be a
viable solution to strengthen, nurture and encourage the community's ability to maintain and
use traditional skill, particularly home-based arts and crafts, agricultural produce, traditional
housing and landscaping, in a sustainable manner.
Another category of ethical mass tourism - popularly known as dark tourism - seeks to engage
people to ponder on the afflictions of war, poverty, destruction and death. This form of tourism
that seeks to be a sort of memorial such as Auschwitz in Poland and the Killing Fields
Museum in Cambodia often succeeds in providing an education in an appropriately sensitive
manner. Usually, they are respectfully managed. Even the most controversial forms of dark
tourism, such as disaster tourism involving visits to actual war zones, are not necessarily a
bad phenomenon. Dark tourism must be understood within the context of the fast, globalized
society that we live in today; people have a strong desire to find out what is going on and if a
disaster is on Twitter and other social media sites, they will want to see what has actually
happened. The resulting empathy would spur them to be bastions of morality in this sea of
uncertainty.
Thus, ethical forms of mass tourism though controversial are still of value to society.
Eschewing such tourism would ultimately mean sacrificing the chance of development on any
transformative scale on the altar of environmental and cultural limits.
Adapted from Good Tourist, Bad Tourist by Jim Butcher

trendyline

10

15

20

25

30

Answer Scheme for 2015 CJC GP JC2 Prelim Paper 2


1. Explain why the author compares the mass tourism industry to a high-speed train racing toward a cliff
edge (line 3)? UYOWAFAP [2]
Lifted

Paraphrase

The industry resembles a high-speed train


racing
toward a cliff edge (consequence)
Context:
the fact remains that the current system of mass
international tourism is utterly unsustainable.
International tourism has exploded in size

A) Authors Intention:
To emphasise that the mass tourism industry
is
developing
at
too
fast
a
rate
/
quickly/exponentially
B) Context
that it is almost impossible to be maintained /
supported without any implications /

2. What is the author implying when he uses the word illusive (line 9)? UYOWAFAP [2]
Lifted

Paraphrase

illusive statistics
Context:
Undoubtedly, tourism continues to be a stimulant
of economic recovery, generating more than
$2.1trillion in annual revenues... tourism is now
the primary source of foreign exchange,
employment
and
cash.
This
illusive
statistic masks the quality of jobs generated
through tourism for many developing countries
such as Kenya, which are often low-paid,
seasonal shift work.

A) Definition of Illusive:
The authors intent is to suggest that the statistics
presented
in
support
of
tourism
is deceptive/misleading
B) Context:
As the kinds of employment/jobs created to benefit
the community is of inferior standard
*B - Context should be specific to the job standard,
and not low wages / seasonal work
*Benefits must be tied to quality (substandard,
inferior)

3. What does the word infected (line 16) suggest about the authors attitude towards budget travel?
UYOWAFAP [1]
Lifted

Paraphrase

with the democratisation through budget travel, a A) Attitude: Disapproving, critical, unsupportive
form of myopia has infected us all and we take for Ans: His attitude is disapproving because infected
granted the chance to travel.
suggests that like a virus, budget travel has spread
and negatively impacted us.
The word infected suggests that like a virus,

Get PSJ Private Tutor to Guide you through Exams Now!


Contact www.privatetutor.com.sg

trendyline

budget travel has spread and negatively affected


us. As such his attitude is one of disapproval.

b) Shows understanding of the meaning of the


word infected: LIke a virus, the myopia brought
about by budget travel has spread and
negatively/unfavourably affected us.

4.
[2]

According to the author in para 2, how are travellers different from tourists? UYOWAFAP.

Lifted

Paraphrase

a)Travellers...open to new experience vs tourists


superficial sojourn/ fail to see the bigger
purpose
typically looking but not always seeing, often
photographing instead of thinking, invariably
devoting more energy to worries about finding a
lavatory and somewhere to eat rather than being
open to new experience.
b) Travellers stay longer (inferred)
c) prepare better/ make effort to learn the
language (inferred)
d) Travellers are more discreet when there
e) tourists move in crowds from one must-see to
another

a) Difference in the Mindset of Acquiring New


Things AND / OR Quality of Experience when
they are actually there
Travellers are receptive to fresh/novel encounters
whereas tourists are content with shallow
experiences which are insignificant OR looking
but not seeing/ level of interest
*Notion of openness / attitude to learning must
be captured. In
*Open needs to be rephrased, new and
experience cannot be taken without context
b) Duration of Stay (For the trip / During the
trip)
Tourists go for a brief experience, but travellers
spend more time on the trip
c) Commitment to ready oneself for the trip
(Effort expanded)
Travellers are more committed to ready oneself
for the trip than tourists

d) Regard for the locals way of life


Travellers do not want to be intrusive
on/disrespectful of the locals way of life, but
tourists do not have such reservations.
e) Willingness to break away from group
(Numbers when travelling)
Tourists prefer to stick with their group/remain in

trendyline

their comfort zone during the trip, but travellers


are willing to break away from their group/from
their comfort zone.
*Teaching Point - Answers must have some clear
sense of contrasting language, whether its A1 +
A2 (closely linked), or (A1, B1, C1, + A2, B2, C2)
*Understanding of more must be evident
NB: Each paired point gets 1 mark, 1 mark per pair.
5. Explain what the author means by cheap travel being a right, not a privilege (line 57)?
UYOWAFAP [2]
Lifted

Paraphrase

Technological connectivity and price comparison


engines have shifted purchasing power to
consumers, who have been convinced, by
repeated discounting, that travel is now a right,
not a privilege.

Consumers have been led to believe that cheap


travel is/ has become a fundamental equal
entitlement (1m)
rather than something that is exclusive / open only
to the enjoyment of a few (1m)
*Question is focusing on the notion of travel, it is a
right not because it is cheap, but because it has
become so cheap

6. Explain what the author means by there is no magic wand or silver bullet as used in line 59?
UYOWAFAP [1]
Lifted

Paraphrase

there is no magic wand or silver bullet


Context: Change will need to occur at the
grassroots level, one destination at a time.

Magic bullet means something that solves a


difficult
problem
easily
without
negative
consequences. (1m)
OR there is no quick/fast fix/solution to mass
tourism
OR Mass tourism needs to be transformed slowly/
little by little from the ground up / Alternatives to
mass tourism will not address the issue
immediately/entirely
NB: Students must refer to the subject matter of
mass tourism to obtain the mark.
*Question focuses on the nature of the solution
*Context must be linked to mass tourism

Questions from Passage 2


8. Why does the author insert the phrase still exclusive (line 6) in parenthesis? UYOWAFAP [1]

trendyline

Lifted

Paraphrase

(a) To emphasize/draw special attention to / to


Partly due to the development of mass package add in his opinion
tourism, it now enjoys levels of wealth and
education that enable many Spanish people to (b) that in spite of the easy access to mass
tourism, it remains/continues to be a privilege
join the (still exclusive) club of leisure travellers.
for/ restricted to a few
Must have both (a) and (b) to get the mark
9. Why is ecotourisms claim to be sustainable considered circular logic (Line 11)? UYOWAFAP. [2]
Lifted

revenue
through
ecotourism
means
that
conservation
is
incentivised,
and conservation ensures that the ecotourist
revenue will keep on coming.

Paraphrase
(a) Money earned / financial gains made from
ecotourism
would
encourage
the
protection/preservation of the environment,
(b) hence this will in turn/lead to attract more
money from ecotourists which will in turn
encourage ecotourism.
NB: the sense of a circular logic must be present
to award for (b)
Must have (a) + (b) for the full mark. Students can
only obtain 0m or 2m.(No 1m)

10a. In paragraph 3, why is the word ethical in inverted commas (line 8)? [1]
Lifted

Paraphrase

But can ecotourism sustain the high moral claims


that have been made for it? As a model of
development, ecotourism's claim to be
"sustainable" and "ethical" resides in its ability to
link conservation and development.

The author questions /has skeptical reservations


about the claim that ecotourism is morally right.
Do not accept: Disagree/ Critical/ Disapprove
(Para 3 is written as a counterargument)

10b. Explain the authors use of the word nevertheless in line 13. [1]
Lifted

Paraphrase

But can ecotourism sustain the high moral claims


that have been made for it? As a model of
development,
ecotourism's
claim
to
be
"sustainable" and "ethical" resides in its ability to
link conservation and development. It is part of
the growth of integrated conservation and
development projects that attempt to bring

NOTE: Answer must provide a sense of a contrast


by using these phrases:
(a) Despite his belief that the premise is weak /
although this is the case /
PLUS

trendyline

together these two apparently competing aims


within biodiversity-rich areas. The argument is
carried by its own circular logic: revenue through
ecotourism
means
that
conservation
is
incentivised, and conservation ensures that the
ecotourist revenue will keep on coming.
Nevertheless, ecotourism has proven itself to be
a viable solution to strengthen, nurture and
encourage the community's ability to maintain and
use traditional skill, particularly home-based arts
and crafts, agricultural produce, traditional
housing and landscaping, in a sustainable
manner.

(b) he still recognises / begrudgingly


acknowledges the value of ecotourism as an
answer to the local communitys
economy/sustainability
Teaching point: Authors intention
Nevertheless is a conjunction that reflects a
contrast being made, contrast must be made
between a negative point from before and positive
point

11. What explanations does the author offer for the growth of dark tourism? UYOWAFAP [2]
Lifted

Paraphrase
A. We are increasingly connected/interconnected
B. people yearn/long to learn the issues that are
taking place
C. to watch the event unfold personally / to have a
real /genuine experience / first hand experience
1-2 points = 1m
3 points = 2m

Context: even the most controversial forms of


dark tourism, such as disaster tourism involving
visits to actual war zones
Dark tourism must be understood within the
context of the fast, globalized society that we
live in today;
people have a strong desire to find out what is
going on
and if a disaster is on Twitter and other social
media sites, they will want to see what has
actually happened.

Summary
7. Using material from paragraphs 3 - 5, summarize what the author has to say about the current harms
posed by mass tourism.
Para 3
Misguided focus on growing the tourism
industry

Deluded/ Wrong emphasis / direction / illconceived in developing the tourism industry

Many sectors are running themselves


into the ground financially as margins
dwindle to razor thin

Many sectors are making extreme losses/


bearing too much costs /facing economic
collapse / bankruptcy / on a downward spiral

trendyline

*answer must reflect intensity


thanks to congestion

due to overcrowding or overpopulation

overuse of scarce water and land


resources

over-/ excessive reliance on limited natural


resources
*Accept use if explain over

destroying the natural landscapes...

harming/degrading/ruining the physical


environment / terrain

and (destroying) cultural attractions

and heritage sites / ethnic


*impact on heritage sites needs to be stated

on which they depend

on which they rely

fixated on speeding up development to


attract more tourists

the government is obsessed with increasing


the rate of growth

to the extent that it is unsustainable


(Inferred)

to the point that it cannot be maintained over


the long term/ managed / to a destructive
level

The environmental
damage distances tourists from the
environment

The ecological harm brings them further


away from / creates a rift between /
separates / isolates
(Point J must be contextualized to the
environment)

from the romanticised idyllic and pristine


environment

from the idealised untainted beauty of the


place
*K must come with J (Negativity)
*only need to paraphrase one descriptor

Para 4
inevitably dilute the wondrous experience
of travel

reduces/weakens the marvellous / amazing


pleasure of travel

through sterile

M1

through cold / clinical / mechanical

commodification

M2

Commercialization

sleekly packaged product

alluringly/ cleverly, glossily designed tours

trendyline

that are a result of mass production.


one product is substitutable for another
strategies of homogenisation,
standardisation and automation that
further strip out any remaining vestiges
of difference
the commodification of what should be
revered as unique

let alone mystique

without any wonder/special quality

Tourists "do" places

Tourists merely go through the motion of


travel. They are just going to places/
destinations without much thought.
NB: difference between Pt K and P: Focus
on the way the tourist visits the places

rarely get the chance to stand in awe


and wonder at unique attractions

they do not often have the opportunity to


marvel at distinctive/exceptional attractions

they are no longer subject to the restraint


of home

while touring, they do not need to adhere to


the social norms/rules/expectations of their
country/society

unruly behaviour
Their inconsiderate acts

their disorderly/rowdy/thoughtless behaviours

the goods become replaceable / goods are


manufactured/produced to look identical / the
items become generic
Accepted: Mass production
Lift: Differentiated

Accepted: Misbehaviours / Selfish


have caused disruption to the lives of
the people in the host country.

have disturbed / negatively affected the


locals way of life / adversely/imposed on the
lives of the locals
DNA: Harm, as its lifted from the question
already

Even the sacred historic sites are not


spared

to the extent that the respected/revered holy


places are marred/desecrated by their acts
*Lack of Respect = BOD
DNA: Defacing without context

Such culturally insensitive acts


have earned the ire and frustration of
local residents

Such behaviour has angered and irritated the


locals.
The tourists thoughtless disrespect of the
locals traditions/heritage have angered the
locals

trendyline

Marks

Points

2-3

4-5

6-7

8-9

10-11

12-13

14+

12. Application Question


Anna Pollock argues that mass tourism has reduced the value of travel, while Jim Butcher argues that
some types of mass tourism are beneficial. Which of the two authors are you more inclined to agree with?
Explain the reasons for your choice with reference to you, and your society.
Cap at 3 marks for mentioning only 1 passage
Argument from Passage A

Suggested Analysis

we fail to see the bigger purpose of


travel as a privilege to open ourselves
to new experiences.

EV (Agree): Tourism has become a popular global leisure


activity particularly for those who can afford to leave the
country to enjoy the benefits of leisure time and to get away
from the stressful confines of the workplace. Seen in this
light, mass tourism with its packaged tours and packed
itineraries, allow their clients to visit various key attractions
and sites in the limited time available to them (e.g. 3 days to
two weeks), to take their selfies and photographs and to
boast of having visited some or many significant places of
interest in that time. In such cases, the travel experience
becomes a superficial one and a numbers game, where the
purpose is to have visited a country rather than to have learnt
more about the culture and way of life of the locals. The easy
access to various countries as a result of technology and
growth of the travel industry is easily taken for granted.
(SG): The leave that the working population is entitled to is
prized and when the opportunity arises (e.g. with the
announcement of travel fairs such as the NATAS travel fair),
many Singaporeans attend such fairs in droves with the hope
of taking advantage of the special promotions to buy
packaged tours and to get the best value for the package.
The price and itineraries (number of places slated for each
tour in the given number of days) are often compared and the
better deal would be based on the amount saved on the
package for the same number of places visited. In short, the
more places visited in the shortest amount of time, the
greater the value of the visit.
EV Disagree: A sweeping generalisation made, that ALL fail
to see the bigger purpose of travel and therefore do not open
themselves up to new experiences . The author has not
considered the perspective of different groups of tourists and
their purpose of travel and the role of the tourist industry and

trendyline

different travel operators.


The number of backpackers have also risen and nature of
youth travel has also changed enormously in the past
decade. Young travelers today want, more than ever, to
enrich themselves with cultural experiences, to meet local
people and to improve their employability when they return
home. With young people traveling further, staying away for
longer, spending more, keeping in touch more and integrating
with overseas communities on a scale not seen before, the
industry is becoming far broader than ever
before. http://www.forbes.com/sites/tanyamohn/2013/10/07/th
e-new-young-traveler-boom/

(SG): Different travel operators are in the business of travel


not solely for the purpose of profit. There has been a rise of
boutique travel agencies (e.g. Lightfoot Travel/ Country
Holidays SG/ Eco Adventurers) that recognise that holidays
are more than simply opportunities for a getaway and send
their employees (who also have a passion for travel and look
to grow their knowledge on the various cultures and places of
different countries) to explore, research and make
connections with local agencies and guides. These operators
work together with their clients to plan their itineraries and
offer recommendations so that their clients can obtain the
experience that they are seeking, be it a pilgrimage or to
indulge in their passions for culture, nature and adventures to
shopping and partaking in culinary delights. The bigger travel
agencies also offer free and easy packages which allow the
client to plan their getaway more purposefully. There is also a
growing range of travel and guidebooks available for
purchase to help travellers plan their travel.
Many sectors are running themselves
into the ground financially as margins
dwindle to razor thin

EV (Agree): While not really pertinent to Singapore, this may


be true for countries who are overly-reliant on tourism and the
industry may be hit by uncertain political situations, terrorism
and health epidemic that could cause a huge dip in tourism
arrivals. Tourism is largely seasonal, and businesses in
places like Thailand are struggling from a fall in tourist
numbers due to the military coup and political protests that
have driven tourists away in 2014. The most recent example
being Malaysias Sabah due to the earthquake and Korea as
a result of the Ebola outbreak.
EV (Disagree): Not true because tourism only contributes to
10.9% of Singapores economy/GDP. Also Singapore has

trendyline

changed direction to attract more sophisticated travellers who


tend to stay longer and spend more - and as such, even with
more uncertainties in political situations and aviation
accidents, tourism receipts are still healthy despite a drop in
tourism arrivals and sports events such as F1 help to
rejuvenate the retail sector. According to STB, tourism
increased slightly to $23.6 billion in 2014 from $23 billion the
previous year. The fact that STB is pumping in more money
to galvanise attractions like the Project Jewel and revamp the
Zoo with the River Safari shows that margins have not
dwindled to razor thin.

Singapores tourism is also driven by Singapores Business


Travel and Meetings, Incentive Travel, Conventions and
Exhibitions (MICE) and this industry has continued to thrive.
In 2013, Singapore hosted 3.5 million business visitors, an
increase of 3 per cent from the same period in 2012. These
visitors spent an estimated S$5.5 billion (excluding
sightseeing, entertainment and gaming expenditure).
Singapore continues to establish thought-leadership through
new compelling content in first-in-Asia events and first-inSingapore events in 2013, such as Annual Conference of the
International Association for Philosophy and Literature, World
Pulses Convention and Maison & Objet Asia.
In total, 12 World Congresses and several key meetings were
held on the island-city in 2013, including the World Library
and Information Congress IFLA, CFA Institute Annual
Conference and Regional World Health Summit Asia.
Meanwhile, existing events such as the International
Furniture Fair Singapore and BroadcastAsia continue to
experience strong growth in attendance and/or exhibition
space, signalling Singapores continued popularity as a MICE
destination.

Travel has been reduced to a sleekly


packaged product that is based on
mass-market assembly

EV (Agree): Commercialisation has diluted the authenticity of


the the cultural experiences.
E.g. Hawaii or New Zealand where the essence of the culture
is packaged in a way that it appeals to the tourist
convenience and accessibility. E.g NZs Hangi experience
and the Hawaiians Hula dance is now more dramatized and
presented out of its historical context.
EV (Disagree): Various countries continue to preserve their
rich cultural history and have worked towards preserving the

trendyline

authenticity of the cultural monuments/environment/buildings.


E.g. Cambodians work with Japan and India to restore parts
of Ang Kor Wat in its original form. In Italy, tourists are not
permitted to take photographs or to touch the physical
structures of some churchs to ensure that there is no further
damage to the buildings.
(SG): Efforts have been made by URA to conserve historical
sites/areas such as Little India and Chinatown to retain its
authenticity and so that experience for the tourist remains
uniquely Singapore

The unruly behaviour of tourists also


poses concerns for the host country

EV (Agree): The reports of unruly tourists behaving poorly on


airlines or the host countries have been on the rise. Tourists
very often bring with them their habits which might be
considered cultural norms in their own country which has led
to problems when such tourists disrespect the rules and
cultural norms of the host country.
E.g 1: Mt Kinabalu earthquake in June was blamed on the
foreign tourists who showed disrespect to the sacred
mountain by posing naked on the mountain. Even after
having been arrested, Emil Kaminski who became infamous
among the online community after posting the pictures has
not apologised and since called Masidi Manjun, the minister
for tourism, culture and environment in the East Malaysian
state of Sabah, an "idiot" and "not a minister of tourism" for
blaming the nudists for the quake, which left at least 16
people dead.
Eg 2: Tourists China's economy saw a major boom over the
past decade raising the income levels of its middle class and
their ability to travel out of the country. A total of $102bn
(70bn) was spent by Chinese travellers overseas in 2012,
which makes them the world's biggest spenders followed by
German and US tourists, according to the UN World Tourism
Organization. Earlier this year, several Chinese tourists
reportedly outraged locals in Thailand by drying
undergarments at the airport, defecating in public and kicking
a bell at a temple. News of such reports has embarrassed
China such that the government is set to form a "blacklist" of
its citizens who behave badly as tourists abroad. The
National Tourism Administration (NTA) will create a database
of people who commit offences overseas and their names will
be passed onto police, banks and customs officials. A 64page 'Guidebook for Civilized Tourism' was also written,
advising tourists on appropriate behaviour.
EV (Disagree): Although the concerns raised are valid, the
fear of unruly tourists are only a reflection of a small group of

trendyline

tourists and is not a global phenomena


Although it must be acknowledged that such numbers are on
the rise (particularly on airlines). It is possible that the
worsening situation simply reflects societal changes where
anti-social behaviour is increasingly prevalent. However, what
is deemed acceptable on the ground takes on a completely
different complexion in the confined environment of an aircraft
cabin at 35,000 feet. From 2007 to 2013, the average was
one per 1,600 flights. In 2013, airlines reported more than
8,000 incidents, or one for every 1,370 flights.
(SG): Singaporeans are by and large respectful and are
cognizant of what good behaviour entails. There have not
been any reports of Singaporeans who misbehaved while on
tour. On the part of the SIngapore government though, there
is sufficient concern that the government has pledged to do
its bit to reign in unruly travellers. As part of a global push to
deal with the growing problem of unruly passengers,
Singapore will amend its laws to give police and other legal
bodies here the authority to charge and prosecute
wrongdoers - a process expected to take about two years.
Under current international civil aviation laws - stipulated by
the Tokyo Convention - Singapore is able to take action only
if the culprit arrives on Singapore Airlines (SIA) or other
Singapore carriers. As a result, troublemakers on foreign
carriers usually escape unaffected. The Montreal Protocol
2014, drafted by the global aviation community last year,
aims to plug this gap. Among the offences it lists is refusing to
comply with safety instructions and physically or verbally
abusing cabin crew. A CAAS spokesman said the new
protocol will provide better protection for travellers and air
crew.

Arguments from Passage B

Analysis

the impact of tourism revenues from the 1960s


played an important part in Spain's economic
modernisation, and, some even argue, in breaking
down social conservatism[1]

EX (Agree): Singapore has been recognized as a


tourist destination - indeed it has reaped the
economic benefits of tourism - with SGD 23.5 to
SGD 24 billion expected in tourist receipts this
year, tourism has become one of the countrys

trendyline

major industries.
EV (Agree): Singapore has certainly used the
gains from its tourism industry to cement the
decades-long process of modernization. The
constant upgrading and revamping of Singapores
major tourist landmarks such as the Merlion, along
with the addition of new landmarks such as the
IRs, Gardens by the Bay, ArtScience Museum,
indicate a desire purposefully taken by the
Government to constantly promote Singapore as a
hyper-modern nation to overseas tourists.
EV (Disagree): With the influx of many foreigners,
compounded by the effects of the Population
White Paper, Singaporeans have become
generally more xenophobic, or are expressing an
increasing number of xenophobic perspectives.
Singaporeans perceive these tourists negatively as people coming in to enjoy the fruits of their
labour and then leaving only after a short stint of
time, often enjoying the benefits that the country
has to offer, at the expense of the average
Singaporean.
As a model of development, ecotourism's claim to
be "sustainable" and "ethical" resides in its ability
to link conservation and development

EV (Agree) The argument prima facie is plausible.


Tourism that is informed and derives its inspiration
not only from commercial motivations but also
from moral concerns about the environment can
indeed significantly contribute to its preservation
and pave the way for further progress in this
regard if managed in the right way. Singapore, in
many different ways, is living proof of this. Apart
from being established as a commercial hub, it
has strategically built up and integrated ecological
concerns with economic considerations through its
concept of a Garden city.
And within its national boundaries, despite limited
space, certain areas of land have been set aside
for nature reserves (e.g. Bukit Timah Forest
Nature Reserve) and also its world famous
Botanical Gardens - giving rise to a win-win
situation on both fronts, in terms of generating
millions of tourist dollars and at the same time
developing and consolidating community ecoconsciousness.

trendyline

EV (Disagree) The argument is superficially


plausible. Examined more closely, its claim to
credibility becomes far less tenable and
defensible. Ever since the rise and spread of
capitalism, and its ideology of material greed and
economic exploitation, it has become increasingly
apparent (for anyone who opens their eyes) that
the desires of economic gain and ecological
preservation cannot coexist harmoniously over the
long term, and in the end, every time, the
environment loses out. Why do we now have
global warming? Why are the polar ice caps
melting? Why are we seeing more extreme
weather in this new century? Eco-tourism amounts
to nothing more than sham tourism.
In Singapore, while on the surface, it may appear
to be the case that these two projects can be
successfully played off against each other and
generate immense payoffs in terms of tourist
dollars to aid with further development and
preservation. However, as commercial interests
and the desire for profit continue to consume the
minds of the swarming masses, more and more of
Singapores natural landscape is being hacked
away to make room for more and more flyovers,
more tower blocks of offices, malls, and
apartments. Just look at Sentosa Island with what
it was twenty years ago. What is now left behind
by capitalist developers is the cleverly crafted
illusion of a garden city - flowers draped over a
pedestrian bridge amidst the sprawling concrete.
The time is fast approaching when the only
garden one may be able to see are a few potted
plants on ones balcony.

trendyline

This form of tourism that seeks to be a sort of


memorial often succeeds in providing an
education in an appropriately sensitive manner.

EV (Disagree): The purpose of acting as a


memorial for victims may be eroded due to tourists
acting in a disrespectful or unsympathetic manner.
There may be a propensity for such behaviour
especially when such an experience is so vastly
different from ones own life, when one holds onto
conventional notions of what being a tourist entails
and when one merely goes with the intention of
experiencing something thrilling in a type of
tourism that is distinctly different from other types.
Furthermore, dark tourism may be viewed as an
exploitative tool where the emotional reactions of
the visitors towards deaths are used to generate
profit. This may run contrary to the suggestion that
this is education done in an appropriately sensitive
manner.

trendyline

CANDIDATE NAME

CLASS

6C __ __

INDEX NUMBER

0 0 __ __

DUNMAN HIGH SCHOOL


General Certificate of Education Advanced Level
Higher 1

YEAR 6 PRELIMINARY EXAMINATION


GENERAL PAPER

8807/02
31 August 2015

Paper 2
INSERT

1 hour 30 minutes

READ THESE INSTRUCTIONS FIRST


This Insert contains the passage for Paper 2.

This document consists of 3 printed pages and 1 blank page.

[Turn over
DHS 2015

8807/02/Y6/15/PRELIM/INSERT

trendyline

2
Justin Thyme considers the value of time in the modern world.
1

DHS 2015

Time is such a fundamental concept that one finds it almost impossible to refer to it without
mentioning its name. Yet, it is a notion that still eludes us and scholars have trouble
coming to an agreement on its definition. From day to night, to the changing of the
seasons, every single portion of time carries with it varying significance to our survival.
One thing is for sure though, no one escapes its effects. People can only track animals for
food when there is adequate light from the sun; we need to ward against nocturnal
predators when the light goes out; the warmth of the long summer days brings with it
bountiful harvests; the frigid temperatures of the dreadfully long winter nights remind us to
ration our caches well. Even the recurring periods of dusk and dawn compel us to adopt a
daily regime. It is no wonder that throughout civilisation, we are constantly trying to perfect
our measurement of the flow of time from crude sundials and hourglasses to the precise
millisecond afforded by cesium atomic clocks just so we can live to see another day.
As much as time is a natural phenomenon, we have slowly come to realise that it is of
equal part a man-made concept as well. We gradually learn to mitigate its damaging
effects while at the same time harnessing its potential. Whether to strike while the iron is
hot or to bide ones time, we are constantly on the lookout for opportune moments to turn
the tides of war to our advantage: Richard the Lionheart led a sudden shock attack after
suffering many hours of arrow rain to decimate the Saracens in the Battle of Arsuf during
the Third Crusade; the destruction of three Roman legions in the Teutoburg Forest by
Arminius and a troop of Germans remains one of the most successful and consequential
ambushes in history. Taking a moment to observe and reflect also proves useful kinks in
opponents defence become obvious and our own flaws can be ironed out. The ones in
control of the larger situation take things one step further. Relying on Mans penchant for
forgetting details over time, unscrupulous politicians embellish stories from their countries
past to persuade the masses to support them. Although time makes fools of the majority of
us, it bestows some with unparalleled power.
For all commodities that are important and scarce we attach higher price tags to them,
time included. With the penetration of technology on a global scale, economies are now
inter-linked. It does not matter whether we are on Greenwich Mean Time or Pacific Time;
as long as someone is awake in one part of the world, the rest must follow suit or risk
losing the chance to make more money. Playing the stock market is essentially playing the
game of time we monitor trends as closely as possible, making sure that we buy and sell
at the right moment to strike a windfall, or at the very least, not plunge into bankruptcy. The
media industry exploits our obsession with time, churning out classics like Back to the
Future, Doctor Who, and The Girl who Leapt through Time, with the common theme of
protagonists trying to control time. Authors of self-help books also jump on the bandwagon
by playing on our fear of inefficiency, admonishing us about squandering away time on our
vices and then urging us to toil to make every second count. Ultimately, it is deeply
ingrained in us that time is money (as espoused by Benjamin Franklin).
For that reason, we buy cars so that we might reach our destination faster, and invest in
newer and faster laptops so that we might complete our work sooner. We outsource work
that seems mundane or trivial, so that we can redeem time that we think would otherwise
be wasted on worthless tasks. As such, we pay our domestic helpers to cook for us so that
we can focus on the more important task of educating our children, leave our canine
friends with dog-walkers at $20 for a 20-minute session so that we can just play with them
(presumably loving ones pet does not include being personally involved in taking care of
its basic needs), let concierge services remind us of our loved ones birthdays and help us
get the perfect present just in time. After all, are we not looking for the fastest and most
efficient way to show our love and respect for the ones we care for? Unsurprisingly, certain
time-saving service industries have become increasingly popular in major cities; they are
particularly patronised by young people who are mesmerised by the time-is-money notion.

8807/02/Y6/15/PRELIM/INSERT

trendyline

10

15

20

25

30

35

40

45

50

3
Indeed, service providers have embarked on an insidious conspiracy to convince us that
our time is worth the exorbitant prices, but only if we can afford them.
5

DHS 2015

That we might exchange money for time is the prerogative of those who are more affluent;
we think we have the capital and capability to fill up our pot, or more accurately, pots of
gold quickly. Typically, we believe that the exchange is a well-calculated move which
creates more productive time for us to roll in more personal profits. This obsession to count
every minute can be attributed fundamentally to meritocratic societies and individualistic
cultures which emphasise, reward and celebrate personal achievements. We have come to
wear our ranks like little scouts with multiple badges. As economies grow and incomes rise,
our finite time becomes even more valuable and we start to get more anxious about the
degree to which we have used time profitably and meaningfully. In wealthier cities like New
York, higher wages and soaring costs of living raise the value of peoples time even further
which explains why rich city-dwellers are thriftier with their minutes than residents of
Nairobi. Within cities, in the corporate world, executives everywhere are afflicted with a
perennial time-scarcity problem. Gallup, a polling company, reported in 2011 that the more
cash-rich working Americans are, the more time-poor they feel. Professionals today are
twice as likely to work long hours as their less-educated peers. Lunches (and even dinners)
now tend to be efficiently sloshed down at ones desk, with an eye on the screen and its
multiple windows. When we finally dare to leave the office, the constant beeping of our
smartphones reminds us that our work never gets done.
As we pack in more tasks in a day to save time, we invariably let time slip like sand through
our fingers. We shrug our shoulders and accept the refrain: we win some and we lose
some. Modernisation is akin to a Trojan Horse, bringing us some victories but possibly
destroying us in the midst. But who has time for such complex soul-searching, especially
when it promises no returns at the end of lengthy reflection? And so, a generation of us
continue to run lifes treadmill, hit our milestones and help our children reach theirs without
really thinking about what Life is or should be. Alas, if Time be our Idol, we would not idle.

8807/02/Y6/15/PRELIM/INSERT

trendyline

55

60

65

70

75

BLANK PAGE

Acknowledgements:
Justin Thyme 2015.

DHS 2015

8807/02/Y6/15/PRELIM/INSERT

trendyline

DUNMAN HIGH SCHOOL


General Certificate of Education Advanced Level
Higher 1

YEAR 6 PRELIMINARY EXAMINATION


CANDIDATE
NAME
CLASS

INDEX
NUMBER

0 0

GENERAL PAPER

8807/02
31 August 2015

Paper 2

1 hour 30 minutes

Candidates answer on the Question Paper.

READ THESE INSTRUCTIONS FIRST


Write your Class, index number and name on all the work you hand in.
Write in dark blue or black pen.
Do not use staples, paper clips, glue or correction fluid.
DO NOT WRITE IN ANY BARCODES.
Answer all questions.
The Insert contains the passage for comprehension.
Note that up to 15 marks out of 50 will be awarded for your use of language.
The number of marks is given in brackets [ ] at the end of each question or part question.

AQ Mark
For Examiners Use
Content

/ 35

Language

/ 15

Total

/ 50

This document consists of 6 printed pages and 1 Insert.

[Turn over
DHS 2015

8807/02/Y6/15/PRELIM

trendyline

2
Read the passage in the Insert and then answer all the questions. Note that up to fifteen marks will
be given for the quality and accuracy of your use of English throughout this Paper.

For
Examiners
Use

NOTE: When a question asks for an answer IN YOUR OWN WORDS AS FAR AS POSSIBLE and
you select the appropriate material from the passage for your answer, you must still use your own
words to express it. Little credit can be given to answers which only copy words and phrases from
the passage.

Explain the irony in lines 13.

[1]
2

In the first paragraph, explain the effects of time on our survival. Use your own words as far
as possible.

[3]
3

Explain how the examples given in lines 1521 show the ways we can turn the tides of war to
our advantage.

[2]
4

Suggest two reasons why the author compares playing the stock market to playing the game of
time in lines 3133.

[2]

DHS 2015

8807/02/Y6/15/PRELIM

trendyline

3
5

In paragraph 5, explain why the author thinks that those who are more affluent are more likely to
exchange money for time. Use your own words as far as possible.

For
Examiners
Use

[3]
6 What is the significance of the word typically in line 56?

[1]
7

In paragraph 5, what differences are there between life in New York and Nairobi? Use your
own words as far as possible.

[2]
8

What does the phrase run lifes treadmill in line 77 suggest about how we live our life?

[1]
9

Suggest why the author uses the sentence Alas, if Time be our Idol, we would not idle in line
78. Use your own words as far as possible.

[2]

DHS 2015

8807/02/Y6/15/PRELIM

trendyline

4
10 Using material from paragraphs 34 only, summarise the ways in which time is associated with
monetary value.

For
Examiners
Use

Write your summary in no more than 120 words, not counting the opening words which are
printed below. Use your own words as far as possible.

We place monetary value on time because

[8]
[ ______ words]

DHS 2015

8807/02/Y6/15/PRELIM

trendyline

5
11

In this article, Justin Thyme shares his observations on how the modern world views time. How
valuable is time to you and your society?

DHS 2015

8807/02/Y6/15/PRELIM

trendyline

For
Examiners
Use

6
For
Examiners
Use

[10]

DHS 2015

8807/02/Y6/15/PRELIM

trendyline

DUNMAN HIGH SCHOOL


General Certificate of Education Advanced Level
Higher 1

YEAR 6 PRELIMINARY EXAMINATION


CANDIDATE
NAME
CLASS

INDEX
NUMBER

6 C

GENERAL PAPER

8807/02
31 August 2015

Paper 2

1 hour 30 minutes

Candidates answer on the Question Paper.


Additional Materials:

1 Insert

SUGGESTED ANSWER KEY AND MARK SCHEME

DHS 2015

8807/02/Y6/15/PRELIM

trendyline

Question 1
Explain the irony in lines 1-3. [1]

Lifted

Inferred

Time is such a fundamental concept that one


finds it almost impossible to refer to it without
mentioning its name. Yet, it is a notion that still
eludes us and scholars have trouble coming to
an agreement on its definition.

People
do
not
have
a
standardised
understanding of time and we would not have
expected people to be clear about what we are
referring to and yet we use the term time as if
we all know what we all mean.
We expect people to have a clear definition of
time when we always talk about it but the truth is
that we do not even have a standardised
understanding of time.
OR
We have multiple definitions for time but we
have only one name for it.
Key idea: There is shared understanding and yet
we have problems defining it.
Students need to try to close the gap
between the text and understanding of the
word
irony.
Mere
paraphrase
of
fundamental is insufficient.

Question 2
In paragraph 1, explain the effects of time on our survival. Use your own words as far as
possible. [3]
Lifted

Paraphrased

(a) People can only track animals for food when


there is adequate light from the sun;
(b) we need to ward against nocturnal predators
when the light goes out;
(c) the warmth of the long summer days brings
with it bountiful harvests;
(d) the frigid temperatures of the dreadfully long
winter nights remind us to ration our caches well.
(e) Even the recurring periods of dusk and dawn
compel us to adopt a daily regime.

a) People could only hunt when there is enough


sunlight / in the day
b) we need to guard against / protect ourselves
from animals in the night
c) heat in summer is needed to grow food in
abundance
(reference to seasons required; sunlight
only not acceptable)
d) extreme cold in winter signals for us to
portion / budget/ save our supplies
(reference to seasons; no need degree)
(reference to seasons)
e) even the transitional/ diurnal periods of the
day force us into a routine
1-2 points = 1m
3-4 points = 2m
5 points = 3m

Get PSJ Private Tutor to Guide you through Exams Now!


Contact www.privatetutor.com.sg
DHS 2015

8807/02/Y6/15/PRELIM

trendyline

3
Question 3
Explain how the examples given in lines 15 21 show the ways we can turn the tides of war to our
advantage. [2]
Lifted

Inferred

Whether to strike while the iron is hot or


to bide ones time, we are constantly on the
lookout for opportune moments to turn the tides
of war to our advantage:

Key elements: surprise and patience


Should be about strategy and not the result
Needs to be explained in relation to the
example/ context. Can collapse 2 examples.

Richard the Lionheart led a sudden shock attack


after suffering many hours of arrow rain to
decimate the Saracens in the Battle of Arsuf
during the Third Crusade;

Richard the Lionhearts unexpected charge /


offense shows how we need to act at the right
moment or risk losing the chance forever. [1m]

the destruction of three Roman legions in the


Teutoburg Forest by Arminius and a troop of
Germans remains one of the most successful
and consequential ambushes in history.

Arminius lying in wait to trap the enemy shows


how we should wait patiently for a good
opportunity to act. [1m]

Question 4
Suggest two reasons why the author compares playing the stock market to playing the game of time
in lines 32 33. [2]
Lifted

Suggested Answer
We follow economic changes across time so as
to make calculated decisions.
In both, actions have consequences. A move at
the wrong time will lead to bad consequences.

Playing the stock market is essentially playing


the game of time
we monitor trends as closely as possible,
making sure that we buy and sell at the right
moment to strike a windfall at the very best,
or at the very least,
not plunge into bankruptcy.

Both require us to be alert/ watchful to know the


precise moment to act.
Both need to strategise/ need tactical thinking
to seize the opportune moment.
We compete with each other for time and in
playing games.
1m each (max 2 m)
Must have time, stock market and quality of
game (parallel to game must be clear).
Element of time must be capture in the
answer.
stock market need not be paraphrased.

DHS 2015

8807/02/Y6/15/PRELIM

trendyline

4
Question 5
In paragraph 5, explain why the author thinks that those who are more affluent are more likely to
exchange money for time. Use your own words as far as possible. Use your own words as far as
possible. [3]
Lifted

Paraphrased

We (a) think we have the (b) capital and (c)


capability to fill up our pot, or more accurately,
pots of gold quickly.

They (a) imagine/ perceive that they have the


(b) resources/ financial resources/ money and
(c) ability/ talent to make even more money at a
fast pace.

Typically, we (a) believe that the exchange is a


(d) well-calculated move which creates more
productive time for us to roll in more personal
profits.

They are also (a) certain that they have


(d) well thought of ways to achieve success
more quickly.

As economies grow and incomes rise, our


finite time becomes even (e) more valuable and
we start to (f) get more anxious about the degree
to which we have used time profitably and
meaningfully.

As our societies become richer, our limited time


becomes (e) more precious, and we are (f) more
worried if we are maximising the time.
1-2 points = 1m
3-4 points = 2m
5-6 points = 3m
point (a) = imagine/ assume/ certain; should only
be credited once. believe = 0m

Question 6
What is the significance of the word typically in line 56? [1]
Lifted

Inferred

That we might exchange money for time is the


prerogative of those who are more affluent; we
think we have the capital and capability to fill up
our pot, or more accurately, pots of gold quickly.
Typically, we believe that the exchange is a wellcalculated move which creates more productive
time for us to roll in more personal profits.

It tells us that most people/ normally share the


opinion that richer people should use money to
exchange for time so that they might make
even more money at a faster pace.
[need meaning of typically and context)

Question 7
In paragraph 5, what differences are there between life in New York and Nairobi? Use your own
words as far as possible. [2]
Lifted

Paraphrased

In (a) wealthier cities like New York, higher


wages and (b) soaring costs of living raise the
value of peoples time even further which
explains why rich city-dwellers are (c) thriftier
with their minutes than residents of Nairobi.
Within cities, in the corporate world, executives
DHS 2015

(a) Rich vs poor/ High vs low salary


(b) Expensive vs Cheaper living expenses
(c) Busy vs Free / Lack time vs Have time

8807/02/Y6/15/PRELIM

trendyline

5
everywhere are afflicted with a perennial timescarcity problem. Gallup, a polling company,
reported in 2011 that the more cash-rich
working Americans are, the more time-poor they
feel. Professionals today are twice as likely to
work long hours as their less-educated peers.
Lunches (and even dinners) now tend to be
efficiently sloshed down at ones desk, with an
eye on the screen and its multiple windows.
(d) When we finally dare to leave the office, the
constant beeping of our smartphones reminds us
that our work never gets done.

(d) Afraid of losing their jobs vs Free from


worries
1-2 differences = 1m
3-4 differences = 2m
MUST COMPARE

Question 8
What does the phrase run lifes treadmill in line 77 suggest about how we live our life? [1]
Lifted

Suggested Answer

And so, a generation of us continue to run lifes


treadmill, hit our milestones and help our
children reach theirs without really thinking about
what Life is or should be.

It suggests that we have to complete our


personal list of endless cycle OR
tedious of tasks OR futile (busy yet gets
nowhere).

Question 9
Suggest why the author uses the sentence Alas, if Time be our Idol, we would not idle in line 78.
Use your own words as far as possible. [2]
Lifted

Paraphrased

Alas, if Time
be our Idol, we
would not idle.

(a) When we worship time,


(b) we would therefore think that it is crucial to make full use of time/ use it stingly.
(c) (Authors intent) The author then bemoans/ laments that we are not spending
more time on things which are meaningful.
Will accept: The author wants to emphasise that we should be spending more
time on things which are meaningful.
1-2points = 1m
3points = 2m

DHS 2015

8807/02/Y6/15/PRELIM

trendyline

6
Question 10
Using material from paragraphs 3-4 only, summarise the ways in which time is associated with monetary
value.
Write your summary in no more than 120 words, not counting the opening words which are printed
below. Use your own words as far as possible.
Paragraph 3
a
b
c
d
e
f
g
h
i
j
k
l
m
n

o
p

q
r
s
t

DHS 2015

Lifted

Paraphrased
We place higher monetary value on time because

For all commodities that are important


and scarce we attach higher price tags to
them, time included.
With the penetration of technology on a
global scale, economies are now interlinked.
as long as someone is awake in one part of
the world, the rest must follow suit
or risk losing the chance to make more
money.
Playing the stock market is essentially
playing the game of time we monitor
trends as closely as possible,
making sure that we buy and sell at the
right moment to strike a windfall, or at the
very least, not plunge into bankruptcy.
The
media
industry
exploits
our
obsession with time,
churning out classics with the common
theme of trying to control time.
Authors of self-help books (play) on our
fear of inefficiency,
admonishing us about squandering away
time on our vices
and then urging us to toil to make every
second count.
Ultimately, it is deeply ingrained in us that
time is money
Paragraph 4
we buy cars .. and invest in newer and
faster laptops
so that we might reach our destination
faster so that we might complete our
work sooner/ looking for the fastest and
most efficient way
We outsource work that seems mundane
or trivial,
so that we can redeem time that we think
would otherwise be wasted on worthless
tasks.
are we not looking for the fastest and most
efficient way to show our love and respect
for the ones we care for?
certain time-saving service industries have
become increasingly popular in major
cities;
service providers have embarked on an
insidious conspiracy to convince us
that our time is worth the exorbitant prices,
but only if we can afford them.

it is significant and limited.


As technology becomes widespread, markets are
now inter-dependent.
Hence, we need to stay up all day/ watch the market
all day
in order to acquire more money/ earn more profits.
Playing the stock market means that we observe
patterns intently
and ensure that our decisions are opportune/
executed in a timely manner.
The film business capitalises on our fascination with
time,
providing us with numerous movies with the similar
idea of attempting to master/ manipulate time.
Writers of self-help books take advantage of our
insecurity about our incompetence,
warn us about wasting time on bad habits,
and exhort us to work hard to maximise our use of
time.
Consequently, the value of time is firmly entrenched
in us/ we strongly believe in the value of time.
We purchase advanced gadgets
as they help us to finish our work more quickly.
NB: comparative element must be present.
We pay for others to do our work
in order to regain/ free time/ in order to exchange for
time that is otherwise lost on useless jobs.
NB: Student to get point if able to paraphrase
redeem or wasted
These are done to show affection/ appreciation/
care for our dear ones.
Particular service providers which help us to increase
efficiency/ productivity have become more in
demand in large cities.
Service companies collude to persuade those
with deep pockets to pay excessive amounts for
their services.

8807/02/Y6/15/PRELIM

trendyline

7
Point-Mark Table for Summary Question
Number of Points
12
34
56
78
9
10 11
12 13
> 14

Marks Awarded
1
2
3
4
5
6
7
8

Question 11
In this article, Justin Thyme shares his observations on how the modern world values time. How
valuable is time to you and your society? [10]
Ideas from Paragraphs
Para 1- introduction
Everyone understands the meaning of time even though there is no common definition.
Keeping track of time is crucial to our survival.
Para 2
We have learnt to effectively use time to our advantage (in times of war and conflict and in politics).
Para 3
Time is scarce and hence we attach a high price tag to it.
Due to globalisation, we need to seize time to make money.
The media exploit our obsession with time; authors exploit our fear of being inefficient.
The belief that time is money is firmly entrenched.
Para 4
We believe that time is money and spend money on gadgets and services so that:
- we do not waste time on trivial pursuits; and
- we can use the time freed up for more worthwhile tasks.
But only if we can afford to pay the hefty prices for these.
Para 5
The more affluent believe that they can buy time with money as they have the resources and ability to do
so.
Our obsession to make every second count stems from meritocratic societies and individualistic cultures
which value personal achievements
High costs of living make us more obsessed with making full use of time.
This is why in wealthier cities, people rush through their days and they constantly worry about not getting
sufficient work done.

DHS 2015

8807/02/Y6/15/PRELIM

trendyline

8
Ideas from Paragraphs
Para 6 Conclusion
We are trapped in this constant cycle of doing and achieving more when we should be spending time
more meaningfully.

DHS 2015

8807/02/Y6/15/PRELIM

trendyline

HWA CHONG INSTITUTION


JC2 Preliminary Exam
Higher 1
CANDIDATE
NAME

CT GROUP

14A/S

GP TUTOR
CENTRE
NUMBER

INDEX
NUMBER

General Paper

8807/02

Paper 2

26 August 2015

Additional Materials: Insert

1 hour 30 minutes

INSTRUCTIONS TO CANDIDATES
1) Write your name, CT class and GP tutors name clearly in the spaces at the top of this page.
2) Write in dark blue or black pen in the spaces provided in the answer booklet.
3) Do not use paper clips, highlighters, correction fluid or tape.
4)

Answer ALL questions.

5) You may attach additional pieces of writing paper if necessary.


There are 12 questions in this paper.
The number of marks is given in brackets [ ] at the end of each question or part question.
Note that up to 15 marks out of 50 will be awarded for your use of language.

You are reminded of the importance of legible handwriting and good presentation.
Staple the passage to the back of this answer booklet at the end of the examination.
For Examiners Use
Content

/ 35

Language

/ 15

Total

/ 50

This answer booklet consists of 6 printed pages.

trendyline

Read the passage and then answer all the questions which follow below. Note that up to fifteen marks
will be given for the quality and accuracy of your use of English throughout this paper.
NOTE: When a question asks for an answer IN YOUR OWN WORDS AS FAR AS POSSIBLE and you
select the appropriate material from the passage for your answer, you must still use your own words to
express it. Little credit can be given to answers which only copy words or phrases from the passage.
1

Why does the author refer to the pedestrian as an extremely fragile, virtually extinct species (lines
3-4)? Use your own words as far as possible.
..............................................................................................................................................
........................................................................................................................................................[1]

In paragraph 2, the author describes civic spaces as stages for our public lives (line 10) and bridge
building places (line 12). Explain what she means in each case. Use your own words as far as
possible.
..............................................................................................................................................
..............................................................................................................................................
..............................................................................................................................................
........................................................................................................................................................[2]

Why does the author say that walking through certain urban areas can be an unnerving experience
(line 19)? Use your own words as far as possible.
..............................................................................................................................................
........................................................................................................................................................[1]

How did the introduction of a pedestrian district transform Copenhagens city centre? Use your own
words as far as possible.
..............................................................................................................................................
..............................................................................................................................................
..............................................................................................................................................
........................................................................................................................................................[2]

Explain the authors use of the word trumpet in line 43. Use your own words as far as possible.
..............................................................................................................................................
........................................................................................................................................................[1]

trendyline

For
Examiners
Use

Using material from paragraphs 4 and 5 only, summarise what the author has to say about what is
wrong with cities today, the changes that need to be made and the benefits they would bring.
Write your summary in no more than 120 words, not counting the opening words which are printed
below. Use your own words as far as possible.
Cities need to change because.................................................................
..............................................................................................................................................
..............................................................................................................................................
..............................................................................................................................................
..............................................................................................................................................
..............................................................................................................................................
..............................................................................................................................................
..............................................................................................................................................
..............................................................................................................................................
..............................................................................................................................................
..............................................................................................................................................
..............................................................................................................................................
..............................................................................................................................................
..............................................................................................................................................
..............................................................................................................................................
..............................................................................................................................................
..............................................................................................................................................
..............................................................................................................................................
..............................................................................................................................................
..........................................................................................................................................[8]

What similarity does the author draw between cave paintings and Creative Placemaking? Use
your own words as far as possible.
..............................................................................................................................................
..............................................................................................................................................
..............................................................................................................................................
........................................................................................................................................................[2]

trendyline

For
Examiners
Use

Genuinely green citiesprettify shopping malls (lines 68-70). What does this sentence tell us
about the authors opinion of current attempts to make cities look greener? Use your own words
as far as possible.
..............................................................................................................................................
..............................................................................................................................................
..............................................................................................................................................
........................................................................................................................................................[2]

In paragraph 8, show how the author employs an extended metaphor to illustrate the fact that cities
are in urgent need of transformation. Use your own words as far as possible.
..............................................................................................................................................
..............................................................................................................................................
..............................................................................................................................................
........................................................................................................................................................[2]

10

According to the author in paragraph 9, what are the main obstacles that currently stand in the
way of transforming cities? Use your own words as far as possible.
..............................................................................................................................................
..............................................................................................................................................
..............................................................................................................................................
..............................................................................................................................................
..............................................................................................................................................
........................................................................................................................................................[3]

11

Explain the relevance of the final paragraph to the title of the article. Use your own words as far
as possible.
..............................................................................................................................................
........................................................................................................................................................[1]

trendyline

For
Examiners
Use

12

Alissa Walker severely criticises modern cities in general and argues that there is an urgent need
to make them more liveable. Discuss the relevance of her observations and recommendations to
your country in particular.
..............................................................................................................................................
..............................................................................................................................................
..............................................................................................................................................
..............................................................................................................................................
..............................................................................................................................................
..............................................................................................................................................
..............................................................................................................................................
..............................................................................................................................................
..............................................................................................................................................
..............................................................................................................................................
..............................................................................................................................................
..............................................................................................................................................
..............................................................................................................................................
..............................................................................................................................................
..............................................................................................................................................
..............................................................................................................................................
..............................................................................................................................................
..............................................................................................................................................
..............................................................................................................................................
..............................................................................................................................................
..............................................................................................................................................
..............................................................................................................................................
..............................................................................................................................................
..............................................................................................................................................
..............................................................................................................................................
..............................................................................................................................................
..............................................................................................................................................

trendyline

For
Examiners
Use

..............................................................................................................................................
..............................................................................................................................................
..............................................................................................................................................
..............................................................................................................................................
..............................................................................................................................................
..............................................................................................................................................
..............................................................................................................................................
..............................................................................................................................................
..............................................................................................................................................
..............................................................................................................................................
..............................................................................................................................................
..............................................................................................................................................
..............................................................................................................................................
..............................................................................................................................................
..............................................................................................................................................
..............................................................................................................................................
..............................................................................................................................................
..............................................................................................................................................
..............................................................................................................................................
..............................................................................................................................................
..............................................................................................................................................
..............................................................................................................................................
..............................................................................................................................................
..............................................................................................................................................
..............................................................................................................................................
..............................................................................................................................................
..............................................................................................................................................
..............................................................................................................................................
......................................................................................................................................................[10]

trendyline

For
Examiners
Use

HWA CHONG INSTITUTION


JC2 Preliminary Exam
Higher 1
CANDIDATE
NAME

CT GROUP

CENTRE
NUMBER

INDEX
NUMBER

General Paper

14A/S

8807/02

Paper 2

26 August 2015

INSERT

1 hour 30 minutes

INSTRUCTIONS TO CANDIDATES
This Insert contains the passage for Paper 2.

This document consists of 3 printed pages and 1 blank page.

trendyline

RECLAIMING OUR CITIES


1

Today, the majority of mankind lives in soul-crushing cities plagued by belching traffic, bloated roads,
emaciated sidewalks, and shrivelled trees. In a hostile environment where the automobile is the alpha
predator, the pedestrian has become an extremely fragile, virtually extinct species. Under the right
conditions, this creature thrives and multiplies, but creating those conditions requires urgent attention to
a broad range of challenging criteria before our urban environments degenerate into post-apocalyptic 5
nightmares. Success in the transformation of our cities will depend upon the adoption of a holistic
approach that integrates efforts to make them engaging, enriching, and liveable.

Civic spaces are key elements of individual and social well-being, the places of a communitys
collective life, expressions of their cultural richness and a foundation of their identity. When they work
well, they serve as stages for our public lives: they can be the settings where celebrations are held, 10
where social and economic exchanges take place, and where friends run into each other. Civic spaces
are also bridge-building places which draw a diverse population that can include the elderly, teenagers
and children, as well as an ethnic and cultural mix. A successful civic space acts as a common ground
which encourages people to integrate, get involved and take pride in the area. While this ambitious
mixture of attributes is a complex puzzle to piece together, it is increasingly sought after by civic 15
leaders who are desperately trying to reknit the unravelling fabric of urban life.

While only a century ago public spaces almost everywhere were crowded with people, many are nearly
empty now. Walking through certain communities can be an alienating, not to mention extremely
unnerving, experience, as if the whole place had been evacuated for an emergency that no one told you
about. Cultures and climates differ all over the world, notes Jan Gehl, but people are the same. They 20
will gather in public if you give them a good place and reason to do it. Gehl, an international consultant
and professor of urban design, has charted the progress of Copenhagen's central pedestrian district
since it opened in 1962. At that time, the pavements were deserted and cars infested the streets. The
pedestrian zone was conceived as a way to revitalise a moribund city centre. It has been expanded
incrementally each year ever since, with parking spaces gradually removed, and biking and public 25
transport facilities improved. Sidewalk cafes, once thought to be exclusive to the Mediterranean, have
become the life blood of Copenhagen's social life places of encounter, conversation and debate. The
pedestrian district is now the pulsing heart of a reinvigorated city.

Too many cities suffer from streets dedicated to moving multitudes of cars quickly a goal that
effectively eliminates foot traffic and precludes the social intercourse vital for healthy street life. Rich 30
rewards, Jeff Speck, author of Walkable City argues, await cities that move to tame traffic and put
pedestrians first by creating attractive streetscapes with congenial environments: in other words, truly
walkable places. More than a utopian notion, the walkable city is an eminently feasible solution to a
number of pressing problems that affect our overall health as a society. Much more than a faddish
amenity, walkability is an ecological imperative, and to an increasing extent, as fuel costs spiral, a 35
financial one as well. Revitalising streets for walking, gathering and shopping also makes broader
economic sense. In New York, greenmarkets are not only helping to bring conviviality to
neighbourhoods but also providing opportunities for small-scale entrepreneurship. In Arkansas, Little
Rocks River Market has helped bring the citys downtown to life, spurring the development of cultural
amenities such as museums and galleries as well as a host of recreational facilities such as a 40
skateboarding park and a sports arena.

Truly walkable streets need to be lined with aesthetically appealing buildings, not the cold monolithic
monstrosities with grim facades that most modern cities erect to trumpet their global status. This is
where the conscientious conservation of heritage buildings plays a crucial role in injecting architectural
charm, testifying to a citys unique historical personality and celebrating its rich cultural identity. But 45
meaningful conservation is not merely about preserving the faade of a building, much less converting
vital places into mouldy museums. Whilst they have tremendous value as historical icons, heritage
buildings must also remain organically connected to city life. This requires not only careful resurrection
but also judicious repurposing to maintain them as living entities that breathe character, proclaiming
the identity of vibrant communities where people still live and work. Such edifices not only strengthen a 50
sense of belonging by forging emotional bonds to places but also sustain a sense of collective memory
even as we move into an uncertain future. They anchor people securely amidst the swirling currents of
globalisation which threaten to cast them adrift.
trendyline

For cities to thrive, their inhabitants need to encounter art, to enjoy performances and to participate in
a wide range of cultural activities. From Shakespeare in the Park to string quartets at a downtown 55
plaza, good places foster and enhance a citys cultural dynamic. In America, Creative Placemaking is
a new concept pioneered by a number of foundations such as ArtPlace America and the National
Endowment for the Arts which involves reimagining and reinventing public spaces using arts and
culture as the lynchpin in building vibrant urban communities. In San Jose, a grant was awarded to a
project to turn an abandoned park into an urban living room for the arts. Whether on a local, state or 60
national level, Creative Placemaking is not only a potent strategy for civic renewal, but also a window
into something deeper. Starting with the first cave paintings 40,000 years ago, we have been using art
to transform places that feel dark and menacing into places that are inviting and vibrant. From primitive
hand prints of blue and red pigment on gloomy cavern walls to Project Storefront in Connecticut which
transforms abandoned and derelict New Haven storefronts into artists studios, the creative process 65
that informs placemaking is an unbroken chain. What defines us, and the places where we live and
work, is our art.

Finally, our dismally grey cities desperately need to go green. Genuinely green cities are a far cry from
those of today where token trees camouflage congested roads or manicured shrubs in concrete
troughs vainly attempt to prettify shopping malls. The Urban Greenspaces Institute of Vancouver 70
wants to create liveable and loveable cities where the built and natural environments are interwoven,
not set apart. Collaborating with government agencies, businesses, architects and landscape
architects to achieve its mission, it is committed to the restoration of a vibrant green infrastructure in
our cities comprising interconnected systems of healthy ponds, parks and recreational nature trails. Its
motto, In Livable Cities is Preservation of the Wild, reflects its philosophy that a prerequisite of any 75
well-designed city should be that it is nature-rich for the ecological health, civic vitality and overall
quality of community life.

Cities today are social, cultural, aesthetic and environmental wastelands. Transforming such
inhospitable and arid landscapes requires equal parts sensitivity and rigour. Sensitivity, because
planners need to realise that every city is not only a unique cultural and historical product, but also an 80
evolving organism. Rigour, because drastic measures need to be applied and scrupulous attention paid
to enable their present revival and ensure their eventual blossoming.

But before the resuscitation of the city can commence, we need to first rid ourselves of the soulless
planners whose cookie cutter designs suffocate them, banish the faceless corporations whose colossal
towers smother them, and outlaw the gas-guzzling brutes that choke them. We need to return the city to 85
its rightful owners the people. As American author and urbanist Jane Jacobs so astutely cautioned
several decades ago: "Cities have the capability of providing something for everybody, only because,
and only when, they are created by everybody."
Adapted from an article by Alissa Walker

trendyline

BLANK PAGE

trendyline

Hwa Chong Institution


2015 JC2 Preliminary Examination
Paper 2 Answer Key
1. Why does the author refer to the pedestrian as an extremely fragile, virtually extinct species (line 3)? (1)
Lift from Passage
Paraphrase/appreciation
In a hostile
Either: By using the phrase alpha predator, the author presents an image of the car
environment where
prowling cities in a menacing way, preying on vulnerable people who are too fearful of
the automobile is the
their lives to venture outside/walk the streets. Hence, pedestrians are analogous to an
alpha predator
endangered species.
(students might also mention the emaciated sidewalks and belching traffic as further
evidence of a hostile environment for pedestrians in terms lack of safety and pollution)
Or (put more simply):
Cars dominate cities and people do not want to walk the streets anymore for fear of
being run over hence the pedestrian is an increasingly rare sight/ fast disappearing from
cities.
(Any sensible answer = 1 mark)
2. In paragraph 2, the author describes civic spaces as stages for our public lives (line 10) and bridge-building places
(line 12). Explain what she means in each case. Use your own words as far as possible.
Explanation
Lift from Passage
they serve as stages for our collective lives: they can Physical facilitation (STAGE)
They are the platforms/backdrops/settings/focal
be the settings where celebrations are held, where
social and economic exchanges take place, and where points/venues where we perform/enact our
personal/group/communal interactions (with the added
friends run into each other.
implication of others being the audience) (1)
Civic spaces are also bridge building places which
draw a diverse population that can include the elderly,
teenagers, and children, as well as an ethnic and
cultural mix. A successful civic space acts as a
common ground which encourages people to
integrate, get involved and take pride in the area.

Social facilitation (BRIDGE)


They bring together different groups/sections of the
community (with different interests, perhaps conflicting
views/attitudes), thus giving them the opportunity to
communicate/mingle and thereby bond/connect/come
together as a harmonious community. (1)

3. Why does the author say that walking through certain urban areas can be an unnerving experience (line 19)? Use
your own words as far as possible. (1)
Explanation
Lift from Passage
Walking through certain communities can be an
It can be an unsettling/ discomforting/ disconcerting/
alienating, not to mention extremely unnerving,
daunting feeling / instil a sense of foreboding (1/2)
experience, as if the whole place had been
evacuated for an emergency that no one told you
as the area is inexplicably and ominously/ eerily desolate
about.
(like a wasteland), so people instinctively feel that
something is very wrong/something terrible has
happened. (1/2)
4. How did the introduction of a pedestrian district transform Copenhagens city centre? Use your own words as far
as possible. (2)
Paraphrase
Lift from Passage
At that time, the pavements were deserted and cars Before:
infested the streets. The pedestrian zone was
Dead/dying/lifeless/in decline (1/2)
conceived as a way to revitalise a moribund city
Because:
centre.
It was overrun by traffic and there were no pedestrians/
It has been expanded incrementally each year ever
the sidewalks were empty. (1/2)
since, with parking spaces gradually removed, and
biking and public transport facilities improved.
After:
Sidewalk cafes, once thought to be exclusive to the
ReMediterranean, have become the life blood of
energised/resurrected/revived/revitalized/vibrant/lively
Copenhagen's social life places of encounter,
(1/2)
conversation and debate. The pedestrian district is
now the pulsing heart of a reinvigorated city.
Because:
Get PSJ Private Tutor to Guide you through Exams Now!
Contact www.privatetutor.com.sg

trendyline

(with private transport reduced/restricted and public


transport increased) there was an influx of cafes which
created a bustling hive of activity where people
converged to meet, talk and exchange opinions. (1/2)
*information in brackets not required for mark
5. Explain the authors use of the word trumpet in line 43. (1)
Lift from Passage
Inference
to trumpet their
The use of the word suggests that the buildings were used to
global status
loudly/arrogantly/boastfully/proudly proclaim/vaunt/broadcast that the cities have arrived
or attained a certain/elevated standing/position in the world.
(Allow show off, boast)
(do not accept announce/declare as these do not adequately capture the negative
connotation of brash arrogance or crowing about an achievement in a strident manner)
6. Using material from paragraphs 4 and 5 only, summarise what the author has to say about what is wrong with
cities today, the changes that need to be made and the benefits they would bring. Write your summary in no more
than 120 words, not counting the opening words which are printed below. Use your own words as far as
possible.

Cities need to change because


Lift from Passage
suffer from streets dedicated to
moving multitudes of cars quickly a
goal that effectively eliminates foot
traffic
and precludes the social intercourse
vital for healthy street life

Paraphrase
roads exclusively allocated to moving large amounts
of/droves/hordes of traffic/vehicles (allow cars) essentially
exclude/prohibit pedestrians/stop people walking on the streets
which prevents/bars/stops/obviates the communal/societal
gatherings/events/activities crucial/essential/imperative for a
vigorous/lively/robust community life (allow street life)

tame traffic and put pedestrians first

We need to curb/control/severely restrict/regulate vehicles in favour


of pedestrians/walkers/and prioritise pedestrians

creating attractive streetscapes with


congenial environments

Fashioning/establishing appealing street scenes with


pleasant/warm/welcoming/inviting/friendly/sociable
surroundings/settings

truly walkable places

spaces genuinely suited for walking

overall health as a society

general well-being as a community

An eminently feasible solution to a


number of pressing problems
ecological imperative

A viable/practical/workable/practicable/realisable answer to several


urgent/critical/acute difficulties
environmental necessity/ crucial/ vital/ critical environmental
obligation/requirement

as fuel costs spiral, a financial one as


well

And, as energy/gas/petrol prices continues to sky-rocket/rapidly


rise/increase, a monetary/economic/fiscal one as well

10

Revitalizing streets broader


economic sense

Re-energising/reviving streets (allow) has a greater/wider/larger


financial/monetary benefit/advantage

11

bring conviviality to neighbourhoods

giving warmth/friendliness/geniality to local communities/areas

12

providing opportunities for small-scale


entrepreneurship

Offering/affording openings/chances/ prospects for business


ventures/new enterprises/start-ups

13

bring the citys downtown to life

Creating vibrant city centres / revitalizing/ re-energising/ reviving


inner cities

14

spurring the development of cultural

Stimulating/encouraging/propelling/galvanizing/ incentivizing/driving

trendyline

amenities such as museums and


galleries
a host of recreational facilities

the growth/expansion of facilities supporting the arts

16

lined with aesthetically appealing


buildings

filled with visually/artistically attractive /beautiful


structures/edifices/buildings (allow)

17

not the cold monolithic monstrosities


with grim facades

As opposed to colossal/massive eyesores with


unfriendly/inhospitable/unwelcoming/forbidding grey/uninviting
frontages/fronts/appearances

18

conscientious conservation of heritage


buildings
injecting architectural charm

Careful/thoughtful preservation of historical/old buildings

20

testifying to a citys unique historical


personality

attesting to/affirming a citys distinct/special/one-of-a-kind


identity/character (allow historical)

21

and celebrating its rich cultural identity

and commemorating/saluting/honouring/paying tribute to/lauding its


long/rich/diverse traditions/heritage

22

But meaningful conservation is not


merely about preserving the faade of
a building, much less converting vital
places into mouldy museums
Whilst they have tremendous value as
historical icons

but preservation should go beyond just/only/simply/purely


maintaining/keeping frontages or changing living places into
stale/drab/lifeless public building (allow museum)

15

19

23
24

A multitude of entertainment/leisure/amusement amenities

giving/infusing quaint appeal/ picturesque/enchanting character to


buildings

Although precious as universally recognized/renowned and admired


symbols (be generous here)

remain organically connected to city life They should also continue to be natural /living parts of the city/ part
of its life blood

25

Careful resurrection
(repetition of point 18)
judicious repurposing

26

living entities that breathe character

Vital/functioning buildings that exude/emanate personality

27
28

proclaiming the identity of vibrant


communities where people still live
and work
strengthen a sense of belonging

Evincing/showing/broadcasting the character of


dynamic/vigorous/flourishing/thriving/healthy
neighbourhoods/districts
Reinforce/fortify kinship/affiliation/togetherness

29

by forging emotional bonds to places

By building/constructing/creating/fashioning close/intimate
attachments

30

but also sustain a sense of collective


memory

Preserve/maintain an awareness of our


shared/cumulative/communal past

31

They anchor people securely amidst


the swirling currents of globalisation
which threaten to cast them adrift

They firmly sustain our ties/attachment to a place, (mooring us


firmly/steadily/strongly to prevent us from being swept away by the
forces of globalization)

astute/thoughtful/careful re-use / employing buildings for a different


use or function

*the words in brackets not necessary for mark


18 + points
7 - 8 points

8
4

15 - 17 points
5 - 6 points

7
3

12 - 14 points
3- 4 points

6
2

9 -11 points
1 - 2 points

5
1

trendyline

7. What similarity does the author draw between cave paintings and Creative Placemaking? Use your own words
as far as possible. (2)
Lift from Passage
Paraphrase
Starting with the first cave paintings 40,000 years ago, Both use art to radically change areas/places/settings
we have been using art to transform places that feel
which seem bleak/desolate and
dark and menacing
threatening/intimidating/foreboding (1)
into places that are inviting and vibrant.

into areas which are friendly/welcoming and lively. (1)

8. Genuinely green citiesprettify shopping malls (lines 68-70). What does this sentence tell us about the authors
opinion of current attempts to make cities look greener? Use your own words as far as possible. (2)
Lift from Passage
Inference
Genuinely green cities are a far cry
The author thinks the current attempts to do so are woefully /
from those of today where token
pathetically inadequate and completely unsuccessful. (1)
trees camouflage congested roads
(accept critical, scornful but not sceptical or doubtful)
or manicured shrubs in concrete
troughs vainly attempt to prettify
Because they are insincere/half-hearted/inauthentic (merely
perfunctory/cosmetic/paying mere lip-service) - totally different from
shopping malls.
what a truly/real green city should be encompass. (1)
9. In paragraph 8, show how the author employs an extended metaphor to illustrate the fact that cities are in urgent
need of transformation. Use your own words as far as possible. (2)
Lift from Passage
Language appreciation
Cities today are social, cultural,
She employs natural imagery, comparing cities to living
aesthetic and environmental
organisms/landscapes which are dying, describing them as bleak/
wastelands. Transforming such
barren/ desolate/ empty/ neglected wastelands (1)
inhospitable and arid landscapes
EITHER:
requires equal parts sensitivity and
She subsequently uses the adjective arid to reinforce the same image
rigour. Sensitivity, because planners
of cities being
need to realize that every city is not
parched/barren/lifeless deserts
only a unique cultural and historical
product, but also an evolving
organism. Rigour, because drastic
OR: (preferred)
She proceeds to develop this natural image by prescribing remedies
measures need to be applied and
for their revival and eventual blossoming (1)
scrupulous attention paid to enable
their present revival and ensure their
eventual blossoming.
Alternative Answer
She employs natural imagery, comparing cities to
changing/growing/developing/embryonic living things/entities (i.e.
evolving organisms) (1)
She subsequently describes the measures that will guarantee their
eventual blossoming. (1)
NB. If students explain the central metaphor in general without
showing how the image is extended, award no mark.
NB. If students merely describe/paraphrase the extended metaphor
without citing specific images, then award 1 mark only.
10. According to the author in paragraph 9, what are the main obstacles that currently stand in the way of transforming
cities? Use your own words as far as possible. (3)
Lift from Passage
Paraphrase
The main obstacles are:
the soulless planners whose cookie
1. uninspiring/characterless architects/developers whose designs are
cutter designs suffocate them,
uniform/ identical/ unvaried/ indistinguishable/ homogeneous/ mass
produced/ lack originality/ all the same (1)
the faceless corporations whose
colossal towers smother them, and

2. impersonal/anonymous business organisations/companies whose


gigantic/vast/massive buildings/skyscrapers overwhelm/stifle cities (1)

the gas-guzzling brutes that choke


them.

3. the fuel-wasting automobiles/vehicles/cars congesting/clogging/


polluting cities (1)
trendyline

11. Explain the relevance of the final paragraph to the title of the passage. Use your own words as far as possible.
(1)
Lift from Passage
Appreciation
we need to first rid ourselves of the soulless
In this paragraph, the author says the city has been
planners whose cookie cutter designs suffocate them, appropriated/seized/taken over by technocrats/big
banish the faceless corporations whose colossal
business/automobiles and must be returned to its
towers smother them, and outlaw the gas-guzzling
original and legitimate (rightful) owners the people brutes that choke them. We need to return the city to
from whom it has been stolen. (1/2)
its rightful owners - the people. As American author
The title Reclaiming our Cities encapsulates this idea
and urbanist Jane Jacobs so astutely cautioned
reclaiming meaning retrieving or recovering
several decades ago: "Cities have the capability of
providing something for everybody, only because, and something once taken that is rightfully yours and the
plural possessive pronoun our referring to the
only when, they are created by everybody."
people/inhabitants/denizens of cities in general (1/2)
Be generous here not all of the above detail is
required but the connection must be clearly
understood.
12. Alissa Walker severely criticises modern cities in general and argues that there is an urgent need to make them
more liveable. Discuss the relevance of her observations and recommendations to your country in particular. (10)

trendyline

INNOVA JUNIOR COLLEGE


JC 2 PRELIMINARY EXAMINATION 2

in preparation for General Certificate of Education Advanced Level

Higher 1
CANDIDATE
NAME
CLASS

INDEX NUMBER

GENERAL PAPER

8807/02

Paper 2

31 August 2015
1 hour 30 minutes

Candidates answer on the Question Paper.


READ THESE INSTRUCTIONS FIRST
Write your name and class on all the work you hand in.
Write in dark blue or black pen on both sides of the paper.
Do not use staples, paper clips, highlighters, glue or correction fluid.
Answer all questions.
The Insert contains the passages for comprehension.
Note that up to 15 marks out of 50 will be awarded for your use of
language.
At the end of the examination, fasten all your work securely together.
The number of marks is given in the brackets [ ] at the end of each question or part
question.

For Examiners Use


Content

/35

Language

/15

Total

/50

This document consists of 7 printed pages.


Innova Junior College

[Turn over

trendyline

Read the passages in the Insert and then answer all the questions. Note that up to fifteen marks will be
given for the quality and accuracy of your use of English throughout this Paper.
NOTE: When a question asks for an answer IN YOUR OWN WORDS AS FAR AS POSSIBLE and you
select the appropriate material from the passage for your answer, you must still use your own words to
express it. Little credit can be given to answers which only copy words and phrases from the passage.

Why do the authors begin the passage with the quotation in lines 1-2?
.
.
.
.[2]

Why do the authors place inverted commas around conspiracy (line 12)?
.
.
.
.[2]

According to the authors, how can humour lead to exclusion (line 13)? Use your own words as
far as possible.
.
.
.
.[2]

Explain how humour might be associated with either higher self-esteem (line 39) or intelligence
(line 40).
.
.[1]

trendyline

According to the authors, when are comedians considered good comics (line 45)? Use your own
words as far as possible.
.
.
.
.[2]

Explain how using humour to deal with normally unacceptable (lines 52-53) content is like a
sugar coating to bitter medicine (line 54)?
.
.
.
.[2]

Explain what the authors mean by Even though the audience may laugh it off, the cogs will have
started spinning in their minds (lines 63 64).
.
.
.
.[2]

In paragraph 9, why do the authors suggest that there is a place for offensive humour (line 72)?
Use your own words as far as possible.
.
.
.
.[2]

Explain why the authors use the word old in line 81.
.
.
.
.[2]

trendyline

10

Using material from paragraphs 2 to 5 of the passage, summarise what the authors have to say
about the benefits of humour.
Write your summary in no more than 120 words, not counting the opening words which are
printed below. Use your own words as far as possible.
Humour is beneficial to us because .........

......
......
......
..
......
......
......
......
..
......
......
......
......
..
......
..[8]

trendyline

11

Louis Franzini and Nichole Force write about the value of humour. How important is humour to you
and your society? In your answer, develop some of the points made by the authors and give your
own views and some account of the experiences which have helped you to form them.

trendyline

.[10]
End of Paper

trendyline

Louis Franzini and Nichole Force write about humour.


1

The philosopher Ludwig Wittgenstein said, A serious and good philosophical work could be
written consisting entirely of jokes. Despite the buffoonish imagery that comes to mind when
one considers the joker, the clown or the pie-in-the-face comedian, humour is more than
mere silliness. It is an advanced intellectual means of developing new perspectives and
5
coping with extreme circumstances.

Humour is a quintessentially social phenomenon. Jokes and other humorous utterances are a
form of communication that is usually shared in social interaction. The topics and themes
people joke about are also generally central to the social, cultural and moral order of a
society or a social group. Humour not only is a sign of closeness among friends, it is an
effective way of forging social bonds too, even in situations not very conducive to closeness: 10
it breaks the ice between strangers, unites people in different hierarchical positions, and
creates a sense of shared conspiracy in the context of illicit activities like gossiping or joking
about superiors. The ip side of this inclusive function of humour is exclusion. Those who do
not join in the laughter, because they do not get the joke, or even worse, because the joke
15
targets them, will feel left out, shamed or ridiculed.

For individuals, the direct benefits of humour lie in the bodys chemical reaction to laughter.
Among other things, laughter has been shown to reduce stress and boost the immune
system. Rather than taking antidepressants, one can self-medicate by watching a funny
movie, going to a comedy show or playing a fun game. For the rejected lover or laid off
worker, this self-induced boost provided by humour activates a neurochemical reaction that 20
enhances their ability to tolerate the stress response and think creatively of coping options.
Theorist Martin Armstrong, who wrote about the function of laughter in society, may have said
it best when he wrote, For a few moments, under the spell of laughter, the whole man is
completely and gloriously alive: body, mind and soul vibrate in unison the mind flings open
25
its doors and windows.

In fact, humour has also been known to possess healing powers. Some medical settings offer
a special room for patients and their family where a variety of materials with humorous
content is available for their use as desired. Sometimes an aide brings a comedy cart to the
patients room, which typically includes choices of funny movies, tapes, joke books and other
amusing materials. It is an extraordinarily powerful and effective way to use humour to 30
alleviate pain and increase the cheerfulness of patients residing in institutions that are often
much less comfortable for them than being at home.

A keen sense of humour is a quality nearly all of us claim in ourselves and one we seek and
highly value in others. We love a sense of humour in our dates and mates, our children and
basically everyone with whom we have social contact. When our humour-making is 35
successful, we are drawn closer to people and share a bonding experience with them. We
enjoy life more and our troubles seem to lessen instantly. Most important, we like that person
even more. Laughter and humour can buffer stress and reduce experiences of pain. The use
of humour has been shown to be associated with a positive self-concept, higher self-esteem
40
and intelligence, and even the enhancement of enjoyment of positive life events.

Given the benefits of humour, it is no wonder why people are willing to pay to watch
comedies. Most people find comedy as merely funny, but comedy is more than just a
laugh. Beneath the humour lies a rich layer of social commentaries ranging from the political,
to gender, to class and to other social issues. While comedians will make everyone
uncomfortable at some point, good comics play an important function in society by holding up 45
a mirror and forcing us to confront realities that we would often prefer to ignore. For minority
groups, humour also serves as a tool to neutralise the power of stereotypes that obstruct their
path to equal participation in society. Comedy can give social critique and instigate
transformation in a way that leaves many audience members wanting more.

trendyline

Comedy regularly draws criticism for being offensive and for perpetuating negative
stereotypes. This, however, is a sign of a healthy comedy culture because it means that 50
comedians are pushing social boundaries. Stories and expressions that are normally
unacceptable are met with laughter and agreement when they are told on stage. The fact that
the content is encrusted in humour is like a sugar coating to bitter medicine. The laugh takes
away the sting. As Mary Hirsch the humourist once commented, Humour is a rubber
swordit allows you to make a point without drawing blood. Observational comedy, 55
situational comedy, slapstick comedy, comedy that both enlightens and offendsthese are
forms of creative destruction at their height and in their depths, and they have long allowed
us to talk about things that taboos, or at the very least taste, might otherwise preclude.

Comedy can also take on a political face. While oftentimes, such political comedy focuses on
more trivial matters such as a politicians appearance or personality, political humour also has 60
its serious side that sometimes provides political, social or economic commentary. It can
attack the character, policy or even the larger electoral system. Even though the audience
may laugh it off, the cogs will have started spinning in their minds. It is this type of political
humour that politicians, institutions and authority figures over the centuries have feared the
most.
65

The fear of humour points to the fact that humour can be destructive, cruel, belittling,
humiliating, relationship destroying, blatantly biased, extreme in its effects, unpleasant to
nearly all listeners in addition to the butts of the joke, and simply outrageous in its disregard
for its consequences and lack of boundaries. Humour that is racist, sexist, ageist, obscene
and otherwise politically incorrect can at times be as funny as it is offensive. However, there 70
is a place for such offensive humour. Some audiences love high-intensity controversy and
sarcasm, while others prefer only safe non-controversial topics. It is no wonder why
comedians develop the skill of using humour as a lens through which they can examine all
kinds of topics and observations about life, subjects that may be otherwise unremarkable,
unpleasant or just unfunny. Humour is the Philosophers Stone that turns everyday lead into 75
comedy gold. It makes the mundane, the awful, the sad or the embarrassing into something
we can all laugh at and find funny.

10

As comedy began to do a better job of reflecting the world, it began, as well, to take on the
responsibilities associated with that reflection. It began to recognise the fact that the long
debate about the things comedy owes to its audiences and itselfthe old hey, Im just 80
making a joke line of logiccan be partially resolved in the idea that nothing, ultimately, is
just a joke. Humour has moral purpose. Humour has intellectual heft. Humour can change
the world.

trendyline

1. Why do the authors begin the passage with the quotation in lines 1-2? [2]
Answer
Passage
A serious and good philosophical work
a) The authors are setting the tone
could be written consisting entirely of
for the whole passage
jokes.
b) which looks at humour as a
grave/not a trivial subject matter
humour is more than sillinesscoping
OR
with extreme circumstances.
a) The quotation is
aligned/consistent with/related
to/shows from the
outset/beginning
b) their view that humour is a
grave/not a trivial subject matter
2. Why do the authors place inverted commas around conspiracy (line 12)? [2]
Answer
Passage
a) The word is used in jest/tonguecreates a sense of shared conspiracy in
in-cheek//rather inappropriately/
the context of illicit activities like
not used in the usual sense
gossiping or joking about superiors
(either of these must be paired
with the correct context in b) [1]
b) because activities like gossiping
and joking about superiors are not
really/too mild to be considered
unlawful/very harmful/
treacherous. [1]
3. According to the authors, how can humour lead to exclusion (line 13)? Use your
own words as far as possible. [2]
Passage
Answer
Those who do not join in the laughter,
because they do not get the joke, or
even worse, because the joke targets
them, will feel left out, shamed, or
ridiculed.

a) Those who do not share the


humour
b) because they do not understand
it
c) or they are at the receiving end of
it
[(a), (b) or (c) = 1m]
d) will experience the sense of not
being part of/included in the
group,
e) embarrassment/ humiliation
f) or a sense of being
mocked/made fun of.
[(d) must be paired with (e) or (f) to
earn 1m]
[(d), (e) or (f) alone = 0m]

4. Explain how humour might be associated with either higher self-esteem (line 39) or
intelligence (line 40) [1]
Passage
Answer
A keen sense of humour is a quality
a) Humour can lead to a higher selfGet PSJ Private Tutor to Guide you through Exams Now!
Contact www.privatetutor.com.sg

trendyline

nearly all of us claim in ourselves


and one we seek and highly value in
others. We love a sense of humour in
our dates and mates, our children, and
basically everyone with whom we have
social contact Most important, we like
that person even more.
The use of humour has been shown to
be associated with a positive selfconcept, higher self-esteem and
intelligence

esteem because it makes a person


well-liked/popular/a valued
member in any group. [1]
OR
b) Humour shows intelligence
because to make people laugh, a
person must be able to perceive
the mindset of his audience,
predict their response, and
accordingly tune the joke and the
manner he conveys it. [1]
Alternative answer for (b):
c) A certain level of cognitive
ability/competence is needed to
understand jokes/comedy/humour
Accept any relevant answer for
either.

5. According to the authors, when are comedians considered good comics (line 45)?
Use your own words as far as possible. [2]
Passage
Answer
Comedians are good comics
a) when they perform a significant/
While comedians will make everyone
essential/necessary/valuable role
uncomfortable at some point, good
comics play an important function in
in society
society by holding up a mirror and
b) by making us reflect on
forcing us to confront realities that we
ourselves/our lives//showing us
would often prefer to ignore.
our true selves//highlight the
actual situation
c) and compelling us to face
situations we would frequently
rather/want to/choose to
disregard/dismiss/not pay
attention to.
Any 2p = 2m
6. Explain how using humour to deal with normally unacceptable (lines 52 53)
content is like a sugar coating to bitter medicine (line 54)? [2]
Passage
Answer
This, however, is a sign of a
a) Just as medicine which tastes awful {but
healthy comedy culture because it
is good for us} is made more
means that comedians are pushing
palatable/tasty/easy to swallow by a
social boundaries. Stories and
sweet coating/external layer, [1]
expressions that are normally
[Cannot lift: sugar or bitter]
unacceptable are met with laughter
b) so content which may be {socially
and agreement when they are told
significant but} usually hard to take is
on stage. The fact that the content
more easily received/accepted when it
is encrusted in humour is like a
is funny/laced with humour. [1]
sugar coating to bitter medicine.
The laugh takes away the sting.

Note: Attempt at (a) must be present for (b) to be


awarded.

trendyline

7. Explain what is meant by Even though the audience may laugh it off, the cogs will
have started spinning in their minds. (line 63 64). [2]
Passage
Answer
a) The people listening to political
political humour also has its serious
humour
side that sometimes provides political,
[context of political must be present to
social, or economic commentary. It can
award (a)]
attack the character, policy or even the
b)
may
dismiss it as merely
larger electoral system. Even though
funny/humorous/amusing,
the audience may laugh it off, the
c) but they are already beginning to
cogs will have started spinning in
think about the matter.
their minds. It is this type of political
humour that politicians, institutions, and
1-2p=1m
authority figures over the centuries
3p=2m
have feared the most.
8. In paragraph 9, why do the authors suggest that there is a place for offensive
humour (line 72)? Use your own words as far as possible. [2]
Passage
Answer
Some audiences love higha) Some people like/enjoy/are
intensity controversy and
entertained by
sarcasm
b) extreme
c) debatable/disputable/contentious
arguable issues
d) and ironic/mocking/ contemptuous
language//caustic/sneering/cutting
witticism/cynical
1p = 0
2p = 1m
3-4p = 2m
9. Explain the authors use of the word old in line 81. [2]
Passage
Answer
It began to recognize the fact that the long
He uses the word old to make the point
debate about the things comedy owes to its
that
audiences and itselfthe old hey, Im just
(a) the idea/reasoning/thinking that
making a joke line of logiccan be
comedians are
partially resolved in the idea that nothing,
merely/only/simply being funny
ultimately, is just a joke.
[1]
(b) is no longer relevant/true
(Inferred from nothing, ultimately,
is just a joke) [1]
The context what is no longer relevant- must be correct.

10. Using material from paragraphs 2 to 5 of the passage, summarise what the author has to say
about the benefits of humour.

trendyline

Write your summary in no more than 120 words, not counting the opening words which are printed
below. Use your own words as far as possible.
Humour is beneficial to us because

Lifted
From Paragraph 2
1

The topics and themes people joke about


are generally central to the social,

2
3
4

cultural
moral order of a society or a social group
a sign of closeness among friends

5a

it is an effective way of

5b

forging social bonds too


OR
it breaks the ice between strangers
OR
unites people in different hierarchical
positions
OR
creates a sense of shared conspiracy
in the context of illicit activities
OR
(from paragraph 5) we are drawn closer
to people and share a bonding experience
with them.
even in situations not very conducive to
closeness

From Paragraph 3
7

laughter has been shown to reduce


stress
OR
(from paragraph 3) tolerate the stress
response
OR
(from paragraph 5) Laughter and humour
can buffer stress

and boost the immune system.

Rather than taking antidepressants, one


can self-medicate
OR
this self-induced boost provided by
humour
think creatively of coping options

10
11

the whole man is completely and


gloriously alive

Paraphrased
Its content is usually essential/very
important/key/fundamental/primal to human
interrelationships
traditions/customs
ethics/conventions/mores of a community
a symbol/an indication/an expression of
intimacy among peers/buddies
it is a powerful/efficient/successful
method/technique for
building/creating/developing/establishing
relationships/connections/people to come
together

even under circumstances that are not very


suitable/appropriate/helpful/comfortable to
facilitate tight friendships/intimacy
Humour has been proven to
decrease/alleviate/mitigate
anxiety/distress/strains/tensions
OR
Humour has been proven to enhance our abilities
to cope with anxiety/distress/strains/tensions
OR
Humour can help to be a defence/shield an
individual from anxiety/distress/strains/tensions
and enhance/increase the bodys
defences/bodys ability to fight diseases
Humour is a form of treatment that one can
administer/dispense/provide for/give to
oneself/himself // We are able to cure / treat our
own illness / condition
think innovatively of / Think of new/fresh
ways/methods/suggestions to
adapt/readjust/handle the situation
with humour, an individual is fully/totally and

trendyline

12

the mind flings open its doors and


windows

From Paragraph 4
13

possess healing powers

14

extraordinarily powerful and effective


way

15

to alleviate pain
OR
(from paragraph 5) reduce experiences
of pain
increase the cheerfulness of patients
residing in institutions that are often
much less comfortable for them than
being at home

16
17

From Paragraph 5

19

one we seek and highly value in others


OR
We love a sense of humour in our dates
and mates, our children, and basically
everyone with whom we have social
contact
OR
we like that person even more
We enjoy life more

20

our troubles seem to lessen instantly

21

a positive self-concept

22

higher self-esteem

23

Intelligence

24

the enhancement of enjoyment of


positive life events

18

greatly/remarkably
rejuvenated/refreshed/engaged
Psychologically/mentally
liberated/freed/illuminated
has/contains the ability to cure/mend/ an
individual
greatly/immensely/exceptionally/remarkably
potent/capable/efficient/successful/strong
/useful method
to lower suffering/discomfort/hurt //
to relieve/lessen agony/suffering /discomfort
heighten/elevate the happiness/joy of patients
who stay in/live in places that are not as
conducive/pleasant/cosy/snug as their own
dwellings
makes a person more
desirable/appreciated/popular/
charming/attractive

We can better relish life // We can have a better


time // Increase our pleasures in life // We relish
life to a larger extent
helps to reduce our
anxieties/sufferings/pain/distress immediately
a healthy/good/favourable image of the
individual // a healthy/good/favourable way in
which one views/sees himself
elevated/increased confidence/respect/regard
for themselves // more faith in themselves
(Inferred) Others perceive you as
brilliant/sharp/smart/clever
OR
Using humour develops a sharp mind
[Must be framed as a benefit]
increase the delight we get from our good
experiences

11. Louis Franzini and Nichole Force write about the value of humour. How important is
humour to you and your society? In your answer, develop some of the points made
by the authors and give your own views and some account of the experiences which
have helped you to form them.

trendyline

JURONG JUNIOR COLLEGE


JC 2 Preliminary Examination 2015
CANDIDATES NAME
CLASS

GP TUTORS NAME

General Certificate of Education


GENERAL PAPER

8807/2
27 August 2015

Paper 2
Candidates answer on the Question Paper.

1 hour 30 minutes

Additional Materials: 1 Insert


READ THESE INSTRUCTIONS FIRST
Write your name, civics class and GP tutors name on all the work you hand in.
Write in dark blue or black pen on both sides of the paper.
Do not use paper clips, highlighters, glue or correction fluid.
Answer all questions.
The Insert contains the passage for comprehension.
Note that 15 marks out of 50 will be awarded for your use of language.
At the end of the examination, fasten all your work securely together.
The number of marks is given in brackets [ ] at the end of each question or part question.

EXAMINERS USE
Content

35 Comments:

Language

15

Total

50
This document consists of 6 printed pages.

trendyline

Read the passage in the Insert and then answer all the questions. Note that up to fifteen marks
will be given for the quality and accuracy of your use of English throughout this Paper.
NOTE: When a question asks for an answer IN YOUR OWN WORDS AS FAR AS POSSIBLE
and you select the appropriate material from the passage for your answer, you must
still use your own words to express it. Little credit can be given to answers which only
copy words or phrases from the passage.
1 a Why is a common tongue more important than ever (line 3)? Use your own words as far
as possible.
........
........
... [2]
1b Suggest what is causing languages to become more standardised (lines 5 6).
.
... [1]
2

What is the authors purpose in providing us the series of examples in lines 9 to 10?
.
.
... [1]

Explain what the words realities and seductive imply about the pressures on minority
language speakers in each case (line 12).
.
.
... [2]

How does paragraph 3 illustrate the rapid die-off of languages (line 15)?
.
.
.
... [3]

trendyline

For
Examiners
Use

Explain the futurists reaction to the rapid die-off of languages (line 21).
.
.
... [2]

Why does the author describe the common wisdom about globalisation as undeniable
(line 26)? Use your own words as far as possible.
.
.
.
[2]

Why is the word global in inverted commas (line 30)?


.
.
... [2]

In paragraph 9, how does the author feel about the survival of minority languages in the
future and how does he illustrate his attitude?
.
.
.
... [2]

trendyline

For
Examiners
Use

Using material from paragraphs 6 to 8 only, summarise what the author has to say about
why the extinction of minority languages is not a certainty. Write your answer in no more
than 120 words, not counting the opening words which are printed below. Use your own
words as far as possible.
The extinction of minority languages is not a certainty as .....

.
.
.
.
.
.
.
.
.
.
.
.
.
.
.
.
.
.
[8]

trendyline

For
Examiners
Use

10 In this article, Eric Garland writes about the pressures of globalisation on minority
languages and how their extinction is not a certainty. How far would you agree with Eric
Garlands observations? Relate your opinions to your society.
.
.
.
.
.
.
.
.
.
.
.
.
.
.
.
.
.
.
.
.
.
.
.
.
.
.

trendyline

For
Examiners
Use

.
.
.
.
.
.
.
.
.
.
.
.
.
.
.
.
.
.
.
.
.
.
.
.
.
.
.
..... [10]

trendyline

For
Examiners
Use

JURONG JUNIOR COLLEGE


JC 2 Preliminary Examination 2015
CANDIDATES NAME

GP TUTORS NAME

CLASS

General Certificate of Education


GENERAL PAPER

8807/2

Paper 2

27 August 2015

INSERT

1 hour 30 minutes

READ THESE INSTRUCTIONS FIRST


This insert contains the passage for Paper 2.

This document consists of 3 printed pages and 1 blank page.


[Turn over

trendyline

2
Eric Garland writes about minority languages.
1

Globalised economics and the media are changing the face of culture around the globe,
reducing the number of languages that humans speak. As the world economy becomes
more integrated, a common tongue has become more important than ever to promote
commerce, and that puts speakers of regional dialects and minority languages at a distinct
disadvantage. In addition, telecommunications has pressured languages to become more 5
standardised, further squeezing local variations of language.

Over the past 500 years, as nation states developed and became more centralised, regional
dialects and minority languages have been dominated by the centrist dialects of the ruling
parties. Cornish has given way to English, Breton to French, Bavarian to High German, and
Fu-jian-wa to Cantonese. Linguists concur that minority languages all over the world are 10
giving way to more dominant languages, such as English, Mandarin, and Spanish, among
others. The realities of commerce and the seductive power of world pop culture are placing
pressure on speakers of minority languages to learn majority languages or suffer the
consequences: greater difficulty doing business, less access to information...

These pressures are inducing a rapid die-off of languages around the world. Languages 15
have been disappearing steadily, with 3,000 of the worlds languages predicted to disappear
in the next 100 years. According to the United Nations Environment Programme, there are
5,000 to 7,000 spoken languages in the world, with 4,000 to 5,000 of these classed as
indigenous, used by native tribes. More than 2,500 are in danger of immediate extinction,
and many more are losing their link with the natural world, becoming museum pieces rather 20
than living languages. Futurists have noted this loss with no little despair, for significant,
culturally specific information may disappear along with a language. For instance, knowledge
about unique medicines and treatments used by aboriginal groups could be lost forever if the
language used to transmit that information is banned by a majority culture.

The common wisdom is that globalisation is the wave of the future, and in many respects this 25
is undeniable. For centuries, dialects and languages have been unifying to facilitate national
identity, scientific research, and commerce. Without question, there will be a need for
common languages, as standardisation allows growth in software and in people. However,
swept up in this conventional wisdom is the notion that languages and cultures will simply
cease to exist, and people will instead choose global cultures and languages that will 30
transcend boundaries. This is not the only potential scenario. It is possible for globalisation
and new technology to safeguard cultural identity while simultaneously allowing free
exchanges of ideas and goods. Global prosperity and new technologies may also allow
smaller cultures to preserve their niches. It is clear from several modern examples that a
dying or dead language can turn around and become vibrant again, depending on peoples 35
determination and the government policies that are put in place.

The idea of saving languages is very modern. When linguistics scholar Joshua A. Fishman
first wrote of reversing language shift in his book of that title in 1990, one reviewer actually
laughed at the notion. The conventional wisdom among linguists, historians, and sociologists
was that, if your culture and language were on the way out, their doom was assured in a 40
globalised world. After all, the prevailing trends are toward globalisation and a unified world.
Tiny dialects such as Breton, the Celtic language spoken in Brittany, a province on the
north-western coast of France are not a benefit in the global economy, since they are

trendyline

difficult to learn, poorly adapted to modern life, and unintelligible to almost everyone beyond
a small region.
45
6

Learning or relearning a native language is often a political statement, an act of selfdefinition, one that brings solidarity with our neighbours. It is political power, cultural
reverence, and perhaps a feeling of control in a world where political and cultural borders are
collapsing all around us. Minority languages may also have a place alongside majority forms
of communication. The International Committee for the Defence of the Breton Language 50
suggests that early bilingualism can help prepare young people to master several languages,
which will be an advantage if not a necessity for the future in Europe.

Changing world geopolitics is already reforming the pressures on languages. The fall of the
Soviet Union actually spurred a trend toward reversing language loss. In many of the former
Soviet republics, older Turkic languages have been revived, now that the Russian influence 55
is gone. Turkey is spending US$1.5 billion to encourage the resurgence of Turkish
throughout the region. Language is power, economic and otherwise, and the Turks are
capitalising on the possibility of extending their reach, causing a reverse of language shift in
the region. It is becoming clear that, when people have a strong cultural reason to reverse
language shift, they can effectively resist the onslaught of majority languages. Moreover, the 60
mass media technologies that allowed the one-way dialogue of majority languages to drive
out minority languages and dialects are now helping those silenced languages to make a
comeback. Speakers of these smaller languages can use interactive technologies such as
websites, e-mail, and message boards to talk back to the world by creating and distributing
65
media in their own language to a global diaspora.

Globalised commerce and the media are not necessarily the death knell for local languages,
because certain trends support their preservation. Whereas one-way mass media
technologies such as TV, radio, and print served to support majority languages, todays
computer technology is turning the tables. It is considerably less expensive now to produce
video and audio in any language, and communications technologies allow you to transmit 70
these media to a diaspora anywhere in the world. In the future, with lower prices for powerful
computers and dramatic advances in broadband Internet (such as IPv6 architecture that will
soon turn any Internet connection into a broadcast device), majority languages may no
longer possess an advantage in distributing information to the public. In the future, anybody
anywhere on Earth could conceivably receive the evening news in Welsh or Irish Gaelic. 75
Also, the availability of cheap, powerful multimedia will allow teachers to translate
educational materials into a local language more easily. These educational technologies will
be essential to the survival and prosperity of languages in the future. Only education of the
youth assures the continuity of a language.

The pressures of globalisation on minority languages are undeniable, and many will likely 80
disappear. However, extinction is not a certainty. The trend toward the homogeneity of global
culture has stimulated many people to search for their native roots and hold tighter to their
cultural identity. The availability of government services in a chosen language is the only
path to its legitimacy in a political sense, but even more critical is for a minority language to
be used in commerce. We are living in interesting times, linguistically, as powerful national 85
languages encounter fierce resistance in their drive to dislodge local languages. New
technologies are offering people greater freedom to choose their own cultural identity, and
many are choosing minority local languages. The linguistic giants will not be the only choice
in the future.

trendyline

BLANK PAGE

trendyline

JC 2 Preliminary Examination 2015


Paper 2 Answer Scheme
1. (a) Why is a common tongue more important than ever (line 3)? Use your own words as far as possible. [2]
Lifted

Paraphrased

As the world economy becomes more integrated

(a) As the world becomes more interconnected,

to promote commerce

(b) a common tongue will make it easier to carry out


trade or other economic activity with people from
other countries,

..that puts speakers of regional dialects and minority


languages at a distinct disadvantage.

(c) without which we would lose our economic edge over


others.

1=1m, 2-3 = 2m
1. (b) Suggest what is causing languages to become more standardised (lines 5 6). [1]
Lifted

Paraphrased

In addition, telecommunications has pressured


languages to become more standardised

A common language makes


communication to take place.

it easier

for

global

Note: Any other logical answers related to the features of


telecommunications and how it has pressurised
languages should be accepted as well.
1 = 1m
2. What is the authors purpose in providing us the series of examples in lines 9 to 10? [1]
Lifted

Inferred

Cornish has given way to English, Breton to French,


Bavarian to High German, and Fu-jian-wa to Cantonese

He wants to emphasise/ highlight

minority languages have been dominated by the centrist


dialects

how majority languages are prevailing over the minority


ones.
OR
the prevalence in which minority languages are being
replaced.
Note: No marks to be awarded if the answer does not
contain emphasise/ highlight or if the student writes
trying to emphasise/ highlight. Student must also show
the degree of the extent.

1 = 1m

Get PSJ Private Tutor to Guide you through Exams Now!


Contact www.privatetutor.com.sg

trendyline

3. Explain what the words realities and seductive imply about the pressures on minority language speakers in each
case (line 12). [2]
Lifted

Inferred

The realities of commerce

(a) The word realities suggests that there is no choice /


there are harsh repercussions for those who do not
learn majority languages. (push factor)

learn majority languages or suffer the consequences:


greater difficulty doing business, less access to
information...
the seductive power of world pop culture

(b) The word seductive suggests that world pop culture


offers a strong attraction/ impetus for minority
language speakers to learn the common/ popular
language. (pull factor)
Note: Extent of the impact on minority languages needs
to be captured.

2 = 2m
4. How does paragraph 3 illustrate the rapid die-off of languages (line 15)? [3]
Lifted

Paraphrased
(a)

The author uses examples, statistics to show (as


long as the technique is shown)

3,000 of the worlds languages predicted to disappear in


the next 100 years

(b)

how nearly half of the worlds languages are


expected to vanish within the next century

More than 2,500 are in danger of immediate extinction

(c)

with more than half local dialects expected to be


lost soon.

and many more are losing their link with the natural
world, becoming museum pieces rather than living
languages

(d)

There are also countless others that are losing


their relevance/ place in todays society
OR
Numerous languages have become mere archival
records with no practical use in todays context.

1=1m, 2=2m, 3-4=3m


5. Explain the futurists reaction to the rapid die-off of languages (line 21). [2]
Lifted

Paraphrased

Futurists have noted this loss with no little despair,

(a) They are greatly dismayed/devastated at the loss of


languages

for significant, culturally specific information may


disappear along with a language. For instance,
knowledge about unique medicines and treatments used
by aboriginal groups could be lost forever if the language
used to transmit that information is banned by a majority
culture.

(b) as it meant the loss of important and often exclusive


wisdom that is beneficial to humankind.

Note: Students need to be accurate about the emotion or


tone to get the full 2 marks.

2 = 2m

trendyline

6. Why does the author describe the common wisdom about globalisation as undeniable (line 26)? Use your own
words as far as possible. [2]
Lifted

Paraphrased

For centuries,

(a) This is because, for the longest of time

dialects and languages have been unifying to facilitate


national identity,

(b) dialects and languages are sought to ease the


creation of a sense of a nation as a cohesive whole,

scientific research,

(c) allow for greater collaboration in scientific inquiry

and commerce.

(d) and business.

Without question, there will be a need for common


languages, as standardisation allows growth in software
and in people.

(e) It is also obvious how a common language has


brought about advance and progress.
Note: Students are allowed to capture either (c) or (d).
Four points out of five needed for 2m.

1 3 = 1m, (a) + (b) + (c) or (d) + (e) = 2m, 4-5 = 2m


7. Why is the word global in inverted commas (line 30)? [2]
Lifted

Paraphrased

Without question, there will be a need for common


languages, as standardisation allows growth in software
and in people. However, swept up in this conventional
wisdom is the notion that languages and cultures will
simply cease to exist, and people will instead choose
global cultures and languages that will transcend
boundaries.

(a) The author wants to highlight/emphasise


(b) that simply because these languages are widely
used/accepted, it does not mean that a homogenised
culture is going to become a reality.
Note: No marks to be awarded if the answer does not
contain emphasise/ highlight or if the student writes
trying to emphasise/ highlight.

2 = 2m
8. In paragraph 9, how does the author feel about the survival of minority languages in the future and how does he
illustrate his attitude? [2]
Lifted

Inferred

We are living in interesting times, linguistically, as


powerful national languages encounter fierce resistance
in their drive to dislodge local languages. New
technologies are offering people greater freedom to
choose their own cultural identity, and many are choosing
minority local languages. The linguistic giants will not be
the only choice in the future.

(a) The author is fairly/cautiously optimistic/ hopeful/


confident.
(b) This can be seen from his choice of words such as
fierce resistance, many are choosing and will not
which indicate the certainty with which people would
fight to preserve their minority tongue
(c) and how the latest technology ensure that people can
and are opting for languages other than the majority
languages.

Note: If students describe the attitude for (a), the mark


should be awarded. Also, if students do not pick out the
phrases for point (b), they should be awarded the mark.
(a) = 1m, (a) + (b) or (c) = 1m, 3 = 2m

trendyline

9. Using material from paragraphs 6 to 8 only, summarise what the author has to say about why the extinction of
minority languages is not a certainty. Write your answer in no more than 120 words, not counting the opening
words which are printed below. Use your own words as far as possible. [8]
The extinction of minority languages is not a certainty as

From the text

Possible paraphrase

Learning or relearning a native language is often


a political statement (line 46)

The use of such languages can drive home certain political


messages,

It is political power (line 47)


2

an act of self-definition (line 46),

shapes the identity of a nation

one that brings solidarity with our neighbours.


(line 47)

and possibly improve the relationship with other countries


through a common goal.

(It is) cultural reverence (line 47)

The use of minority languages also shows our devotion/


respect for our traditions

and perhaps a feeling of control in a world where


political and cultural borders are collapsing all
around us. (line 48)

and gives us a sense of being in charge in a context where


there seems to be less order in society due to globalisation.

Minority languages may also have a place


alongside majority forms of communication. (line
49)

Minority languages may be able to complement the majority


ones,

The International Committee for the Defence of


the Breton Language suggests that early
bilingualism can help prepare young people to
master several languages, (line 50)

possibly laying the foundation to being multi-lingual

which will be an advantage (line 52)

which not only gives people an edge in this age,

if not a necessity for the future in Europe


(line 52)

but is also fast becoming an essential prerequisite for


sustainable development.

10

Changing world geopolitics is already reforming


the pressures on languages. (line 53)

The dynamic geopolitical trends are transforming the tension


that globalisation places on languages

11

The fall of the Soviet Union actually spurred a


trend toward reversing language loss. In many
of the former Soviet republics, older Turkic
languages have been revived, now that the
Russian influence is gone. (line 53)

(Inferred from examples)

Turkey is spending US$1.5 billion to encourage


the resurgence of Turkish throughout the region.
(line 56)

(Inferred from example)

13

Language is power, economic and otherwise,


(line 57)

Language has the capacity to influence both economics and


politics

14

and the Turks are capitalising on the possibility


of extending their reach (line 57-58)

(Inferred from example)

12

in some cases inducing/ providing the impetus for the


restoration of minority languages and

some countries expend on campaigning for such restoration.

and can be used by countries to spread their influence in other


societies.
15

It is becoming clear that, when people have a


strong cultural reason to reverse language
shift, (line 59)

Preservation of our heritage provides solid grounds to use


minority tongues

16

they can effectively resist the onslaught of


majority languages. (line 60)

and that can help us prevent the homogenisation of culture


and languages.

trendyline

17

Moreover, the mass media technologies that


allowed the one-way dialogue of majority
languages to drive out minority languages and
dialects are now helping those silenced
languages to make a comeback. (line 61)

The same communication technologies that help majority


languages dominate can also aid in the revival of minority
ones

18

Speakers of these smaller languages can use


interactive technologies such as websites, e-mail,
and message boards to talk back to the world by
creating and distributing media in their own
language to a global diaspora. (line 63)

as increasingly more users are generating and disseminating


content in their native tongues to the rest of the world using
such technologies.

Globalised commerce and media are not


necessarily the death knell for local languages,
because certain trends support their preservation.
Whereas one-way mass media technologies such
as TV, radio, and print served to support majority
languages, todays computer technology is
turning the tables. (line 66)
19

It is considerably less expensive now to


produce video and audio in any language, (line
69)

The lower cost

20

and communications technologies allow you to


transmit these media to a diaspora anywhere in
the world (line 70)

and ease in dissemination

21

In the future, with lower prices for powerful


computers and dramatic advances in broadband
Internet (such as IPv6 architecture that will soon
turn any Internet connection into a broadcast
device), majority languages may no longer
possess an advantage in distributing
information to the public. In the future,
anybody
anywhere
on
Earth
could
conceivably receive the evening news in Welsh
or Irish Gaelic. (line 71)

as well as advances in accessibility

Also, the availability of cheap, powerful


multimedia will allow teachers to translate
educational materials into a local language more
easily. (line 76)

Such technologies make it more convenient for educators to


tap on resources in minority tongues

23

These educational technologies will be essential


to the survival and prosperity of languages in
the future. (line 77)

which is important and instrumental in the preservation of any


language

24

Only education of the youth assures the


continuity of a language (line 78)

and especially when the younger generation sees the same


value (in languages, minority or otherwise) as well.

22

means that we no longer need a lingua franca to communicate


with the world

Note: The ease of availability must be captured.

trendyline

Points

Marks

13 and above

12

10 - 11

8-9

6-7

4-5

2-3

Table of Specifications:
No.

Question Types

Question Number

Mark Allocation

Direct Literal

1a, 4, 6

Inferential Technique, Punctuation

1b, 2, 7, 8

Extended Vocabulary

3, 5

Summary

AQ

10

10

Subtotal

35

Language
Total

15
50

trendyline

10. In this article, Eric Garland writes about the pressures of globalisation on minority languages and how their
extinction is not a certainty. How far would you agree with Eric Garlands observations? Relate your opinions to
your society. [10]
Main Ideas:
Paragraph 1

Globalised economics and the media are changing the face of culture around the globe, reducing the
number of languages that humans speak.

Paragraph 2

The realities of commerce and the seductive power of world pop culture are placing pressure on
speakers of minority languages to learn majority languages or suffer the consequences.

Paragraph 3

These pressures are inducing a rapid die-off of languages around the world and futurists have noted
this loss with no little despair, for significant, culturally specific information may disappear along with a
language.

Paragraph 4

Globalisation not only causes languages to die off but it can also help dying or dead language to turn
around and become vibrant again.

Paragraph 5

The idea of saving languages is very modern because in the past this would not have been possible
(refer to previous paragraph about globalisation).

Paragraph 6

There are various benefits to learning or relearning a native language/ several languages.

Paragraph 7

It is becoming clear that when people have a strong cultural reason to reverse language shift, they can
effectively resist the onslaught of majority languages.
Moreover, mass media technologies are now helping minority languages to make a comeback.

Paragraph 8

Globalised commerce and the media are not necessarily the death knell for local languages, because
certain trends support their preservation.
The availability of cheap, powerful multimedia will assure a continuity of minority languages.

Paragraph 9

The extinction of minority languages is not a certainty because:


the trend toward the homogeneity of global culture has stimulated many people to search for
their native roots and hold tighter to their cultural identity and
new technologies are offering people greater freedom to choose their own cultural identity.

Note: For this AQ, minority languages will be defined as


Languages traditionally used within a given territory of a state by nationals of that state who form a group
numerically smaller than the rest of the states population (the European Charter for Regional or Minority
Languages (ECRML))
i.e. A language spoken by a minority of the population of a territory.

trendyline

Main Ideas

Examples from Singapore

Paragraph
2

Globalisation
pressuring speakers
of
minority
languages to learn
majority languages
or
suffer
the
consequences

Use of English in Singapore


Official language medium in Singapore is English, English as the lingua franca.
English is the dominant language of commerce and government.
Information released by the government is primarily in English on government
websites and supplemented with Mandarin, Malay and Tamil (at times) but the nonMandarin Chinese dialects are not used for official communication purposes.
Bilingual education policy teaches English and the vernacular language.
The Ministry of Education places great emphasis on English and believes that the
mastery of English is vital to Singapores pupils because English is the language of
administration, education, commerce, science, technology, and global
communication.
As the only country in the South East Asian region to use English as the working
language, it offers us a comparative advantage as compared to other countries in the
region.
Balance
English was also chosen to be the working language in the early years of our
independence in order to give all races equal opportunities through a common
language to learn.
Bilingual education policy stresses on the importance of mastering ones Mother
Tongue as well as English this has mainly affected the Chinese to stop speaking
dialects and speak Mandarin instead.
Proficiency in Malay and Tamil are still emphasised by the government.

Paragraph 3
Significant, culturally
specific information
may disappear along
with a language

General Impacts
Declining language proficiency.
Changing language attitudes.
Chinese dialects
The use of Chinese dialects such as Hokkien, Teochew, Cantonese, Hainanese and
Hakka has been declining over the last two decades, although they are still being
used especially by the older generations of the Chinese population.
Much of the younger generations of Singaporeans are neither fluent nor conversant
in their dialects (due to lack of use, exposure and education).
The Media Development Authority (MDA) states that on National Television, all
Chinese programmes, except operas or other programmes specifically approved by
the Authority, must be in Mandarin. Dialects in dialogues and songs may be allowed
provided the context justifies usage and is sparingly used.
The clan associations of Singapore (Singapore Hokkien Huay Kuan, Teochew Poit Ip
Huay Kuan, Singapore Kwang Tung Association, Nanyang Khek Community Guild,
The Singapore Hainan Kwee Kuan, Sam Khiang Huay Kwan and the Singapore
Foochow Association, all of which are under the Singapore Federation of Chinese
Clan Associations) are active in keeping the Chinese dialects alive but these
associations do not seem to be popular to the young, whom Garland argues in line
78 are crucial to the continuity of a language.
On the other hand, such clans are a platform for minority languages as they promote
understanding of Chinese language, culture and values and are a testament to how
important it is to learn local and vernacular culture in a globalised world.
Malay
There has been a marked increase in the number of sermons delivered in English at
mosques during Friday prayers.
Greater use of English in MUIS religious knowledge programmes at kids, teens and
youth levels.

trendyline

Tamil

Paragraphs 4 & 5 Globalisation


not
only
causes
languages to die off
but it can also help
dying
or
dead
language to turn
around and become
vibrant again.
Paragraph 9
Extinction of minority
languages not a
certainty
because
the
trend
of
homogeneity
of
global culture has
stimulated
many
people to search for
their native roots and
hold tighter to their
cultural identity

Paragraph 6
There are various
benefits to learning
or
relearning
a
native
language/
several languages.

Spoken by 60% of Singapores Indian population, other widely spoken Indian


languages are Punjabi, Malayalam, Hindi and Telugu.
Acknowledged by many to be facing a number of crises:
Tamil as a home language is not being maintained by the better-educated.
Policy studies have shown that the shift towards using English as a home
language is the most obvious among the Indians in Singapore.
Indian education in Singapore is not living up to the expectations many
people have for it.
More than half of the Indians surveyed said that they learn about their religion
(Hinduism) mainly through English.
Tamil is thought of as a coolie language.

In Singapore, the force of globalisation has stirred nostalgia and a desire to cement
Singapores heritage cultural identity and to improve inter-generational communication
this has resulted in small-scale ground-up initiatives in reviving minority languages.

A series of books with the back-to-basics approach has been published and sold in
Singapore by Koh Kuan Eng, a creative director in advertising turned social worker.
There are currently 5 books in this series that covers 5 Chinese dialects Hokkien,
Teochew, Hainanese, Cantonese and Hakka (from sibeynostalgic.com).

Along with this series of books, a series of tote bags have also gained popularity. They
feature various objects and items with how they are spoken in various dialects (from
sibeynostalgic.com).

Dialects are also featured in many local Jack Neo films, further cementing the Singapore
identity.

The Peoples Association has rolled out dialect-related activities in several Community
Centres island-wide. There are dialect singing courses and a dialect initiative piloted in
2011 to reach out to young people through courses such as Learn Cantonese Language
through Forgotten Cantonese Cuisine.

The Chinese Theatre Circle has older Chinese Singaporean members and supporters
who turn to opera for opportunities to speak dialects with one another. The Glowers
Drama Group whose members are all above 50, use a mix of Cantonese, Mandarin and
English in their plays.

An act of self-definition, one that brings solidarity with our neighbours.


Singapore identity and heritage.
Pre-schoolers learn about Singapores heritage through Mother Tongue languages
(pilot programme in 2015 by National Heritage Board).
Can be used as a tool to differentiate between Singaporeans and foreigners due to
the use of Singlish.
Singlish amalgamates various minority languages with English as it consists of words
originating from Malay, Hokkien, Teochew, Cantonese, Tamil and etc.
Singaporeans are proud of Singlish even though it is heavily discouraged officially but
continues to be used in daily life and even in local dramas and comedies.
Celebrated during the SG50 National Day Parade as one of the distinguishing factors
of being a born-and-bred Singaporean.
Code-switching is a skill that Singaporeans have been repeatedly urged to master so
that we are able to revert to using Standard English as well as our mother tongues in
formal situations.

Can help young people to master several languages which could be an advantage.

trendyline

For Singaporeans, the advantage comes from having better inter-generational


communication with the older population.
There is not much economic or political advantage for Chinese dialects; however, the
Singapore government constantly urges Chinese Singaporeans to be conversant and
proficient in Mandarin as such a proficiency would be beneficial to us due Chinas
huge economic role in the world today.
For Malay and Tamil and the other minority Indian languages, some economic and
political advantages could be present (e.g. Taufik Bautisah, first ever winner of
Singapore Idol, could break into the Malaysian entertainment scene by singing in his
mother tongue).

Paragraph 7 It is becoming clear


that when people
have
a
strong
cultural reason to
reverse
language
shift,
they
can
effectively resist the
onslaught of majority
languages.
Moreover,
mass
media technologies
are now helping
minority languages
to
make
a
comeback.
Paragraph 8 Globalised
commerce and the
media
are
not
necessarily
the
death knell for local
languages, because
certain
trends
support
their
preservation.

Due to Singapores unique geographical location, some are stressing on the importance
to learn or be proficient in Malay.
All of Singapores Prime Ministers (as well as some key ministers) could
converse fluently in Malay as it is crucial in establishing important political ties (as
well as fluency in Mandarin and English).
Remembering that Malay is Singapores national language could help us to better
connect with our historical heritage.

Contrary to what Garland asserts, there is no need to resist any onslaught of majority
languages in Singapore and thus reverse language shift any resurgence of minority
language in Singapore mostly comes from the desire to retain Singapores culture and
heritage, in which many minority language played a crucial role in.

It is not very clear or obvious that it is the multimedia that will assure a continuity of
minority languages in Singapore. Rather, it is the very essence of Singapores multiracial and multi-ethnic social fabric that is a larger determinant of the continuity of minority
languages in Singapore as each generation, backed by political and social rhetoric, seeks
to pass down the traditions, culture and language associated to each of the minority
languages (though it is not so much the case for Chinese dialects).

The internet and social media platforms like YouTube and Facebook allow the younger
generation to have access to greater variety of programmes and videos that contain
minority languages.
While local broadcasting channels censor dialects from their programmes, one would
be able to find many short clips containing these censored portions on YouTube.
There are even videos put up by young people that aim to teach others how to speak
dialects.

The availability of
cheap,
powerful
multimedia
will
assure a continuity
of
minority
languages.

trendyline

2015 Prelim Paper 2 Standardisation (AQ)


Script A

There are limited attempts to evaluate and link to the students society
However, the development of ideas is not really coherent
Student rehashes the text towards the end of the AQ response
Fits into Band 3

Mark: 3/10
Language: 9/15
Better than Script B because there are fewer grammar errors
Some lifting in summary
Script B

Like Script A, there are attempts to evaluate and link back to Singapore society but content does not
reflect reality
Misinterpretation of the text in the second paragraph

Mark: 2/10
Language: 8/15
Errors are frequent but there are consistent attempts to rephrase so the language does not fit into Band
C
Script C

The response has features that fit into Band 2 it has valid evaluation and there is a basic
understanding of the text
Not high Band 2 because the overall response does not really answer the question

Mark: 4/10
Language: 11/15
There are consistent attempts to paraphrase
Decent fluency in AQ
Varied sentence structures and few glaring grammar errors
Script D

Fits into Band 2 because there are attempts to answer the question and provide evaluation not merely
restating the text
Development of ideas in paragraphs is not really coherent in the first half of the response, and some
contradiction in arguments
Last body paragraph offered a better quality response
Comparable to C, but same grade is given for different reasons

Mark: 4/10
Language: 11/15
Organisation of ideas is rather weak (from AQ)
Language is more succinct than C but there are more instances of lifting than script C

trendyline

Some expression inaccuracies and lifting

Script E

Overall response fits into Band 2


There is coherence throughout the whole response and consistent attempt to answer the question
Script E has more scope and coherence as compared to other scripts, hence a higher grade
However, there seems to be confusion between culture and language at some points
Development of ideas is not really coherent throughout paragraph 2
Text referencing has to be more explicit

Mark: 5/10
Language: 12/15
Technical errors are rare
There is overall fluency throughout the script
Expression is succinct and more precise than all the other scripts

trendyline

NANYANG JUNIOR COLLEGE


JC2 PRELIMINARY EXAMINATION 2015

Candidates Name: __________________

Class: __________ GP Tutor: ____________

GENERAL PAPER

8807/2

Paper 2:

2 September 2015
1 hour 30 minutes

ANSWER BOOKLET
INSTRUCTIONS TO CANDIDATES
Write your name, class and GP tutors name in the spaces at the top of the page.
Write in dark blue or black pen on both sides of the paper.
Do not use staples, paper clips, highlighters, glue or correction fluid.
Answer all questions.
The Insert contains the passage for the comprehension.
Note that 15 marks out of 50 will be awarded for your use of language.
At the end of the examination, fasten all your work securely together.
The number of marks is given in brackets [ ] at the end of each question or part question.
SAQs

/17

Summary

/8

Application Qn

/10

Language

/15

Total

/50

This answer booklet consists of 7 printed pages including this page


trendyline

Read the passage in the Insert and then answer all the questions. Note that up to
fifteen marks will be given for the quality and accuracy of your use of English
throughout this Paper.
NOTE: When a question asks for an answer IN YOUR OWN WORDS AS FAR AS
POSSIBLE and you select the appropriate material from the passage for your answer,
you must still use your own words to express it. Little credit can be given to answers
which only copy words or phrases from the passage.

Explain in your own words as far as possible how clothes shopping is much like
social media (line 3).
.
....

. [2]

Explain what the writer means by hedonic treadmill (line 9). Use your own words as
far as possible.

[2]

Explain the writers use of the word engineer (line 20).

[1]

trendyline

For
Examiners
Use

What is the writer implying about fast fashion brands by the use of the word notorious
(line 26)?

[1]

Explain in your own words as far as possible the difference in the ways fast fashion
labels and mid-market and luxury brands play off consumers desire for a bargain (line
29).

... [2]

What are the conditions that make it easy for people to buy things that they dont
need or even really want (lines 38 39)? Use your own words as far as possible.

... [3]

Explain what the writer means by this glut of clothing is having effects beyond stuffing
our closets (line 40). Use your own words as far as possible.

...
... [2]

trendyline

For
Examiners
Use

In paragraph 7, what conclusions can be easily made and what are not, when studying
the consumption patterns of Americans?
.

[2]

How does the popularity of YouTube haul videos (line 62) demonstrate millennials love
for shopping?
.
.
.[1]

10

Identify one phrase in paragraph 1 which echoes Its consumerism as entertainment


(line 95).
...
. [1]

trendyline

For
Examiners
Use

11

Using material from paragraphs 10 13, summarise what the writer has to say about
compulsive shopping and the backlash against mindless overconsumption as well as
his response to the backlash.
Write your summary in no more than 120 words, not counting the opening words
which are printed below. Use your own words as far as possible.
Compulsive shopping

.
.
.
......
. [8]

trendyline

For
Examiners
Use

12

Marc Bain discusses some issues around the subject of clothes shopping. How far
would you agree with his observations, relating your arguments to your society?

trendyline

For
Examiners
Use

[10]
THE END

trendyline

For
Examiners
Use

NANYANG JUNIOR COLLEGE


JC2 PRELIMINARY EXAMINATION 2015

GENERAL PAPER
PAPER 2:
INSERT

8807/2
2 September 2015
1 hour 30 minutes

READ THIS INSTRUCTION FIRST

This insert contains the passage for Paper 2.

________________________________________________________________
This insert consists of 4 printed pages including the cover page.

trendyline

Marc Bain writes about clothes shopping.


1

In wealthy countries around the world, clothes shopping has become a widespread
pastime, a powerfully pleasurable and sometimes addictive activity that exists as a
constant presence, much like social media. The Internet and the proliferation of
inexpensive clothing have made shopping a form of cheap, endlessly available
entertainmentone where the point isnt what you buy so much as it's the act of
shopping itself.
This dynamic has significant consequences. Secondhand stores receive more clothes
than they can manage and landfills are overstuffed with clothing and shoes that dont
break down easily. Consumers run the risk of ending up on a hedonic treadmill in which
the continuous pursuit of new stuff leaves them unhappy and unfulfilled. For most,
breaking the cycle isnt as easy as just vowing to buy nothing. Its no accident that
shopping has become such an absorbing and compulsive activity: The reasons are in
our neurology, economics, culture, and technology.
Shopping is a complex process, neurologically speaking. In 2007 a team of researchers
looked at the brains of test subjects using fMRI technology as they made decisions while
out buying clothes. They found that pleasure kicks in not only from the act of looking, but
also from purchasing, or more specifically, getting a bargain. It's whats called
transactional utility says Tom Meyvis, a professor of marketing at NYUs Stern School
of Business. If seeing items you want and getting a bargain both elicit waves of shopping
joy, you couldnt engineer a more pleasurable consumer culture than the modern,
globalised West.
Fast fashion perfectly feeds this neurological process. First, the clothing is incredibly
cheap, which makes it easy to buy. Second, new deliveries to stores are frequent, which
means customers always have something new to look at and desire. Zara stores get two
new shipments of clothes each week, while H&M and Forever21 get clothes daily. These
brands are notorious for knocking off high-end designers, allowing the customer to get
something at least superficially similar to the original at a small fraction of the cost, and
theyre priced lower than the rest of the market, making their products feel like a bargain.
Mid-market and luxury brands play off consumers desire for a bargain as well, with
many seeming to be perpetually holding sales. To facilitate the frequent markdowns they
offer, several now inflate their initial retail prices. Theyre able to protect their margins
and let customers believe theyre getting a deal, enticing them to buy more.
Overall, clothes have been getting cheaper for decades, ever since apparel
manufacturing started moving to developing countries, where production costs are
significantly lower. The spread of fast-fashion chains has also helped spur the process.
Data from the U.S. Bureau of Labor Statistics Consumer Price Index shows that clothing
prices have generally decreased. This means Americans are able to buy more clothing,
and as incomes have increased overall, they spend less of their money on it. These
conditions make it easy for people to buy things they dont need or even really want.
This glut of clothing is having effects beyond stuffing our closets. About 10.5 million tons
of clothes end up in American landfills each year, and secondhand stores receive so
much excess clothing that they only resell about 20 percent of it. The remainder is sent
to textile recyclers, where its either turned into rags or fibers, or, if the quality is high
enough, its exported and cycled through a cutthroat global used-clothing business.

trendyline

10

15

20

25

30

35

40

10

11

12

13

Determining exactly how much time people spend shopping for clothing isnt simple. The
U.S. Bureau of Labor Statistics conducts an American time use survey, but clothes
shopping is lumped in with shopping for everything else except groceries and gas. It is
clear, however, that more and more Americans are shopping online, and early evidence
suggests that they are shopping more often. Andrew Lipsman, vice president of
marketing and insights at the Internet research firm ComScore, says that mobile
shopping in particular has exploded.
Mobile, in fact, is now the primary way people buy online, and one ComScore study on
mobile shopping in five key European countries found that purchases of clothing and
accessories led all other categories. Lipsman also points out that this mobile browsing
didnt necessarily lead to purchases. Browsing is also about research and entertainment.
It is more than just transactional, he says.
The obsession with looking at products, even if no purchase is intended, is especially
prevalent among Millennials, the generation that grew up in the age of the Internet. A
report by the Urban Land Institute concluded that 45 percent of Millennials (called
Generation Y in the report) spend more than an hour each day looking at retail sites. A
series of separate reports on millennials reached similar conclusions: They love to shop,
even if theyre not buyingalthough plenty are buying, too. YouTube haul videos, which
feature mostly teen girls posting their scores from shopping trips, have become so
popular that the bigger names, such as Bethany Mota, are now bona-fide social media
stars.
Studies of how the Internet plays into compulsive buying are in their early stages, but the
evidence so far suggests there may be a link. One of the few relevant longitudinal
studies on compulsive shopping, published in 2005, looked at the way East Germans
integrated into Western society after the fall of the Berlin Wall. The study found that, as
East Germans settled into Western consumer culture, they showed a marked increase
in compulsive buying. The authors concluded that postmodern consumer societies
create an atmosphere which supports the rise of compensatory and compulsive
buying.
April Lane Benson, a psychologist and the author of To Buy or Not To Buy: Why We
Overshop and How To Stop, specialises in treating compulsive shopping. When she
describes the reasons for people constantly browsing as entertainment, she makes it
sound like an existential crisis. I think that it has something to do with the pace that we
live our lives at and the paucity of time that so many of us spend in pursuits that really
feed our souls, she says. Shopping is a way that we search for our selves and our
place in the world. A lot of people conflate the search for self with the search for stuff.
Shopping therefore becomes a quick fix, as she puts it, for other problems.
There has been a backlash against what some perceive as mindless overconsumption.
In the past few years a slow fashion movement has emerged which emphasises buying
less clothing and sticking to garments made using sustainable, ethical practices. The
recent book by Japanese organisational guru Marie Kondo, The Life-Changing Magic of
Tidying Up, has led to whats been described as a cult of decluttering, with her acolytes
boasting of shedding piles of clothing.
Lets take a breath here. Residents of industrialised societies are not all doomed to

trendyline

45

50

55

60

65

70

75

80

85

endless compensatory shopping just because our brains seem to enjoy it and our
cultures are set up for it. The five-minute break from work you take to look at clothes
doesnt necessarily mean youre searching for your identity in a pair of pants, or that
youre trying to fill a void. The evidence does suggest, however, that shopping has taken
on a new role in our society and in our lives. Its no longer just a transaction, a way to
procure necessities or luxuries, but rather has become an end in itself. Its a leisure
activity, much like watching TV. Its consumerism as entertainment.

trendyline

90

95

1. Explain in your own words as far as possible how clothes shopping is much like
social media (line 3). [2] (PARAPHRASING)
Like social media, clothes shopping
is
a widespread pastime,

a. a common/popular

that exists as a constant presence

ALSO ACCEPT:
an activity one can engage in at any
time/anywhere
an activity that is always
available/accessible
an activity that many people engage
in
(Clothes shopping is)
everywhere/ever
present/omnipresent/prevalent
(problematic expression, though)

a powerfully pleasurable and

b. highly enjoyable/satisfying and


NOTE: powerfully must be paraphrased

sometimes addictive activity

c. absorbing/compelling/compulsive/
habitual leisure activity
ALSO ACCEPT: an activity that we are
hooked on/we find impossible or difficult
to stop engaging in/we cannot do
without/we find irresistible/keeps us
wanting more
DO NOT ACCEPT: develop an attachment
to

Mark scheme: 1-2 points 1m; 3 points 2m

Get PSJ Private Tutor to Guide you through Exams Now!


Contact www.privatetutor.com.sg

trendyline

2. Explain in your own words as far as possible what the writer means by hedonic
treadmill (line 9). [2] (EXPLAIN WHAT THE WRITER MEANS/PARAPHRASING)
Consumers run the risk of ending up on a
hedonic treadmill in which
the continuous pursuit of new stuff
leaves them unhappy and unfulfilled.

ANSWER I:
a. The relentless/never-ending
consumption of new stuff [1]
b. leaves
consumers dissatisfied/discontented. [1]
ANSWER II:
a. The relentless/never-ending
consumption of new stuff [1]
b. does not make us happier or more
satisfied/contented [1]

3. Explain the writers use of the word engineer (line 20). [1] (USE OF LANGUAGE)
If seeing items you want and getting a
bargain both elicit waves of shopping joy,
you couldnt engineer a more pleasurable
consumer culture than the modern,
globalized West.

The writer wants to show that the


enjoyable culture of shopping OR
consumer culture (CONTEXT)
is (artificially) created/designed/planned.
(MEANING)
DO NOT ACCEPT: manipulated
NOTE:
Both context and meaning must be
correct to secure the mark
enjoyable is not necessary

trendyline

4. What is the writer implying about fast fashion brands by the use of the word
notorious (line 26)? [1] (USE OF LANGUAGE)
These brands are notorious for knocking The writer is implying that fast fashion
off high-end designers,
brands are
(a) infamous/well or widely known
(MEANING)
(b) for their
unethical/despicable/unscrupulous
practice of copying/imitating clothes of
high fashion brands. OR
for their
unethical/despicable/unscrupulous
practices OR
for copying/imitating clothes of high
fashion brands. (CONTEXT)
NOTE: knocking off high-end designers
can be lifted
DO NOT ACCEPT:
famous or recognised (positive
connotation)
well known despite knocking off high
end designers (misinterpretation)

trendyline

5. Explain in your own words as far as possible the difference in the ways fast fashion
labels and mid-market and luxury brands play off consumers desire for a bargain
(line 29). [2] (CONTRAST/DIFFICULT PARAPHRASING)
These brands are notorious for knocking a(i) Fast fashion labels copy designer
off high-end designers, allowing the clothes and
customer to get something at least
ALSO ACCEPT: sell clothes that are alike
superficially similar to the original
to/are identical to/ are the same
at a small fraction of the cost, and as/imitate/replicate/resemble clothes of
theyre priced lower than the rest of the high-end designer brands
market, making their products feel like a
a(ii) and sell them at an extremely low
bargain.
price, [1]
NOTE: Both (i) and (ii) must be correct to
secure the mark
To facilitate the frequent markdowns
they offer, several now inflate their initial
retail prices. Theyre able to protect their
margins and let customers believe
theyre getting a deal, enticing them to
buy more.

b(i) Mid-market and luxury brands


set extremely high original list prices
b(ii) to make their discounted prices
more alluring/appealing [1]
OR
b(i) Mid-market and luxury brands
set extremely high original list prices,
b(ii) only to lower them later. [1]
NOTE: Both (i) and (ii) must be correct to
secure the mark

trendyline

6. What are the conditions that make it easy for people to buy things that they do
not need or want (lines 38 39)? Use your own words as far as possible. [3]
(PARAPHRASING)
Overall,
clothes
have
getting cheaper for decades,

been a. Clothes today are far more affordable


because

Data from the U.S. Bureau of Labor NOTE: Candidates can lift prices
Statistics Consumer Price Index shows
that
clothing
prices
have
generally decreased.
ever since apparel manufacturing
started moving to developing countries,
where production costs are significantly
lower.

i. Clothes manufacturers have relocated


to less developed countries to capitalise
on the much reduced costs of production
and
ACCEPT: production costs in developing
countries are much reduced
DO NOT ACCEPT: production costs have
been reduced without stating the reason
NOTE:
Candidates
can
lift
apparel
manufacturers,
developing
countries and production costs
significantly must be paraphrased

The spread of fast-fashion chains has ii. The presence of fast fashion labels
also helped spur the process.
is growing.
ALSO ACCEPT: the opening of more/new
fast fashion stores
DO NOT ACCEPT:
the opening of fast fashion stores
the rising popularity of fast fashion
stores
This means Americans are able to buy b. At the same time, people are earning
more clothing, and as incomes have more / more affluent.
increased overall, they spend less of their
money on it.

trendyline

Mark scheme: 1-2 points 1m; 3 points: 2m; 4 points: 3m


Candidates will not get any mark for (i) and/or (ii) if (a) is incorrect (very unlikely,
though). (b) is independent of (a).
7. Explain what the writer means by this glut of clothing is having effects beyond
stuffing our closets (line 40). Use your own words as far as possible. [2] (EXPLAIN
WHAT THE WRITER MEANS/PARAPHRASING + INFERENCE)
This glut of clothing

The excessive amount of clothes that


people own/ Buying too many clothes [1]
ALSO ACCEPT: Excessive shopping for
clothes
DO NOT ACCEPT
Shopping for clothes
(Excessive) Shopping for things
Shopping

is having effects beyond stuffing our


closets. About 10.5 million tons of clothes
end up in American landfills each year,
and secondhand stores receive so much
excess clothing that they only resell
about 20 percent of it. The remainder is
sent to textile recyclers, where its either
turned into rags or fibers, or, if the
quality is high enough, its exported and
cycled through a cutthroat global usedclothing business.

has consequences/ impacts/


ramifications that do not only affect the
individual
OR
are far-reaching (inferred) [1]
ALSO ACCEPT: has consequences/
impacts/ ramifications beyond a full
closet/cupboard/wardrobe (no marks if
the closet is less than full!)
DO NOT ACCEPT:
adverse
consequences/repercussions
macro impacts

trendyline

8. In paragraph 7, what conclusions can be easily made and what are not, when
studying the consumption patterns of Americans? [2] (IDENTIFICATION)
The conclusions that can be easily made
are
It is clear, however, that more and more a(i) that more and more Americans are
Americans are shopping online,
shopping online and
and early evidence suggests that they are a(ii) that they are shopping more often/
shopping more often.
spend more time on shopping
Determining exactly how much time The conclusions that cannot be easily
people spend shopping for clothing isnt made are
simple. The U.S. Bureau of Labor
Statistics conducts an American time use (b)(i) how much time people spend
survey, but clothes shopping is lumped in shopping for clothing
with shopping for everything else except
groceries and gas.
Andrew Lipsman, vice president of (b)(ii) Mobile shopping in particular has
marketing and insights at the Internet exploded
research firm ComScore, says that mobile
shopping in particular has exploded.
NOTE: Paraphrasing is not necessary
Mark scheme: 1-2 points 1m; 3-4 points: 2m
9. How does the popularity of YouTube haul videos (line 62) demonstrate millennials
love for shopping? [1] (USE OF EXAMPLES)
A series of separate reports on
millennials reached similar conclusions:
They love to shop, even if theyre not
buyingalthough plenty are buying, too.
YouTube haul videos, which feature
mostly teen girls posting their scores
from shopping trips, have become so
popular that the bigger names, such as
Bethany Mota, are now bona-fide social
media stars.

A large number of millennials love


shopping
so
much
that
they
derive enjoyment/pleasure/satisfaction/
happiness from watching other people
buy
DO NOT ACCEPT: They enjoy shopping so
much that they show off their latest buys
(wrong perspective)

trendyline

10. Identify one phrase in paragraph 1 which echoes Its consumerism as


entertainment (line 95). [1] (IDENTIFICATION)
In wealthy countries around the world,
clothes
shopping
has
become
a widespread pastime, a powerfully
pleasurable and sometimes addictive
activity that exists as a constant
presence, much like social media. The
Internet and the proliferation of
inexpensive clothing have made shopping
a form of cheap, endlessly available
entertainmentone where the point
isnt what you buy so much as it's the act
of shopping itself.

widespread pastime
a
powerfully
pleasurable
sometimes addictive activity

and

powerfully pleasurable
(sometimes) addictive activity
cheap,
endlessly
entertainment

available

ANY ONE

trendyline

11. Using material from paragraphs 10 13, summarise what the writer has to say
about compulsive shopping and the backlash against mindless overconsumption as
well as his response to the backlash.
Write your summary in no more than 120 words, not counting the opening words
which are printed below. Use your own words as far as possible. [8]
Compulsive shopping
Material from text

Section 1: what author has to say about compulsive shopping

Suggested Answer

how the Internet plays into


compulsive buying the evidence so
far suggests there may be a link
(l66-67)

is (a) possibly
(b) caused/encouraged by or due
to the Internet

postmodern consumer societies


create an atmosphere which supports
the rise of compensatory and
compulsive buying.
(l71-73)

and todays materialistic society/


todays society that believes in the
constant acquisition of goods/ is
attached to possessions.

it has something to do with the pace


that we live our lives at and
the paucity of time
(l77-78)

We shop compulsively

that so many of us spend in pursuits


that really feed our souls
(l78-79)

so much so that we rarely


strive for/seek spiritual fulfilment
OR
find meaning/purpose in life
OR
fill the void/emptiness in our lives

DO NOT ACCEPT:
affected by
technology

because speed dominates our lives


OR
because we are too busy/lack the
time
OR
To seek relief from our hectic lives

DO NOT ACCEPT: therapeutic relief


5

is a way that we search for our selves


(l79)

Shopping is a means to find


out/understand who we are
OR establish our personal
identity/individuality

trendyline

and our place in the world.


(l79-80)

our role/purpose in society/the world

Shopping becomes a quick fixfor


other problems
(l80)

It is a
(a) convenient/expedient/temporary/
superficial (b) solution/ remedy/ to our
problems.

DO NOT ACCEPT: status, position

escape/distraction
Section 2: the backlash against mindless overconsumption

a slow fashion movement has


emerged which emphasizes buying
less clothing
(l83-84)

As a response against compulsive


shopping,
there is now (a) a campaign/drive that
(b) encourages/raise awareness about
purchasing fewer clothes/ discourages
excessive purchase of clothes
(allow the lifting or omission of slow
fashion)

and sticking to garments made


using sustainable, ethical practices.
(l84)

and choosing clothes that are


produced using environmentallyfriendly
OR
morally/socially
acceptable/humane/right methods.
DO NOT ACCEPT: correct

10 a cult of decluttering with her


acolytes boasting of shedding piles of
clothing
(l86-87)

There is also (a) a


(strong) following/support for the
movement that advocates
(b) discarding clothes.

Section 3: authors response to the backlash

11 Lets take a breath here.


(l88)

The author, however, thinks there is


no cause for alarm/ we should not
over-react. (inferred)

12 Residents of industrialised societies


are not all doomed to endless
compensatory shopping
(l88-89)

The persistence of compulsive


shopping is (a) not all (b) bad
and inevitable
NOTE: doom means to destine,

trendyline

especially to an adverse fate


OR
We are (a) not always
(b) vulnerable/susceptible or do not
always fall victim to compulsive
shopping
NOTE: not all must be included in the
answer
13 just because our brains seem to enjoy
it
(l89)

just because people like to shop/a


bargain
OR
people derive neurological pleasures
from shopping
ALSO ACCEPT (albeit begrudgingly):
our minds appear to like shopping
(-_-)

14 and our cultures are set up for it.


(l89)

and their (a) lifestyles/ways of life


(b) allow/encourage it.
DO NOT ACCEPT: ready for it

15 The five-minute break from work you


take to look at clothes
doesnt necessarily mean
youre searching for your identity in a
pair of pants.
(l90-91)

He does not think we (a) always shop


to (b) assert our individuality/acquire
our sense of self

16 or that youre trying to fill a void.


(l91-92)

or make up for something missing in


our lives
OR
find meaning/purpose in life. (inferred)

17 shopping has taken on a new role in


our society and in our lives.
(l92-93)

He thinks shopping however has


a different function today.

18 Its no longer just a transaction


(l93)

Shopping is (a) no longer (b) merely (c)


a business deal/economic activity/an
exchange of money for goods

NOTE: necessarily must be


paraphrased

(allow the lifting of no longer and

trendyline

just)
19 or a way to procure necessities or
luxuries
(l93-94)

or a way to acquire things (to satisfy


our needs and wants/desires)

20 but rather has become an end in itself. People shop for the sake of shopping.
(l94)
21 Its a leisure activity, much like
watching TV.
(l94-95)

They find
it relaxing/enjoyable/satisfying.

Its consumerism as entertainment.


(l95)
Mark allocation
1-2 pts:
5-6 pts:
9-10pts:
13-14pts:

1m
3m
5m
7m

ALSO ACCEPT: derive happiness from


shopping

3-4 pts:
7-8 pts:
11-12pts:
15pts and above:

2m
4m
6m
8m

trendyline

12. Marc Bain discusses some issues around the subject of clothes shopping. How far
would you agree with his observations, relating your arguments to your society? [10]
Point From Passage
The Internet and the
proliferation of inexpensive
clothing have made
shopping a form of cheap,
endlessly available
entertainment (Paragraph 1)

Evaluation
WHY I AGREE
The internet has indeed made clothes shopping
more accessible to consumers. (EVALUATION)
People are able to browse through arrays of
clothing at any time of the day as long as they
have an internet connection, unlike physical retail
shops where shopping is restricted by specific
operating hours. (ELABORATION)
In Singapore, online shopping is an activity easily
available to majority of the population as 88% of
Singaporeans have access to the internet in 2014
according to the Infocomm Development
Authority Singapore.
From established international brands like
Forever21 and ASOS to local online shops like Love
Bonito and Her Velvet Vase, Singaporeans now
have no lack of choice when it comes to buying
affordable and fashionable clothing.
(EXEMPLIFICATION)
Online shopping websites have also burgeoned over
the past few years, giving rise to fiercer competition
between clothing brands. (ELABORATION) It is no
wonder the prices of clothes have fallen given that
supply has increased dramatically. (EVALUATION)
Sale gimmicks employed by online shops to
increase sales have also led to significant savings
for consumers. (ELABORATION)
For instance, retail giants like H&M and
Abercrombie and Fitch provide online coupon
codes which consumers can use to enjoy
discounts. (EXEMPLIFICATION)
WHY I DISAGREE
While the prices of clothes itself may be low, clothes
shopping is not necessarily cheap when one factors
in the high shipping costs that often accompany
shopping. (EVALUATION)
Shipping costs for Forever21s clothing is
ludicrously expensive US$50 worth of purchases
would cost US$48 to ship to Singapore which is

trendyline

almost equivalent to the price of the clothes!


(EXEMPLIFICATION)
the continuous pursuit of
new stuff leaves [us]
unhappy and unfulfilled.
(Paragraph 2)

WHY I DISAGREE
Admittedly, it would be too far-fetched to claim that
shopping does not bring us any happiness; it does,
though the sense of satisfaction derived from retail
therapy is transient. (EVALUATION)
In Singapore, people work extremely long hours.
To reward themselves for a hard week or months
work, they shop, and they normally get something
expensive (or more than what they would usually
pay for) to make up for the physical and mental
ordeal they go through, which they call work.
They buy yet another luxury watch or designer
handbag also to earn some bragging rights.
Photos of their latest purchases posted on
Facebook and Instagram would invariably garner
the coveted likes and comments of envy. These
can be gratifying until reality sets in (well, work
beckons) and the cycle continues.
(ELABORATION/EXEMPLIFICATION)
WHY I AGREE
But this material pursuit is an ultimately
unsatisfactory one. (EVALUATION)
As evident from the above, Singapore is
a materialistic society, not to mention a highly
competitive one, where everyone struggles to
keep up with, or better yet outdo, everyone else.
Because of their materialistic and competitive
nature, they like some form of tangible evidence
showing that they are successful, and so they go
shopping. Their status symbols are luxury brands
like Louis Vuitton, Chanel and Hermes because
their designs are easily identifiable, and their
hefty price tags put their owners clearly in an
income bracket that not only appreciates but also
deserves the finer things in life. Sadly, it is not
enough for Singaporeans to make a purchase
once and for all and be happy. They must keep up
with the latest fashion or otherwise be seen as
outdated or worse, inferior to their friends and
neighbours. These material wants do not come
cheap, and we have to work harder than we
already are in order to keep up with the pretense.
Unless we learn to stop living to earn and spend

trendyline

money in a continuous cycle, we will never get off


the endless hedonic treadmill that keeps us
down and miserable while constantly looking
forward to the occasional temporary high for the
energy to keep us running on.
(ELABORATION/EXEMPLIFICATION)
you couldnt engineer a
more pleasurable consumer
culture than the modern,
globalized West.
(Paragraph 3)

WHY I AGREE
Without a doubt, the mall-shopping experience is
designed for maximum comfort, convenience and
service to be enjoyed by the customer.
(EVALUATION)
Air-conditioning to beat the heat outside; bright
lights highlighting exciting retail displays; music to
set the atmosphere, encouraging patronage from
the desired type of clientele; easy access to
transportation, entertainment and food so that
the shopping experience is one continuous stretch
of ease and comfort without interruption. It is no
surprise, then, that Singaporeans love to shop
and actually buy things while theyre at it.
Mall design aside, product offering is also key to
an enjoyable shopping experience. This is why
Singapore is widely touted around the world as a
shoppers paradise. In terms of mall design and
product offering, the shopping experience here is
as close to mall-shopping in the West as the
neighbouring region can get. The astounding
variety of brands available in Singapore, not to
mention the relatively more affordable prices
(thanks to our lower import taxes), is unrivalled in
Southeast Asia, attracting tourists from Malaysia,
Thailand and Indonesia by the hordes to do their
shopping here. (ELABORATION &
EXEMPLIFICATION)
WHY I DISAGREE
While the consumer culture is engineered to induce
satisfaction, ultimately getting people to shop to
their hearts content, shoppers may not always find
the experience pleasurable. (EVALUATION)
The biggest problem with the vast majority of
shopping malls in Singapore is a lack of
differentiation they look and feel as if they are
all made from the same mould. With the
exception of high-end malls, such as Orchard Ion,

trendyline

Ngee Ann City and Paragon, the rest have more or


less the same tenants you can expect to see a
Uniqlo, H&M and/or Charles & Keith in any largescale shopping centre. After a while, shopping
becomes boring, and Singaporeans find it more
satisfying to shop online or do it overseas.
(ELABORATION & EXEMPLIFICATION)

They found that pleasure


kicks in not only from the act
of looking, but also from
purchasing, or more
specifically, getting a
bargain
These brands are notorious
for knocking off high-end
designers, allowing the
customer to get something
at least superficially similar
to the original at a small
fraction of the cost, and
theyre priced lower than
the rest of the market,
making their products feel
like a bargain. Mid-market
and luxury brands play off
consumers desire for a
bargain as well, with many
seeming to be
perpetually holding sales. To
facilitate the frequent
markdowns they offer,
several now inflate their
initial retail prices. Theyre
able to protect their margins
and let customers believe
theyre getting a deal,
enticing them to buy more.
(Paragraphs 3 & 4)

WHY I AGREE
Unless one has pockets as deep as those of Jamie
Chua, a Singapore socialite whose impressive
Hermes Birkin collection would even make Victoria
Beckham swoon with envy, almost everyone wants
the best value for their money. This explains why
low-end and mid-range brands like COS, H&M, Zara
and Massimo Dutti, which copy high-end fashion
and sell it for only a mere fraction of the original
price, are doing roaring business worldwide.
(EVALUATION)
Singapore may be one of the most affluent
countries in the world, but her people are
also highly pragmatic. The majority of
Singaporeans, while indulging in an occasional
Chanel 2.55, do not shop at luxury boutiques on a
regular basis. They prefer seeking out sartorial
bargains at the likes of Zara where they can look
like a million dollars without actually burning a
hole in their pocket. Packed stores during the sale
season are a testament to Singaporeans love of
all things cheap and good. (ELABORATION &
EXEMPLIFICATION)
WHY I DISAGREE
While fast fashion looks like it is here to stay, high
fashion is becoming a rising trend, as income
increases and sartorial taste becomes more
sophisticated. While low-end to mid-range fashion
brands will continue to dominate the wardrobe of
the man on the street, the middle class are turning
to luxury brands mostly for the prestige factor
snapping up bags, shoes and accessories, even at full
retail prices. (EVALUATION/ELABORATION)
Although most luxury brands, with the exception
of Chanel, Goyard, Hermes and the like, hold end-

trendyline

of-season sales twice a year, the majority of


coveted items, especially the classic colours and
styles, never go on sale. They also happen to be
the most popular with customers and are usually
sold out soon after they are displayed on the shelf.
(EXEMPLIFICATION)

Overall, clothes have been


getting cheaper for
decades
incomes have increased
overall These
conditions make it easy for
people to buy things they
dont need or even really
want. (Paragraph 5)

WHY I AGREE
The prices of clothes have indeed been falling over
the years (EVALUATION), as many producers
outsource production to developing countries like
China and Thailand where costs are much lower.
Furthermore, cheap clothes can be easily
purchased via online shopping platforms like
Taobao that sell relatively inexpensive clothing.
(ELABORATION)
We have also been growing more affluent
(EVALUATION), thanks to our rapid consistent
economic growth over the years since our
independence. (ELABORATION)
It indeed becomes easy to make unnecessary and
mindless purchases when the value of such
purchases is deemed to be insignificant.
(EVALUATION)
Research agency Frost and Sullivan says Singapore
was the largest e-commerce market in South-east
Asia last year, generating revenues of US$1.7
billion (S$2.1 billion). This is confirmed by a recent
survey of shopping habits here by Japanese ecommerce giant, Rakuten. (EXEMPLIFICATION)
WHY I DISAGREE
While it is true these conditions make it easy for
people to buy more things unnecessarily, a more
significant reason for why people make unnecessary
purchases these days can be attributed to the rise of
mobile shopping. (EVALUATION)
With most Singaporeans owning a 3G enabled
mobile phone as well as more retailers opening
up more online store options, including mobile
apps created just for it, shopping has never been

trendyline

more convenient. However, the danger of such


convenience is that it makes it easy for people to
make immediate purchases to gratify their
shopping impulses, even irrational ones.
(ELABORATION)
It is clear, however,
that more and more
Americans are shopping
online,
Mobile, in fact, is now the
primary way people buy
online, and one ComScore
study on mobile shopping in
five key European countries
found that purchases of
clothing and accessories led
all other categories.
(Paragraphs 7 & 8)

WHY I AGREE
There is no denying that the internet is fast
becoming an integral part of our lives we depend
heavily on it for news and information, interpersonal
communication and of course, shopping.
(EVALUATION)
Due to the hectic pace of life today (Singaporeans,
work an average of 2402 hours a year, the longest
in the world), many do not have the time to trawl
malls in pursuit of sales and good buys and hence
embrace the convenience of online shopping. This
enables them to find good buys without having to
jostle with the crowds as well as lug their
purchases home as the purchases will be
delivered to their very doorstep.
Younger shoppers also want more unique clothes
to stand out from the masses. As a result they
turn to online shops that have varied options as
well as those who have more eclectic tastes such
as vintage and antique clothes. Many seek out
stores such as Etsy and eBay for vintage fashion.
In addition, with the changing diet as well as
lifestyle choices available today the average
Singaporean no longer fits the petite stereotype.
A more Western and global diet has resulted in
average body sizes expanding. According to Straits
Times, the average female in Singapore is a UK 12
which is equivalent to the beginning of the plus
size range. In spite of that, many retailers are
catering only to smaller sized women. Only a few
do Dorothy Perkins, H & M, River Island. Many
plus-sized women (and men) are hence seeking
out online options, such as Forever21 Plus, Asos
Curve, The Curve Cult, where size options are
almost limitless. Emotionally, it is also less
degrading for plus-sized shoppers to purchase
online as they no longer need to be told in the
face that the store does not cater to plus sizes or
humiliation when sales assistants their size behind
their back.
Online retailers are also aggressively promoting

trendyline

their shops with attractive discounts, member


coupons and with free delivery as well as returns
and refunds. Online shops like Zalora and Asos
have regular promotions and offers, and some
have even tied up with companies that offer cash
back schemes like Shopback. And until recently
most physical shops in Singapore did not accept
refunds and even an exchange in size was
frowned upon. Due to competition from eretailers, some of these shops, such as Uniqlo and
CottonOn, have started accepting refunds but the
vast majority still does not. Hence, Singaporean
shoppers are beginning to fully embrace the
attractive offers available online to get good deals
and offers.
In 2014, online spending here exceeded $1 billion,
up 13 per cent year on year beating the 12 per
cent pace set in 2013. Conversely, sales at many
malls in Singapore have reported a significant
drop in number of customers and the shopping
belt in Singapore, Orchard Road is losing its shine
amongst consumers. (ELABORATION &
EXEMPLIFICATION)
WHY I DISAGREE
No doubt, shopping for clothes online is a rising
trend worldwide, but compared with their Western
counterparts, Singaporean shoppers are still rather
traditional in their approach towards clothes
shopping. This is especially true for older shoppers
the middle aged and elderly. (EVALUATION)
Many still enjoy the experience of trawling malls
and physically trying on clothes.
They are also fearful of purchasing clothes that
may not fit them and the hassle of having to mail
them back. According to The Straits Times:
Western shoppers are more ready to purchase
clothes while browsing online compared with
Asian shoppers, according to Asian e-commerce
giant Zalora and that buyers in Asia tend to spend
a lot more time considering their purchases. Due
to this e-retailers are coming up with
sophisticated software to try on clothes virtually
via avatars and fit determiners.
Furthermore, they have fears of using their credit
card information to make payment online (the
usual method of making payment for online

trendyline

shops) due to the possibility of credit card theft


and the find other options available such as using
Paypal and other such middleman services,
daunting due to the complexity involved in setting
up such accounts. (ELABORATION &
EXEMPLIFICATION)
There has been a backlash
against what some perceive
as mindless
overconsumption. In the
past few years a slow
fashion movement has
emerged which emphasizes
buying less clothing and
sticking to garments made
using sustainable, ethical
practices (Paragraph 12)

WHY I AGREE
Of late, the slow fashion movement has been
steadily gaining momentum in many societies
around the world, including Singapore.
(EVALUATION)
As people are more educated today and more
aware of global issues and environmental issues,
they have started to realise that the shopping
culture is affecting the environment and the
livelihood of the poor workers in developing
countries and sought alternatives that are more
eco-friendly, such as secondhand clothes or
clothes that made of recycled materials.
(ELABORATION)
There are more eco-friendly brands emerging in
Singapore such as H & M Conscious, Biro, Honest
etc.
H&M is also the first fashion company to have a
garment collecting initiative. Collecting boxes are
placed at every cash point in all their stores,
receiving unwanted pieces of any brand which will
eventually be re-worn, reused or recycled. This is
part of H&M's ongoing efforts to promote
economic, social and environmental sustainability.
It is ranked as 64 on the Global annual list of 100
of the most sustainable companies in the world.
This shows how fast fashion need not necessarily
harm the environment. (EXEMPLIFICATION)
WHY I DISAGREE
More shoppers may have become more
environmentally and socially conscious, but by and
large the vast majority of people are not active
supporters of the slow fashion movement,
(EVALUATION PART 1) as they are not particularly
concerned about the implications, which do not affect
their lives in any significant way.
Most still feel that vintage fashion is taboo as the
more superstitious older Singaporeans, for

trendyline

example, feel that it is not advisable to wear used


clothing due to paranoia (associations with dead
previous owners, hygiene etc.)
In addition, there is a common perception
that eco-fashion is not fashionable or of good
quality. Most would prefer to purchase from
regular stores.
Furthermore, often eco-friendly fashion uses
alternative sources (hemp, bamboo, organic
cotton etc.) which are more costly due to the
complexity of the production methods, with the
exception of a few like H&M. Many charge
exorbitant prices for their eco-friendly pieces.
Often the majority of these eco-friendly brands
are also high fashion brands that are trying to
woo the wealthier crowd (Stella McCartney, Gucci
and Saint Laurent etc). The hefty price tags mean
that only a few can afford and relate to the cause.
(ELABORATION & EXEMPLIFICATION)

Even for the minority of shopper who are indeed


turning to vintage items or even eco-friendly fashion
as an alternative, it is more due to the fact that do
not want to look common and instead want to gain
style credit by being unique and owning these often
one of a kind gems. (EVALUATION PART 2)
Many, like popular bloggers, love mixing their fast
fashion purchases with vintage or eco fashion to
create their own looks. They feel that picking up
an off-the-rack, mass-produced outfit at a highstreet store will make them resemble everyone
else. Hence they seek out these alternatives but
not because of the fact that they are trying to
deliberately be socially or environmentally
conscious. (ELABORATION & EXEMPLIFICATION)

trendyline

PIONEER JUNIOR COLLEGE


JC2 PRELIMINARY EXAMINATION
GENERAL PAPER

8807/2

Higher 1
Paper 2

28 Aug 2015
1 hour 30 minutes

ANSWER BOOKLET

Candidates Name: ______________________________ CT Group: _____________


GP Tutor: ______________
READ THESE INSTRUCTIONS FIRST
Write your name, CT group and GP tutors name on the cover page of this answer
booklet.
Write in dark blue or black pen on both sides of the paper.
Do not use staples, paper clips, highlighters, glue or correction fluid.
Answer all questions in the Answer Booklet.
Note that 15 marks out of 50 will be awarded for your use of language.
The number of marks is given in the brackets [ ] at the end of each question or each
part question.
For Examiners Use
Content

/35

Language

/15

TOTAL

/50

This document consists of 6 printed pages.


PJC 2015

[Turn over]

trendyline

For
Examiners
Use

PAPER 2 (50 marks)


Read the passage and then answer all the questions which follow below. Note that up to
fifteen marks will be given for the quality and accuracy of your use of English throughout
this paper.
Note: When a question asks for an answer IN YOUR OWN WORDS AS FAR AS
POSSIBLE and you select the appropriate material from the passages for your answer,
you must still use your own words to express it. Little credit can be given to answers
which only copy words or phrases from the passages.
1

In lines 2-3, what similarity do the authors see between the Age of Loneliness and the
ages that precede it? Use your own words as far as possible.

[2]
2

What is the claim made by Thomas Hobbes in lines 7-8 and why do the authors
disagree with it? Use your own words as far as possible.

[3]
3

What do the words just as (line 14) tell you about the effect of the epidemic on older
people?

[1]
4

What is implied by the shifts (line 20) described in the sentence Today, people travel
by car instead of buses and use YouTube rather than the cinema (line 19)?

[2]

trendyline

Explain the phrase to fight like stray dogs over a dustbin (line 31).

For
Examiners
Use

[2]
6

Explain the income-happiness paradox in lines 39-40. Use your own words as far as
possible.

[2]
7

What does the word assailed in line 73 tell us about how loneliness affects the rich?

[1]
8

(a) What does this in line 76 refer to?

[1]
(b) Why does the author repeat the phrase for this in lines 76 and 78?

[1]
9

What is the authors opinion about schemes like Men in Sheds and Walking Football in
line 80?

[1]
10

Why is Hobbes mentioned again in the first line of paragraph 12?

[1]

trendyline

11

Using material from paragraphs 7 to 9, summarise what the authors have to say about
the harmful effects of loneliness on people.

For
Examiners
Use

Write your summary in no more than 120 words, not counting the opening words which
are printed below. Use your own words as far as possible.
Loneliness leads to

[8]

trendyline

12 George Monbiot and Philippa Perry argue that the Age of Loneliness is killing us.
How far would you agree with their observations? Relate your arguments to your own
society.

trendyline

For
Examiners
Use

For
Examiners
Use

[10]

END OF PAPER

trendyline

Insert
George Monbiot and Philippa Perry argue that the Age of Loneliness is killing us.
1

What do we call this age? It is not the information age: the collapse of popular education
movements left a void filled by marketing and conspiracy theories. Like the stone age, iron age
and space age, the digital age says plenty about our artefacts but little about society. The
anthropocene, in which humans exert a major impact on the biosphere, fails to distinguish this
century from the previous twenty. What clear social change marks out our time from those that
precede it? To me, it is obvious. This is the Age of Loneliness.

When Thomas Hobbes claimed that in the state of nature, before authority arose to keep us in
check, we were engaged in a war of every man against every man, he could not have been
more wrong. We were social creatures from the start, mammalian bees, who depended entirely
on each other. The hominins of east Africa could not have survived one night alone. We are 10
shaped, to a greater extent than almost any other species, by contact with others. The age we
are entering, in which we exist apart, is unlike any that has gone before.

Recently, we read that loneliness has become an epidemic among young adults. Now we learn
that it is just as great an affliction of older people. A study by Independent Age shows that severe
loneliness in England blights the lives of 700,000 men and 1.1 million women over 50, and is 15
rising with astonishing speed. Social isolation is as potent a cause of early death as smoking 15
cigarettes a day; loneliness, research suggests, is twice as deadly as obesity. Psychological and
physical ailments become more prevalent when connections are cut. We cannot cope alone.

Today, people travel by car instead of buses and use YouTube rather than the cinema. However,
these shifts alone fail to explain the speed of our social collapse. These structural changes have 20
been accompanied by a life-denying ideology, which enforces and celebrates our social isolation.
The war of every man against every man competition and individualism, in other words is the
religion of our time, justified by a mythology of lone rangers, sole traders, self-starters, self-made
men and women, going it alone. For the most social of creatures, who cannot prosper without
love, there is no such thing as society, only heroic individualism. What counts is to win. The rest 25
is collateral damage.

British children no longer aspire to be train drivers or nurses more than a fifth say they just
want to be rich: wealth and fame are the sole ambitions of 40% of those surveyed. A government
study in June revealed that Britain is the loneliness capital of Europe. We are less likely than
other Europeans to have close friends or to know our neighbours. Who can be surprised, when 30
everywhere we are urged to fight like stray dogs over a dustbin? We have even changed our
language to reflect this. Our most cutting insult is loser". We no longer talk about people. Now
we call them individuals. So pervasive has this alienating, atomising term become that even the
charities fighting loneliness use it to describe the bipedal entities formerly known as human
beings.
35

One of the tragic outcomes of loneliness is that people turn to their televisions for consolation:
two-fifths of older people report that the one-eyed god is their principal company. This selfmedication aggravates the disease. Research by economists at the University of Milan suggests
that television helps to drive competitive aspiration. It strongly reinforces the income-happiness
paradox: the fact that, as national incomes rise, happiness does not rise with them. Aspiration, 40
which increases with income, ensures that the point of arrival, of sustained satisfaction, retreats
before us. You have only to think of the generalised obsession with fame and wealth, the
pervasive sense, in watching it, that life is somewhere other than where you are, to see why this
might be.

trendyline

Lonely people are nearly twice as likely to die prematurely as those who do not suffer feelings of 45
isolation. Being lonely it seems, is a lot more worrying for your health than obesity. Its dramatic
consequences on health is more pressing for the elderly. In a US report, the effect of satisfying
relationships on the elderly was measured and the report concluded that the lonely elderly was
adversely affected in developing their resilience and their ability to bounce back after adversity.

Not surprisingly, there is no corresponding good news for those less well connected to other 50
people. Feeling isolated from others can disrupt sleep, raise blood pressure, lower immunity,
increase depression, lower overall subjective well-being and increase the stress hormone
cortisol. The Lonely Society, in its 2010 report commissioned by The Mental Health Foundation,
cited a link between our "individualistic society" and the increase in common mental health
disorders in the last 50 years. It also drew on research showing that mental health problems 55
occur more frequently in unequal societies where lonely people are often left behind. By
squandering "social capital" in the individualistic pursuit of greater wealth, or treating social
networks as incidental, are we neglecting a part of life that makes us happy and keeps us healthy
for longer?

Loneliness is often the core feeling that gives rise to mood swings. Lonely people frequently feel 60
that they are disliked, are often self-obsessed and lack empathy with others. They fear rejection
and keep themselves at a distance, which feeds the loneliness. People who are lonely often think
that everyone else is doing OK while they are not. They think they are the only ones carrying a
burden. Clients talk about putting their "game face" on rather than sharing truthfully about
themselves and it can be difficult to know when it is appropriate to make the move from the 65
former to the latter.

10

So what is the point? What do we gain from this war of all against all? Competition drives growth,
but growth no longer makes us wealthier. It is noted that while the income of company directors
has risen by more than a fifth, wages for the workforce as a whole have fallen in real terms over
the past year. Even if competition did make us richer, it would make us no happier, as the 70
satisfaction derived from a rise in income would be undermined by the aspirational impacts of
competition. Even the rich are not happy. A survey by Boston College of people with an average
net worth of $78m found that they too were assailed by anxiety, dissatisfaction and loneliness.
Many of them reported feeling financially insecure: to reach safe ground, they believed, they
75
would need, on average, about 25% more money.

11

For this, we have ripped the natural world apart, degraded our conditions of life, surrendered our
freedoms and prospects of contentment to a compulsive, atomising, joyless hedonism, in which,
having consumed all else, we start to prey upon ourselves. For this, we have destroyed the
essence of humanity: our connectedness. Yes, there are palliatives, clever and delightful
schemes like Men in Sheds and Walking Football developed by charities for isolated older 80
people. But if we are to break this cycle and come together once more, we must confront the
world-eating, flesh-eating system into which we have been forced.

12

Hobbess pre-social condition was a myth. But we are entering a post-social condition our
ancestors would have believed impossible. Our lives are becoming nasty, brutish and long.

trendyline

Pioneer Junior College


JC2 Preliminary Examination 2015
Paper 2 Answer Scheme
1. In lines 2-3, what similarity do the authors see between the Age of Loneliness and the ages
that precede it? Use your own words as far as possible. [2]
Lifted from passage
Paraphrased
Like the stone age, iron age and space age, a lot about things/ articles/objects that belong
the digital age says plenty about our to a particular time in the past
artefacts (line 2-3)
but little about society. (line 3)
but not much about people /ourselves
2. What is the claim made by Thomas Hobbes in lines 7-8 and why do the authors disagree with
it? Use your own words as far as possible. [3]
Lifted from passage
claimed that in the state of nature, before
authority arose to keep us in check, we were
engaged in a war of every man against
every man (line 7-8)
he could not have been more wrong. We
were social creatures from the start,
mammalian bees, who depended entirely
on each other. (lines 8-10)
We are shaped, to a greater extent than
almost any other species, by contact with
others. (line 10-11)

What:

Paraphrased / Inferred

Each man for himself is an innate tendency


Why:
But, we are people who relied on others/ need
other people/ communal beings
Moulded / Influenced by our interactions with
others

3. In paragraph 3, what do the words just as tell you about the effect of the epidemic on older
people? [1]
Lifted from passage
Paraphrased
Loneliness just as great an affliction of Loneliness is as much a source of misery/
older people (line 14)
hardship/ suffering for the elderly/ same extent

4. What is implied by the shifts (line 21) described in the sentence Today people travel by car
instead of buses and use YouTube rather than the cinema in line 20? [2]
Lifted from passage
Paraphrased / Inferred
People travel by car instead of buses
a. Choices that people make in their daily lives /
use YouTube rather than the cinema (line Our daily preferences/ change in peoples
behaviour
20)
b. have shifted from a group setting to an
individual setting / become more individualistic/
increasing social isolation

Get PSJ Private Tutor to Guide you through Exams Now!


Contact www.privatetutor.com.sg

trendyline

5. Explain the phrase to fight like stray dogs over a dustbin (line 32). [2]
Lifted from passage
Paraphrased / Inferred
to fight like stray dogs over a dustbin (line a. In the struggle for survival,
32)
b. we regard others as competitors.
OR
Just as stray dogs compete for food in their
fight for survival, so do people regard others as
competitors.

6. Explain the income-happiness paradox in lines 40-41. Use your own words as far as
possible. (2m)
Lifted from passage
as national incomes rise, happiness does not
rise with them. Aspiration, which increases
with income, ensures that the point of arrival,
of sustained satisfaction, retreats before us.
(line 40-41)

Paraphrased / Inferred
a: It seems contradictory that happiness does
not increase with salary/wages. (focus is on the
contradiction)
b: Yet, the truth is that as our salary increases,
the increased ambitions that come along with it
cannot be fulfilled leading to happiness
becoming elusive. (focus is on the reasoning)

7. What does the word assailed in line 74 tell us about how loneliness affects the rich? [1]
Inferred from passage
strongly affected / troubled greatly/ more troubled/ overcome/ affect deeply
Note: idea of strongly is necessary for awarding of mark.
As long as the degree is shown, award the mark.

8. a) What does this in line 77 refer to? [1]


Inferred from passage
competition to acquire / accumulate/ gain wealth
8. (b) Why does the author repeat the phrase for this in lines 77 and 79. [1]
Lifted from passage
He wants to emphasise/ highlight/ show our foolishness in being willing to sacrifice what makes
us human, for the trivial.

trendyline

9. What are the authors opinion about schemes like Men in Sheds and Walking Football in line
82? [1m]
Lifted from passage
Yes, there are palliatives, clever
delightful

Paraphrased / Inferred
and They do not think that they are the solution/
Do not tackle the root cause/
Appear to be impressive or effective but are not
Or any other sensible answers.

10. Why is Hobbes mentioned again in the first line of paragraph 12? [1]
From the passage
(Para 2) When Thomas Hobbes claimed that in
the state of nature, before authority arose to keep
us in check, we were engaged in a war of every
man against every man, he could not have been
more wrong.
(Last
Paragraph)Hobbess
pre-social
condition was a myth. But we are entering a postsocial condition our ancestors would have
believed impossible. Our lives are becoming
nasty, brutish and long.

Paraphrased / Inferred
It is to give a sense of continuity (begins and
ends with a reference to Hobbes)
Or
While the authors disagreed with Hobbes claim,
Hobbes is mentioned again to show that the
claim may become true/ To show that the claim
was untrue in the past but seems to be valid
now. (possibility of claim becoming true must be
shown or that it was untrue in the past.)
(Note: Do not accept answers which only states
that the authors want to debunk Hobbes claim/
the authors want to reinforce their stand that
they disagree with Hobbes.)

11. Summary
Using material from paragraphs 7 to 9, summarise what the authors have to say about the
harmful effects of loneliness on people.
Write your summary in no more than 120 words, not counting the opening words which are
printed below. Use your own words as far as possible. (8m)
Loneliness leads to
#
1
2

3
4

Lift
nearly twice as likely to die
prematurely as those who do not
suffer feelings of isolation.
a lot more worrying for your health
than obesity.(FOCUS IS ON
COMPARISON TO OTHER
MEDICAL CONDITIONS).
more pressing for the elderly
the lonely elderly was adversely
affected in developing their
resilience
OR
and their ability to bounce back
after adversity

Lines Paraphrase
46
Cause of early death/ more prone to early
death.
47

Medically, is more troubling / a greater


cause for concern than other illnesses/
ailments

48

More serious for the elderly

50

As they were less able


hardship / difficulty

to recover from

50

trendyline

Lift

can disrupt sleep, raise blood


pressure, lower immunity, increase
depression, lower overall subjective
well-being and increase the stress
hormone cortisol. (VS PT 2, FOCUS
IS ON HEALTH EFFECTS).

53

Leads to physical
complications

increase in common mental


health disorders in the last 50
years.

56

We see more
ailments/illness

unequal societies

58

Unfair / non-inclusive societies

where lonely people are often left


behind.

58

as lonely people are not given the chance to


go forward / marginalised/ sidelined

neglecting a part of life that makes


us happy and keeps us healthy for
longer?

60

10

gives rise to mood swings

62

11

feel that they are disliked

63

12

self-obsessed

63

13

lack empathy with others.

14

fear rejection

15

keep themselves at a distance

16

feeds the loneliness.

17

often think that everyone else is


doing OK while they are not.

18
19
20

Lines Paraphrase

cases

health

of

issues

psychological

Note: isolation [0]


Disregarding/ ignoring / forsaking the very
things that ensure our well-being.
Note: health and happiness come together
Leads to volatile/ unstable/ unpredictable
emotions / temperamental
Think people detest them

Place too much focus on themselves /


narcissistic
63
Do not show understanding/ compassion /
apathy towards others
63-64 Are afraid / scared /frightened about not
being accepted by others
64
Isolate themselves / cut themselves off from
the community / alienate them
64
Reinforces / adds to/ compounds the
loneliness
65
Think they are the only ones suffering /
facing difficulties

OR

65

they are the only ones carrying a


burden.
putting their "game face" on

66

Put on a brave front / pretending to be brave

66

Instead of being open/ honest with others

67

Unsure of when to confide in others

rather than sharing truthfully about


themselves
difficult to know when it is
appropriate to make the move from
the former to the latter.
No. of Points
1
2- 3
4- 5
6-7

Marks Awarded
1
2
3
4

trendyline

8
9- 10
11-12
>13

5
6
7
8

12. Application Qn
George Monbiot and Philippa Perry argue that the age of loneliness is killing us.
How applicable are their observations to yourself and your own society? (10m)
#

Point

Lonely people are nearly


twice as likely to die
prematurely as those who do
not suffer feelings of isolation.
"Being lonely it seems, is a lot
more worrying for your health
than obesity."
Feeling isolated from others
can disrupt sleep, raise blood
pressure, lower immunity,
increase depression, lower
subjective well-being and
increase the stress hormone
cortisol.

Agree/ Disagree
Ex
Eg
Agree
Ex
Loneliness is becoming a major cause for mental
health concerns in highly urban or developed
societies, potentially leading to disorders or even
premature deaths.
E.g.
A 2009 - 2011 NUS study commissioned by the
Ministry of Social and Family Development found that
loneliness shortens the lifespan of the elderly, with
data showing that those who professed to be lonely
were found to have died within 2 years of the initial
interview. Also, according to Singhealth, social
isolation leads to lowered immunity and increased
possibility of medical conditions such as cancer and
heart disease.

"(there is)...a link between our


"individualistic society" and the
increase in common mental
health disorders...

Agree

(Mental health
disorders)occur more
frequently in unequal
societies where lonely
people are often left behind."

E.g.
As one of the most competitive nation in Asia,
Singapore makes for a highly individualistic, selfcentred society which prizes getting ahead of others
over empathy and camaraderie today. This is evident
in our 'kiasu' culture, which sees the young and old
inadvertently forgo the creation of genuine or deep
emotional bonds with family, peers and community in
their pursuit of success in academics, career and life.

Ex
Feelings of loneliness are elicited from social isolation
common in competitive and individualistic societies,
leading to mental illnesses such as depression and
even suicide.

trendyline

For example, the Samaritans of Singapore reported in


July 2013 that there was an 80% increase in suicide
rates in the age group 20 29 years old from 2012
and this is attributed to struggles with stressful
studies or work, social interactions and feelings of
loneliness.
Disagree
Ex
Altruism is still present in individualistic societies with
the government and community organizations
pitching in to create opportunities for interaction,
reducing the likelihood of loneliness and mental
illnesses.
E.g.
Estates with a high percentage of elderly residents
often see Senior Activity Centres set up at the foot of
their HDB blocks, such as that of the Lions
Befrienders Senior Activity Centre in Toa Payoh and
Bendeemer. The elderly residents can chat with
friends, take part in mass exercise sessions and
handicraft classes or help prepare meals for each
other once in a while.

By squandering 'social capital'


in the individualistic pursuit of
greater wealth or treating
social networks as incidental,
are we neglecting a part of
life that makes us happy and
keeps us healthy for
longer?"

Other Eg: The Silver Ribbon Project raises


awareness of the stigma of mental illnesses
Diasgree
Ex
Recent times have shown that there is an increased
inclination towards building a greater work-life
balance amongst Singaporeans, especially amongst
adults with young children and/or elderly parents. We
are increasingly aware that we should not neglect
what makes us happy and healthy.
E.g.
Public campaigns such as Eat with Your Family Day,
which fell on 29 May this year saw a day set aside for
all workers as well as students to get off work or
school early and spend quality evening time with their
families.

life-denying ideology, which


enforces and celebrates our
social isolation. The war of
every man against every man
competition and
individualism

Disagree
Ex: Competition and individualism do not necessarily
enforce and celebrate social isolation. The authors
simply assume that all forms of competition are
unhealthy when in fact competitions can come in
various forms and manners with desirable
consequences.
Competition
can
promote
collaborative effort where people work with one
another. Competing against other groups also
engenders bonding rather than isolation.

trendyline

E.g. Unilever Future Leaders League (UFLL) is a


dynamic global competition that brings together
students from all over the world to experience what
tomorrow's leadership is all about through
collaboration.
5

For this (accumulation of


wealth), we have ripped the
natural world apart, degraded
our conditions of life,
surrendered our freedoms and
prospects of contentment to a
compulsive, atomising, joyless
hedonism, in which, having
consumed all else, we start to
prey upon ourselves. For this,
we have destroyed the
essence of humanity: our
connectedness.

Disagree

Yes, there are


palliatives, clever and
delightful schemes like Men in
Sheds and Walking Football
developed by charities for
isolated older people.

The author is mocking the schemes and indicating


their ineffectiveness. However, this may not be true in
Singapore.

Ex: The efforts made towards the accumulation of


wealth do not necessarily destroy our connectedness.
In fact, we can use this to help people.
E.g.
social
entrepreneurships
(KerbsideGourmet, 18Chefs)

in

Singapore

KerbsideGourmet is modelled on a BAMGAM (Buy A


Meal Give A Meal) basis. For every main meal sold,
one is given to needy families. The organisation
works with Volunteer Welfare Organisations like
Prison Fellowship Singapore (PFS) and South
Central Community Family Service Centre and also
collaborates with Grand Hyatt Singapore to deliver
food to beneficiary families. Thus far, it has received
great support.

Ex: In fact, the Government may have pre-empted


that isolation is divisive and have thus implemented
sustainable policies or activities that help make our
society more inclusive.
E.g. Care Corner Senior Activity Centre.
Schools with tie-up programmes with eldercare
associations that are on a long-term basis.

TV aggravates the disease of


loneliness by
People turn to their TVs for
consolation (especially old
people)

Agree
Ex: People turn to television to curb their loneliness.
This is especially so in Singapore for the Pioneer
Generation living on their own. For this particular
group, the television is arguably more ubiquitous than
the internet. Furthermore, this is user-friendly to the
tech-illiterate.
E.g. The TV is also able to entice since MediaCorp is
rather established in its programmes for the Chinese
speaking community.

trendyline

Raffles Institution
2015 Year 6 Preliminary Examination
General Certificate of Education Advanced Level
Higher 1

Candidates Name

CT Group

GP Tutor

GENERAL PAPER
Paper 2
Candidates answer on the Question Paper.

8807/02
31 August 2015
1 hour 30 minutes

READ THESE INSTRUCTIONS FIRST


Write your name, CT group and GP tutors name in the spaces at the top of this page.
Write in dark blue or black pen.
Do not use staples, paper clips, glue or correction fluid.
Answer all questions.
The Insert contains the passage for comprehension.
Note that up to 15 marks out of 50 will be awarded for your use of language.
The number of marks is given in brackets [ ] at the end of each question or part question.

For Examiners Use


Content

/35

Language

/15

TOTAL

/50

____________________________________________________________________________
This document consists of 7 printed pages, 1 blank page and 1 Insert.

trendyline

Read the passage in the Insert and then answer all the questions which follow. Note that up to
fifteen marks will be given for the quality and accuracy of your use of English throughout this
Paper.
NOTE: When a question asks for an answer IN YOUR OWN WORDS AS FAR AS POSSIBLE
and you select the appropriate material from the passage for your answer, you must still use
your own words to express it. Little credit can be given to answers which only copy words or
phrases from the passage.

1.

From paragraph 1, what is the authors purpose in sharing her personal experience?

For
Examiners
Use

.......[1]

2.

How do the two questions in line 3 demonstrate the authors excessive anxiety?

.......[1]

3.

And the shouldering of that one task can snowball into responsibility for the whole
assembly line of child-minding. (lines 6-8)

i.
What does snowball suggest about the nature of child-minding?

...

.......[2]

ii.
Why does the author describe child-minding as an assembly line?

.......[1]

trendyline

For
Examiners
Use

4.

From paragraph 3, what are the consequences in the workplace for the mother as a result
of her worry work (line 16) at home? Use your own words as far as possible.
..
.... .
..
..
....
.... [3]

5.

Why does the author find the situation of gender discrimination towards household
supervision surprising (line 19)? Use your own words as far as possible.
..
.... .
..
....[2]

6.

What does this insertion in parentheses (lines 22-24) reveal about womens new approach
towards housework?
..
...... [1]

trendyline

For
Examiners
Use

7.

From lines 32-33, show how mothers excessive concerns about their children reveal a
contradiction in their own situation. Use your own words as far as possible.
..
.... .
..
.....[2]

8.

And when I say we, you know who I mean. (lines 33-34)

i.
Identify the tone used by the author when she says And when I say we, you know
who I mean.

..
...... [1]

ii.
What is the authors intention in saying this?

..
...... [1]

9.

What do lines 38-41 suggest about how men and women see their contributions to
housework?

..

...... [1]

trendyline

10.

Suggest one reason why the author ends the passage with the sentence Just ask the
Finnish mothers. (line 85)

For
Examiners
Use

..
...... [1]
11.

Using material from paragraphs 8 to 12 only (lines 47-70), summarise what the author has
to say about why mothers get stuck with the micromanagement of child-minding and end
up worrying more than fathers about their children.
Write your summary in no more than 120 words, not counting the opening words which
are printed below. Use your own words as far as possible.
Mothers get stuck with micromanagement because ...
..
.... .
..
..
.... .
..
..
.... .
..
..
.... .
..
..
.....[8]
No of words: ___________

trendyline

12.

For
Examiners
Use

Judith Shulevitz argues that mothers play more of an active role in their childrens lives than
fathers. How far would you agree with the authors views? Relate your views to both you
and your society.
..
.... .
..
..
.... .
..
..
.... .

..
.... .
..
..
.... .
..
..
.... .

..
..
.... .
..
..
.... .

..
6

trendyline

..

.... .

For
Examiners
Use

..
..
.... .
..
..
.... .

..
..
.... .
..
..
.... .

..
.... .
..
..
.... .
..
..
.... .

..
..
[10]

trendyline

BLANK PAGE

trendyline

Raffles Institution
2015 Year 6 Preliminary Examination
General Certificate of Education Advanced Level
Higher 1

GENERAL PAPER

8807/02

Paper 2

31 August 2015

INSERT

1 hour 30 minutes

READ THESE INSTRUCTIONS FIRST


This Insert contains the passage for Paper 2.

This document consists of 3 printed pages and 1 blank page.

trendyline

Judith Shulevitz writes that child-minding duties are not equal between mothers and fathers.
1

Theres a story my daughter loves to hear me tell: the day after I came home from the hospital with her
big brother, my first child, I was seized by the certainty that I was about to die. I sobbed uncontrollably. I
asked my husband, But who will keep him in socks? Wholl make sure hes wearing his little socks?

New parenthood, of course, does things to your brain. But I was on to something, even in my deranged,
postpartum way. I should state for the record that my husband is perfectly handy with socks. Still, the
parent more obsessed with the childrens hosiery is the one wholl make sure its in stock. And the
shouldering of that one task can snowball into responsibility for the whole assembly line of childminding. She who buys the bootees will surely buy the bottle washer, just as shell probably find the
babysitter and pencil in the class trips. I dont mean to say that shell be the one to do everything, just
that shell make sure that almost everything gets done. Sociologists sometimes call the management of
familial duties worry work, and the person who does it the designated worrier, because you need
large reserves of emotional energy to stay on top of it all.
I wish I could say that fathers and mothers worry in equal measure. But they dont. Disregard what your
two-career couple friends say about going 50-50. Sociological studies of couples from all strata of
society confirm that, by and large, mothers draft the to-do lists while fathers pick and choose among the
items. And whether a woman loves or hates worry work, it can scatter her focus on what she does for
pay or clean off a career path. This grind of apprehension and organisation may be one of the least
movable obstacles to womens equality in the workplace.

Its surprising that household supervision resists gender reassignment to the degree that it does. In the
United States today, more than half of all women work, and women are 40 percent of the sole or
primary breadwinners in households with children under 18. The apportionment of the acts required to
keep home and family together has also been evening out during the past 40 years (though, for
housework, this is more because women have sloughed it off rather than because men have taken it
on). Nonetheless, one of the last things to go is women keeping track of the kind of non-routine details
of taking care of children: when they have to go to the doctor, when they need a permission slip for
school paying attention at that level, says the social psychologist Francine Deutsch, author of
Halving It All: How Equally Shared Parenting Works.

The amount of attention that must be paid to such details has also ballooned in the past few decades.
This is because of our commitment to what the sociologist Annette Lareau calls concerted cultivation.
We enrol children in dance classes, soccer, tutoring often three or four extracurricular activities a
week. These demand reluctant effort, obviously, but also have less visible time costs: searching the
web for the best programme, ordering equipment, packing snacks and so on. We fret that were
overscheduling the children, but dont seem to realise that were also overscheduling ourselves. And
when I say we, you know who I mean. A 2008 study by Dr. Lareau and the sociologist Elliot B.
Weininger found that while fathers often, say, coach games, its mothers who perform the behind-thescenes labour that makes kids sports and other pursuits possible.

Of course, sweeping generalisations about who does what always have a near-infinite number of
exceptions. There are many more men in charge of child care than there were 20-odd years ago. How
many more depends on whether you ask men or women: half of the men surveyed in a Families and
Work Institute study from 2008 said they were either the responsible parent or shared the role equally
with their spouse, while two-thirds of the women said they were the one in charge.

And then there are the stay-at-home dads: two million of them in 2012, up from 1.1 million in 1989,
although only around a fifth of those fathers stay home for the children. The other four-fifths are
unemployed, ill, in school or retired. Some of these fathers serve as primary caregivers. On average,
however, men who are out of work eke out slightly under three hours a day of housework and child care
combined less than working women do (3.4 hours a day).

trendyline

10

15

20

25

30

35

40

45

One reason women like me get stuck with the micromanagement is that we dont see it coming, not at
first. Pamela Smock, a sociologist at the University of Michigan, tells a story about the students in her
Women and Work class. Mostly women, they spend a semester reading about the gendered division
of domestic labour. And yet in their presentations, even they slip up and talk about men helping out.
As long as the phrase he helped is used, says Dr. Smock, we know we have not attained gender
equality.
No matter how generous, helping out isnt sharing. I feel pinpricks of rage every time my husband
fishes for praise for something Ive asked him to do. On the other hand, my friends and I have never
gotten around to drawing up the List of Lists and insisting that we split it. Even though women tell
researchers that having to answer for the completion of domestic tasks stresses them out more than
any other aspect of family life, I suspect theyre not always willing to cede control.

Ive definitely been guilty of maternal gatekeeping rolling my eyes or making sardonic asides when
my husband has been in charge but hasnt pushed hard enough to get teeth brushed or bar mitzvah
practice done. This drives my husband insane, because hes a really good father and he knows that I
know it. But women cant help themselves. They have standards, helicopter-ish though they may be.

10

11

12

50

55

60

Allow me to advance one more, perhaps controversial, theory about why women are on the hook for
what you might call the human-resources side of child care: women simply worry more about their
children. This is largely a social fact. Mothers live in a world of other mothers, not to mention teachers
and principals, who judge us by our children. Or maybe we just think theyre judging us.

65

But there is also a biological explanation: we have been conditioned to worry. Evidence from other
animals as well as humans makes the case that the female of the species is programmed to do more
than the male to help their offspring thrive. Neurological and endocrinological changes, the production
of hormones such as oxytocin and estrogen during pregnancy and after birth, exert a profound
influence over mothers moods and regulate the depth of their attachment to their children.

70

13

So we worry. When we worry, we coordinate. When we coordinate, we multitask. We text about a play
date while tending to a spreadsheet. And we underestimate how many minutes we rack up on stuff
were not being paid to do. Smartphones are particularly dangerous in this regard, because they make
multitasking seem like no work at all.

14

But what is to be done? When Martin Luther King, Jr. had the Dream in inspiring the civil rights
movement, we enacted laws that broke the attitudes on segregation, painful as they were in the
beginning. We created avenues that allowed blacks and whites to come together in meaningful ways,
awkward as it was for both groups. Most of all, King created a vision never seen before. I want that
for mothers!

75

15

All this may change as men as well as women chafe against the lengthening and increasingly
unpredictable workday foisted upon us by globalisation and the Internet, among other forces. It should
be said, however, that planning for equality is not the same as achieving it. The realities of child rearing
the shortage of time and sleep, the fraying of tempers, the pressure on women to be the right kind of
mother and on men not to let family affect career tend to define equality down. Its about time things
get easier when Mom and Dad feel happier looking after junior. Just ask the Finnish mothers.

80

Adapted from the New York Times Sunday Review

trendyline

85

BLANK PAGE

trendyline

Yr6GPPrelimAnswerScheme
1. From paragraph 1, what is the authors purpose in sharing her personal experience? [1]
From the passage
Theres a story my daughter loves to hear me tell:
The day after I came home from the hospital with
her big brother, my first child, I was seized by the
certainty that I was about to die. I sobbed
uncontrollably; I asked my husband: But who will
keep him in socks? Wholl make sure hes
wearing his little socks?

Inferred
Note: Must have both points a and b
a

Mark
1

Function/Intention
To introduce the issue
To engage/get the reader to think
about the issue of
To connect with the reader/establish a
rapport with the reader

Context/Explanation
of mothers
minding

anxiety

about

child-

2. How do the two questions in line 3 demonstrate the authors excessive anxiety? [1]
From the passage
But who will keep him in socks? Wholl make sure
hes wearing his little socks?

Inferred
Focus: content of her questions
They reveal her
something trivial.

anxiety

by

Mark
1
asking

3. And the shouldering of that one task can snowball into responsibility for the whole assembly line of
child-minding. (lines 6-8)
i.

What does snowball suggest about the nature of child-minding? [2]

From the passage


[Cause]
And the shouldering of that one task
[Effect]
can snowball into responsibility for the whole
assembly line of child-minding.

ii.

a
b

Paraphrase + Inferred
It suggests that bearing responsibility
for one task

Mark
1

can lead to an avalanche of other


duties connected to that initial task

Why does the author describe child-minding as an assembly line? [1]

From the passage


And the shouldering of that one task can cascade
into responsibility for the whole assembly line of
child-minding.

Inferred
Both
are
routine/repetitive/mundane.

equally

Mark
1

OR
Both involve different/many things
being put together.

Get PSJ Private Tutor to Guide you through Exams Now!


Contact www.privatetutor.com.sg

trendyline

Yr6GPPrelimAnswerScheme

4. From paragraph 3, what are the consequences in the workplace for the mother as a result of her
worry work (line 16) at home? Use your own words as far as possible. [3]
From the passage
scatter her focus on what she does for pay or
[distracting grind of apprehension and
organisation]
clean off a career path

a
b

This grind of apprehension and organisation


may be one of the least movable obstacles to
womens equality in the workplace.

Re-exp
Her ability to pay full attention at work is
compromised/She cannot pay attention
to her job
and
this
also
destroys
her
professional/job prospects. OR This
limits her access to certain job
opportunities.

Mark
1

The tedium of worrying makes it


[worrying] the least changeable/most
difficult barrier to overcome gender
bias.

5. Why does the author find the situation of gender discrimination towards household supervision
surprising (line 19)? Use your own words as far as possible. [2]
From the passage
In the United States today, more than half of
all women work, and women are 40 percent
of the sole or primary breadwinners in
households with children under 18.

Re-exp
Expectation (Work)
a

It is surprising because although a large


proportion/majority of women are now
supporting
their
families
financially/holding jobs

Mark
1

Note: either one of the above


Its surprising that household supervision
resists gender reassignment to the degree
that it does.

Reality (Domestic)

[inf] they still hold the main responsibility


of household /child-minding chores.

6. What does this insertion in parentheses (lines 22-24) reveal about womens new approach towards
housework? [1]
From the passage
The apportionment of the acts required to keep
home and family together has also been
evening out during the past 40 years (though,
for housework, this is more because women
have sloughed it off rather than because men
have taken it on).

Inferred
It shows that women have also learnt to be
less
attentive/less
obsessive
about
housework.

Mark
1

trendyline

Yr6GPPrelimAnswerScheme

7. From lines 32-33, show how mothers concerns about their children reveal a contradiction in their own
situation. Use your own words as much as possible. [2]
From the passage
We fret that were overscheduling the
children,
but dont seem to realise that were also
overscheduling ourselves.

Inferred
If
mothers
worry
about
excessively/over-planning their
lives,

Mark
1

planning
childrens

they should also see how they shouldnt do


the same to their own lives but they dont.

Note: 2m or nothing
8.

And when I say we, you know who I mean. (lines 33-34)
i.

Identify the tone used by the author when she says And when I say we, you know who I mean.
[1]

Ans: The authors tone is sarcastic/ sardonic /wry/ironic/tongue-in-cheek.


ii.

What is the authors intention in saying this? [1]

From the passage


And when I say we, you know who I mean.

Inferred
The author intends/wants to reinforce/show
the inequality of gender roles in family duties.

Mark
1

9. What do lines 38-41 suggest about how men and women see their contributions to housework? [1]
From the passage
There are many more men in charge of child
care than there were 20-odd years ago. How
many more depends on whether you ask men
or women: half of the men surveyed in a
Families and Work Institute study from 2008
said they were either the responsible parent or
shared the role equally with their spouse, while
two-thirds of the women said they were the one
in charge.

Inferred
This suggests that men and women both
overestimate their contribution.

Mark
1

10. Suggest one reason why the author ends the passage with the sentence Just ask the Finnish
mothers. (line 85) [1]
From the passage
Its about time things get easier when Mom and
Dad feel happier looking after junior. Just ask
the Finnish mothers.

Inferred
To show how they are enjoying
fruits/benefits of equal participation.

the

Mark
1

Note:
Accept any reasonable answer that includes a
reasonable rationale

trendyline

Yr6GPPrelimAnswerScheme

11. Using materials from paragraphs 8 to 12 only (lines 47-70), summarise what the author has to say
about why mothers get stuck with the micromanagement of child-minding and end up worrying more
than fathers about their children.
Write your summary in no more than 120 words, not counting the opening words which are printed
below. Use your own words as far as possible. [8]
Mothers get stuck with micromanagement because

From the passage (lines)

Paraphrase

we don't see it coming (line 47)

unaware that it will happen

not at first (lines 47-48)

initially / at the start/beginning

(Mostly womenspend a semester reading


about the gendered division of domestic
labour) even they (lines 49-50)

[Inf] and even women who are aware of this


inequality

slip up and talk about men helping out. (line


50)

make the mistake of seeing husbands as


providing assistance only when asked/needed

my friends and I have never gotten around to


drawing up the List of Lists (lines 54-55)

Mothers do not spell out responsibilities/tasks

insisting that we split it (line 55)

and demand that these be divided.

having to answer for the completion of


domestic tasks stresses them out more than
any other aspect of family life, (lines 56-57)

Despite the enormous pressure


accountable for domestic tasks,

I suspect theyre not always willing to cede


control (line 57)

mothers fear relinquishing child-minding authority

guilty of maternal gatekeeping (line 58)

[Inf] and are critical of paternal involvement

But women cant help themselves (line 61)

However, mothers can do little to change their


anxiety over their children.

They have their standards (line 61)

They have exacting /demanding requirements


that they feel only they can meet

helicopter-ish though they may be (line 61)

even if these might be excessive/obsessive.

of

being

women simply worry more about their


children. (lines 63-64)

Mothers just/basically/fundamentally agonise over


their children more than fathers.

This is largely a social fact (line 64)

This is primarily a social issue/socially true

Mothers live in a world of other mothers, not to


mention teachers and principals (lines 64-65)

as mothers feel that other mothers and educators/


women/people

trendyline

Yr6GPPrelimAnswerScheme

who judge us by our children (line 65)

assess them based on the behaviour and actions


of their children

Or maybe we just think theyre judging us. (line


65)

or are paranoid that they do.

biological explanation: we have been


conditioned to worry (line 66)
the female of the species is programmed to
do more than the male (lines 67-68)

Women have also biologically evolved to agonise


over their children

to help their offspring thrive (line 68)

and assist in their success / flourishing.

Neurological and endocrinological changes, the


production of hormones such as oxytocin and
estrogen during pregnancy and after birth (lines
68-69)

Changes in the brains chemistry and the release


of hormones

exert a profound influence (lines 69-70)

have a deep influence/significantly influence

over mothers moods (line 70)

mothers temperaments fundamentally

and regulate the depth of (line 70)

and control the intensity of the feelings

their attachment to their children (line 70)

that bind them to their children

Total available: 24 points


Mark allocation
Points
1-2
3-4
5-6
7-8
9-10
11-12
13-14
15 and above

Mark
1
2
3
4
5
6
7
8

trendyline

Yr6GPPrelimAnswerScheme

Application Question
12. Judith Shulevitz argues that mothers play more of an active role in their childrens lives than fathers.
How far would you agree with the authors views? Relate your views to both you and your society.
[10]
Points to raise:

Womens role in the public sphere (workforce) vs their role in the private sphere (main care-givers)
Patriarchal society and how this affects parental involvement
o Patriarchy, rather than eradicated, has evolved into a more subtle and nuanced form of
oppression. We should not be fooled into thinking that patriarchy is at its twilight in SG just
because there are more women in the workforce, including more women working in male
professions.
o In advanced patriarchies, patriarchy does not simply mean that men rule. It is a value system
that is liable to recur because it serves to keep birthrates high among the affluent, while also
maximising parents investments in their children.
Gender norms and parenting in SG
o Do cultural definitions of manliness prevent men and fathers from optimising their role as
fathers?
o Or are we seeing changing cultural norms and the rise of new age fathers, especially
amongst the millennial set? Even if this is the case, has this resulted in shifting perceptions
towards parenthood?
o Increasing cognizance of fathers role in parenting;
Both mothers and fathers contribute uniquely to their childs development but there is
a growing realisation that fathers generally remain an under-tapped resource
Platforms such as the Centre for Fathering and Dads for Life Movements
Fathers@Schools Programme were conceived to mobilise this resource to ensure
the best outcomes for children.
Attitudes towards motherhood in SG?
Parentocracy and Tiger mums - competitive, anxious parenting of middle- and upper-class SG
mothers?
Is there adequate governmental and policy to support greater paternal involvement?
Role of domestic workers and grandparents in child-minding?

trendyline

RIVER VALLEY HIGH SCHOOL


YEAR 6 PRELIMINARY EXAMINATION II 2015
CANDIDATE
NAME
INDEX
NUMBER

CLASS

GENERAL PAPER

8807/02

Paper 2

August 2015
1 hour 30 minutes

Candidates answer on the Question Paper.


Additional Materials:
1 insert

READ THESE INSTRUCTIONS FIRST


Write your class, index number and name on all the work you hand in.
Write in dark blue or black pen on both sides of the paper.
Do not use staples, paper clips, highlighters, glue or correction fluid.
Answer all questions.
The Insert contains the passage for comprehension.
Note that up to 15 marks out of 50 will be awarded for your use of language.
At the end of the examination, fasten all your work securely together.
The number of marks is given in brackets [ ] at the end of each question or part question.

For Examiners Use


Content

/35

Language

/15

Total

/50

This document consists of 6 printed pages, 1 blank page and 1 insert.


River Valley High School
EL/GP Department

[Turn over

trendyline

2
Read the passage and then answer all the questions. Note that up to fifteen marks will
be given for the quality and accuracy of your use of English throughout this Paper.

For
Examiners
Use

NOTE: When a question asks for an answer IN YOUR OWN WORDS AS FAR AS
POSSIBLE and you select the appropriate material from the passage for your answer,
you must still use your own words to express it. Little credit can be given to answers
which only copy words and phrases from the passage.
1. Why does the author describe the person as kind, if slightly perverse (line 2)?

[2]
2. What does the author mean when he describes reality as binary (line 13)? Use your
own words as far as possible.

[2]
3. In paragraph 3, what explanations does the writer give to show that our inability to
accept the possibility of change can make us cruel (lines 2021)? Use your own
words as far as possible.

[3]

trendyline

4. Explain the irony about how we react to chaos in paragraph 5. Use your own words
as far as possible.

For
Examiners
Use

[2]
5. Explain the authors use of the word conversely in line 42.

For
Examiners
Use

[2]
6. Explain the metaphor of the prison (lines 3840).

[1]
7. What are the similarities between our exploitation by politicians in paragraph 7 and
our reliance on the mystical industries in paragraph 9? Use your own words as far
as possible.

[3]

trendyline

4
For
Examiners
Use

8. Using material from paragraphs 8 and 9 (lines 61-79), summarise what the author
has to say about the consequences of our unwillingness to change and the
characteristics of people who are open to change.
Write your summary in no more than 120 words, not counting the opening words
which are printed below. Use your own words as far as possible.
Our unwillingness to be open to change

[8]

9. Identify which of the authors arguments are supported by the illustration of the boxes
in the first and last paragraph.

[2]

trendyline

5
10. Al Kennedy writes about how we are reluctant to change and how being open to
change can make us happier and better people. How far do you think you and your
society should embrace change?

trendyline

For
Examiners
Use

For
Examiners
Use

[10]

trendyline

BLANK PAGE

trendyline

RIVER VALLEY HIGH SCHOOL


YEAR 6 PRELIMINARY EXAMINATION II 2015

GENERAL PAPER

8807/02
August 2015

Paper 2
INSERT

1 hour 30 minutes

READ THESE INSTRUCTIONS FIRST


This insert contains the passage for Paper 2.

This document consists of 4 printed pages.


River Valley High School
EL/GP Department

[Turn over]

trendyline

2
Al Kennedy writes about change.
1

Imagine three identical boxes. Two are empty and one contains your heart's desire,
perhaps love, perhaps a nice cup of tea. A kind, if slightly perverse, person says that
you can pick one box and own its contents. Let's say you select Box A. The person
then shows you Box B is empty. So either Box A your choice, or Box C a
mystery, contains your happiness. Now, you can change your choice to Box C, or
stick with Box A. But what gives you the better chance? Should you change or not?
If you are like me, you will not want to change. Even if things were not wonderful, we
would rather stay with what is familiar. Why meddle with something for which there
is a Latin, and therefore authoritative, term: the status quo? I studied dead
languages at school (no chance of sudden changes in grammar or vocabulary
there), so I am aware that the term status quo has roots in the longer phrase "in
statu quo res erant ante bellum", which means in the state in which things were
before the war. Of course, the implication of this phrase is that reality is binary:
without the status quo, there will be chaos and war. Who in his right mind would
choose to change the status quo?
People like me tend to ignore the possibility of positive change. We are inclined to
think that things will continue the way they are forever and ever, and that whatever
we do will not make much of a difference in the larger scheme of things. The poor
will always be with us, and human suffering in the form of poverty and hunger is
systemic and unchangeable, an aspect of the landscape we live in. In this way, our
inability to accept the possibility of change can make us cruel: the homeless man
wandering in the street is inevitable and a fact of life.
Even as we dig our heels in against positive change, we are not immune to
inevitable changes in our lives. Things change, whether or not we want it people
come and go, economies rise and fall, our luck waxes and wanes. But instead of
really embracing inevitable change, we change by making only superficial
alterations we buy the latest iThings and change our bags, shoes, hairstyles, and
the like seeking what is new and newer. Yet this behaviour is just as rigid as
being opposed to all types of change, as the changes we welcome are not really
changes.
The inability to face changes paralyses us, preventing us from responding
appropriately. We can interpret new situations and new people as unwelcome, if not
threatening, change, which is bad news for how we police crowds, carry out
overseas peace-keeping missions, or treat unfamiliar people. Yet our overreaction
to what feels like chaos can actually produce real chaos, and when we are
overwhelmed by change, we can get so frightened that we forget other humans are
human, and so we behave badly.
When change-avoidance forms a prison, walking out of that prison can seem
daunting. We cannot find it in ourselves to accept something new, even when we

trendyline

10

15

20

25

30

35

3
are dissatisfied with the current state. Being without love or novel interactions might
be awful, but we fear doing anything about it because we do not know what to
expect should it come our way. Conversely, when we are stuck in an unfavourable
situation, an unhappy marriage for example, we cannot summon the courage to rid
ourselves of the thorn, choosing instead to suffer many years in silence (or not). For
many of us, this saying holds true: better the devil you know than the devil you do
not.
7

Our reluctance to change is exploited by politicians, and they play to our fears and
offer us stasis. They build us shark cages for our time in the ever-changing water
consoling little pens which cannot protect us when something huge and horrible
arrives and we end up like Richard Dreyfuss in Jaws all at sea. They offer
eternal values eternity being unchanging and therefore reliable the
brotherhood of men, the wisdom of the free market, the evil of outsiders, and the
rewards of obedience. During an election, an incumbent government will warn
against "changing horses in midstream", and we may accept a dire status quo
because of how much worse we think the alternative might be. When we are told
that something threatens "our way of life", it can make us fearful and encourage us
to tolerate all manner of prohibitions, oppressions and compulsions. Populations
encouraged to fear change may only pursue it when their situation is already
savage, perhaps reassured by some of those eternal values. The more savage the
situation, the easier it is to offer equally savage solutions.
As individuals and nations, our unwillingness to be open to change can prevent us
from being merciful and finding good solutions. If we cannot have the humility to
imagine that we could become like the homeless man in the street (or to imagine
that he could become like us), we need not try to make the world safer as he is
beyond help. Even when we convince ourselves to help, we usually cling to familiar,
failed templates. Nations are as inflexible as possible in worryingly fluid situations.
How easy it is for governments and personnel-in-combat to avoid subtleties and
imaginative solutions, and choose to rely on old, failed strategies. If the international
community can offer something better something truly beyond self-interest that
would be a beautiful change.
Every analysis of what makes happy people happy demonstrates their ability to
adapt fast and well to new situations and people. They are able to roll with the
punches and deal with seismic changes in their lives and their surroundings. They
do not try to impose stillness on a universe which is in motion, and they know that
real security involves a degree of exposure. They are not so frightened as to indulge
in the mystical industries good luck charms and tarot readings which only give
a false sense of security and lock us in with their predictions. To be free and happy
is to realise that it is impossible to guarantee anything in this life. We must be able
to cope with risk.

10 So what is the best solution to that three-box problem? Remember we picked Box A
of the three. Box B was empty. Now we can stick with A, or change to C. But should

trendyline

40

45

50

55

60

65

70

75

80

4
we? Yes, we should. Switching from Box A to Box C will not guarantee success, but
will massively improve its odds even though it is counter-intuitive. Our intuition does
not like change either, but we can overcome it. Approaching changing reality with
sensible flexibility is the best strategy for happiness. I may not want to believe it, but
it is true. If I can change my mind, I can change anything else I need to.
Adapted from A Point of View: Why Embracing Change is the Key to Happiness,
BBC (September 2013)

trendyline

85

2015 RVHS Y6 Prelim II


Answer Scheme for Paper 2
1. Why does the author describe the person as kind, if slightly perverse (line 2)? [2]
From text
kind
if slightly
perverse

Paraphrase/ Inference
He is giving you the possibility of your hearts
desire/happiness/the contents of the box,
but he is making a game out of your hearts desire/ the odds
are stacked against you/ there is the possibility of choosing
wrongly and bringing it upon ourselves/ choosing wrongly

2. What does the author mean when he describes reality as binary (line 13)? Use your
own words as far as possible. [2]
From text
binary
Without the status quo,
there will be chaos and
war

Paraphrase/ Inference
He means that there are only two possible options:
things remain the same or they are in discord/
disarray/ conflict.

OR: He means that if things do not remain the same, there will necessarily be
discord/disarray/ conflict. (2 marks)
(Necessarily explains binary. Do not accept automatically.)
3. In paragraph 3, what explanations does the writer give to show that our inability to
accept the possibility of change can make us cruel? Use your own words as far as
possible. [3]
From text
Paraphrase/ Inference
People like me tend to [A1] ignore the
A1. We disregard the prospect of
possibility of positive change. We are
things becoming better, OR
inclined to [A2] think that things will
A2. think that things will remain
the same.
continue the way they are forever and
ever, and that [B] whatever we do will not
make much of a difference in the larger
B. We do not think that we can
scheme of things. The poor will always be
make a change to how things
with us, and [C] human suffering in the
are
C. and think that social problems
form of poverty and hunger is systemic
and unchangeable, an aspect of the
are ingrained/ inherent/
landscape we live in. In this way, our
perpetuated by institutions and
inability to accept the possibility of change
governments. (not present)
can make us cruel: [D] the homeless man
D. (implied) This makes us ignore/
wandering in the street is inevitable and a
be apathetic about real people
fact of life.
who are facing hardship.
1-2 points for 1 mark
3 points for 2 marks
4 points for 3 marks
Get PSJ Private Tutor to Guide you through Exams Now!
Contact www.privatetutor.com.sg

trendyline

4. Explain the irony about how we react to chaos in paragraph 5. Use your own
words as far as possible. [2]

From text
Paraphrase/ Inference
Yet our overreaction to
[expectation]
what feels like chaos can We think that when we respond to [A] what we
actually produce real
perceive as chaos, [B] we are actually solving it,
chaos,

[reality]
but what we are doing is to [C] create a chaos that
was not there in the first place.

A + C = 1 mark
B + C = 1 mark
A + B = 0 marks
A + B + C = 2 marks

5. Explain the authors use of the word conversely in line 42. [2]

Being without love or novel interactions might be awful, but we fear doing anything
about it because we do not know what to expect should it come our way.
Conversely, when we are stuck in an unfavourable situation, an unhappy marriage
for example, we cannot summon the courage to rid ourselves of the thorn, choosing
instead to suffer many years in silence (or not).

From text
Paraphrase/ Inference
Conversely
The author is trying to show
how the reverse/opposite
situation is also true. (1 mark)
Being without love or novel interactions might We not only are unwilling to
be awful, but we fear doing anything about it
pursue things we want and do
because we do not know what to expect should not have,
it come our way.



Conversely, when we are stuck in an
We are also unwilling to cut off
unfavourable situation, an unhappy marriage
things that we have but do not
for example, we cannot summon the courage to want.
rid ourselves of the thorn, choosing instead to
suffer many years in silence (or not).
Both points required for 1 mark.

6. Explain the metaphor of the prison (lines 3840). [1]

From text
Paraphrase/ Inference
Even when change-avoidance forms (literal)
a prison, walking out of the prison
Just as a prison traps us,
can seem daunting. We cannot find
it in ourselves to accept something (metaphorical)
new, even when we are dissatisfied being fearful of change keeps us from
with the current state.
progressing in life/ trying something new.

Both points for 1 mark.

trendyline

7. What are the similarities between our exploitation by politicians in paragraph 7 and
our reliance on the mystical industries in paragraph 9? Use your own words as far
as possible. [3]

From text
Paraphrase/ Inference
Our reluctance to change is exploited by politicians, Both politicians and the
and [A] they play to our fears and offer us stasis.
mystical industries
They build us [C] shark cages for our time in the
A. manipulate our
ever-changing water[B] consoling little pens
insecurities/ anxieties.
(not make us fearful)
which cannot protect us when something huge and
horrible arrives and we end up like Richard Dreyfuss B. They cannot deliver
in Jaws[D] all at sea.
what they promise/
cannot keep us safe.

They are [A] not so frightened as to indulge in the
C. They make us less
mystical industriesgood luck charms and tarot
free/ restrict our
readingswhich only give [B] a false sense of
actions,
security and [C] lock us in with their predictions. [D] D. making us miserable.
To be free and happy is to realise that it is


impossible to guarantee anything in this life. We
1 point for 1 mark
must be able to cope with risk.



8. Using material from paragraphs 8 and 9 (lines 61-79), summarise what the author
has to say about the consequences of our unwillingness to change and the
characteristics of people who are open to change.

Write your summary in no more than 120 words, not counting the opening words
which are printed below. Use your own words as far as possible. [8]

trendyline


Lifted
As individuals and nations, our unwillingness to be
open to change can prevent us from being [A]
merciful
and [B] finding good solutions.
If we cannot [C] have the humility to
[D] imagine that we could become like the homeless
man in the street (or to imagine that he could
become like us),

we need [E] not try to make the world safer
Even when we convince ourselves to help, [F] we
usually cling to familiar,
[G] failed templates.
Nations are as [H] inflexible as possible in
worryingly fluid situations.
How easy it is for governments and personnel-in-
combat to [I] avoid subtleties and
[J] imaginative solutions, and choose to rely on old,
failed strategies.
If the international community can offer something
better something truly [K] beyond self-interest
that would be a beautiful change.

Every analysis of what makes happy people [L]
happy
demonstrates their [M] ability to adapt fast and well
to new situations and people. They are able to roll
with the punches and
[N] deal with seismic changes in their lives and
their surroundings.
They [O] do not try to impose stillness on a
universe which is in motion, and they know that
real security involves a degree of exposure.
They are [P] not so frightened

as to [Q] indulge in the mystical industriesgood
luck charms and tarot readingswhich only give
[R] a false sense of security


[S]and lock us in with their predictions. OR To be
free
is to [T] realise that it is impossible to guarantee
anything in this life.
We must be able to [U] cope with risk.

[21 points]

Points
1-2
Marks
1

3-4
2

5-6
3

7-8
4

Suggested response
Our reluctance to be receptive to change stops us
from being compassionate (not gracious)
and coming up with effective answers
If we are prideful
and refuse to believe that our circumstances could
worsen/ or that his can become better
OR
are not empathetic towards the less privileged
we will not bother bettering/improving our
society/ helping others
In our attempts to help, we will stick to the
usual/what we are used to/ what we are
comfortable with/ what we know (not
comfortable proposals)
and this may be ineffective/unsuccessful /lacking
models.
Countries will remain rigid in a volatile world
They will rely on obvious/unrefined
and uncreative answers/measures
The international community will be selfish/self-
centric.
People who are open to change are joyful
They react/adjust quickly and effectively to the
foreign
and can cope with major changes
They do not resist changes
are brave
and do not rely on fortune
as these only lull them into believing they are safe
They are not entrapped/confined
They acknowledge that there are no promises in
life
Happy people must be able to handle /deal with
uncertainty/failure

9-10
5

11-13
6

14-16
7

17 +
8
trendyline

9. Identify which of the authors arguments are supported by the illustration of the
boxes in the first and last paragraph. [2]

From text
Paraphrase/ Inference
even though it is counter-intuitive
We do not like to change.
(line 82)

If you are like me, you will not want
to change. (line 7)
massively improve its odds (line 7) If we can bring ourselves to change, it can

bring about positive effects.
Approaching changing reality with
sensible flexibility is the best

strategy for happiness. (lines 83-84)
1 mark for each point


- Answer cannot simply explain the
illustration of the boxes.
- Argument must be stated clearly, not just
He is arguing whether or not changing can
bring positive effects. (0 marks)



10. Application Question: [10]

Al Kennedy writes about how we are reluctant to change and how being open to change
can make us happier and better people. How far do you think you and your society
should embrace change?

trendyline

ST ANDREWS JUNIOR COLLEGE


JC2 PRELIMINARY EXAMINATION
Name: ____________________________ (

Civics Group: ______________________

GENERAL PAPER

8807/02

Paper 2

Thursday, 27 August 2015


1 hour 30 minutes

ANSWER BOOKLET

READ THESE INSTRUCTIONS FIRST


Write your name, Civics Group and register number on all the work you hand in.
Write in dark blue or black pen on both sides of the paper.
Do not use staples, paper clips, highlighters, glue or correction fluid.
Answer all questions.
The insert contains the passages for comprehension.
(Note that 15 marks out of 50 will be awarded for your use of language.)
At the end of the examination, fasten all your work securely together.
The number of marks is given in brackets [ ] at the end of each question or part question.

For Examiners Use


Content

/35

Language

/15

Total

/50

This document consists of 7 printed pages and 1 insert.

trendyline

Read the passages in the insert and then answer all the questions which follow. Note that up
to fifteen marks will be given for the quality and accuracy of your use of English throughout this
paper.
NOTE: When a question asks for an answer IN YOUR OWN WORDS AS FAR AS POSSIBLE
and you select the appropriate material from the passages for your answer, you must still use
your own words to express it. Little credit can be given to answers which only copy words or
phrases from the passages.

From Passage 1
1. In paragraph 2, explain why guilt may be an understandable response to the host of
problems we face.
.
.
.
. [2]
2. How does the dolphin-safe logo illustrate the importance of individual choice in paragraph
3?

[2]
3. Why does the author use the word but in paragraph 5? Use your own words as far as
possible.

[2]
4. we of course have more power than just our purchasing power. (lines 33-34). What is the

alternative power suggested by the author?

[1]

trendyline

For
Examiners
Use

5. Using your own words as far as possible, explain the paradox in lines 35-36.

[2]
6. Using material from paragraphs 8 to 9 only, summarise what the author has to say about the
differences between guilt and shame, and how shame can be used to bring about positive
change.
Write your summary in no more than 120 words, not counting the opening words which
are printed below. Use your own words as far as possible.
Compared to shame, guilt is ....

[8]

trendyline

7. In paragraph 10, why does the author believe that shame is seemingly less effective than
guilt? Use your own words as far as possible.

[1]

From Passage 2
8. Identify two words in paragraph 2 which resonate with the idea that we are perfectly poised
between condemnation and celebration (line 9).

[1]
9. How do the examples in paragraph 3 show that we are better off without shame? Use your
own words as far as possible.

[2]
10. In paragraph 5, explain how guilt and shame maintain morality respectively.

[2]
11. According to the author in paragraph 6, why might Christians feel more guilt than others?
Use your own words as far as possible.

[1]

trendyline

12. In paragraph 8, how may shaming be used negatively? Use your own words as far as
possible.

[1]
13. Jennifer Jacquet argues that shaming can lead to positive social action while Julian Baggini
believes that acts of shaming need to be used with caution. To what extent are the authors
views convincing and which view is more applicable to you and your society?

trendyline

trendyline

[10]

trendyline

ST ANDREWS JUNIOR COLLEGE


JC2 PRELIMINARY EXAMINATION

GENERAL PAPER
8807/02
Paper 2

27 AUGUST 2015
1 hour 30 minutes

INSERT

READ THESE INSTRUCTIONS FIRST

This Insert contains the passages for Paper 2.

This document consists of 4 printed pages.

trendyline

PASSAGE 1 Jennifer Jacquet argues that shaming can lead to positive social action
1 Guilt-free shopping will not solve the worlds problems. It is time for more shame.

2 Guilt may be an understandable response to environmental destruction, poor labour standards, animal
cruelty or carbon emissions from budget holiday flights. Its up to me to change, we have been
encouraged to think. I can vividly remember the first time I felt this kind of guilt: I was nine years old and
saw a haunting black-and-white photo of a dead dolphin that had been hoisted on to a tuna boat. Fishing
for tuna was killing dolphins, which would get caught in the nets and drown. I ate tuna, so apparently I,
too, was to blame.

3 I insisted that my family boycott tuna, and we were not alone. After a large-scale boycott, tuna certified as
dolphin-safe or dolphin-friendly was introduced and we started buying tuna again. My guilt was
assuaged and I did not think about it again for more than a decade. When I revisited it as a graduate 10
researching overfishing, I realised the problem was more complicated. I could see that the tuna boycott
had been powerful, but the dolphin-safe logo of 1990, which eased my conscience, arose during the antiregulatory Reagan-Thatcher era, which underlined the importance of individual consumer choice, rather
than government oversight over large scale producers. If demand changed, the argument went, supply
would respond.
15
4 Those who felt concerned about the state of the planet, or guilty about sweatshops overseas or factory
farming, were reminded that they are not innocent bystanders: they were part of the problem as a result
of their own patterns of consumption. This was, of course, true. We were encouraged to engage with our
guilt primarily as consumers; with ethical shopping, or taking fewer flights, we could feel better and be
part of the solution through the power of our pockets. The rise of dolphin-safe, cage-free, organic, fair- 20
trade, shade-grown, sustainable, cruelty-free, carbon-neutral products suggested that responsibility for a
wide array of problems was in the hands of individuals, rather than being something that society, or the
political system, needed to address.
5 But here is the thing: most consumers continue to buy the same old stuff. Only the portion of the industry
that wants to cater to consumers with consciences has changed; the rest of the industry continues to use
pesticides, or unfair trade, or destructive fishing gear and can sell those products at lower prices to
people who do not feel bad about buying them. This is how entire industries shirk reform, while niche
products ease the consciences of the few.
6 Collective problems such as the use of pesticides and excessive levels of carbon emissions cannot be
solved through individual choice. If I buy organic foods, but pesticides are in everybody elses food, those
pesticides will still leach into our shared water supply. If I stop flying and everyone else continues, carbon
emissions continue to increase. Although guilt is an appropriate response to social and environmental
problems, the way we have addressed our guilt is not. As individuals, we of course have more power
than just our purchasing power.

25

30

7 The guilt that might mobilise a minority to activism has been co-opted and used as a marketing tool to
distract that same minority with easy acts of consumption. Over the past quarter-century, the movements
to encourage better environmental protection, labour standards and animal welfare have all, to varying
degrees, used guilt and guilt-alleviation products that engage people primarily as consumers rather than
as citizens. Guilt tactics have eclipsed the older strategy, shame.

35

8 It is tempting to think about shame as something Western societies have outgrown. Shaming
punishments expose a transgressor to public disapproval dunce caps, whipping poles, or hot-iron
branding come uncomfortably to mind. Most Western countries have abolished such punishments,
especially shaming by the state. There are observable differences between guilt and shame that are
worthy to note. Psychologists suggest that guilt is largely a Western phenomenon (many Eastern cultures
do not even have a word for it). Compared to shame, it is also more recent (Shakespeare used the word
guilt only 33 times, while he used shame 344 times). Guilt requires in the first place a conscience and
makes you feel bad if you transgress your own standards. Because guilt requires a conscience, its use is
limited to individuals, since groups, such as the tuna-fishing industry, lack a conscience and therefore

40

trendyline

45

cannot feel guilty. Yet, I believe we need shame now more than ever in this self-promoting and selfindulgent world, and that it is possible to employ shame to serve us in new, larger ways.
9 Unlike guilt, which is primarily a private emotion, shame can be used to influence the way groups even
entire industries behave. Shame can also be used by the weak against the strong. Environmental
groups traced the responsibility for mountaintop removal in Appalachia to nine banks that financed the
coal companies involved, and singled them out year after year (by 2014, the fifth year of the campaign,
both Wells Fargo and JPMorgan Chase have pledged to end their financial relationship with mountaintop-removing coal companies). Shaming can also be used against entire countries. Human rights groups
persuaded the US government to stop executing juvenile offenders by pointing out that only seven
countries apart from America (Bangladesh, Iran, Iraq, Nigeria, Pakistan, Saudi Arabia and Yemen) had
executed juveniles since 1990. In 2005, the US Supreme Court outlawed juvenile executions.
10 Shaming makes a difference, but it is more difficult to make a profit employing shame than it is with guilt.
There are no financial returns from singling out bad products, bad banks or the worst air polluters. In
contrast, products or industries marketed as guilt-free can be sold at higher prices. It is time to realise,
however, that although guilt-free shopping might be a morally correct thing to do, it is not making enough
of a difference. It does not lead to large-scale reform. We need more shaming.

50

55

60

PASSAGE 2 Julian Baggini believes that acts of shaming need to be used with caution.
1 The act of shaming has in recent years surfaced in unexpected places, such as social media platforms. In
California, one form of such shaming is #droughtshaming. People found using excessive amounts of
water when the state is as dry as a bone find themselves facing trial by hashtag. The actor Tom Selleck is
the latest target, accused by online citizens of taking truckloads of water from a fire hydrant for his thirsty
avocado crop.

2 Most people seem happy to harness the power of shame when the victims are the rich and powerful. But
our attitudes to shame are actually much more ambivalent and contradictory. That is why it was a stroke
of genius to call Paul Abbotts BBC series Shameless. We are at a point in our social history where the
word is perfectly poised between condemnation and celebration.
3 Shame, like guilt, is something we often feel we are better off without. The shame culture is strongly
associated with oppression. So-called honour killings are inflicted on people who bring shame to their
families, often for nothing more than loving the wrong person or, most horrifically, for being the victims
of rape. In the case of gay people, shame has given way to pride. To be shameless is to be who you are,
without apology.

10

4 And yet in other contexts we are rather conflicted about the cry of shame. You can protest against honour
killings one day, then name and shame tax-evading multinationals the next. When politicians are called
shameless, there is no doubt that this is a very bad thing. Shame is like rain: whether it is good or bad
depends on where and how heavily it falls.

15

5 There should be no question that we need shame. Morality is in essence the means by which we control
the way we treat each other to maintain as much peace, fairness and social harmony as possible. Both
guilt and shame are central to this. Guilt works from the inside out, emerging in the privacy of your own
conscience. You can feel guilty about something no one else ever finds out about. Shame works the
other way around. Shame is all about how you are perceived in the eyes of others. This is why the
innocent can be made to feel shame, and why the guilty who evade detection can evade shame.
6 Anthropologists distinguish between guilt and shame cultures, depending on which is more important.
The broad, simplistic generalisation is that guilt is more prevalent in Western, Christian cultures, whereas
shame is more potent in Asia insofar as societies in the latter are more collectivistic and what is right is
often enshrined in the community. The link between guilt and certain religions such as Christianity is not
trendyline

20

25

accidental. Guilt has most power when you have a sense of a divine eye who sees what your peers might
not a soul that can be stained without any physical sign of defilement. In that sense, guilt is a kind of
internalisation of shame.

30

7 As Christianity loses its power, we might then expect guilt also to loosen its grip. And if conscience
provides a weaker motivation to behave morally, we might need to rely more on the overtly social
mechanism of shame. If we want people to pick up litter, pay their employees a fair wage or worry about
whether their noise bothers others, shaming them might be the only way.

35

8 If we are to use shame positively, however, we must be mindful of how easy it is to abuse it. Because
shame is a social mechanism, it can all too easily become a tool of bullying, a psychological form of mob
violence. This is what Jon Ronson warns against in his book So Youve Been Publicly Shamed, in which
he worries about the causal cruelty of social media turning against people often on the basis of flimsy or
no evidence.

40

9 That is why shame is a dish best served cold. But if we are to use it, we ought to stop and think whether it
is really merited. Hot-headed indignation too often leads to hasty judgment and the vile scene of the
masses turning on the vulnerable. Shame is a strong moral medicine and, as with any pharmaceutical,
applying the wrong dose is worse than not using it at all.

trendyline

2015 Prelim Paper 2


From Passage 1
1. In paragraph 2, explain why guilt may be understandable response to the host of
problems we face. [2]
Text
Inferred
Its up to me to change, we have been A: We are conditioned to believe that
encouraged to think (lines 3-4)
we are responsible for bringing about
change. [1]
I can vividly remember the first time I felt this
kind of guilt: I was nine years old and saw a B: We are guilty of contributing
haunting black-and-white photo of a dead towards the problems by our
dolphin that had been hoisted on to a tuna consumerist behaviour. [1]
boat. Fishing for tuna was killing dolphins,
which would get caught in the nets and
drown. I ate tuna, so apparently I, too, was to
blame. (lines 4-5)
2. How does the dolphin-safe logo illustrate the importance of individual choice in
paragraph 3? [2]
Text
I insisted that my family boycott tuna, and we were
not alone. After a large-scale boycott, tuna certified
as dolphin-safe or dolphin-friendly was
introduced and we started buying tuna again. My
guilt was assuaged and I did not think about it again
for more than a decadeI could see that the tuna
boycott had been powerful, but the dolphin-safe
logo of 1990, which eased my conscience, arose
during the anti-regulatory Reagan-Thatcher era,
which underlined the importance of individual
consumer choice, rather than government oversight
over large scale producers.

Inferred
A: The tuna boycott by consumers
led to the dolphin-safe production
of tuna [1]
Key idea: individual choice
influences suppliers actions
B: The dolphin-safe logo allows
the consumers to exercise their
choice to buy only tuna with the
dolphin-safe logo OR to not buy
tuna without the logo. [1]
Key idea: see logo buy or
dont buy?
(Key idea: dolphin-safe, ecofriendly products that do not harm
animals)
Do not accept:
The dolphin-safe logo allows
consumers to exercise their
choice. [0] clear context needed

Get PSJ Private Tutor to Guide you through Exams Now!


Contact www.privatetutor.com.sg

trendyline

3. Why does the author use the word but in paragraph 5? Use your own words as far
as possible. [2]
Text

Paraphrased
A: (function/authors
purpose/intention)
The author is contrasting possibility
vs reality OR
emphasising/highlighting the
limitations of the assumption OR
emphasising the irony of the
situation.
Do not accept:
Rebuttal

The rise of dolphin-safe, cage-free, organic,


fair-trade, shade-grown, sustainable, crueltyfree, carbon-neutral products suggested that
responsibility for a wide array of problems
was in the hands of individuals, rather than
being something that society, or the political
system, needed to address.

B1: (context)
Consumers have the power/ability to
solve/alleviate a vast range of
environmental and social problems.

But here is the thing: most consumers


continue to buy the same old stuff. Only
the portion of the industry that wants to cater
to consumers with consciences has changed;
the rest of the industry continues to use
pesticides, or unfair trade, or destructive
fishing gear and can sell those products at
lower prices to people who do not feel bad
about buying them.

B2: (context)
However, most of them choose to
persist in their old environmentally
unfriendly habits.
2 or 0 otherwise it wont make
sense/no contrast.
Some form of reference to authors
purpose is okay to get A.

4. we of course have more power than purchasing power. (lines 33-34). What
is the alternative power suggested by the author? [1]
The power of shaming
Answer must encapsulate guilt.
X activism
X positive change

trendyline

5. Using your own words as far as possible, explain the paradox in lines 35-36. [2]
Text
The guilt that might mobilise a minority to
activism has been co-opted and used as
a marketing tool to distract that same
minority with easy acts of consumption.

Paraphrased
A. The few who feel that they should
shop ethically [1]
B. Fall prey to the ploy that they
have done their part/enough by
their effortless buying choices. [1]

7. In paragraph 10, why does the author believe that shame is seemingly less effective
than guilt? Use your own words as far as possible. [1]
Text
There are no financial returns from singling out bad
products, bad banks or the worst air polluters. In
contrast, products or industries marketed as guilt-free
can be sold at higher prices. It is time to realise,
however, that although guilt-free shopping might be a
morally correct thing to do, it is not making enough of a
difference. It does not lead to large-scale reform.

Paraphrased
Shaming does not lead to
monetary incentives for
producers.

From Passage 2
8. Identify two words in paragraph 2 which resonates with the idea that we are
perfectly poised between condemnation and celebration (line 9). [1]
Ambivalent and contradictory. [1 or 0]

9. How do the examples in paragraph 3 show that we are better off without shame?
Use your own words as far as possible. [2]
Text
Shame, like guilt, is something we often feel we
are better off without. The shame culture is
strongly associated with oppression. So-called
honour killings are inflicted on people who bring
shame to their families, often for nothing more
than loving the wrong person or, most
horrifically, for being the victims of rape.

Paraphrased
The examples show that
A: shame has led to cruel and
unjust treatment of others [1],
B: even causing deaths [1].

C: Also, ostracised groups of


In the case of gay people, shame has given way people can enjoy freedom from
to pride. To be shameless is to be who you are, stigma. [1]
without apology.
Accept any 2 possible points.

trendyline

10. In paragraph 5, explain HOW guilt and shame maintain morality respectively. [2]
Text
Morality is in essence the means by which we
control the way we treat each other to maintain
as much peace, fairness and social harmony as
is possible. Guilt works from the inside out,
emerging in the privacy of your own conscience.
You can feel guilty about something no one else
ever finds out about. Shame works the other
way around. Shame is all about how you are
perceived in the eyes of others.

Inferred
A: Guilt makes us change our
behaviour because we know we
are wrong/we are pricked by our
conscience.
B: Shame makes us change our
behaviour/keeps us on our toes
because we are afraid of how
others view us.

11. According to the author in paragraph 6, why might Christians feel more guilt than
others? Use your own words as far as possible. [1]
Text
Guilt has most power when you have a sense of a
divine eye who sees what your peers might not
a soul that can be stained without any physical sign
of defilement.

Paraphrased
This is because they are
convicted by an omnipresent
God who knows when they
have done wrong, even if it is
not visible to others. [1]
Key idea:
Accountability to God
God is watching

12. In paragraph 8, how may shaming be used negatively? Use your own words as far
as possible. [1]
Text
Because shame is a social mechanism, it can all too
easily become a tool of bullying, a psychological
form of mob violence. This is what Jon Ronson
warns against in his book So Youve Been Publicly
Shamed, in which he worries about the causal
cruelty of social media turning against people often
on the basis of flimsy or no evidence.

Paraphrased
Shaming can be employed to
intimidate others.

trendyline

Summary
Using material from paragraphs 8 to 9, summarise the differences between guilt and shame,
and how shame can be used to bring about positive change. Write your summary in no more
than 120 words, not counting the opening words which are printed below. Use your own
words as far as possible.
Compared to shame, guilt is
Text
Differences
A Psychologists suggest that
guilt is largely a Western
phenomenon (many
Eastern cultures do not
even have a word for it)

Paraphrased

Main Idea

Guilt is mainly a Western

Western
concept

Compared to shame, it is
also more recent
(Shakespeare used the
word guilt only 33 times,
while he used shame 344
times).

more modern/contemporary concept.

More recent

Guilt requires in the first


place a conscience

It requires an inner moral voice/a sense


of right and wrong

Conscience

and makes you feel bad


if you transgress your
own standards.

which chastises you when you fail to live


up to your own moral principles.

Transgression
of standards

Because guilt requires a


conscience, its use is
limited to individuals,

In this way, guilt is only applicable /


restricted to individuals.

Limited
application
(individuals)

since groups, such as the


tuna-fishing industry, lack a
conscience and therefore
cannot feel guilty.

Hence, groups do not possess a


collective moral voice and do not have
the capacity to experience guilt.

Does not
apply to
groups

Yet, I believe we need


shame now more than ever
in this self-promoting and
self-indulgent world,

In this self-serving/self-obsessed times


/ selfish culture,

(context)

and that it is possible to


employ shame to serve us
in new, larger ways.
Unlike guilt, which is
primarily
a
private
emotion, shame can be

shame can be put to use in novel and


more wide-reaching/bigger ways.

New, larger
ways

Guilt is a feeling known only by the


individual,

Private
emotion

trendyline

Positive Change
J (shame is) used to
influence the way groups
even entire industries
behave.

and can even be employed to shape


the actions of large corporations

Influence
how larger
(business)
groups
behave/act
Empower the
weak

Shame can also be used


by the weak against the
strong.

Shame can empower/be employed by


the vulnerable to take action against the
powerful

Environmental groups
traced the responsibility for
mountaintop removal in
Appalachia to nine banks
that financed the coal
companies involved, and
singled them out year after
year (by 2014, the fifth year
of the campaign, both Wells
Fargo and JPMorgan
Chase have pledged to end
their financial relationship
with mountain-top-removing
coal companies).

(generalised)
It has succeeded in stopping the
profiteering ways of large
companies/industries at the expense
of the environment.

Past success
stopping
corporations
profiteering
ways

Shaming can also be used


against entire countries.

It can also be employed to stop


governments/states/nations

Influence
how
countries
behave
(can lift
countries)

Human rights groups


persuaded the US
government to stop
executing juvenile
offenders by pointing out
that only seven countries
apart from America
(Bangladesh, Iran, Iraq,
Nigeria, Pakistan, Saudi
Arabia and Yemen) had
executed juveniles since
1990. In 2005, the US
Supreme Court outlawed
juvenile executions.

(generalized)
from violating the basic rights of
their citizens.

Past success
stopping
countries
violation of
human rights

trendyline

Application Question
Jennifer Jacquet argues that shaming can lead to positive social action while Julian Baggini
believes that acts of shaming need to be used with caution. To what extent are the authors
views convincing and which view is more applicable to you and your society?
Passage 1
Text
Para 3
I could see that the tuna
boycott had been powerful, but
the dolphin-safe logo of 1990,
which eased my conscience,
arose during the antiregulatory Reagan-Thatcher
era, which underlined the
importance of individual
consumer choice, rather than
government oversight over
large scale producers. If
demand changed, the
argument went, supply would
respond.
Para 4
that responsibility for a wide
array of problems was in the
hands of individuals, rather
than being something that
society, or the political system,
needed to address.

Para 5
Only the portion of the industry
that wants to cater to
consumers with consciences
has changed; the rest of the
industry continues to use
pesticides, or unfair trade, or
destructive fishing gear and
can sell those products at
lower prices to people who do
not feel bad about buying
them. This is how entire
industries shirk reform, while
niche products ease the
consciences of the few.
Para 6
Although guilt is an
appropriate response to social

Evaluation
Not Convincing:
This largely did not affect our society and we were not
large tuna consumers and the boycott of 1990s of tuna
did not really affect Singaporean consumers choice as
far as tuna was concerned.
Convincing:
But there was concern in other areas especially in the
consumption of Sharks Fin

Not Convincing:
Singaporeans are rather a passive community and do not
see themselves as making a difference.
Most, look to the government to lead the way in the
solving of environmental problems. Singaporeans think
that without government support and encouragement
nothing can be done.
Most Singaporeans are also indifferent or passive.
Consumption is a way of life in Singapore. We do not see
the processes and therefore do not care where or how
the food comes from, we just consume.
Not Convincing:
Those who buy organically grown food are also few.
Singaporeans being practical do not want to pay higher
prices for these goods.
The industry therefore is not at all concerned as there is
no pressure form the consumer. So largely it does not
cater to the small number of activists.
Convincing:
Those who look for the products are environment friendly
are very, very few.

Not Convincing:
In Singapore, where its citizens have been politically
passive, any kind of activism was frowned upon. It had

trendyline

and environmental problems,


the way we have addressed
our guilt is not. As individuals,
we of course have more power
than just our purchasing
power.

Para 7
..used guilt and guiltalleviation products that
engage people primarily as
consumers rather than as
citizens. Guilt tactics have
eclipsed the older strategy,
shame.

become a way of life to be apathetic, that one person


cannot change society. We look to the government to
bring about changes as the government has always
taken the lead to do the right thing. So the Singaporean
does not have any guilt, nor can he be shamed into doing
the right thing.
Singaporeans may not know that they have more than
just purchasing power. Singaporeans also tend not really
care and would unlikely be co-operated in any boycotts.
Not Convincing:
Guilt cannot be used on the Singaporean consumer as, if
we extend the idea of guilt, fall kinds of products need to
be boycotted.

Passage 2
Text
Para 1
The act of shaming has in
recent years surfaced in
unexpected places, such as
social media platforms.
find themselves facing trial
by hashtag

Evaluation
Convincing
A valid observation of recent trends of online behavior as
there has been a rise of such acts of online vigilantism
across the world e.g. China human flesh search
engine.

Para 3
Shame, like guilt, is
something we often feel we
are better off without. The
shame culture is strongly
associated with
oppression.

Convincing
Shaming of people who are different can lead to social
discrimination and ostracism in society. On a personal
level, the individual is made to be a misfit or an outcast.

Para 5

Convincing

The prevalence of such keyboard warriors show that


people are emboldened by the anonymity afforded by
online platforms and are taking to them to humiliate those
who fall short of societal norms. Such online vigilantism
is problematic because it has the potential of escalating
into an online lynch mob.

Not convincing
There is value to shaming when it is used appropriately:
While it might be linked to oppression, when used in an
appropriate manner, shaming people can be a powerful
approach to induce the kind of behavioral change that
programme designers seek as it is a negative emotion
that people work hard to avoid. In order to avoid being
shamed, people would change their personal behavior.

trendyline

There should be no question


that we need shame. Morality
is in essence the means by
which we control the way we
treat each other to maintain
as much peace, fairness and
social harmony as is
possible. Both guilt and
shame are central to this.

Shame effectively deters people from behaving


inappropriately, which socialises people to conform to
social values and norms.

Para 8
we must be mindful of how
easy it is to abuse it. Because
shame is a social mechanism,
it can all too easily become a
tool of bullying, a
psychological form of mob
violence.

Convincing

But if we are to use it, we


ought to stop and think
whether it is really merited.
Hot-headed indignation too
often leads to hasty judgment
and the vile scene of the
masses turning on the
vulnerable.

Not convincing
Shame can be relentless, much more so in todays wired
society. Public shaming is corrosive to our social fabric
as netizens grow increasing obsessed with uploading
and criticising even the most frivolous wrongdoing. This
is rather cruel and is no different from bullying.

Our reaction to those who have violated social norms


(spoken or otherwise) reveals much about ourselves, and
the kind of society we are. Using shame to expose and
punish the follies of others whether they were intended
or not reflects a certain malice and brutality that is
incongruous to the civilised and humane society that
Singapore is aspiring towards. A hallmark of a civilised
society is one that is gracious in its response to the
wrongdoer and just and even-handed in meting out the
punishment, especially if the offender is from a
vulnerable group of society (eg. young, poor, mentally
unsound) and if his or her actions were not intentionally
hurtful. This is particularly true if the significance or
impact of the misconduct is debatable.
Unconvincing
Far from being a tool of bullying, shaming is necessary to
enforce certain standards of respect, decency and
morality that are important to a particular society.

Para 9
Shame is a strong moral
medicine and, as with any
pharmaceutical, applying the
wrong dose is worse than not
using it at all.

Convincing
The use of shame to correct any form of misdemeanour
can be seen as a setback to other important values in
society, such as democracy and freedom of speech etc.
It can also lead to societal backlash and undermine a
countrys s global image.

trendyline

Name:

Civics Group:

GP Tutor:

SERANGOON JUNIOR COLLEGE


JC2 Preliminary Examination 2015

GENERAL PAPER
Paper 2
Insert & Question Paper
Candidates answer on the Question Paper.
No Additional Materials are required.

8807/02
Friday 14th August 2015
1 hour 30 minutes

READ THESE INSTRUCTIONS FIRST


Write in dark blue or black pen in the spaces provided on the Question Paper.
Do not use paper clips, highlighters, glue or correction fluid/paper.
DO NOT WRITE IN THE MARGINS.
Answer all questions.
The Insert contains the passage for comprehension.
Note that 15 marks out of 50 will be awarded for your
use of language.
At the end of the test, fasten all your work securely together.
The number of marks is given in brackets [ ] at the
end of each question or part question.

For Examiners Use


Short-Answer
Questions

/17

Summary

/8

Application
Question

/10

Content

/35

Language

/15

Total

/50

This Question Paper consists of 6 printed pages and 1 insert.


[Turn over

trendyline

Read the passage in the insert and then answer all the questions which follow below. Note that up
to fifteen marks will be given for the quality and accuracy of your use of English throughout this
Paper.

For
Examiners
Use

NOTE: When a question asks for an answer IN YOUR OWN WORDS AS FAR AS POSSIBLE and
you select the appropriate material from the passage for your answer, you must still use
your own words to express it. Little credit can be given to answers which only copy words
or phrases from the passage.
1. In paragraph 1, what are the ways in which sports has produced something more than just the
usual trappings (line 4)? Use your own words as far as possible.
.......................................................................................................................................................
.......................................................................................................................................................
.......................................................................................................................................................
..................................................................................................................................................[3]
2. What are the two ways in which the dialogue about sports has evolved (lines 7 8)? Use your
own words as far as possible.
....
....
...[2]
3. Suggest how the explosion of the blogosphere (lines 13 14) has led to an intensified
discussion and debate about sports.
.........................................................................................................................................................
.........................................................................................................................................................
... [1]
4. What does the word so-called (line 19) tell you about the experts?
.....
... [1]

5. In paragraph 3, what are the functions of sports? Use your own words as far as possible.
trendyline

For
Examiners
Use

....
....
....
...[3]
6. Explain the authors use of the word presumably (line 28).
.....
.....
...[1]
7. Using material from paragraphs 46 only, summarise what the author has to say about
the value of sports and his criticisms of it.
Write your summary in no more than 120 words, not counting the opening words,
which are printed. Use your own words as far as possible.
Sports is valuable because ..........................................................................................
........................................................................................................................................................
........................................................................................................................................................
........................................................................................................................................................
........................................................................................................................................................
........................................................................................................................................................
........................................................................................................................................................
........................................................................................................................................................
........................................................................................................................................................
........................................................................................................................................................
........................................................................................................................................................
........................................................................................................................................................
........................................................................................................................................................
... [8]

Number of words: ___________


8. According to the author, how has sports taken up the slack of religions retreat from modern
society (lines 61-62)? Use your own words as far as possible.
trendyline

For
Examiners
Use

.....
.....
.....
.....
... [2]
9. What does the phrase oasis of truth (lines 66-67) suggest about sports?
.....
.....
.....
... [2]
10. Explain the irony in lines 74-75. Use your own words as far as possible.
.....
.....
.....
...[2]

11. In this article, Joshua R. Keefe discusses the functions of sports, its appeal and flaws.
How far do you agree with his observations? Relate your opinions to your own society.

trendyline

For
Examiners
Use

For
Examiners
Use

trendyline

[10]
END OF PAPER

trendyline

Name:

CG:

GP Tutor:

SERANGOON JUNIOR COLLEGE


JC2 Preliminary Examination 2015

GENERAL PAPER
Paper 2
Insert

8807/2
14th August 2015
1 hour 30 minutes

READ THESE INSTRUCTIONS FIRST


Write your name and CG in the spaces at the top of this page.
Submit the insert together with your answer booklet at the end of the examination.

This Insert Booklet consists of 3 printed pages including this page


[Turn over

Joshua R. Keefe discusses the functions of sports, its appeal and flaws.

trendyline

Sports has become far more than contests with rules played on fields, pools, or courts. Our
current conception of sports is more than just a ball moving between groups of athletes, or a
struggle for a finish line, or an effort to impress judges, as various critical studies will attest.
Sports has produced something more than just the usual trappings of sports that bleed into
other aspects of society: the refereeing and record keeping, the public accounts of events,
the hero worship of the victors, and the training of youth.
Sports has produced, especially in the information age, a remarkable evolution in the
dialogue about sports. What was fifty years ago a sports section of a local newspaper and
perhaps a radio hour, and later, a local news update, has become an entire wing in the
marketplace of ideas. There are now twenty-four hour sports networks on radio and
television, periodicals and books of every kind, and endless websites devoted to sports in all
of its forms. Very few of these outlets are actually concerned with reporting about sporting
events, at least in an objective sense. Especially with the sudden explosion of the
blogosphere, the recent trend in sports is not simple enjoyment of the contests on some
aesthetic level, but a constant need to debate and argue over the underlying meaning of the
games. The forms these arguments take are as diverse as the games themselves. There
are radio talk shows, devoted mostly to second guessing coaches and allowing fans to vent
their frustrations on an emotional level.
There are TV shows where panels of socalled experts debate increasingly hypothetical
situations. There are those who seek meaning in numbers, pouring over and creating new
stats, and arguing which translate into indicators of real world success. Then there are those
who seek to disengage from the subjective experience of sports and seek to understand the
field in an academic way that appreciates sports place within society. All of these
perspectives penetrate to different levels of sport, and all have their own functions, from the
angry fan driving to work who needs a morning distraction and outlet, to the intellectual who
uses sports to explore mans social needs. What is interesting is that such a wide and
diverse strata of society use sports, and the dialogue about sports, for so many different
ends, and that they all presumably continue to find something of worth in those pursuits.

Today, sports teaches that effort leads to victory, a useful lesson in a work-oriented society.
Sports also helps people navigate the tension between team loyalty and individual glory.
Sports culture has influenced students: it discourages whining, and rewards self-discipline. It
teaches self-control and its own form of justice, which has a more powerful effect than
anything taught in the classroom.

Sports is also most certainly an industry. Wherever large amounts of capital are created and
concentrated, both in the hands of the owners and the hands of athletes themselves, there
is going to be society wide interest. Sports has become a mostly corporate entity, and
athletes are essentially bought and sold on an open market. The intrinsic character building
lessons that have long been held up as sports shining virtue have been replaced by
corporate interests. Dr. Stanley Eitzen, a professor of sociology at the Colorado State
University, notes that sports has been transformed from an activity for individuals involved
in sports for its own sake, to one where they do it for work, and where loyalty to players,
coaches, and owners is a quaint notion that is now rarely held.

Sports is increasingly becoming a product to be bought and sold, and the huge increase in
sports dialogue can be seen as a testament to that. It has come to the point,
Jay J. Coakley, author of Sports in Society, observes, where participants can prove
themselves in sports through their ability to consume as well as their ability to master
physical skills. Seasons in sports have become too long and the arenas too gargantuan.
Athletes have become a separate gladiator class, and the recruitment process gives them
an undue sense of their own worth. Spectators have been reduced to an anonymous mass
of passive consumers of other peoples excellence. Coaches have a greater incentive to
satisfy the braying crowd with victories than to teach good habits. Yet, while it is useful to
look at modern sports as a commodity, it is not an all-inclusive perspective. Many do still

trendyline

10

15

20

25

30

35

40

45

50

play sports for the sake of playing, and it would seem that judging sports as just another part
of the entertainment industry misses much of the emotional connections, cultural bridges
and cultural walls that sports creates and reinforces.
7

To explain the depth of devotion to sports and various teams, sports is often analogously
compared to organised religion, and the comparison is an apt one: sports has its saints and
its devils, its shrines and its customs, its prayers (the concept of rooting), and like religion,
sports both informs the culture at large and is moulded by it. While it is easy to compare
sports to religion, it is another thing to call sports a religion. But sports may very well be a
kind of progeny of religion, as a social function that has taken up the slack of religions
retreat from modern society. In industrial society, sports has overtaken many of the previous
functions of an organised religion. In a segmented society, big-time sports is one of the few
avenues for large-scale communal participation, often crossing class lines. Such an avenue
induces large numbers of people in a region to share common emotional experiences.
Another perspective on the place of sports in a modern society is that of a kind of oasis of
truth. Sports has clear winners and losers, clear rules that give an opportunity for us to view
objective truth forged somewhere between brute physical force and aesthetic beauty
captured by human form. The popular public conception is that sports create an equal
playing field that strips away social and political advantages. Modern sports assume
equality. In a society like ours that prides itself on egalitarian values but is often forced to
painfully confront its own inequalities and injustices, the allure of the sports as centre of truth
is particularly compelling. Mass sporting sports are the emotional hubs at the centre of vast
networks of analysis, criticism and conversation. They generate loyalties and emotional
debates that are at once completely meaningless and totally consuming.
Many scholars argue that sports reflects and informs society. This, I believe, is true. But to
then assume that sports accomplishes nothing but the reduction of the population to a
position of complete passivity is to assume that the same is true of society itself. There are
moments of transcendence in sports, just as there are in society, and there are revolutions
in sports, just as there are in society. Sports, like any large and important social institution, is
a complex weaving of forces and concerns just as the culture itself is. Sports allows us to
view the very best, and the very worst, of ourselves.

trendyline

55

60

65

70

75

80

Suggested Answers for 2015 JC2 Prelims Paper 2


1.

In paragraph 1, what are the ways in which sports has produced something more than just the usual trappings' (line
4)? Use your own words as far as possible. [3]
Lifted

Paraphrased

Sports has produced something more than just the usual The ways are
trappings of sports that bleed into other aspects of society:
(a) (1) Judging
(a1) the refereeing and (a2) record keeping, (b) the
(2) count/ score tracking / keeping track
public accounts of events, (c) the hero worship of the
(b) Provision of public/official records/ media reports of
victors, and (d) the training of youth
activities
(c) the idolisation of successful people,
(d) and the drilling / disciplining of the young people in
society / coaching / nurturing / conditioning of the
young
Range marking
2 points 1 m
3 points 2 m
4/5 points 3 m
Accept
a1) the job of those making sure players are adhering to
the rules and regulations, practice of being the judge, being
an umpire, regulating and enforcing rules, ensuring fair play,
regulating/umpiring/officiating the game
a2) those taking note of the time and score, taking down the
score, noting of one's achievements
b) widespread coverage, broadcasting, citizen journalism of
the play,
c) intensity admiration, (great) devotion, great recognition,
reverence, being devoted fans, glorification, high regard,
d) [so long as there is a sense of forward progression, and
there isnt a reference to a micro-session]
teaching, strengthening, preparing, developing, preparing
the young with the necessary skills, guiding,
shaping/moulding/culturing of the young, grooming,
education for the newer generation, the physical preparation
of adolescents,
Do not accept
a1) managing the game, having judges
b) those who manage the publicity of the game, sharing
news of these activities
c) obsession, fanatic fans, devoted towards successes
(wrong subject, winners as role models,
d) practice of the young, lessons for the young, the practice
that young people have to go through, the drills/practices
young people have to go through,

2.

What are the two ways in which the dialogue about sports has evolved (lines 7 8)? Use your own words as far as
possible. [2]
Lifted

Paraphrased

Get PSJ Private Tutor to Guide you through Exams Now!


Contact www.privatetutor.com.sg

trendyline

What was fifty years ago a sports section of a local


newspaper and perhaps a radio hour, and later, a local
news update, has become an entire wing in the marketplace
of ideas. There are now twenty-four hour sports networks
on radio and television, periodicals and books of every
kind, and endless websites devoted to sports in all of its
forms is not simple enjoyment of the contests on some
aesthetic level, but a constant need to debate and argue
over the underlying meaning of the games.

a)
b)
c)
d)

duration
From short segments on radio, sports coverage has
expanded into all-day sports channels.
coverage
Sports used to be covered in traditional media only but
it is now a feature in a range of media
types of sports
Sports coverage has now expanded from a small range
of sports to an extensive range of sports.
nature of discussion
In the past, discussions were based on the game itself
but now, there is contentious/heated
discussion/exchange about the purpose/worth of the
game.

Any 2 ways
There must be a comparison to present the evolution.
There must be a match between the past and present ways.
3.

Suggest how the explosion of the blogosphere (lines 13 14) has led to an intensified discussion and debate about
sports. [1]
Lifted

Paraphrased

Especially with the sudden explosion of the blogosphere, the The explosion of the blogosphere has enabled
recent trend in sports is not simple enjoyment of the
contests on some aesthetic level, but a constant need to greater reach to sports news and information,
debate and argue over the underlying meaning of the
games.
OR
greater accessibility of sports news and information,
OR
more people can create blogs, expressing themselves
freely
such that more people are participating in the discussions.
Accept:
Any suitable and logical inferences but must reflect the large
increase in the reach / increase in number of people, as
implied by the word explosion
Do not accept
Increased efficiency/coverage/variety, much easier

4.

What does the word so-called (line 19) tell you about the experts? [1]
Lifted

There are TV shows where panels of socalled experts


debate increasingly hypothetical situations.

Paraphrased
These experts are not widely recognized / not
professionals who are familiar with the analysis of
sports / not fully qualified to provide their views on sports.
OR
They are not as knowledgeable about sports as they
claim to be.
*Sports is a necessary context
Do not accept
mere negation without any context of sports
may or may not be experts in sports understanding of socalled has not been clearly clarified

trendyline

not professional sports players incorrect context

5.

In paragraph 3, what are the functions of sports? Use your own words as far as possible. [3]
Lifted

Paraphrased

All of these perspectives penetrate to different levels of


sport, and all have their own functions, from the (a) angry
fan driving to work who needs a morning distraction and
(b) outlet, to the (c) intellectual who uses sports to explore
mans social needs.

The two functions are


for angry fan/frustrated fanatic:
(a) to have an avenue to escape / to be entertained [1]
(b) Vent/catharsis [1]
for intellectuals/sociologist/social
scientist/researcher:
(c) to uncover the social/interpersonal desires/wants of
man through the understanding of how sports
works/to interact/to bond [1]
Important: must have the function + to whom
NO CONTEXT = NO MARKS
The players need not be paraphrased.
Do not accept
a) divert attention

6.

Explain the author's use of the word 'presumably'. (line 28) [1]?
Lifted

What is interesting is that such a wide and diverse strata of


society use sports, and the dialogue about sports, for so
many different ends, and that they all presumably find
something of worth in those pursuits.

Paraphrased
The author highlights the
likelihood / possibility / uncertainty
OR
people are likely
to find something worthy in sports (necessary context)

trendyline

7.

Using material from paragraphs 4 6 only, summarise what the author has to say about the value of sports and his
criticisms of it.
Write your summary in no more than 120 words, not counting the opening words, which are printed. Use your own
words as far as possible.
Lifted

Paraphrased

Sports is valuable because it [benefits of sports]


(a) Today, sports teaches that effort leads to victory, Imparts / demonstrates/ propagates that hard work
brings success,
(b) a useful lesson in a work-oriented society.

which is important in a labour-focused economy/society


*Award only if POINT A is present/attempted

(c) (1) Sports also helps people navigate the tension


(2) between team loyalty and individual glory.

(1) aids/enables/allows people in mediating the conflict


(2) between group solidarity and personal success /
achievement OR promotion

(d) it discourages whining

It disincentivises / reduces complaining / lamenting


*Context of students/school is required

(e) rewards self-discipline

Incentivises / awards / celebrates self-regimentation


*Allow for lift of self
*Context of students/school is required

(f)

teaches self-control

coaches one to have mastery over himself / selfrestraint /manage ones temper
*Allow for lift of self
*Context of students/school is required

(g) its own form of justice

Gives people their just deserts / what they deserve /


doing what is right/just/fair
*Context of students/school is required

(h) which has a more powerful effect than anything


taught in the classroom

with greater efficacy / more compelling than formal


instruction / conventional education in schools
*Allow for lift of more
*Context of students/school is required
*Award only if POINT G is present / attempted

(i)

Wherever large amounts of capital are created


and concentrated both in the hands of the
owners and the hands of athletes themselves,
there is going to be society wide interest

[INFERRED POINT]
Sports has an economic value

HOWEVER, [criticisms of sports]


(j)

Sports has become a mostly corporate entity,


and athletes are essentially bought and sold on
an open market.

[INFERRED POINT]
Athletes are viewed as goods / no longer regarded as
people.

OR
Sports is increasingly becoming a product to be
bought and sold
(k) intrinsic character building lessons that have

The inherent teaching of values Sports fundamental /

trendyline

long been held up as sports shining virtue

inherent moral influence


* context of CHARACTER BUILDING LESSONS must be
present
* Award POINT K only if POINT L is present / attempted

(l)

have been replaced by corporate interests

(m) transformed from an activity for individuals


involved in sports for its own sake

substituted / supplanted by profit-centred motivations /


profit-driven agendas.
* Award POINT L only if POINT K is present / attempted
Sports has changed / evolved from being an activity for
people to engage in for enjoyment / its intrinsic value
* context of INDIVIDUALS must be present
* Award POINT M only if POINT N is present / attempted

(n) to one where they do it for work

and treated as a job / career / profession

(o) and where loyalty to players, coaches, and


owners is quaint notion that is now rarely held

Having an allegiance / Displaying faithfulness to a team,


a coach and to the sponsors is an antiquated concept / no
longer a practice / seldom practised today.

(p) Seasons in sports have become too long

There is an excessive number of rounds of play / are


competition periods are exceedingly lengthy
* Degree is compulsory

(q) and the arenas too gargantuan.

And the stadiums are unnecessarily large today


* Degree is compulsory

(r)

Athletes are now elites / distinct group

Athletes have become a separate gladiator


class,

(s) and the recruitment process gives them an


undue sense of their own worth

The selection criteria disproportionately boost their own


egos / prides.

(t)

The audience / fans are now treated merely as a nameless


/ unidentifiable group of people

Spectators have been reduced to an anonymous


mass

OR
The viewers become just a crowd. [BOD]

(u) of passive consumers of other peoples


excellence.

that blindly support / simply buy into the athletes


outstanding performance
[INFERRED POINT]
* Award POINT U only if POINT T is present / attempted
- DO NOT ACCEPT: inactive

(v) Coaches have a greater incentive to satisfy the


braying crowd

Coaches now have more motivation to simply pander to


the preferences of the audience / fans
OR
Coaches rather feed the fans [Script B]

trendyline

(w) with victories than to teach good habits.

with success / admirable / commendable results rather


than to impart desirable practices / manners /
sportsmanship in their players.
* Award POINT W only if POINT V is present / attempted

judging sports as just another part of the entertainment


industry misses much of the
(x) emotional connections,

(x) Sports in fact forges bonds,


(y) facilitates cultural exchange/ties, and

(y) cultural bridges and


(z) cultural walls that sports creates and reinforces.

Marking range:

1 2 points
3 4 points
5 6 points
7 8 points
9 points
10 11points
12 13 points
14 points or more

(z) preserves / strengthens traditions / cultures

1 mark
2 marks
3 marks
4 marks
5 marks
6 marks
7 marks
8 marks

trendyline

8.

According to the author, how has sports 'taken up the slack of religion's retreat from modern society' (lines 62 63)?
Use your own words as far as possible. [2]
Lifted

But sports may very well be a kind of progeny of religion, as


social function that has taken up the slack of religions
retreat from modern society. In industrial society, sports has
overtaken many of the previous functions of an organised
religion. In a segmented society, big-time sports
is one of the few avenues for
(a) large-scale communal participation,
(b) often crossing class lines. Such an avenue induces
(a) large numbers of people in a region to
(c) share common emotional experiences.

Paraphrased
Sports has managed to
in a society that is segmented [context is not necessary],
(a) LARGE QUANTITY OF PEOPLE / PARTICIPATION
Sports that allow mass gathering / community to come
together
(b) ACROSS CLASS
Frequently joins people of different walks of life / social
strata together
(c) SHARE COMMON EMOTIONAL EXPERIENCE
To enjoy similar / communal
emotional encounters
OR
feelings
1 pt = 0m
2 pts = 1m
3pts = 2m
Marks can be awarded simply by identifying the how,
without the context

9.

What does the phrase 'oasis of truth' (line 68 69) suggest about sports? [2]
Lifted

Another perspective on the place of sports in a modern


society is that of a kind of oasis of truth.

Paraphrased
Sports (necessary context) offers / allows us
(a) OASIS
a safe haven / respite
OR
positively attracts / draws us [1]
(b) TRUTH
where clarity / unbiased / objective information is
assured [1]

10. Use your own words as far as possible, explain the irony in lines 77-78. [2]
Lifted

Paraphrased

They generate loyalties and emotional debates that are at


once completely meaningless and totally consuming.

EXPECTED:
MEANINGLESS AND SO WOULDNT CONSUME TIME/
EFFORT

ACCEPT ALSO:

It is expected that when something is completely


worthless, it would not cause a person to spend
much time / effort / place much attention to it.

EXPECTED
CONSUMES TIME/EFFORT AND SO, WLD BE
MEANINGFUL

ACTUAL
CONSUMES TIME/EFFORT BUT MEANINGLESS

ACTUAL:
MEANINGLESS BUT ALSO CONSUME TIME/EFFORT
However, the loyalties here are capable of fully
occupying us / our hearts / making us waste a lot of
time on them.

trendyline

ALL OR NOTHING
Context of loyalties/emotional debates is necessary.
IF IRONY PRESENTED BUT NOT PARAPHRASED.

trendyline

TEMASEK JUNIOR COLLEGE


PRELIMINARY EXAMINATION 2015

CANDIDATES NAME
CLASS

GP TUTORS NAME

GENERAL PAPER

8807/02

Paper 2

27 August 2015
1 hour 30 minutes

Candidates answer on the Question Paper.


Additional Materials: 1 Insert

READ THESE INSTRUCTIONS FIRST


Write your name, class and GP tutors name on all the work that you hand in.
Write in dark blue or black pen.
Do not use staples, paper clips, highlighters, glue, correction fluid or correction tape.
Answer all questions.
The Insert contains the passage for comprehension.
Note that up to 15 marks out of 50 will be awarded for your use of language.
At the end of the examination, fasten all your work securely together.
The number of marks is given in brackets [ ] at the end of each question or part question.

For Examiners Use


Content

/35

Language

/15

Total

/50

This document consists of 6 printed pages.

[Turn over
trendyline

Read the passage in the insert and then answer all the questions. Note that up to fifteen
marks will be given for the quality and accuracy of your use of English throughout this Paper.
NOTE: When a question asks for an answer IN YOUR OWN WORDS AS FAR AS POSSIBLE
and you select the appropriate material from the passage for your answer, you must still use
your own words to express it. Little credit can be given to answers which only copy words or
phrases from the passage.

1 Why is the promise of continual human progress and improvement alluring (line 8)? Use
your own words as far as possible.
.
.
[2]

2 What is the authors intention in asking a series of questions in lines 911?


.
[1]

3 Why are the words failures, errors, fallacies and naiveties (line 15) in inverted
commas?
.
[1]

4 What is the author suggesting about the nature of philosophy in the last sentence of
paragraph 2?
.
[1]

5 Explain what the author means by saying that failure reveals just how close our existence
is to its opposite (lines 2425). Use your own words as far as possible.
.

. [3]
[Turn over
trendyline

For
Examiners
Use

6 Why does the author use the phrase the most self-aware or enlightened excepted (lines
4041)?
.
[1]

7 Explain the authors use of the word may in line 63.


.
.
.
[2]
8 Why does the author argue that we will be virtually perfect and essentially dead (lines 71
72) if science solves all our problems? Use your own words as far as possible.

.
.
.
[2]

9 What does biological failure (line 81) refer to and why have we pretended not to see it
(line 83)? Use your own words as far as possible.
.

. [3]

10 What is the authors intention in comparing Tolstoys Ivan in paragraph 9 with Bergmans
Block in paragraph 10?
.
[1]

[Turn over
trendyline

For
Examiners
Use

11 Using material from paragraphs 4 to 6 (lines 34 to 57), summarise why the author believes
that failure is important.
Write your summary in no more than 120 words, not counting the opening words which
are printed below. Use your own words as far as possible.
Failure holds immense importance for human beings because ......
..
..
..
..
..
..
..
..
..
..
..
..
..
..
..
.. [8]

[Turn over
trendyline

For
Examiners
Use

12

Costica Bradatan argues strongly in favour of failure. How applicable are the authors
observations to you and your society? Illustrate your answer by referring to the ways in
which you and your society regard failure.
..
..
..
..
..
..
..
..
..
..
..
..
..
..
..
..
..
..
..
..
..
..
..
..
..
[Turn over
trendyline

For
Examiners
Use

..
..
..
..
..
..
..
..
..
..
..
..
..
..
..
..
..
..
..
..
..
..
..
..
..
..
..
... [10]
trendyline

For
Examiners
Use

TEMASEK JUNIOR COLLEGE


PRELIMINARY EXAMINATION 2015

GENERAL PAPER

8807/02

Paper 2

27 August 2015

INSERT

1 hour 30 minutes

READ THESE INSTRUCTIONS FIRST


This Insert contains the passage for Paper 2.

This document consists of 4 printed pages.


[Turn over

trendyline

Costica Bradatan writes about the importance of failure.

We are firmly in an era of accelerated progress. We are witness to advancements in


science, the arts, technology, medicine and nearly all forms of human achievement at a
rate never seen before. We know more about the workings of the human brain and of
distant galaxies than our ancestors could have imagined. The design of a superior kind of
human being healthier, stronger, smarter, more handsome, more enduring seems to
be in the works. Even immortality may now appear feasible, a possible outcome of better
and better biological engineering.
Certainly the promise of continual human progress and improvement is alluring. But there
is a danger there, too that in this more perfect future, failure will become obsolete. Why
should we care? And more specifically, why should philosophy care about failure?
Doesnt it have better things to do? The answer is simple: Philosophy is in the best
position to address failure because it knows it intimately. The history of Western
philosophy at least is nothing but a long succession of failures, if productive and
fascinating ones. Any major philosopher typically asserts herself by addressing the
failures, errors, fallacies or naiveties of other philosophers, only to be, in turn,
dismissed by others as yet another failure. Every new philosophical generation takes it as
its duty to point out the failures of the previous one. It is as though, no matter what it
does, philosophy is doomed to fail. Yet from failure to failure, it has thrived over the
centuries. As Emmanuel Levinas memorably put it in an interview with Irish philosopher
Richard Kearney, the best thing about philosophy is that it fails. Failure, it seems, is
what philosophy feeds on, what keeps it alive. As it were, philosophy succeeds only
insofar as it fails.
So, allow me to make a case for the importance of failure. Failure allows us to see our
existence in its naked condition. Whenever it occurs, failure reveals just how close our
existence is to its opposite. Out of our survival instinct, or plain sightlessness, we tend to
see the world as a solid, reliable, even indestructible place. And we find it extremely
difficult to conceive of that world existing without us. It is entirely impossible for a thinking
being to think of its own non-existence, of the termination of its thinking and life,
observed German writer Johann Wolfgang von Goethe. Self-deceived as we are, we
forget how close to not being we always are. The failure of a plane engine could be more
than enough to put an end to everything. Even a falling rock or a cars faulty brakes can
do the job. And while it may not always be fatal, failure does carry a certain degree of
existential threat.
Failure is the sudden explosion of nothingness into the midst of existence. To experience
failure is to start seeing the cracks in the fabric of being, and that is precisely the moment
when, properly digested, failure turns out to be a blessing in disguise. For it is this lurking,
constant threat that should make us aware of the extraordinariness of our being: the
miracle that we exist at all when there is no reason that we should. Knowing that gives us
some dignity.
In this role, failure also possesses a distinct therapeutic function. Most of us (the most
self-aware or enlightened excepted) suffer chronically from a poor adjustment to
existence. We compulsively fancy ourselves much more important than we are and
behave as though the world exists only for our sake. In our worst moments, we place
ourselves as being like infants at the centre of everything and expect the rest of the
universe to be always at our service. We insatiably devour other species, denude the
planet of life and fill it with trash. Failure could be a panacea against such arrogance and
hubris, as it often brings humility.
[Turn over
trendyline

10

15

20

25

30

35

40

45

10

Our capacity to fail is essential to what we are. We need to preserve, cultivate, even
treasure this capacity. It is crucial that we remain fundamentally imperfect, incomplete,
erring creatures. In other words, that there is always a gap left between what we are and
what we can be. Whatever human accomplishments there have been in history, they
have been possible precisely because of this empty space. It is within this interval that
people and individuals, as well as communities, can accomplish anything. Not that we
have turned suddenly into something better; we remain the same weak, faulty material.
But the spectacle of our shortcomings can be so unbearable that sometimes it shames us
into doing a little good. Ironically, it is the struggle with our own failings that may bring out
the best in us.
The gap between what we are and what we can be is also the space in which utopias are
conceived. Utopian literature, at its best, may document in detail our struggle with
personal and societal failure. While often constructed in worlds of excess and abundance,
utopias are a reaction to the deficits and precariousness of existence. They are the best
expression of what we lack most. Sir Thomas Mores book Utopia is not so much about
some imaginary island, but about the England of his time. Utopias may look like
celebrations of human perfection, but they are just spectacular admissions of failure,
imperfection and embarrassment.
And yet, it is crucial that we keep dreaming and weaving utopias. If it were not for some
dreamers, we would live in a much uglier world today. But above all, without dreams and
utopias we would dry out as a species. Suppose one day, science solves all our
problems: We will be perfectly healthy, live indefinitely, and our brains, thanks to some
enhancement, will work like computers. On that day, we may be something very
interesting, but I am not sure we will have what to live for. We will be virtually perfect and
essentially dead. Ultimately, our capacity to fail makes us what we are. Our being
essentially failing creatures lies at the root of any aspiration. Failure, fear of it and learning
how to avoid it in the future are all part of a process through which the shape and destiny
of humanity are decided. That is why, as I had hinted earlier, the capacity to fail is
something that we should absolutely preserve, no matter what the professional optimists
may say. Such a thing is worth treasuring, even more so than artistic masterpieces,
monuments or other accomplishments. For, in a sense, the capacity to fail is much more
important than any individual human achievement: It is that which makes them possible.
We are designed to fail. No matter how successful our lives turn out to be, how smart,
industrious or diligent we are, the same end awaits us all: biological failure. The
existential threat of that failure has been with us all along, though in order to survive in a
state of relative contentment, most of us have pretended not to see it. Our pretence,
however, has never stopped us from moving toward our destination; faster and faster, in
inverse ratio to the square of the distance from death, as Tolstoys Ivan Ilyich expertly
describes the process. Yet Tolstoys character is not of much help here. The more
essential question is rather how to approach the grand failure, how to face it and embrace
it and own it something poor Ivan fails to do.
A better model may be Ingmar Bergmans Antonius Block, from the film The Seventh
Seal. A knight returning from the Crusades and plunged into a crisis of faith, Block is
faced with grand failure in the form of a man. He does not hesitate to engage Death headon. He does not flee, does not beg for mercy he just challenges him to a game of chess.
Needless to say, he cannot succeed in such a game no one can but victory is not the
point. You play against the grand, final failure not to win, but to learn how to fail.

[Turn over
trendyline

50

55

60

65

70

75

80

85

90

11

Bergman the philosopher teaches us a great lesson here. We will all end in failure, but
that is not the most important thing. What really matters is how we fail and what we gain
in the process. During the brief time of his game with Death, Antonius Block must have
experienced more than he had done all his life. Without that game he would have lived
for nothing. In the end, of course, he loses, but accomplishes something rare. He not only
turns failure into an art, but manages to make the art of failing an intimate part of the art of
living.

trendyline

95

100

2015 Preliminary Examination


GENERAL PAPER
Paper 2 Suggested Answers
1. Why is the promise of continual human progress and improvement alluring
(line 8)? Use your own words as far as possible. [2]
Lifted Answer

Suggested rephrase
It is alluring because human beings are
making breakthroughs in knowledge /
understanding our world at an
unprecedented pace

We are witness to advancements and


nearly all forms of human achievement
at a rate never seen before. (l.1-3)

The design of a superior kind of


human being healthier, stronger,
smarter, more handsome, more
enduring seems to be in the works.
(l.45)
Even immortality may now appear
feasible (l.6)

likely to endow ourselves with/ acquire


better/desirable physical traits

experiencing greatly increased life-span/


life expectancy

**any 2 of the 3 points


2. What is the authors intention in asking a series of questions in lines 911? [1]

Lifted Answer
Why should we care? And more
specifically, why should philosophy
care about failure? Doesnt it have
better things to do? (l.1011)

Suggested rephrase
The author asks a series of questions
to stimulate further interest [1] OR
to provoke the reader to think more
deeply about the topic [1] OR
to introduce the role of philosophy in an
interesting manner [1] OR
to pre-empt questions that readers might
have [1] OR
to highlight the importance of failure/
lead the reader to realise that
eradicating failure is not all beneficial [1]

3. Why are the words failures, errors, fallacies and naiveties (line 15) in
inverted commas? [1]

Lifted Answer
Any major philosopher typically asserts
herself by addressing the failures,
errors, fallacies or naiveties of
other philosophers, only to be, in turn,
dismissed by others as yet another
failure. (l.1315)

Suggested rephrase
The words are in inverted commas
to suggest that the opposite is true [1]
OR
to show that they are not really failures
and errors [1] OR
to point out that it is incorrect/inaccurate
to label them as such [1] OR

Get PSJ Private Tutor to Guide you through Exams Now!


Contact www.privatetutor.com.sg

trendyline

because the author questions the


meaning/ usage of these words [1] OR
not everyone may agree on the meaning
of these words/ the meaning of these
words are subjective [1]

4. What is the author suggesting about the nature of philosophy in the last
sentence of paragraph 2? [1]

Lifted Answer
As it were, philosophy succeeds only
insofar as it fails. (l.22)

Suggested rephrase
The author is suggesting that the nature
of philosophy is paradoxical / conflicting/
contradictory. [1]

**ironic = 0 (nature of something cannot be


ironic)

5. Explain what the author means by saying that failure reveals just how close our
existence is to its opposite (lines 2425). Use your own words as far as possible.
[3]

Lifted Answer
Out of our survival instinct, or plain
sightlessness, we tend to see the
world as a solid, reliable, even
indestructible place. (l.2527)

Suggested rephrase
We are inclined/ predisposed to regard
the world as a secure/ stable/
everlasting/ enduring/ unshakeable
place/ think that nothing can cause the
world to become unsafe/ damaged. [1]

Failure allows us to see our existence


in its naked condition.
(l.2324)

However, failure enables us to view the


world/ reality in its undisguised/ true
state/ situation. [1]

The failure could be more than


enough to put an end to everything
(l.3031). while it may not always be
fatal, failure does carry a certain
degree of existential threat (l. 3233)

and failure can result in danger/ risk to


our lives// the elimination of/ wipe out/
humans. [1]

**Note: Award 1 mark for each of the above


points. The points are independent of each
other

6. Why does the author use the phrase the most self-aware or enlightened
excepted (lines 4041)? [1]

Lifted Answer
Most of us (the most self-aware or
enlightened excepted) (l.4041)

Suggested rephrase
The author uses the phrase
to establish rapport with the readers/ to
make the readers think he is one of
them. [1] OR

trendyline

so as not to offend readers who regard


themselves as among the most selfaware or enlightened/who believe that
they do not need the therapeutic
function of failure. [1] OR
to show that there are people who do
not think they suffer chronically from a
poor adjustment to existence. [1] OR
to avoid making an absolute statement/
qualify his claim about peoples poor
adjustment to existence. [1]

7. Explain the authors use of the word may in line 6364). [2]

Lifted Answer
Utopias may look like celebrations of
human perfection, but they are just
spectacular admissions of failure,
imperfection and embarrassment
(l.6365)

Suggested rephrase
Utopias only appear/seem to be
triumphs of human ideals [1]
but expose/highlight the flaws/ fiascos/
shortcomings and awkwardness/
discomfort/ uneasiness of current
existence/ in reality/ the real world/
current society [1]

** Note: To get second mark, answers


MUST contrast with current reality

8. Why does the author argue that we will be virtually perfect and essentially
dead (lines 7172) if science solves all our problems? Use your own words as
far as possible. [2]

Lifted Answer
we will be perfectly healthy, live
indefinitely, and our brains will
work like computers (l.6970)
but I am not sure what we will have to
live for (l.71)
our being essentially failing creatures
lies at the root of any aspiration (l.73
74).

Suggested rephrase
This is because human beings will be
faultless/ unblemished/ flawless/ have
the best attributes when science solves
all our problems. [1]
(**perfect =0 lifted)
However, this will cause us to lose the
motivation to live/ to have nothing to
strive for/ no goals/ ambition to work
towards. [1]
** Note: Must make explicit link to virtually
perfect and essentially dead

trendyline

9. What does biological failure (line 81) refer to and why have we pretended not to
see it (line 83)? Use your own words as far as possible. [3]

Lifted Answer
No matter how successful our lives turn
out to be, how smart, industrious or
diligent we are, the same end awaits
us all: biological failure. (l.81)
The existential threat of that failure
has been with us all along, though in
order to survive in a state of relative
contentment, most of us have
pretended not to see it. (l.8183)

Suggested rephrase
The author is referring to death [1]

and we ignore/avoid
confronting/discussing the fact that it is
ever present/always there [1]

to remain seemingly happy and


satisfied/ so that we can live in
relative/apparent bliss. [1]

10. What is the authors intention in comparing Tolstoys Ivan in paragraph 9


with Bergmans Block in paragraph 10? [1]

Lifted Answer
Suggested rephrase
The
authors
intention is
how to approach the grand failure, how
to face it and embrace it and own it
to contrast their differing attitudes
something poor Ivan fails to do (l.87
towards death.
88).
OR
Block is faced with grand failure does to highlight importance of embracing
not hesitate to engage Death head-on
death like Bermans Block instead of
does not flee, does not beg for mercy
trying to avoid it/ fleeing from it like
(l.9092).
Tolstoys Ivan.
** Note: Answers MUST show
comprehension that grand failure refers to
death learning from failure etc = ZERO

11. Using material from paragraphs 4 to 6 (lines 34 to 57), summarise why the
author believes that failure is important.
Write your summary in no more than 120 words, not counting the opening
words which are printed below. Use your own words as far as possible. [8]
Failure holds immense importance for human beings because
From the passage
Suggested rephrase
A1 when it happens, there is an abrupt/
Failure is the sudden explosion of
nothingness into the midst of
unexpected sense of emptiness/ void
existence (l. 34)
in our lives and
A2 we realise that life is/ humans are not
start seeing cracks in the fabric of
being (l. 35)
perfect.
A3 When we have come to terms with
that is precisely the moment, when
properly digested (l. 3536)
this// assimilated/ absorbed/
processed this fact,

trendyline

From the passage


failure turns out to be a blessing in
disguise (l. 3637)
For it is this lurking, constant threat
(l. 37)
that should make us aware of the
extraordinariness of our being (l.
3738)
the miracle that we exist at all
when there is no reason that we
should (l. 3839)

A4

A5

A6

A7

Knowing that gives us some


dignity (l. 39)

A8

failure also possesses a distinct


therapeutic function (l. 40)
Most of us suffer chronically
from a poor adjustment to
existence (l. 4142)
we compulsively fancy ourselves
much more important than we are
(l. 42)

B1

B2

B3

behave as though the world exists


only for our sake (l. 43)
in our worst moments, we place
ourselves as being like infants at the
centre of everything (l. 4344)
expect rest of the universe to be
always at our service (l. 4445)
We insatiably devour other
species, denude the planet of life
and fill it with trash (l. 4546)

B4

B5

Failure could be a panacea against


such arrogance and hubris (l. 46
47)
as it often brings humility (l. 47)
Our capacity to fail is essential to
what we are (l. 48)

B6

C1

It is crucial that we remain


fundamentally imperfect,
incomplete, erring creatures that
there is always a gap left between
what we are and what we can be (l.
4951)
Whatever human accomplishments
there have been in history, they have

C2

C3

Suggested rephrase
the misfortune of failure becomes
something good/beneficial
The persistent/ continual danger of
failure
makes us appreciate/treasure our
existence/ realise how remarkable/
unusual our existence is
and that we are alive despite the
circumstances/ adverse
circumstances/ circumstances
indicating that it should have been
otherwise.
This knowledge/ understanding gives
us self-worth/ self-esteem/ selfrespect/pride.
Failure has clear/ evident healing/
curative properties,
(inferred) enabling us to come to
terms with reality/ life and
(inferred) stop having a false
perception of self-worth/ an inflated
ego// we realise that we are not as
great as we think.
(inferred) Failure also makes us less
self-centred/ selfish/ egocentric/more
considerate to others/ learn to coexist with others

(inferred) and care more about our


environment/ be more
environmentally conscious/ be less
environmentally destructive.
Failure makes us less prideful/
conceited// reduces our sense of selfimportance.
[Note: humble = LIFT]
Our ability to fail is vital/ important/
indispensable to our existence/ lives/
makes us human.
(inferred) and for us to constantly
strive for improvement/ to do better.

Human achievements/ successes


were due to our quest to overcome

trendyline

From the passage


been possible precisely because of
this empty space (l. 5152)
It is within this interval that people
and individuals, as well as
communities, can accomplish
anything (l. 5354)
the spectacle of our shortcomings
can be so unbearable that (l. 55)
it shames us into doing a little
good (l. 5556)
it is the struggle with our own
failings that may bring out the best
in us (l. 5657)

Total = 20 points
Mark scheme
12 points 1 mark
3 points 2 marks
45 points 3 marks

Suggested rephrase
failure/ better our performance.
[Note: focus must be on closing the
gap and this point is about society or
humanity in general]
C4

C5
C6

The display/ manifestation/


demonstration of our faults/
deficiencies/ weaknesses can be so
intolerable/ agonising
that it makes us embarrassed enough
to do something useful
and this enables us to show/ exhibit
our finest/ greatest qualities.
** Note: C4-C5-C6 build on one another
and apply more to individuals

6 points 4 marks
78 points 5 marks
9 points 6 marks

1011 points 7 marks


12 points 8 marks

12. Costica Bradatan argues strongly in favour of failure. How applicable are the
authors observations to you and your society? Illustrate your answer by
referring to the ways in which you and your society regard failure. [10]
OBSERVATION 1
The observation that Failure allows us to see our existence in its naked condition.
Whenever it occurs, failure reveals just how close our existence is to its opposite.
(lines 23-25) OR failure does carry a certain degree of existential threat. (line 33)
is very applicable to the Singaporean society. Past failures to keep our society safe
forced us confront the precariousness of our lives. When we fail to defend our country, our
survival as a people comes under threat. Even when facing various crises and
emergencies and in times of disaster, we realize our vulnerability and the fragility of our
lives.
Singapores vulnerability during WW2 was due to failure of security/military defense
resulted in the Japanese Occupation. In addition, the MacDonald House bombing
from the Konfrontasi era further reinforces the authors observation.
Such costly lessons are not easily forgotten and post independent Singapore has
implemented various policies to better safeguard ourselves against such failures.
NS and SAF(3G SAF) - Singapores defense budget will increase by 5.7% in
nominal terms to $9.5b for fiscal year (FY) 2015 ... IHS Janes Defence Weekly
reports that defense expenditure will increase as a proportion of GDP for the first
time since 2009, rising from 3.2% of GDP in 2014 to 3.3% for 2015
Importance of National Education and Total Defence Military, Civil, Economic,
Social & Psychological
Singapore is now better prepared and reacts quickly to terrorist threats (e.g.
prevention of JI threat of bombing after 9/11; two youths arrested for links to ISIS)

trendyline

Singaporeans vulnerability to threats and dangers. These accidents and disasters


show just how close our existence is hanging by a thread at times.
Eg. MI185 crash in Palembang, Indonesia in 1997 (human error/failure)
Eg. SQ006 accident in Taipei, Taiwan in 2000 (human error/failure)
Eg. hijacking of SQ117 in 1991(failure/security lapse at point of embarkation
Kuala Lumpur by 4 Pakistan Peoples Party members. But Singapores
Commandos mounted a successful rescue operation and killed all terrorists)
Eg. SARS (initial failure/difficulties in managing the spread but quick action by govt
and healthcare sector contained the epidemic) and dengue epidemics (failure of
people in keeping a clean environment)
Eg. sudden deaths of avid athletes/marathoners eg. Thaddeus Cheong (failure in
providing immediate aid in some cases has pushed organizers to step up on-site
first aid care and to require participants to declare health status pre-event)

OBSERVATION 2
The author argues that To experience failure is to start seeing cracks in the fabric of
being, and that is the moment when, properly digested, failure turns out to be a
blessing in disguise. OR The possibility of failure should make us aware of the
extraordinariness of our being: the miracle that we exist at all when there is no
reason that we should. Knowing that gives us some dignity. (lines 34-39).
This observation is definitely applicable and relatable to the Singapore narrative
because our nation was born of failure, and did experience failure in its infancy. This
makes our current success even more amazing to behold.
Independence was thrust upon us because of the failed merger with Malaysia. LKY
famously described the split as a moment of anguish, and his grief is immortalized in
the video recording of his interview with members of the press after the split was
announced. He, and many of the PAP (including Dr Toh Chin Chye, S. Rajaratnam
and Mr. Ong Pang Boon), felt that it was a disaster and that separation was not the
right thing to do.
Not only was our independence a result of failure, we still experienced strained race
relations, and the failure to live harmoniously together resulted in race riots in the late
1960s - early 70s after independence.
The knowledge of our uncertain and tumultuous beginnings make for an even more
striking contrast with Singapores considerable economic growth and development
today, as well as her racial harmony. It is something that all Singaporeans can be
proud of.
However, it may not be as applicable in modern day Singapore where anything
other than the absolute best is considered a failure, and the cost of failure is seen
to be high, the perceived enormity of failure may be may be too indigestible
(unacceptable) for people to bounce back from, contributing to the increase in
stress and anxiety related mental issues, and to high suicide rates. There is no
dignity in failure, only shame, and the desire to escape it.
901 people were arrested for attempting suicide in 2014, and the suicide rate has
fluctuated over the years in the 400 range (415 in 2014)
The most recently publicized case of suicide involved a straight-As student committing
suicide because of two B grades at O levels (The New Paper, July 27 2015: Straight A
student commits suicide over O-level results, mum takes her own life months later).
Her perception of herself as a failure because of B grades, her inability to accept less

trendyline

than stellar results, led her to find a permanent way out of living instead of deriving
greater appreciation of life, developing and growing from failure.
Nevertheless, there are positive signs that at least some Singaporeans are starting
recognize our lack of resilience in coping with failure, and our unhealthy and
extreme definition of failure, and are trying to be more change this. More and more
we see greater emphasis on multiple pathways and a more varied definition of what
it means to succeed. We can see this observation is at least becoming applicable, if
only for small groups of individuals.
E.g. The Yellow Ribbon Project success stories: There are many ex-convicts whom by
societys definition have failed in life, who have nevertheless managed to learn from
their past mistakes, mend their ways, and in their own way assimilate back into society
and become successes. Society has come to celebrate their success stories, which
are made all the more significant because of them having failed before.
Eighteen Chefs Benny Se Teo is an ex-convict, who struggled with drug addiction
(and failed) many times in his youth. His first few business ventures after his
rehabilitation and freedom also failed, but he kept going. He now owns a franchise and
is committed to hiring ex-offenders and youths-at-risk. http://goo.gl/3G9lpu
Glenn Lim, former drug addict and wayward youth, now successful motivational
speaker, youth counselor, organizational consultant and curriculum developer. A
champion of social enterprises
Adam Khoo did poorly at PSLE and ended up in neighbourhood school where he
continued to fail by societys standards. He was infamous for misbehavior, poor results
and addiction to arcade games. However, he reflected on his life and managed to
change its progression. He topped NUS and became self-made millionaire at 26, a
best-selling author (I Am Gifted, So Are You) and corporate trainer.
Other failures who bounced back from setbacks to inspire others: eg. Aishah Samad
(2003 SEA Games bronze medallist had all 4 limbs amputated due to severe bacterial
infection); Dr William Tan (contracted polio at 2, paralyzed from waist down, survived
Stage 4 chronic lymphocytic leukaemia); Jason Chee (navy serviceman who lost both
legs and left arm after a ship accident)
These individuals accepted the failure of their bodies and managed to succeed in
carving out a definition of success for themselves.
Comments
Scope for personal anecdote about overcoming failure and lessons learnt
GCE 2012 Paper 2 Report
Many dismissed the authors views as unscientific and certainly not to be found in
Singapore, whilst others offered powerful personal testament to its effectiveness,
such as the candidate who recounted how favourite music was played continually
to his grandmother who was suffering with Alzheimers and she eventually
remembered her grandchildrens names.
This illustrates another fruitful
approach to the application question an apt personal anecdote can add a
different dimension to what can so often be an impersonal, unengaged
response.

OBSERVATION 3
The authors observation that Failure could be a panacea against arrogance and
hubris, as it often brings humility. (lines 46-47) may not be entirely applicable to the
Singaporean context. Failure only brings about humility if people are willing to admit

trendyline

mistakes and change for the better. If people are obstinate, they may dig in their heels and
refuse to acknowledge mistakes, or try to shift the blame to others.
Examples - Singapore companies or government bodies refusing to acknowledge the
severity of problems/issues/mistakes through the use of euphemisms:
2011 Flooding -> Initially described by PUB as ponding
Current train repair works -> urgent / unscheduled maintenance
Problems in the new DBSS projects, new condominium projects Failure to deliver
quality products and no one is taking responsibility
2012 SMRT Bus Driver strike -> on the first day of strike, initially described by all
major local newspapers as wage dispute, protest, action, and (the icing on the
cake) episode.
2014 Workplace Happiness Survey -> under happy (in between happy and
unhappy)
Gushcloud Singtel fiasco failure to adhere to ethical standards during the
promotion of one of Singtels services. Gushcloud had its influencers launch a
smear campaign against their competitors StarHub and M1. When found out, both
parties tried to shift the blame to each other in the beginning before proof came out
that a Singtel employee had told Gushcloud to badmouth its competitors in the
advertising for the service.
WPs failure in the management of Aljunied Town Councils funds/lapses in
financial protocols no humility was shown when the issue came to light. Instead,
the WP was very defensive, trying to minimize blame, and circumventing issue in
parliament.
The use of euphemisms may suggest that instead of admitting failure or
acknowledging the severity of situations, there is a tendency to downplay them in order
to minimise backlash and criticism, which in no way serves as a panacea against
arrogance and hubris or brings humility.

While some higher-ups have acknowledged the inappropriate use of some of these
terms (e.g. Minister of the Environment and Water Resources Vivian Balakrishnan
criticised PUBs use of the term ponding to describe the 2011 flooding), the fact that
the euphemism was even used in the first place already suggests a particular
unremorseful, defensive attitude.

While the train operators always do apologise for the inconvenience to commuters
whenever the trains break down, the continued use of urgent maintenance /
unscheduled maintenance may suggest the admission of failure is mere lip service;
more a move to placate Singaporeans ire, rather than a genuine expression of
remorse.
Also, the consequences of such failures are usually too light a slap on the wrist to
motivate the companies involved to improve. Thus, there is no humility, and
arrogance/hubris is allowed to continue existing. E.g. Telco outages and blame on
technical failure, no compensation for consumers. The fine they have to pay is small
change. MRT breakdowns the same as well as well penalties are usually peanuts.
After failures, no clear mending of ways is seen.
Examples
BreadTalks soya bean fiasco. Failure of the company to be honest. Videos and
complaints went viral and company had to apologize and tell the truth.
Escape of Mas Selamat. Failure of the security forces to keep watch. Failure of
persons-in-charge to take responsibility. Failure of the govt to provide explanation.
Increase in foreign worker deaths and exploitation of foreign workers. Failure of

trendyline

companies/employers to look after the welfare of employees. Some employers


have been named and shamed for ignoring the sick, abandoning the ill and
providing poor living conditions. Govt has stepped up efforts to ensure that
employers provide a safe working environment and clean living quarters for their
workers.
OBSERVATION 4
The author argues that there is always a gap left between what we are and what we
can be. It is within this interval that people and individuals, as well as
communities, can accomplish anything. (lines 50-53) OR The capacity to fail is
what makes [achievement] possible. (lines 78-79). This may be applicable to only
some aspects Singapore, as seen from the huge contrast between what the country was
like at independence in 1965 and our varied achievements now, half a century later. We
were spurred on to work even harder to achieve success after our failed merger, growing
from strength to strength.
The lyrics to this years national day song-writing competition winning entry These are
the Days aptly reflect the sentiments of this observation: there was a time When all
we could believe was all we had to give, but Now what we only dreamed is what we
live it draws attention to the stark difference between all we did not have in the past,
and how we have closed the gap between our aspirations and our desired reality.
This is also evident in the great progress the Malay community in Singapore has made
in terms of achieving an equal standard of living. Historically, there has been a gap
between livings standards of Malays and other races, indicating a failure to provide
equal standards of living for all citizens. The government has worked hard to close the
gap by implementing various policies e.g. free education for Malays as well as self help
groups like MENDAKE and MUIS.
This year, in Straits Times, Minister-in-charge of Muslim Affairs Yaacob Ibrahim says
that Malays have done well for themselves http://goo.gl/Sijzel
Mr. Masagos Zulkifli, on being promoted from Senior Minister of State for Home Affairs
and Foreign Affairs to full Minister, said:
It is good to see more and more Malays doing very well in education, doing very
well in all fields of their professions and even in Government http://goo.gl/R8VoZP
The Association of Muslim Professionals (AMP) commissioned a perception study in
2011 on the Malay/Muslim community.
The survey found that the community was satisfied with the direction that the
country is taking and was optimistic of the progress made by local Malay/Muslims
over the last five years. 76% indicated confidence in the communitys prospects
over the next decade (2011 2021)
There were also indications that there were some areas to work on (e.g. many
Malays felt there was still a gap when it came to cost of living, competition for
employment
against
foreigners,
compared
to
other
Singaporeans) http://goo.gl/hqyqhL
However, this approach to failure has taken on an unhealthy nature in modern
Singapore as a whole, where our competitive nature, and our culture of comparing
with others and keeping up with appearance (the Chinese notion of saving face)
has led to many endeavours devolving into a rat race jobs/careers, education, the
pursuit of material indicators of financial success
This leaves little room to accommodate any failure, because no one dares to even
allow a gap to be created in order to be filled. Continuous, sustained success has
become the norm, as citizens are trapped in a never-ending cycle of one-upmanship.

trendyline

In fact, a favourite descriptor of Singaporeans is the Hokkien/Singlish word kiasu,


which literally translates as scared to lose out (to others) (i.e. fail).
Our fear of failure and our unwillingness to appear to have failed can be seen in
various aspects of our life
E.g. The pursuit of the 5Cs (Cash, Car, Condo, Credit Card, Country Club) to signal
wealth and power, and to appear successful.
The growing number of young couples (with little spending power) splurging on
weddings and incurring debt, which can sometimes amount to hundreds of
thousands of dollars in order to appear successful.
Education
All schools chasing awards/achievements in order to appear to be good schools
SYF distinction so common it is now the norm, the desire to succeed and the fear
of failing to achieve have driven performing arts groups to put in ever more effort to
achieve distinction.
Billion dollar tuition industry that support the education system thrives on parents
willingness to invest/spend in/on their children in order to avert failure in schools,
and to get into branded schools.
The 2014 Household Expenditure Survey found that families spent $1.1 billion a
year on tuition - almost double the $650 million spent a decade ago and a third
more than the $820 million spent just five years ago.
The Department of Statistics, which polled more than 11,000 households between
October 2012 and September last year, released the latest survey in September.
The average household spending on tuition rose from $54.70 a month 10 years ago,
to $79.90 in the latest survey.
Our Singapore Success Story, while born of failure, may also ironically have
contributed to low tolerance of failure in our paternalistic government, who feels the
pressure of maintaining its enviable track records. Thus, failure for Singapore is
largely perceived to be not an option (siege mentality), and the government stresses
this fact.
The price of failure may be too high to accept. We are constantly reminded that
Singapore is vulnerable and cannot afford to make a wrong move.
Bilahari Kausikan, Singapores ambassador-at-large:
clear common understandings of our circumstances, chief of which is the
inherent irrelevance of small states in the international system and hence the
constant imperative of creating relevance for ourselves by pursuing extraordinary
excellence http://goo.gl/3G9lpu
Other than our national reserves, Singapore has no hinterland or other assets to fall
back on if we fall behind economically.
As of 31 March 2014, the Official Foreign Reserves managed by MAS was S$343
billion and the size of Temaseks portfolio was S$223 billion.
It is the size of the Governments funds managed by GIC that are not published.
What has been revealed is that GIC manages well over US$100 billion.
Further, our reserves are a strategic asset, and especially for a small country with
no natural resources or other assets. They are a key defence for Singapore in
times of crisis, and it will be unwise to reveal the full and exact resources at our
disposal.

Nevertheless, although many Singaporeans do shy away from failure, the


Government is perhaps slowly recognizing that failure is inevitable, and is trying to

trendyline

change Singaporeans perceptions of failure, to accept it and to be resilient enough


to get back up and close the gap between failure and aspiration after a fall.
This year, the government has released many YouTube videos featuring Singaporeans
who demonstrate resilience, getting back up after a fall so the gap between failure
and success is demonstrated, and it documents how some Singaporeans have
succeeded in closing that gap.
E.g. Eddies Singapore Spirit Jun Yuans Singapore Spirit YouTube videos used
during the Lego lesson guidance period. Jun Yuan opened a Bak Kut Teh stall, but had
to close down due to manpower issues. Instead of giving up after the failure of his
business, he bounced back and became an app developer, developing an app (11th
Hour) to reduce inefficiency and food wastage in the F&B industry.
OBSERVATION 5
The author is advocating that we ought to face biological failure (line 88), that is
death, head-on, so that we can accept this inevitable end and as a result, take better
ownership of our lives and learn to lead our lives in a happier and more blissful
manner, instead of being bitter and resigned to our fate. The author believes that it is
only when we can accept this inevitable failure that we can better cherish our lives and
live more meaningfully. This observation is not so applicable to the Singaporean
society.
Singapore is a conservative society and death has always been a taboo subject. The
issue of death often brings about much doom and gloom, and thus most people avoid
discussing it so that they do not need to face their ultimate fear: the fear of dying the
finality of their existence.
Eg. Hungry Ghost Festival much superstition about death, life after death and ghouls
become even more apparent during the Lunar 7th month when people believe the
spirits from the underworld roam. The superstitious avoid swimming and other more
risky activities for fear of allowing the spirits to claim ones life/soul
Eg. due to discomfort with confronting death and the dying, many people protest
against hospices built near their homes
Eg. prospective residents of an upcoming Build-To-Order residential project in
Sengkang were up in arms regarding the building of a Chinese temple with a
columbarium next to their estate
Eg. Exit International, the pro-euthanasia group from Australia, has tried and failed on
several occasions to seek permission from the government to engage Singaporeans
on issues of dying with dignity. It seems that our society is not ready to openly discuss
such issues.
However, with the rapidly ageing population, more people are now more willing to
talk about death and how to face it / prepare for it and being aware of the choices
they can make. While the stigma surrounding death still remains in Singapore, this
observation is SLOWLY becoming more applicable to our society. Even the
government has recognized the need for more open discussions on end-of-life
matters. People are gradually more open to talk about such issues and to dispel the
stigma surrounding the subject matter of death.
More publicity and awareness of the Advance Medical Directive Act (passed in 1996)
in Singapore. More and more people are now making an informed decision to sign up,
being aware of the document and the conditions, and thinking about ethical and legal
issues surrounding end-of-life care for terminal illnesses.
More people are also aware of other legalities with regards to appointing a Lasting
Power of Attorney (LPA).

trendyline

The scandal involving Mdm Chung Khin Chun who was cheated of her money by
former Chinese tour guide Yang Yin, whom she appointed her legal guardian and gave
LPA, has created awareness of the danger of not being adequately protected and
prepared for old age when ones physical and mental faculties are failing.

trendyline

TAMPINES JUNIOR COLLEGE

H1

JC2 PRELIMINARY EXAMINATION

CANDIDATES
NAME
CIVICS GROUP

1 4

TUTOR

GENERAL PAPER

8807/ 02

Paper 2

28 Aug 2015
1 hour 30 minutes

Candidates answer on the Question Paper


Additional Materials: 1 Insert

READ THESE INSTRUCTIONS FIRST


Write in dark blue or black pen on both sides of the paper.
Do not use staples, paper clips, highlighters, glue or correction fluid.

TOTAL:

Answer all questions.


The Insert contains the passage for comprehension.
Note that up to 15 marks out of 50 will be awarded for your use of language.

/50

At the end of the examination, fasten all your work securely together.
The number of marks is given in brackets [ ] at the end of each question or part question.
For Examiners Use
Qn 1

Qn 7

MARKER 1

Qn 2

Qn 8

MARKER 2

Qn 3

Qn 9

MARKER 3

Qn 4

Qn 10

MARKER 4

Qn 5

Qn 11

MARKER 5

Qn 6

Sub-total

/ 35

AVE LANG MARK

.. / 15

This document consists of 7 printed pages


trendyline

Read the passage in the Insert and then answer all the questions which follow. Note that
up to 15 marks will be given for the quality and accuracy of your use of English throughout
this paper.

For
Examiners
Use

NOTE: When a question asks for an answer IN YOUR OWN WORDS AS FAR AS
POSSIBLE, and you select the appropriate material from the passage for your answer you
must still use your own words to express it. Little credit can be given to answers which only
copy words and phrases from the passage.

Give two reasons from paragraph 1 why the author says that Gorlitzer Park has a
seedy air (line 1). Use your own words as far as possible.
..
..
..
..[2]

What are the authors purposes in beginning his opening paragraph with the example of
Gorlitzer Park? Use your own words as far as possible.
..
..
..
..[2]

Explain the authors use of the word flock in line 10. Use your own words as far as
possible.

..
..
..
..[2]

trendyline

In lines 16-17, what claims about British youth are exaggerated and explain how these
are now ludicrous (line 18). Use your own words as far as possible.

For
Examiners
Use

..
..
..
..
..
...[3]

In paragraph 4, what contrasts does the author make between the youthful excesses
(line 28) of the past and the temperance (line 30) of youth today? Use your own
words as far as possible.
..
..
..
..[2]

trendyline

Using material from paragraphs 5 - 7 only, summarise the reasons for the change in the
behaviour of todays youth.

For
Examiners
Use

Write your summary in no more than 120 words, not counting the opening words
which are printed below. Use your own words as far as possible.
There are many reasons for the change in youth behaviour. Firstly..
..
..
..
..
..
..
..
..
..
..
..
..
..
..
..

..
........ [8]

trendyline

In paragraph 8, what explanations does the author offer for why better behaviour of the
young has not translated into greater happiness (lines 73-74)? Use your own words
as far as possible.

For
Examiners
Use

..
..
..

..
..[3]

What is the author implying by the phrase carefully crafted profiles in line 80?

.[1]

Explain the authors attitude towards the change in youthful tendencies and behaviour
from the phrase Perhaps there has been progress (line 82).
..
..[1]

10 Which phrase in the first paragraph echoes the authors point in the last paragraph
about Berliners not wanting to bring back the decadent past (line 84).
..
.[1]

trendyline

11 In this article, Michael Butler makes a number of observations about how youth in
todays Western world are no longer indulgent, aimless and rebellious. How far is this
true of you and the youth in your society?

For
Examiners
Use

..
..
..
..
..
..
..
..
..
..
..
..
..
..
..
..
..
..
..
..
..
..
..
.........
..

trendyline

..
..
..
..
..
..
..
..
..
..
..
..
..
..
.
..
..
..
...
..
..
..
..
..
..
..

[10]

trendyline

trendyline

GENERAL PAPER
TPJC Preliminary Examination
Paper 2 INSERT

8807/02
28 August 2015
1 hour 30 minutes

Michael Butler writes that todays young people are not the unhappy, alienated, violent
failures they are often made out to be...
1

Grlitzer Park has a seedy air. Its tall walls are covered in graffiti. Near the entrances,
young African men stand around hassling bystanders, asking if they want to buy some
kiffen. Yet in many respects, the drug park does not live up to its ugly reputation. On
a Saturday afternoon, it is mostly full of 20-somethings sitting around on the grass in
groups sipping coffee. Young parents pass by with pushchairs. University students on 5
picnic blankets peer into their textbooks. Over the course of an hour or so, not a single
one of the drug dealers in view seems to make a deal. For most of the locals, they are a
hassle not a service.

Few European cities do youth culture and hedonism better than Berlin. Young people
flock here from all over the world. The nightlife runs until dawn, techno beats flood its 10
streets. Yet as with Grlitzer Park, the wild appearance belies reality. The impression of
Berlin is that it is a city of youthful partying, exciting nightlife and heroin addicts. But that
city has disappeared. In its place is a city where the young come to study, work and
boost their creative careers, not just party.

Similar trends are seen across the Western world. In 2008, Time magazine described 15
Britains youth as unhappy, unloved and out of control; a nation gripped by an
epidemic of violence, crime and drunkenness was scared of its wayward youth. This
claim of British youth, hyperbolic then, has since become ludicrous. In the last ten years,
the teenage murder rate quietly plunged. Perhaps most remarkably, Britains notoriously
surly youths are getting more polite. According to one government survey, those born in 20
the early 1990s are less rude and noisy in public places than previous cohorts were at
the same age. In America, the proportion of high-school students reporting bingedrinking has fallen. Cigarette smoking among the young has become uncommon. The
use of other recreational drugs has fallen sharply. Teenage kicks of other sorts also
appear to be on the decline. Teens are waiting longer to have sex than they did and 25
Americas teenage pregnancy rate is half what it was two decades ago.

This sudden outbreak of restraint is doing havoc to businesses which thrive off youthful
excess. Kids these days just want to live in their own little worlds in their bedrooms
watching Youtube videos and becoming obese, fumes a barman in Leeds. The media,
too, are struggling to cope with the rising temperance of youth. Television stations aimed 30
at young people have dropped programmes that glamourise rebellion and high-living,
according to a media company which owns MTV. Kids seem to want to grow up to
make money, one father suggests, looking slightly unnerved.

What is behind this generation of hard-working, strait-laced kids? It is hard to pin down
any single explanation. Lots of interlinking factors contribute to this social trend. In many 35
countries, shock at the earlier extent of youthful hedonism and disorder has led to
dramatic attempts to curb it. In Britain, anti-social behaviour orders were widely used
against the rowdy. In Spain and Italy, drinking in the streets has been met with ferocious
policing in recent years. Australia has put heavy taxes on teen-friendly alcoholic
beverages. But such policies are hardly the whole story; there are much bigger trends. 40

trendyline

The generations known for rebellion and rule-breaking were large in comparison to the
populations of the time, thanks to the post-war baby boom and its echo boom in the
1970s and 1980s. They grew up in young societies. Todays youth by contrast are few in
number and are growing up in ever older societies. In addition, rising fair treatment of the
sexes is also a factor. Women no longer feel discriminated against and therefore it is no 45
longer necessary to prove their mettle by rebelling through being more addicted than
men in terms of drinking and drugs. As well as being more supportive of young women,
most Western societies are also less white than before. Although prejudice, and patterns
of policing, might lead people to think otherwise, surveys show that in most Western
50
countries people from minorities are less likely to drink or use hard drugs.

Added to these social trends are economic ones. The transfer of unskilled jobs to
developing countries and of menial jobs to immigrants has put a new premium on
education: todays rich-world youth has far more schooling than previous generations.
Young people who are studying rather than in paid employment have less money for
hedonism. That has not always, in the past, made universities particularly clean and
sober places. But todays students have more at stake than previous college kids.
American tuition fees have increased and now average $30,000 at a private university.
In many countries, an increasing number of these studentsand of young people in
generalstill live with their parents, who tend to keep careful watch on their spending
and living habits. Clubbing is now more of a luxury good than a way of life for British
teenagers. And it surely helps that there is plenty else to do: video games and social
networking indulged in bedrooms are far better entertainment than cheap cider and
cigarettes consumed at public places. Yet perhaps the best explanation for this youthful
self-control is the way parents bring these young adults up. A combination of
government initiatives, technology, social pressure and reaction against the follies of the
past has improved parenting dramatically. The amount of time parents devote to child
care has increased significantly. According to various studies of the millennial
generation, children born in the 1970s and 1980s were mostly raised by baby-boomer
parents who married young, had children quickly and were often rather blas about the
consequences. By the late 1980s that generation was giving way to a new group of
parents who waited longer to have children and paid more attention when they did.

55

60

65

70

What this adds up to is a generation that is more closely watched and less free to mess
up. Ironically however, better behaviour has not, as yet, translated into greater
happiness. For all their disavowal of inebriation and criminality, young people are still
proving more likely to be diagnosed with depression and anxiety. They are often 75
obsessed with their careers and rarely satisfied. In addition, several studies show that
people who use the internet more tend to be less happy. In helping people to stay
connected to their peers, social media sites such as Facebook let them compare
themselves to people who are doing better than they feel they are (or at least appear to
be, in their carefully crafted profiles). In addition, the young do not trust the institutions or 80
people they live with.

Perhaps there has been progress in the temperament of the youth. But does that make
them boring? For all that young Berliners joke about their city becoming sanitised and
lacking in character, they do not want to bring back the decadent past. They want to
85
build something better.

trendyline

trendyline

2015

Paper 2 (Answers)

GENERAL PAPER TPJC JC2

1. Give one reason from paragraph 1 why the author says that Gorlitzer Park has a seedy
air (Line 1). Use your own words as far as possible. ( 1m)
From passage

Paraphrased
EITHER

Its tall walls are covered in graffiti

Its faade /outer appearance/is vandalised/defaced/


sleazy (1m)

and drug peddlers ply their trade nearby (1m)


Near the entrances, young African men
stand around hassling bystanders , Note: doodles/drawings not accepted because does not
asking if they want to buy some capture seedy
kiffen( drugs)
2. What are the authors purposes in beginning his opening paragraph with the example of
Gorlitzer Park? Use your own words as far as possible. (2m)

From passage

Paraphrased

Yet in many aspects, the drug


park does not live up to its
ugly reputation.

He wanted to show that the perception of the


park as a place of drug culture / youthful
rebelliousness is not correct.
In contrast, he shows that it is only seemingly

Young parents pass by with


pushchairs, University students
on picnic blankets peer into
their textbooks

so/ the reality is that of respectable young


people in the park doing very respectable
activities in contrast to the drug dealers
Note: 1 mark for showing the contrast
1 mark for details
Examples are not acceptable as answer.

Get PSJ Private Tutor to Guide you through Exams Now!


Contact www.privatetutor.com.sg

trendyline

3. Explain the authors use of the word flock in Line 10. Use your own words as far as
possible. (2m)

From passage

Paraphrased

Young people flock here from all Literal meaning : to gather in large numbers at a
over the world
particular location ( 1m)
Figurative meaning :
The young all over the world seem to be lured by
the city night-to-dawn entertainment and
exciting wild happenings,
OR
actually they also came to fulfil serious ambitions
Where the young come to study, through succeeding in studies and careers (1m)
work and boost their creative
careers, not just party.
Few European cities do youth
culture and hedonism better than
Berlin.. the wild appearances
belies reality

trendyline

4. In Lines 16-17,what claims about British youth are exaggerated and explain how these
are now ludicrous (line 18).Use your own words as far as possible (3m)
Exaggerated claims are
an epidemic of violence, crime and
drunkenness ,scared of its wayward
youth

Fighting, inebriation arising from its


aggressive/lawless/ inebriated/ uncontrollable
young (1m)
Note: Any two exaggerations for 1 mark

.(to the extent that) a nation


gripped by (the above)
an epidemic

To the extent that an entire society is paralysed


by total helpless fear /held ransom by/
terrorised by hooligan youths (1m)

Not true because

Ludicrous because the facts are

Fall in violent crimes by the young


(1m)
OR

In the last ten years, the teenage


murder rate quietly dropped

Reverse trend of social etiquette among new


generation, less boorish display ( 1m)
OR

Surly youths are getting more polite

Better civic-mindedness/ more respectful to


observe privacy/consideration for others in
public places (1m)

are less rude and noisy in public


places.

Any 1 reason to prove the claims are ludicrous

trendyline

5. In paragraph 4, what contrasts does the author make between the youthful excesses
(Line 28) of the past and the temperance (Line 30) of youth today? Use your own words
as far as possible. (2m)

From passage

Answer

Kids these days just want to live


in
their
own
little
worldswatching
Netflix
..
fumes a barman in Leeds.

Public drinking and drunkenness is less


common/ problematic/worrying/ of concern
today as youths today instead enjoy stay-athome hobbles/recreation (1m)

Television stations aimed at


young people have dropped
programmes that glamourise
rebellion and high living Kids
want o grow up to make money..

Themes of teenage angst/ rebelliousness/


excessive immorality/riotous living as reflected
in TV dramas have become unpopular/
unfashionable/less welcomed in contrast with
serious ambitions/ adult concerns like money/
livelihood/success ( 1m)
Both answers must show explicitly the
contrasting pair in order to be given credit (1m
each pair) for answer.

trendyline

Summary
5. Using materials from Paragraphs 5-7 only, summarise the reasons for the change in the
behaviour of todays youths. Write your summary in no more than 120 words, not
counting the opening words which are printed below. Use your own words as far as
possible. [8m]
There are many reasons for the change in youth behaviour. Firstly

shock at the earlier extent of


youthful hedonism and
disorder has led to dramatic
attempts to curb it ( Line 3637)

Public

anti-social behaviour orders


were widely used against the
rowdy (Line 37-38)

Stricter laws against public disturbances

drinking in the streets has


been met with ferocious
policing (Line 38-39)

Tough policing against teenage drinking

Todays youth by contrast are


few in number and are
growing up in ever older
societies( Line 43-44)

Demographic changes of an ageing society means

campaigns

to

control

the

young

at

risk/delinquents

declining number of youths today for trouble-making

Women no longer feel


Trend of gender equality resulting in girls succeeding
discriminated against and
in conventional ways rather than challenging the boys
therefore it is no longer
necessary to prove their
in vices.
mettle by rebelling through
being more addicted than men
in terms of drinking and drugs
(Line 45-47)

trendyline

in most Western countries More minorities now in Western societies who are less
people from minorities are less
likely to drink or use hard inclined to alcoholism and substance abuses
drugs. ( line 49-50)

The transfer of unskilled jobs


Economic restructuring led to job losses to cheap
to developing countries and of
menial jobs to immigrants(Line labour/out-sourcing
51-52)

has put a new premium on


education( Line 52-53)

Resulting in much focus and investment in education

Young people who are


studying rather than in paid
employment have less money
for hedonism( line 54-55)

Thus youths have less money to indulge in excesses

an increasing number of these


studentsand of young
people in generalstill live
with their parents ( line 58-59)

Many undergraduates still depend on their parents for

who tend to keep careful


watch on their spending and
living habits( Line 59-60)

Their lifestyles are monitored by their parents /being

Clubbing is now more of a


luxury good than a way of life
for British teenagers (line 6061)

Clubbing is more expensive/ less attractive than social

video games and social


networking indulged in
bedrooms are far better
entertainment ( Line 61-62)

Such leisure activities done at home are far more

14

perhaps the best explanation


for this youthful self-control is
the way parents bring these
young adults up(Line 63-64)

Parenting has become an art/ smarter /better skilled


than the old days

15

The amount of time parents


devote to child care has
increased significantly(line 6667)

More time spent bonding/educating the young at home

10

11

12

13

/vices

shelter /livelihood/support

watched/supervised by their parents

networking/video-watching

enjoyable/relaxing/casual/comfortable

trendyline

16

to a new group of parents who Modern/older parents are opposed to the practices of
waited longer to have children
and paid more attention when easy go-lucky irresponsible parenting
they did. (line 70-71)

Marking guidelines:
Mark allocation:
No of
pts

1-2

3-4

5-6

7-8

9-10

11-12

13-14

15-16

Marks

1m

2m

3m

4m

5m

6m

7m

8m

Sample of a good summary (with 16 points / 8m for content)


Public campaigns to control delinquency have resulted in stricter laws against public
disturbance and tough policing against teenage drinking. An ageing society has reduced
delinquency. Gender equality has enabled girls to succeed on merit rather than challenging the
boys in vices. The presence of more minorities has reduced alcoholism and substance abuses.
Furthermore, economic restructuring has led to job losses, which has resulted in more focus on
education. Hence youths have less money to indulge in excesses. Additionally, many
undergraduates depend on their parents support and their lifestyles are monitored. Clubbing is
less attractive than social networking at home in comfort. Indeed, modern parenting has made
more time for bonding with the young as opposed to past practices of irresponsible parenting.

trendyline

(121 words)

7. In paragraph 8, what explanations does the author offer for why better behaviour of the
young has not translated into greater happiness? (Line 73-74)? Use your own words
as far as possible.

From passage

Paraphrased

young people are still proving more More young people today have mental stress of
likely to be diagnosed with depression varying seriousness/ sense of psychological
and anxiety
wellness absent. (1m)

They are often obsessed with their More and more preoccupation with work and its
careers and rarely satisfied.
achievement but little happiness/contentment/
fulfilment in return for success (1m) (must capture
both parts)

.. social media sites ..let them compare


themselves to people who are doing Social media users subject themselves to
comparison of status with peers, real or
better than they feel they are
otherwise, and find dissatisfaction
/discontent/disappointment with their own
The young do not trust the institutions achievement/attainment/accomplishment s(1m)
or people they live with

The young have misgivings/lost faith in public


establishments/bodies/state organisations and
relationships with other members of society like
the authority, peers, family, etc. (1m) (must capture
both institutions and the people they live with)

Any 3 pts - 3m
1 pt - 1m

trendyline

8. What is the author implying by the phrase carefully crafted profiles in Line 80?(1m)

From passage

Paraphrased

(or at least appear to be, in their It suggests either that the profiles were fake
carefully crafted profiles)
/dreamed up/ artificially created/just a facade (1m)
OR
the profiles may be real but has been heavily
slanted/skewed/tweaked to give the best
impressions rather than showing the mundane
reality (1m)

9. Explain the authors attitude towards the change in youthful tendencies and behaviour
from the phrase Perhaps there has been progress (Line 82) (1m)

From passage

Answer

Perhaps there has been progress He is accepting but hesitatingly so/ partly
in the temperament of the youth
agreeable/ not completely convinced/ has some
reservations /unsure/doubtful if the change is fully
positive/good (1m)

trendyline

10. Which phrase in the first paragraph echoes the authors point in the last paragraph about
Berliners not wanting to bring back the decadent past (Line 84).(1m)

From passage

Paraphrased
does not live up to its ugly reputation (1m)
Or
a hassle, not a service (1m)
No need to paraphrase, lifting acceptable

Application Question
11.
In this article, Michael Butler makes a number of observations about how youths in
todays Western world are no longer indulgent, aimless and rebellious. How far is this true of
you and the youth in your society?
[10m]
Coherence
Candidates should:
- include a sense of a start / beginning (do not begin with a conclusion)
- adopt a consistent viewpoint
- argue logically
- organise their answers into cohesive, themed paragraphs
- link paragraphs to show continuity and direction of argument use of discourse markers
- maintain relevance to the task in everything they write
- end with summative or clearly concluding paragraph / sentence

1)

Para 7 Line 16 17: Youth as unhappy, unloved and out of control resulting in an
epidemic of violence, crime and drunkenness

trendyline

Applicable
In Singapore Society, increasing
stress and materialism of a hypercompetitive result-oriented system at
school/work has resulted in
upbringing of children in the hands of
outsiders like maids/childcare
centres/baby sitters/school teachers
from almost infancy due to dual
income family necessitated by high
costs of living and strife for
survival/success leading to poor
parenting or simply not having time
for parenting roles
Long school hours and overwork has
resulted in little precious time for
family -dinners/after work gettogethers/family bonding
Excessive stress in both places has
led to disenchantment.
dissatisfaction/fatigue/burnt-out
generation and severe threat to
family unity/e.g. rise of divorce and
single parenthood
Evidence of increasing trend of
teenage under-aged smoking and
drinking at Clarke Quay areas and
shop-lifting crimes, even more
serious teenage rioting like gang
attacks
Clear trend of teenage pregnancy
and abortions averaging 2,000 +
cases per year

Not applicable
Not really epidemic of violence and
anarchy as SG is very strict on law and
enforcement even for teen crimes ( Amos
Yee supposed out-of-control as
acknowledged by his parents still face the
full and fair impact of the law
Authorities have adopted multi-prong
approach to reduce teenage problemsSexuality education lessons, compulsory
education, tougher policing against
gatherings, enacting of laws banning
consumption of alcohol in public places by
10 p.m. weekdays and ban on sale from 7
p.m. to 7.a.m weekends and over entire
period of public holidays despite the age of
the drinker )
SG Youths are accustomed at a young
age by competition and Kiasu-ness to
compete on merit, those who did not make
the grade have other pathways to success
based on their aptitudes and talents
Most families although working long hours
try to find time for overseas
holidays( school holidays tours are booked
way in advance ,also super-long weekend
like SG 50 ND holidays)
Even weekends, family outings at ECP
and favourite shopping pastimes are
typical family time to make up for the
week.
However, with older teenagers, greater
sense of competiveness is ever present ,
also increasing trend of teenage
depression and counselling by MOE with
additional resources like ECG set-up

trendyline

So, more social issues rather than criminal


acts by the troubled young.

2)
Line 27 -28: The sudden outbreak of restraint is doing havoc to businesses which thrive
off youthful excesses
Applicable
Businesses like clubbing in Clarke
Quay areas have gone bust as they
depend on young crowd of range
like college teenagers and young
working adults, NS boys, etc .
Nightlife scene in SG very volatile
to depend on youthful
patronage/excesses
WHY sudden restraint? Youths are
becoming more educated, growing
up in Asian conservative society
with stricter governance and
penalties for breaking alcohol
curfew, fear of losing out to peers
and wasting time, lure of new
media esp rise of online gaming
and gambling to replace clubbing
and pubbing
Even relatively less harmful vices
like smoking curbed by yearly
increase in taxes and strict
enforcement plus compulsory
counselling etc, have made the
young think twice about such social
vices.

Not applicable
Youths are not going to be easily controlled
as many believe they are only young once.
Alcohol and tobacco curb only statutory
control as teenagers can still obtain these
resources if they want to ,
Other new vices like online gambling( until
recently not enforceable but now new law
will come in effect ) have thrived along with
new media,
Addiction to gaming is real among hordes of
teenagers ( with money thrown in) as school
counsellors report more cases
Online businesses promoting excesses
such as targeting youth spending are
thriving, e.g. branded bags for girls, K-POP
products, regular expensive concert tickets
amounting to a few hundred dollars

Both voluntary and involuntary


restraint

3)
Line 43- 44: Todays youths by contrast are few in number and are growing up in ever
older societies: the implications

trendyline

Applicable

Not applicable

SG is fastest ageing society in the


world (for a 3 million +
Singaporeans) , for comparison our
median age is 30 compared to 17
for Indonesias huge population or
20 years old for Vietnam

The precious young now as a result of


one/two child family with dual income
parents has resulted in them becoming
dependent/ indulgent/ molly-coddled/ given
too much leeway to do what they want

By 2050, there will be only 2.5 :1


ratio if present unfavourable
procreation rates is not reversed

The term Strawberry generation was


coined by our political leaders to bring
attention to the lack of
resourcefulness/direction/helplessness /
individualism BECAUSE our youths now
enjoy far greater degree of freedom and
resources to indulge themselves , knowing
parents will bail them out of trouble, or
overlook their excesses.

Implications on youthful conduct


- Stress on young workers to be more
productive for labour returns ( longer
working hours, upgrading of education
and skills, higher taxation) Such
demands mean little time to fool around
while the rest of society speeds ahead
-

Resources for the young like education


expenditure increasingly reduced ( e.g.
recently, the quota for hiring new
teachers has been reduced) and
facilities/services for ageing population
takes higher profile, SO the young
should worry about the future even if
they are blas now

Ageing society means shift in


government and corporate attention to
cater to older mature citizens SO the
young are socially/politically aware
through dialogues, forums, seminars at
official level to move ahead in the same
direction as the rest of society towards
SG100

Increasing evidence many youths live off


their parents rather than look for gainful
employment or meaningful preoccupation
for self and societal progress ( e.g. rise in
credit card debts among the young as a
result of parental indulgence)

4)
Line 45-47: Women are no longer discriminated against and therefore it is no longer
necessary to prove their mettle by rebelling through being more addicted than men in terms of
drinking and drugs.
Applicable

Not applicable

trendyline

Girls now have equal access to educational


opportunities and jobs based on fair
competition and merit due to gender equality in
SG society
Women can rise to very high posts in
government service( large number of women
principals, SAF s first Woman BrigadierGeneral, Ho Ching, Halimah Yacob ,Chua Sok
Hoon ,etc)
So girls spend time on educational/career
pursuits and get rewarded for hard work and
merit.
Just like the guys, they dont have time to fool
around as the new battleground of the sexes
is in equal and fair competition in schools and
at work.

Girls in schools/Women at work face the


same kinds of stress their male counterparts
are going through, so no reason WHY they
are exempt from hard drinking at pubs/
excessive social problems like smoking to
let off steam and chill.
NOT A QUESTION OF DISCRIMINATION
but SOCIAL VICES do not recognise
gender. Increasing trend of young girl
under-age drinking due to possible
explanations like in-thing or peer identity,
or even school pressures or failures. Also
increasing trend of young girls rebelling by
becoming prostitutes, including a few high
profile cases.

Drug and Gang culture among Singaporean


females rather unheard of, unlike the male
addicts as evident by large number of
rehabilitative/correctional institutions for males

5)

Line 70-71:
a new group of parents who waited longer to have children and paid more
attention when they did

Applicable

Not Applicable

New generation of Young Singaporean


parents are highly educated, professional
class, focus on Quality of life

Dual Income families are tied down with


work demands and stress, hardly left time
for self-relaxation since Singaporeans
work the longest hours in the week ( 60
hours ) Unenviable tag of the hardest
working nation

Most married later ( medium age of males


is 32, females is 28 & above ) which
means more financial stability ( also with
dual income family) as well as small
nuclear family with one or two offspring
Implications : Dual income family provide
greater financial security and head-start

Most working mothers make use of


foreign domestic maids to take care of
household chores, and child-raising; even
sending 3-month olds to infant care

trendyline

for child with resources for enrichment


and development

centres with strangers, the consequences


are mixed at best

BUT also mean greater expectations and


hope and much closer attention to childs
needs and development

Nevertheless, Cannes Award Winning


ground-breaking local film Iloilo Iloilo
tells of one such social change in our
societal landscape.

Especially with a single-child nuclear


family with all of both parents attention
and nurturing
Parents take leave during PSLE, some
mothers even resign from jobs to
focus/monitor closely the childs
educational progress
Males are also sensitive New-age types
who share in household chores , and
more importantly, want to be involved in
nappy-changing and child upbringing
Centre like Fathers for Life aim as their
mission to promote the best parenting
practices among new and young
Singaporean fathers

Perhaps only well-to-do families are at


liberty to give the demanding attention to
monitor their childrens progress and
development as the mother need not
work for a living to cope with high costs of
affluent society in Singaporeparentocracy
Irreversible trends of Falling nation-wide
birth-rates below replacement levels are
harbingers of the future, and might well
show up the reluctance /fear of young
Singaporeans towards parenting /
unwillingness to make compromises to
lifestyle and aspirations/ realistic
appraisal of our society ,etc.

A few are willing even to be househusbands to takeover full time parenting


and allow spouse to pursue own career
aspirations
Proliferation of high-cost private
kindergartens and child development
centres are result of demand and appeal,
even if the costs are easily more than
$2,000 per month
Young families spending quality time
bonding on weekends at popular family
establishments (McDonalds) or
playgrounds/parks/ attractions

trendyline

VICTORIA JUNIOR COLLEGE


PRELIMINARY EXAMINATION

8807/2

GENERAL PAPER

Monday

1 hour 30 minutes

31 August 2015

INSTRUCTIONS TO CANDIDATES

Write your Name and Civics Class in the spaces provided in the answer paper.
Answer all questions.
If you need to use extra sheets of writing paper for a question, fasten all your work
securely together with the answer sheets.
The number of marks is given in brackets [
question.

] at the end of each question or part

Note that 15 marks out of 50 will be awarded for your use of language.

___________________________________________________________________
This question paper consists of 9 printed pages including this page.

trendyline

Wednesday Martin writes about the state of marriage today


1

Just in time for Valentine's Day, some recent, highly-publicized non-fiction debuts are sure
to get you in the mood for romance. Staying True by Jenny Sanford, chronicles the very
public breakdown of her marriage to South Carolina governor Mark Sanford who was not
hiking on the Appalachian Trail after all. Marry Him by Lori Gottlieb, the subtitle of which the case for Settling for Mr Good Enough - says it all. It accuses you of being too picky and 5
urges you to snap up that guy who is an 8 rather than waiting for the 10. And The
Politician, Andrew Young's new, explosive tell-all about John Edwards, details his infidelity
and exposes, for our lurid delectation, the operatic fights and the second family he started.

But in their own dramatic and overblown ways, these books speak a quieter, less dramatic
truth: marriage is not what we think it is, and it is not easy. Plenty of marriages are not 10
doing well. Why? Much ink has been spilled and much breath has been spent analysing the
problem. In addition, many workshop fees have been forked over in the interest of
understanding what has gone wrong with marriages and how to improve them to make
them more satisfying, equitable, sexually exciting, emotionally healthy, nurturing and
harmonious. Saving marriages is a multi-million dollar industry and many of us know from 15
first-hand experience, that it can work. Marriages - some of them - can be saved.

But Marriage probably cannot. While marriage therapists tell us how to save our
marriages, sociology, anthropology, and human behavioural ecology suggest that it is not
so much married couples as Marriage itself, that is in trouble. The problem with marriages
is really the fundamental problem with Marriage: marriages are falling apart in large part 20
because Marriage is no longer necessary in the way it once was.

Sociologists and historians of marriage tell us that marriage was originally a business
transaction of sorts, rather than an undertaking hinging on the attraction and love
between individuals. Historically in western culture, people from wealthy families were
directed to marry in order to create bonds, alliances and mutual obligation with other 25
powerful families. Marriages even created bonds between nations in the case of royals.
For the lower classes, marriage was a question of creating a labour force to run a farm or
small business. Households were production-centred economies in which men's labour
and women's labour were complementary, and the children they had together or brought
together from previous unions pitched in. Maternal mortality rates were high until the late 30
19th century and remarriage with children after the death of a spouse - a common
occurrence until relatively recently - was considered the most civic-minded thing a man or
woman could do. Marriage was necessary. The household and by extension, all of society
depended on it, after all.

But by the early 20th century, as marriage historian Stephanie Coontz points out, with the 35
notions of the individual, liberty, and equality well-established by the Enlightenment and
the French and American revolutions, and the subsequent rise of the love match, marriage
had become a different animal entirely. Marriage morphed from institutional, in the
famous formulation of sociologist Ernest Burgess, to companionate and now, something
more individualistic. Marriage is now expected to nurture, satisfy and support the 40
members of the couple in a dizzyingly comprehensive variety of ways - emotionally,
sexually, psychologically.

trendyline

At the same time, as sociologist Andrew Cherlin notes, as women came to participate
more in the workforce, household micro-economies changed as well, from production to
consumption-centred. Gender roles become more flexible. Women now have the 45
economic freedom to walk away from unhappy unions. Not to mention the opportunity to
find friendship, empowerment and other potential partners in the workplace.

Times continue to change, and marriage, whether we like it or not, is tethered to our times
and the forces of historical change. For example, marriage is no longer the only acceptable
context for childbearing. Increasingly, couples in the US elect to cohabit rather than marry. 50
In Scandinavian countries like Sweden, these couples are less likely to break up than are
married couples in the US. Many of these cohabiting couples are also having children
outside marriage. And owing to women's increased economic power and the rise of
reproductive technologies, more women can and do elect to have children outside of
55
marriage and even outside of the structure of coupledom entirely.

As for those who suggest that the heterosexual pair bond is part of our evolutionary
history and so "right" and "forever", there is plenty of evidence to the contrary. In many
contemporary foraging cultures, for example, people live much as evolutionary biologists
believe we did in the Pleistocene era: men and women "marry" nine or ten times and
children are much more likely to live in stepfamilies or with single parents than with two 60
parents. The notion that the permanent couple dyad as we now know it is timeless, is one
human behavioural ecologists now regard with scepticism if not outright disbelief.

Many argue that we must be married to be emotionally and physically healthy, and recent
studies claim to prove as much but, there is evidence that marriage is detrimental. The
National Marriage Project found that the percentage of upper middle class white women 65
who described their marriages as happy dropped from 74% to 68% over the last decade.
Other studies find that married women are more likely to be depressed than unmarried
women, and that women with stepchildren are far more likely to be clinically depressed
than those without.

10

Regardless of our moral and ideological convictions and our public policy about what it 70
should be and how we ought to value it, the fact is that marriage is not what it once was
because the world is no longer what it once was: largely agrarian, with a neatly gendered
division of labour within households whose production-centred economies also centred on
the production and contributions of children therein. Those who propose a return to
'traditional' marriages had better also provide time machines, for marriage is married to its 75
historical moment and we have entered a new one.

11

Many of us are familiar with the argument that marriages are not feasible in the way they
used to be because now we live much longer and till death do us part is likely to be five
or six decades rather than one or two. We are also more mobile as a society and so the
forces that historically helped married people stay together, forces like the church and the 80
extended family, have less influence over us. It is simply unreasonable to expect one
relationship to satisfy us in so many ways because, as some argue, we are not wired for
monogamy.

12

Yet people stay in marriage every day and make it work. We are not all Mark Sanford or
John Edwards, and Gottlieb's anti-romantic vision will leave many of us cold or even 85
outraged. But with Marriage less necessary than ever before, the challenge becomes how
do we make our own marriages necessary and relevant? How do we keep Marriage - not
to mention - marriages alive? And should we even try?

trendyline

Candidates Name:

__________________________

Candidates Civics Class:

__________________________

Content

/35

Language

/15

Total

/50

Paper 2 (50 marks)


Read the passages in the insert and then answer all the questions which follow. Note that up
to fifteen marks will be given for the quality and accuracy of your use of English throughout this
Paper.
Note: When a question asks for an answer IN YOUR OWN WORDS AS FAR AS POSSIBLE
and you select the appropriate material from the passages for your answer, you must still use
your own words to express it. Little credit can be given to answers which only copy words or
phrases from the passages.

1. Explain what Lori Gottlieb means by snap up that guy who is an 8 rather than waiting
for the 10 (line 6). Use your own words as far as possible.
[2m]
_______________________________________________________________________
_______________________________________________________________________
_______________________________________________________________________
_______________________________________________________________________

2. In paragraph 1, what is ironic about the authors claim that the recent books Staying
True, Marry Him and The Politician are sure to get you in the mood for romance?
[2m]
_______________________________________________________________________
_______________________________________________________________________
_______________________________________________________________________
_______________________________________________________________________

trendyline

3. Explain what the author means by much ink has been spilled and much breath has
been spent (line 11). Use your own words as far as possible.
[1m]
_______________________________________________________________________
_______________________________________________________________________
_______________________________________________________________________

4.

In paragraph 3, what is the difference between Marriage and marriages?

[2m]

_______________________________________________________________________
_______________________________________________________________________
_______________________________________________________________________
_______________________________________________________________________

5. What do you understand by the authors claim that marriage was originally a business
transaction (lines 22-23)? Use your own words as far as possible.
[2m]
_______________________________________________________________________
_______________________________________________________________________
_______________________________________________________________________
_______________________________________________________________________
_______________________________________________________________________
_______________________________________________________________________

6. What is implied by the authors claim that remarriage was the most civic-minded thing
a man or woman could do (lines 32-33)?
[2m]
_______________________________________________________________________
_______________________________________________________________________
_______________________________________________________________________
_______________________________________________________________________
_______________________________________________________________________

trendyline

7. Explain the authors use of quotation marks for the words right and forever in line 57.
[1m]
_______________________________________________________________________
_______________________________________________________________________
_______________________________________________________________________

8. What is suggested by the authors statement that marriage is married to its historical
moment (lines 75-76)? Use your own words as far as possible.
[2m]
_______________________________________________________________________
_______________________________________________________________________
_______________________________________________________________________
_______________________________________________________________________

9. Why does the author say that marriages are not feasible in the way they used to be
(line 77-78)? Use your own words as far as possible.
[2m]
_______________________________________________________________________
_______________________________________________________________________
_______________________________________________________________________
_______________________________________________________________________

10. What does the author imply by the remark We are not all Mark Sanford or John
Edwards (lines 84-85)?
[1m]
_______________________________________________________________________
_______________________________________________________________________
_______________________________________________________________________

trendyline

11. Using only material from paragraphs 5-7 only, summarise what the author has to say
about the changes in marriage and the reasons for these changes. Write your
summary in no more than 120 words, not counting the opening words which are
printed below. Use your own words as far as possible.
[8m]
By the early 20th century, ______________________________________________
___________________________________________________________________________
___________________________________________________________________________
___________________________________________________________________________
___________________________________________________________________________
___________________________________________________________________________
___________________________________________________________________________
___________________________________________________________________________
___________________________________________________________________________
___________________________________________________________________________
___________________________________________________________________________
___________________________________________________________________________
___________________________________________________________________________
___________________________________________________________________________
___________________________________________________________________________
___________________________________________________________________________
___________________________________________________________________________

trendyline

12. Wednesday Martin argues that notions of marriage are changing. How far would you
agree with her observations, relating your arguments to your own society?
[10m]
___________________________________________________________________________
___________________________________________________________________________
___________________________________________________________________________
___________________________________________________________________________
___________________________________________________________________________
___________________________________________________________________________
___________________________________________________________________________
___________________________________________________________________________
___________________________________________________________________________
___________________________________________________________________________
___________________________________________________________________________
___________________________________________________________________________
___________________________________________________________________________
___________________________________________________________________________
___________________________________________________________________________
___________________________________________________________________________
___________________________________________________________________________
___________________________________________________________________________
___________________________________________________________________________
___________________________________________________________________________
___________________________________________________________________________
___________________________________________________________________________
___________________________________________________________________________
___________________________________________________________________________
___________________________________________________________________________
___________________________________________________________________________
___________________________________________________________________________

trendyline

___________________________________________________________________________
___________________________________________________________________________
___________________________________________________________________________
___________________________________________________________________________
___________________________________________________________________________
___________________________________________________________________________
___________________________________________________________________________
___________________________________________________________________________
___________________________________________________________________________
___________________________________________________________________________
___________________________________________________________________________
___________________________________________________________________________
___________________________________________________________________________
___________________________________________________________________________
___________________________________________________________________________
___________________________________________________________________________
___________________________________________________________________________
___________________________________________________________________________
___________________________________________________________________________
___________________________________________________________________________
___________________________________________________________________________
___________________________________________________________________________
___________________________________________________________________________
___________________________________________________________________________
___________________________________________________________________________
___________________________________________________________________________
___________________________________________________________________________
___________________________________________________________________________
End of Paper

trendyline

VJC 2015 Prelim Paper 2 Answer Scheme


1. Explain what Lori Gottlieb means by snap up that guy who is an 8 rather than waiting for the 10
(line 6)? Use your own words as far as possible.
[2]
From the Text

Possible Paraphrase

snap up that guy who is


an 8 rather than waiting for
the 10.

Gottlieb is suggesting that women should grab a romantic partner


who is of an acceptable standard (1) instead of waiting for a
perfect man (to come along). (1)
Examiners Comments: Average or mediocre are not acceptable
answers, because 8 out of 10 is good/well above average.

2. In paragraph 1, what is ironic about the authors claim that the recent books Staying True, Marry
Him and The Politician are sure to get you in the mood for romance?
[2]
From the Text

Inference
While the author claims that these newly launched books will put one
in the mood for love, it is ironic that the books listed are all about
failed relationships/marriages.
Note: Candidates must give both portions in order to get 2 marks. No
marks given otherwise.

3. Explain what the author means by much ink has been spilled and much breath has been spent
(line 11). Use your own words as far as possible
[1]
From the Text

Possible Paraphrase/Inference

much ink has been


spilled and much
breath spent

A lot has been written about marriage in


books/newspapers/magazines and a lot of discussions on marriage
have taken place in talks /conversations/seminars.
Note: Candidates must give both portions in order to get 1 mark. Students
should also mention the context of marriage/marriage problems.

4. In paragraph 3, what is the difference between Marriage and marriages?

[2]

From the Text

Inference

While marital and couples therapists tell us how to


save our marriages, sociology, anthropology, and
human behavioural ecology suggest that it is not
so much married couples as Marriage itself, that
is in trouble.

Marriage refers to the social


institution/idea/concept/the place of marriage in
society while marriages refer to the individual
unions between people.
Note: Candidates must give both portions in
order to get 2 marks. No marks given otherwise.

Get PSJ Private Tutor to Guide you through Exams Now!


Contact www.privatetutor.com.sg

trendyline

5. What do you understand by the authors claim that marriage was originally a business transaction
(lines 22-23)? Use your own words as far as possible.
[2]
From the Text

Possible Paraphrase/Inference

marriage was
originally a business
transaction

Marriage used to involve the trade or the exchange of freedom/love/sex


(1) for money/ financial stability/ social status/ political benefits. (1)
OR
The union is for the purpose of financial/material gain instead of affection
Examiners Comments: Many students tended to paraphrase the examples
in the passage wealthy families, royals etc. This is not sufficient to get both
marks unless the student also explicitly mentioned that there was an
exchange where marriage/love is traded for such benefits.

6. What is implied by the authors claim that remarriage is the most civic-minded thing a man or
woman could do (lines 32-33)?
[2]
From the Text

Possible Paraphrase/Inference

was considered the most


civic-minded thing a man or
woman could do.

Remarriage was considered a highly responsible/considerate act


(1) that helped preserve the social cohesion of society/ keep the
family together. (1)
Examiners Comments: Remarriage with children refers to
remarrying with children in tow, it does not mean marrying ones
own children(!)

7. Explain the authors use of quotation marks for the words right and forever in line 57?

[1]

From the Text

Inference

...heterosexual pair bond is part of our


evolutionary history and so "right" and
"forever", there is plenty of evidence to
the contrary...

He does not think that heterosexual unions are


unquestionably moral/ethical and that such unions should
never change as time passes.
Note: Candidates must give both portions in order to get 1
mark.
Examiners Comments: Many students simply lifted right
and forever. Although the question does not ask
students to use their own words, students are to answer in
context and unpack what the words mean.

trendyline

8. What is suggested by the authors statement that marriage is married to its historical moment
(lines 75-76)? Use your own words as far as possible.
[2]
From the Text

Possible Paraphrase/Inference

Those who propose a return to traditional


marriages had better also provide time
machines, for marriage is married to its
historical moment and we have entered a
new one.

Marriage is linked/ tied/connected (1) to a


particular point in time/a specific period/era (1).
OR

Marriage evolves or changes (1) with the times/era


(1)
Examiners Comments: Students should not merely
write marriage is linked to the past as that is too
vague. They should explain that it is linked to the
conditions of the past

9. Why does the author say that marriages are not feasible in the way they used to be (line 77-78)?
Use your own words as far as possible.
[2]
From the Text

Possible Paraphrase

because now we live much longer and


'till death do us part' is likely to be five
or six decades rather than one or two.
We are also more mobile as a society
and so the forces that historically
helped married people stay together
have less influence over us.

Compared to the past, our lifespan has extended


further (a) and so, our matrimony will probably last
for many years, instead of a few. (b) We are also
moving around more often as a community (c) and
thus, the institutions that aided wedded couples in in
the past, do not affect us as much as they used to.
(d)

Note: 1-2 pt: 1 mark, 3-4 pt: 2 marks

10. What does the author imply by the remark We are not all Mark Sanford or John Edwards (lines
84-85)?
[1]
From the
Text

Inference
Not all of us are prone to being unfaithful.
OR
We should not use the actions of a few to generalise about the behaviour/the
faithfulness of everyone else.
Note: If adultery or infidelity is not mentioned or implied, the student will not get the
mark.

11. Using only material from paragraphs 5 -7 only, summarise what the author has to say about the
changes in marriage and the reasons for these changes. Write your summary in no more than 120
words, not counting the opening words which are printed below. Use your own words as far as
possible.
[8m]
By the early 20th century

trendyline

From the Text


a the notions of the individual (36)

Possible Paraphrase
ideas about/belief in the significance/identity of a
person
Note: Significance/identity is inferred.

b liberty (36)

freedom/independence/the right to act as one


wishes

c and equality (36)

egalitarianism/parity/similarity in status

d and the subsequent rise of the love


match (37)

the growing trend of marrying for love

e marriage morphed from institutional


(38)

changed/transformed from a legitimate social


practice/custom/expectation
Note: Developed will not be accepted as a
paraphrase for morphed

to companionate (39)

a romantic relationship between two people


Note: Romantic must be included as relationship is
by itself is too vague

g and now, something more


individualistic (40)

more person-centred/focused on the self

h marriage is now expected to nurture,


satisfy and support the members of
the couple (40)

must nourish/build up, fulfil desires and


enable/help

in a dizzyingly comprehensive variety


of ways (41)
OR

Note: Selfish is not accepted as it does not mean the


same thing as individualistic

Note: Students only need to identify any 2 of the 3


above for the point
many possible aspects/a large number of methods
OR
all areas of human need

emotionally, sexually,
psychologically (41,42)
women came to participate more in the
workforce (43,44)

More women entered the job market


OR
women played a greater role in the economy
(inferred point)

k household micro-economies changed


(44)
l from production to consumptioncentred (44,45)
m gender roles become more flexible
(45)
n women now have the economic
freedom (46)

financial needs/activities of families altered


from manufacturing/making (goods and services) to
buying/purchasing
gender roles are pliable/changeable/not strictly
adhered to
Women are financially independent/have the
financial means

trendyline

o to walk away from unhappy unions


(46)
p the opportunity to find friendship
(46,47)
q empowerment (47)
and other potential partners in the
workplace (47)
s marriage is tethered to our times (48)
r

to end/exit bad/dysfunctional marriages


The chance/means to meet new people/widen their
social circle
to be equipped with/to have the autonomy/agency
to determine their own lives/make own choices
and to find prospective spouses at work
(the institution of marriage) is highly dependent
on/affected by/tied to existing customs/culture

OR

OR

the forces of historical change (49)

new global developments/ground-breaking events

no longer the only acceptable context


for childbearing (49-50)

one need not be married to be a parent/marriage is


not a prerequisite for parenthood

u couples elect to cohabit rather than


marry (50)

choose/decide to live together in a sexual


relationship without marriage
Note: Cohabitation must be explained/paraphrased

v owing to womens increased


economic power (53)
w and the rise of reproductive
technologies (53,54)
x

more women can and do elect (54)

y to have children outside marriage


(54,55)

greater financial clout/strength


increase in medical know-how/methods related to
childbearing/procreation
more women are able and choose,
to become mothers without getting married first/to
become single mothers

Mark conversion:
1-2 = 1
3-4 = 2
5-6 = 3
7-8 = 4
9-10 = 5
11-12 = 6
13-14 = 7
15 = 8
12. Wednesday Martin argues that the notions of marriage are changing. How far would you agree
with her observations, relating your arguments to your own society?
[10]
Band
1
810
marks

REQUIREMENTS: systematic reference to the requirements of the question with evidence of a


balanced treatment.
There must be explicit reference to ideas made by the author Martin.
These references must be relevant to the changing notions of marriage.
Students must make the evaluation of the observation of the role reversal as the focus of
their discussion.
Students must refer to experiences and observations relevant to their own
society/country.
EXPLANATION: shows a good or very good understanding of terms and issues.
Interpretation of authors observations must be accurate.

trendyline

Explanation of students evaluation must be clear and relevant to references chosen.


Examples/support must be explained clearly to illustrate/justify their points.
*These ideas for discussion and evaluation in the AQ are distilled from the passage, with the line
references provided for your benefit.
*Key Ideas from Passage
1.
2.
3.
4.
5.
6.
7.

8.
9.
10.

Pickiness when choosing a marriage partner (line 4-6)


Infidelity of public figures (lines 7-8)
Plenty of marriages are not doing well (line 10-11)
Marriage is no longer necessary in the way it once was. (line 21).
Marriage is now expected to nurture, satisfy and support the members of the couple in a
dizzyingly comprehensive variety of ways - emotionally, sexually, psychologically. (line
40-42).
Women now have the economic freedom to walk away from unhappy unions. Not to
mention the opportunity to find friendship, empowerment and other potential partners in
the workplace. (line 45-47).
More women can and do elect to have children outside of marriage and even outside of
the structure of coupledom entirely (Note: Elect means women make a conscious choice
to have children out of wedlock. This does not apply to single mothers who are
abandoned by husbands or boyfriends) (line 54-55).
The notion that the permanent couple dyad as we now know it is timeless, is one human
behavioural ecologists now regard with scepticism if not outright disbelief (line 61-62)
Marriage is detrimental (line 64)
Marriage is no longer feasible (because we live longer, we are more mobile and forces
like the extended family and religion have less influence over us) (line 77)

Sample Development of Selected Points:


Development of Point #1
These days, Singaporeans are less willing to get hitched or are marrying later or they are simply
unable to find a spouse. The high education levels of Singaporean women may be a contributing
factor. The largest proportion of unmarried women in Singapore are university graduates,
according to the 2014 Department of Statistics Population Trends paper (16 per cent of university
graduates aged 40-44 years were single in 2013, compared to 8.3 per cent among females with
below secondary qualifications). This might mean that successful/well-educated Singaporean
women have high expectations of potential marriage partners and would rather remain unmarried
than marry someone they deem as unworthy.
Development of Point #2
Our government has set a very high standard regarding the moral integrity of our leaders and
infidelity is not tolerated at all. Those who contravene these standards are made to step down or
removed from office. For example, the former Speaker of Parliament Michael Palmer issued a
public apology and stepped down from his post when news of his extramarital affair came to light,
and opposition politician Yaw Shin Leong was summarily dismissed by the Workers Party when
he refused to refute rumours of his affair with another WP executive committee member.
Development of Point #3
Counsellors also point to a diminishing social stigma attached to divorce and some couples are
more willing to give up on a marriage in trouble. Growing acceptance of divorce in Singapore. For
example, the 2014 median age is 33.4 , the total number of marital dissolutions (comprising
divorces and annulments) rose four per cent from 7,237 in 2012 to 7,525 in 2013.
http://www.todayonline.com/singapore/fewer-marriages-registered-more-dissolutions-2013.
Development of Point #4
Marriage is no longer necessary in the way it once was. The rising trend of single men and
women in their 30s and 40s in Singapore is evidence of this. Single men and women do not need

trendyline

to get married and can support themselves. HDB policies have changed, allowing singles to
purchase 2-room BTOs and the income ceiling for singles buying BTOs has been raised. This can
be seen as evidence that the government is acknowledging the rising trend of rising singlehood
and the idea that marriage is no longer necessary to complete our lives.
EVALUATION: makes very convincing evaluation by making judgements and decisions and by
developing arguments to logical conclusions, and includes elaboration and support through
personal insight and apt illustration.
1. Students evaluation should be clear, consistent and relevant to the question.
2. There should be balance and examples/evidence to support students claims.
3. Examples/evidence should be contextualised and concrete.
4. Examples/evidence should cover both men and women.
COHERENCE: very clear shape and paragraph organisation and cogent argument.
5. There should be a proper introduction and conclusion.
6. Students evaluation should be clear and should not contradict.
7. The discussion points should be distinct yet connected by a common purpose of writing.
Band
2
47
marks

REQUIREMENTS: covers requirements of the question adequately but not necessarily a


balanced treatment.
EXPLANATION: shows an adequate level of understanding of terms and issues (which may
include minor distortion).
EVALUATION: evaluation is attempted but is not always convincing, and tends to be superficial
with limited development of ideas, and is not as thorough in support.
COHERENCE: paragraphing is sometimes helpful and there is a recognisable over-all shape to
the answer; arguments are generally cogent.

Band
3
13
marks

REQUIREMENTS: an incomplete and/or unbalanced treatment of the requirements.


EXPLANATION: shows very limited degree of understanding and a higher incidence of
misinterpretation.
EVALUATION: tends to be a mere summary or restatement of the text with a few simple and
undeveloped judgements, with very thin support.
COHERENCE: paragraphing and organisation are haphazard; arguments inconsistent or illogical.

Nothing in the answer meets any of the criteria.

Band 2
47
marks

REQUIREMENTS: covers requirements of the question adequately but not


necessarily a balanced treatment.
EXPLANATION: shows an adequate level of understanding of terms and issues
(which may include minor distortion).
EVALUATION: evaluation is attempted but is not always convincing, and tends to be
superficial with limited development of ideas, and is not as thorough in support.
COHERENCE: paragraphing is sometimes helpful and there is a recognisable overall shape to the answer; arguments are generally cogent.

Band 3
13
marks

REQUIREMENTS: an incomplete and/or unbalanced treatment of the requirements.


EXPLANATION: shows very limited degree of understanding and a higher incidence
of misinterpretation.
EVALUATION: tends to be a mere summary or restatement of the text with a few
simple and undeveloped judgements, with very thin support.

trendyline

COHERENCE: paragraphing and organisation are haphazard; arguments


inconsistent or illogical.
0

Nothing in the answer meets any of the criteria.

trendyline

Candidates Name

CTG

YISHUN JUNIOR COLLEGE


JC2 PRELIMINARY EXAMINATION 2015

H1 GENERAL PAPER

8807/02

PAPER 2

14 August 2015
1030h-1200h

YISHUN JUNIOR COLLEGE YISHUN JUNIOR COLLEGE YISHUN JUNIOR COLLEGE YISHUN JUNIOR COLLEGE YISHUN JUNIOR COLLEGE
YISHUN JUNIOR COLLEGE YISHUN JUNIOR COLLEGE YISHUN JUNIOR COLLEGE YISHUN JUNIOR COLLEGE YISHUN JUNIOR COLLEGE
YISHUN JUNIOR COLLEGE YISHUN JUNIOR COLLEGE YISHUN JUNIOR COLLEGE YISHUN JUNIOR COLLEGE YISHUN JUNIOR COLLEGE
YISHUN JUNIOR COLLEGE YISHUN JUNIOR COLLEGE YISHUN JUNIOR COLLEGE YISHUN JUNIOR COLLEGE YISHUN JUNIOR COLLEGE
YISHUN JUNIOR COLLEGE YISHUN JUNIOR COLLEGE YISHUN JUNIOR COLLEGE YISHUN JUNIOR COLLEGE YISHUN JUNIOR COLLEGE
YISHUN JUNIOR COLLEGE YISHUN JUNIOR COLLEGE YISHUN JUNIOR COLLEGE YISHUN JUNIOR COLLEGE YISHUN JUNIOR COLLEGE
YISHUN JUNIOR COLLEGE YISHUN JUNIOR COLLEGE YISHUN JUNIOR COLLEGE YISHUN JUNIOR COLLEGE YISHUN JUNIOR COLLEGE
YISHUN JUNIOR COLLEGE YISHUN JUNIOR COLLEGE YISHUN JUNIOR COLLEGE YISHUN JUNIOR COLLEGE YISHUN JUNIOR COLLEGE
YISHUN JUNIOR COLLEGE YISHUN JUNIOR COLLEGE YISHUN JUNIOR COLLEGE YISHUN JUNIOR COLLEGE YISHUN JUNIOR COLLEGE
YISHUN JUNIOR COLLEGE YISHUN JUNIOR COLLEGE YISHUN JUNIOR COLLEGE YISHUN JUNIOR COLLEGE YISHUN JUNIOR COLLEGE
YISHUN JUNIOR COLLEGE YISHUN JUNIOR COLLEGE YISHUN JUNIOR COLLEGE YISHUN JUNIOR COLLEGE YISHUN JUNIOR COLLEGE
YISHUN JUNIOR COLLEGE YISHUN JUNIOR COLLEGE YISHUN JUNIOR COLLEGE YISHUN JUNIOR COLLEGE YISHUN JUNIOR COLLEGE
YISHUN JUNIOR COLLEGE YISHUN JUNIOR COLLEGE YISHUN JUNIOR COLLEGE YISHUN JUNIOR COLLEGE YISHUN JUNIOR COLLEGE

TIME

1 hour 30 minutes

INSTRUCTIONS TO CANDIDATES
Write your name and CTG in the spaces at the top of this page.
Answer all the questions.
At the end of the examination, fasten the cover page, passages and all your work securely
together.

INFORMATION FOR CANDIDATES


The number of marks is given in brackets [ ] at the end of each question.
(Note that 15 marks out of 50 will be awarded for your use of language.)
Question

1
1m

2
2m

3
2m

4
2m

5
3m

6
1m

7
2m

8
2m

9
2m

10
8m

11
10m

Marks
FOR EXAMINERS
USE
Content

/35

Language

/15

Total

/50

This question paper consists of 9 printed pages including this page.

[Turn over]

trendyline

David Robson examines the issue of boredom and its effects.


1

Sandi Mann is one of the few psychologists to have forayed into the mind-numbing
territoryof boredom. Mann wants to understand the profound effect that boredom may
have on our lives. Its the Cinderella of psychology, she says. After all, admitting that you
study boredom might itself sound a bit, well, boring but that is far from the truth. Boredom,
it turns out, can be a dangerous and disruptive state of mind that damages your health 5
and even cuts years off your lifespan. If that sounds negative, Manns research would also
suggest that without boredom we couldnt achieve our creative feats.

Boredom is such a large part of day-to-day existence that it is somewhat surprising the word
only entered the language with Charles Dickenss Bleak House in 1852. Dickenss study of
Lady Deadlocks suffering she is bored to death by her marriage would end up pre- 10
empting many of the latest findings. Perhaps because of its prevalence in our lives,
scientists had been slow to explore the sensation. When you are swimming in something,
maybe you dont think of it as being noteworthy, says John Eastwood at York University in
Canada, who was one of the first scholars to take an interest.

One of the most common misconceptions is that only boring people get bored. Yet as 15
Eastwood set about exploring the reasons for boredom, he found that there are two distinct
types of personality that tend to suffer from ennui, and neither are particularly dull
themselves. Boredom often goes with a naturally impulsive mindset among people who are
constantly looking for new experiences. For these people, the steady path of life just isnt
enough of a rollercoaster to hold their attention. The world is chronically under-stimulating, 20
says Eastwood. The second kind of bored people have almost exactly the opposite
problem; the world is a fearful place, and so they shut themselves away and try not to step
outside their comfort zone. Out of their high-sensitivity to pain, they withdraw. While this
retreat might offer some comfort, they are not always satisfied with the safety it offers and
chronic boredom results.
25

Almost from the very beginning, it became clear that either of these states could push
people to harm themselves; a proneness to boredom was linked to a tendency to smoke,
drink too much, and take drugs. Indeed, in one study boredom was the single biggest
predictor of alcohol, cigarette and cannabis use among a group of South African teenagers.

Boredom is also linked to more mundane but equally unhealthy behaviours, such as 30
comfort-eating your way through tedium. Boredom at work is propping up the confectionary
industry, says Mann, who is based at the University of Central Lancashire, UK. The overall
effect of boredom on your life expectancy could be drastic, too. When researchers in the
famous Whitehall study followed the lives of middle-aged civil servants in the UK, they
found that the people who are most likely to get bored were 30% more likely to have died 35
over the next three years.

That is something of a puzzle for evolutionary psychologists. Emotions should evolve for our
benefit not to push us to self-destruction. The very fact that boredom is a daily
experience suggests it should be doing something useful, says Heather Lench at Texas
A&M University. Feelings like fear help us avoid danger, after all, while sadness might help 40
prevent future mistakes. So, if true, what does boredom achieve?

Reviewing the evidence so far, Lench suspects that it lies behind one of our most important
traits curiosity. Boredom, she says, stops us ploughing the same old furrow, and pushes
us to try to seek new goals or explore new territories or ideas. That search for an escape

trendyline

could sometimes push us to take risks that eventually hurt us. One research team simply 45
left subjects by themselves in a room for 15 minutes with a button that allowed them to give
themselves an electric shock on the ankle; many did indeed elect to give themselves the
brief buzz of pain, seemingly because it was the only way to break up the tedium. Perhaps
the same search for an escape explains why bored people turn to unhealthy behaviours
but the upside is that it can also increase innovation.
50
8

Mann has found that the ennui people feel when performing mindless tasks boosts their
creativity such as finding innovative uses for everyday objects. She suspects the tedium
encouraged their minds to wander, which leads to more associative and creative ways of
thinking. If we dont find stimulation externally, we look internally going to different places
in our minds, she says. It allows us to make leaps of imagination. We can get out of the 55
box and think in different ways. Without the capacity for boredom, then, we humans may
have never achieved our artistic and technological heights.

Given this benefit, Mann thinks we should try not to fear boredom when it hits us. We
should embrace it, she says a philosophy that she has now taken into her own life.
Instead of saying Im bored when Im stuck in traffic, Ill put music on and allow my mind to 60
wander knowing that its good for me. And I let my kids be bored too because its good
for their creativity.

10 Eastwood is less enthusiastic about boredoms benefits, but admits we should be cautious
about looking for an immediate escape. The feeling is so aversive that people rush to
eliminate it, he says. Im not going to join that war on boredom and come up with a cure, 65
because we need to listen to the emotion and ask what it is trying to tell us to do. For
instance, simply looking for instant gratification on a smartphone or tablet may be counterproductive, he thinks. We live in a tech-driven society where we are overly stimulated we
are constantly yanked around by interruptions, says Eastwood. That puts us on a kind of
treadmill, he says we keep on expecting quicker and easier ways to revive our curiosity. 70
One possibility is that this actually makes people more bored.
11 Instead, he suggests that it would be wiser to question whether there are more serious,
long-term issues that are causing us to feel disengaged. His work, for instance, has shown
that priming people to feel their lives have a greater purpose and meaning tends to make
them less bored during subsequent tests. Although our feelings of tedium during a work 75
meeting or family gathering might seem like superficial annoyances, they could be a
symptom of a deeper existential crisis and need for fulfilment that extends far beyond
immediate circumstances.
12 To feel you can have an effect on the world and that things in life make sense, these are
inherently important things for human beings just like sunlight, fresh air and food, says 80
Eastwood. As we enter the New Year, that could be as good a reason as any to re-evaluate
your life, what you are trying to achieve with it, and to rethink what you actually mean when
you say you are bored.

trendyline

Read the passage and then answer all the questions. Note that up to fifteen marks will be
given for the quality and accuracy of your use of English throughout this Paper.
Note: When a question asks for an answer IN YOUR OWN WORDS AS FAR AS POSSIBLE
and you select the appropriate material from the passage for your answer, you must still use
your own words to express it. Little credit can be given to answers which only copy words or
phrases from the passage.
1. What does the word forayed (line 1) imply about the research on boredom? Use your
own words as far as possible.

[1]

2. Why does Sandi Mann call boredom the Cinderella of psychology in line 3?

[2]
3. Explain what the author means by Dickenss study of Lady Deadlocks suffering she
is bored to death by her marriage would end up pre-empting many of the latest
findings (lines 9-11). Use your own words as far as possible.

[2]
4. According to Eastwood, why had scientists been slow to explore the sensation (line
12)? Use your own words as far as possible.

[2]

trendyline

For
Examiners
For
Use
Examiners
Use

For
Examiners
Use

5. How do the two types of personality in paragraph 3 illustrate the misconception that
only boring people get bored (line 15)? Use your own words as far as possible.

[3]
6. What does Mann think about how we should respond to boredom when it hits us (line
58)?

[1]
7. In paragraph 10, Eastwood claims that we should be cautious about looking for an
immediate escape. How does the remainder of the paragraph illustrate his concern?

[2]
8. Explain the authors use of superficial (line 76) and deeper (line 77) in paragraph 11.
Use your own words as far as possible.

[2]
9. What, according to the author, is as good a reason as any to re-evaluate your life
(lines 81-82)?

[2]

trendyline

10. Using material from paragraphs 48 only (lines 2657), summarise the drawbacks
and benefits of boredom.
Write your summary in no more than 120 words, not counting the opening words
which are printed below. Use your own words as far as possible.
Boredom is harmful when

[8]

trendyline

For
Examiners
Use

11. David Robson highlights a number of research findings on the effects of boredom and
responses to it. How far would you agree with the findings, relating your arguments to
your own society?

trendyline

For
Examiners
Use

For
Examiners
Use

trendyline

For
Examiners
Use

[10]

END OF PAPER

trendyline

Comprehension: Boredom
1. What does the word forayed (line 1) imply about the research on boredom? Use
your own words as far as possible. [1]
From the Passage
Sandi Mann is one of the few
psychologists to have forayed into the
mind-numbing territoryof boredom. (lines
1-2)

Answer
It implies that boredom is an area that has
not yet been thoroughly explored by
researchers/ that the research on boredom
is relatively new. [1]

2. Why does Sandi Mann call boredom the Cinderella of psychology in line 3? [2]
From the Passage
Answer
Its the Cinderella of psychology, she
a. (i)Just like Cinderella was a character
says. After all, admitting that you study
who was neglected but (ii)eventually
boredom might itself sound a bit, well, boring
received the due recognition for her
but that is far from the truth. (lines 3-4)
contribution/ enjoyed a happy ending,
[1]
b. (i)boredom is a state of mind that has
been treated as unimportant but (ii)is
now gaining the interest of
researchers for its influence on our
lives. [1]
(essence of answer focuses on the
apparent versus reality; point b is only
awarded if point a is present)
3. Explain what the author means by Dickenss study of Lady Deadlocks suffering
she is bored to death by her marriage would end up pre-empting many of the
latest findings (lines 9-11). Use your own words as far as possible. [2]
From the Passage
Answer
Dickenss study of Lady Deadlocks suffering a. Dickenss insights into how marital
she is bored to death by her marriage
relations of his fictitious characters can
lead to boredom [1]
would end up pre-empting many of the b. Turns out to have accurately
latest findings. (lines 9-11)
reflected/predicted the outcomes of
recent research on its effects in reality
[1]
4.

According to Eastwood, why had scientists been slow to explore the sensation
(line 12)? Use your own words as far as possible. [2]
From the Passage
Answer
But perhaps because of its prevalence in a. Since it was a state of being/emotion
our lives, scientists had been slow to explore
so commonly experienced by people,
the sensation.
[1]
Get PSJ Private Tutor to Guide you through Exams Now!
Contact www.privatetutor.com.sg
trendyline

When you are swimming in something, b. scientists did not see the value of
maybe you dont think of it as being
researching boredom/believed it was
noteworthy, says John Eastwood at York
unimportant /insignificant. [1]
University in Canada, who was one of the
first scholars to take an interest. (lines 1114)
swimming in something : a situation where
people are so immersed in the issue/
something is so much a part of us

5. How do the two types of personality in paragraph 3 illustrate the misconception that
only boring people get bored (line 15)? Use your own words as far as possible.
[3]
From the Passage
Answer
One of the most common misconceptions is a. People with the two types of
that only boring people get bored. Yet as
personality experience boredom
Eastwood set about exploring the reasons
easily even though they are not
for boredom, he found that there are two
themselves boring. [1]
distinct types of personality that tend to
suffer from ennui, and neither are
particularly dull themselves. (lines 15-18)
Boredom often goes with a naturally b. (i)Those with the first personality type
impulsive mindset among people who are
are adventure-seeking individuals
constantly looking for new experiences.
(ii)who fail to get sufficient/perpetual
For these people, the steady path of life just
exposure to excitement. [1]
isnt enough of a rollercoaster to hold their
attention. The world is chronically understimulating, says Eastwood. (lines 18-21)
The second kind of bored people have c. The second type of personality is
almost exactly the opposite problem; the
(i)frightened of/averse to risky
world is a fearful place, and so they shut
experiences yet (ii)eventually
themselves away and try not to step
becomes discontented with the
outside their comfort zone. Out of their
security/lack of challenge in their
high-sensitivity to pain, they withdraw.
lives. [1]
While this retreat might offer some comfort,
they are not always satisfied with the
safety it offers and chronic boredom
results. (lines 21-25)
6. What does Mann think about how we should respond to boredom when it hits us (line
58)? [1]
From the Passage
Answer
Given this benefit, Mann thinks we should We should welcome it and believe that
try not to fear boredom when it hits us. we can benefit from it/ need not react
We should embrace it, she says a negatively. [1]

trendyline

philosophy that she has now taken into her (Answer should include either a positive
own life. Instead of saying Im bored when or at least neutral response.)
Im stuck in traffic, Ill put music on and allow
my mind to wander knowing that its good
for me. And I let my kids be bored too
because its good for their creativity.
7. In paragraph 10, Eastwood claims that we should be cautious about looking for an
immediate escape. How does the remainder of the paragraph illustrate his concern?
[2]
From the Passage
Answer
For instance, simply looking for instant a. Seeking immediate satisfaction via
gratification on a smartphone or tablet
electronic gadgets could distract us
may be counter-productive, he thinks. We
and work against our original
live in tech-driven society where we are
intention.[1]
overly stimulated we are constantly
yanked around by interruptions, says
Eastwood.
b. The metaphor of the treadmill gives
That puts us on a kind of treadmill, he says
the impression that our continued
we keep on expecting quicker and
efforts to find faster fixes aggravate
easier ways to revive our curiosity. One
the situation/ get us nowhere/ are
possibility is that this actually makes
futile.[1]
people more bored. (lines 66-71)

8. Explain the authors use of superficial (line 76) and deeper (line 77) in paragraph
11. Use your own words as far as possible. [2]
From the Passage
Answer
The author aims to highlight the contrast
between
Although our feelings of tedium during a a. what might appear to be insignificant
irritants [1]
work meeting or family gathering might
seem superficial annoyances,
they could therefore be a symptom of a b. could in fact be manifestations of
crippling,
fundamental
problems
deeper existential crisis and
beyond day-to-day concerns. [1]
OR
requirement
for
long-term
need for fulfilment that extends far beyond c. A
satisfaction that is not being met. [1]
immediate circumstances. (line 75-78)
(contextual use of both superficial and
deeper must be present in the answer)
9. What, according to the author, is as good a reason as any to re-evaluate your life
(lines 81-82)? [2]
From the Passage
Answer
To feel you can have an effect on the
a. It is fundamentally crucial for us to
world and that things in life make sense,
believe that [1]

trendyline

these are inherently important things for


human beings just like sunlight, fresh air
and food, says Eastwood.
As we enter the New Year, that could be as
good a reason as any to re-evaluate your life
(lines 79-82)

b. we are able to make a difference


in society
AND
c. and have an understanding of the
world. [1 mark for BOTH (b) and
(c)]

10. Using material from paragraphs 48 only (lines 2657), summarise the drawbacks
and benefits of boredom. [8]
Write your summary in no more than 120 words not counting the opening words which
are printed below. Use your own words as far as possible.
Boredom is harmful when
S/ Points/Ideas from the
No passage (Essence of the
point/idea in bold)

Paraphrased equivalent

DRAWBACKS
1

push people to harm


themselves; proneness to
boredom was linked to a
tendency to smoke, drink
too much, and take drugs.
(lines 27-28)

a. driven to self-destructive behaviours

...linked to more mundane


but equally unhealthy
behaviours, such as
comfort- eating your way
through tedium. (lines 3031)

b. and commonplace/ routine/ everyday actions that


are just as damaging to their health

Boredom at work is
propping up the
confectionary industry.
(lines 31-32)

Boredom leads to

The overall effect of


boredom on your life
expectancy could be
drastic, too. (lines 32-33)

d. ...and a much shorter / substantial/ severe


reduction in life span ,

(Do not accept binge-eating)

c. (inferred) ... unhealthy consumption habits

BENEFITS

trendyline

S/ Points/Ideas from the


No passage (Essence of the
point/idea in bold)

Paraphrased equivalent

Lench suspects that it lies


behind one of our most
important traits
curiosity. (lines 42-43)

e. Boredom spurs/ underpins/ drives/ propels an


essential/ a vital inquisitiveness/ need to discover

Boredom, she says, stops


us ploughing the same
old furrow, (lines 41-42)

However, boredom

pushes us to try to seek


new goals or explore
new territories or ideas.
(lines 43-44)

g. and challenges us to look for the novel in things


or discover/ venture into unknown places and
question views and understandings / beliefs.

That search for an escape


[from boredom] could
sometimes push us to
take risks that eventually
hurt us. (lines 44-45)

Our attempt to overcome boredom could occasionally

...bored people turn to


unhealthy behaviours
but the upside is that it can
also increase innovation.
(lines 49-50)

Even though boredom can lead individuals to

f. overcomes/ puts an end to routine/repetitiveness,

h. drive us to take chances that are ultimately


harmful (drawback)

i. make injurious choices, (drawback)


j. it can also develop novel applications

10

...when performing
mindless tasks boost their
creativity such as finding
innovative uses for
everyday objects. (lines
51-52)

k. and promotes/ enhances/ encourages new


applications for commonly used items.

11

...tedium encouraged
their minds to wander,
which leads to more
associative and creative
ways of thinking. (lines
52-54)

Boredom actually
l. invigorates/ stimulates the brain to explore which
further develops deeper connections and inventive
methods of the thought processes / inventiveness
in thoughts.

trendyline

S/ Points/Ideas from the


No passage (Essence of the
point/idea in bold)
12

13

- going to different places


in our minds, she says. It
allows us to make leaps
of imagination. We can
get out of the box and
think in different ways.
(lines 54-56)
Without the capacity for
boredom, then, we
humans may have never
achieved our artistic and
technological heights.
(lines 56-57)

Paraphrased equivalent

m. to think in unconventional ways/ to make


previously unknown connections/ to unbridle the
mind/ to pull out all stops/ to not limit/ constrain our
thinking/creativity

n. accomplishing aesthetic beauty and scientific


breakthroughs/ pinnacles/ peaks of excellence/
would be greatly limited in our results / has helped/
enabled us to reach our current level of aesthetic
and innovative accomplishments
(accept lift for artistic and technological)

Suggested 14-points Mark Table (Revised for JC2 Preliminary Examination)


Range of Points

Marks

12 points and above

8 marks

11

10

7-8

5-6

3-4

1-2

Boredom is harmful when it a) encourages self-destructive behaviours and b)


everyday actions that are just as damaging to ones health. Boredom leads to c)
unhealthy consumption habits and d) a much shorter life span, h) driving us to take
chances that are ultimately harmful or i) injurious. However, e) it underpins a vital
inquisitiveness to discover. Boredom f) puts an end to routine and g) challenges us to
look for novel in things or venture into unknown places and question views and beliefs.

trendyline

Our attempt to overcome boredom could occasionally j) help develop novel applications
and k) encourage new applications for commonly used items. Boredom actually I)
invigorates the brain to explore deeper connections and inventiveness in thoughts; and
m) to unbridle the mind to n) accomplish aesthetic beauty and scientific breakthroughs.
(115 words)

trendyline

11. David Robson highlights a number of research findings on the effects of boredom and responses to it. How far would you agree with
the findings, relating your arguments to your own society? [10]

Points
mentioned by
the author

Elaboration/
Explanation

Evidence based on your society

Boredom can be beneficialbut we need to understand the reason for feeling bored so that we are not too quick to seek an
escape from boredom as it could also lead to undesirable side effects.
One benefit is
that boredom
stops us
ploughing the
same old furrow,
and pushes us
to try to seek
new goals or
explore new
territories or
ideas. That
search for an
escape could
sometimes push
us to take risks
that hurt us
(Para 7, line 43)

By this, he
meant that it
prevents us from
remaining in the
rut or simply put,
always sticking
to old set ways
of thinking and
refusing to
change. Instead
boredom pushes
us to try new
things and seek
new pastures
which might give
us new goals in
life but might
also harm us.

Response
Whether boredom can be a benefit or a bane really depends on what risk is being taken and the
opportunity cost.
Evidence 1
Some old people who feel bored after retirement try new things as a way to pass their time, e.g
learning line dances, taking up singing lessons, learning English and even learning to use the
computer. Some also sign up for short trips out of Singapore.
Evaluation
In the context Singapore, old people, especially the pioneer generation who grew up during a
period of hardship and poverty, their youth and almost their entire life is spent working to raise
their children and providing for them. It being the norm for young couples in Singapore to live on
their own, the old might feel bored and empty and many do seek ways out of this boredom through
registering for Residents Committee activities like learning to sing and going on short shopping
trips to factories and JB. These activities enable the elderly who are often rather set in their ways,
especially those who have never worked in their lives, to meet new people from different

trendyline

Points
mentioned by
the author

Elaboration/
Explanation

Evidence based on your society


backgrounds and through their interaction with each other, gain new insights and experiences.
This is beneficial as it not only makes life less boring, it makes them step out of their comfort zone
and make new friends.
Evidence 2
Another category of bored people are those who are bored with doing the same job year in and
year out. They pick up their courage to leave their first jobs in search of new territories or ideas
and the more fortunate ones do find a second career for themselves, especially if they also end up
in jobs for which they have a passion. Examples of mid-life career transitions include switch from
desk jobs to housing agent jobs or becoming entrepreneurs. However, not all these mid-life career
switches end up happily but they do open up a new experience and new world to the people who
make the switch. In that sense it can be regarded as a benefit.
Evaluation:
Singaporeans are quite averse to risks because we leave in a highly competitive society with many
foreign workers queueing to snap up our jobs and there is also the fear of not being able to adapt
to the new job and having to start all over again at a lower income. This is a risk that not all
Singaporeans are prepared to take, however bored they are, either because they are afraid to
move or because of objections from their spouses or responsibilities to their families. Hence,
though some dream of getting out of the rut, many would be hesitant to do so, unless the pull
factors are strong enough to push them to take the risk. However, it could be considered as a
bane too because in trying to escape boredom and seeking a new job, some end up disillusioned.
In a career website, a mid-thirties man who made a switch to a banking job because he was bored
with the routine of his first job, confessed that the change went horribly wrong as he recounted
how he could not catch up with his much younger colleagues in the field. This could hardly be

trendyline

Points
mentioned by
the author

Elaboration/
Explanation

Evidence based on your society


regarded as a benefit. If he had stuck to his routine job, all he had to face was boredom. Now in a
new territory, he faces threats from his younger colleagues and feels insecure, even though he
might be picking up new ideas.
http://news.efinancialcareers.com/sg-en/122411/my-mid-career-change-into-banking-wenthorribly-wrong-dont-make-the-same-mistakes/

We should be
cautious about
looking for an
immediate
escape and we
need to listen to
the emotion and
ask what it is
trying to tell us
to do
(paragraph 10,
line 63)
..looking for
instant
gratification
may be counterproductive(line
68)make us
more bored (line

What is meant
here is that we
should be careful
when we are
bored and
thinking of doing
something to
relieve the
boredom. We
need to ask
ourselves if the
action we are
about to take will
compromise our
security and
result in harm to
ourselves.

Response
True, especially if the escape chosen provides only a short-lived escape and lands you in more
trouble.
Evidence 1
Boredom causes some people to turn to drugs according to the findings by Singapore AntiNarcotics Association. (http://www.healthxchange.com.sg/News/Pages/young-educated-drugtakers-sana.aspx)
Evidence 2
Boredom was also one of the reasons why people start smoking in their adolescent years,
according to Dr Kenneth Chan, a consultant at the department of respiratory and critical care
medicine at Singapore General Hospital. (http://www.healthxchange.com.sg/News/Pages/SGHdoctor-on-smoking-addiction.aspx)
Evidence 3
Daryl Lim Jun Liang,18, was arrested for beating up smaller foreign workers to practise his martial

trendyline

Points
mentioned by
the author
71)

Elaboration/
Explanation

Evidence based on your society


arts skills because he was bored.
Evaluation:
If people do not understand the reasons for their boredom and just think of all ways to escape this
feeling, they could only end up hurting themselves as seen in the case of people who choose to
experiment with drugs and smoke since they are risking their health. In the case of the aggressive
teen, he ended up hurting himself too, besides hurting the foreign workers, as he was detained
and ordered to report to a supervision officer every day and remain indoors from 10pm to 6am for
a year and to perform 150 hours of community service. Moreover, he would now have a criminal
record for life. What these examples prove is exactly what Robinson also tries to highlight that
people need to know the reason for their boredom and be cautious about looking for an immediate
escape.
Evidence 4
The same can be observed among Singapore kids. Parents eager to keep their children occupied
during the holidays so they do not become bored send them for all kinds of coaching classes to
prepare them for the next level of education. In the end, after knowing all the content they need to
learn, they become bored when the teachers teach them the same content in class.
Evaluation:
In Singapore schools, a strict curriculum is followed and students who have already learnt the
content to be taught in class, will still need to sit through them, especially with a class size of 40,
teachers cannot afford to customize lessons for individuals. Hence those who have already learnt
the content in the holidays because they were bored end up being bored in class.

trendyline

Points
mentioned by
the author
Boredom allows
us to make
leaps of
imagination ,
get out of the
box and think in
different ways

Elaboration/
Explanation

Evidence based on your society

Meaning
boredom makes
us think beyond
what we would
normally do and
stretches our
imagination to
conceive
creative ideas.

Response
This could be true if the energy is directed to the right purposes.
Evidence 1
Derrick Kohs own boredom as a child when learning Math through the electronic learning systems
in the late 90s and early 2000s pushed him to think of how to do things differently. Now the owner
of an education chain in Singapore, he conceived his interactive Math learning portal out of
adapting the Neopets, the virtual pets website and the elements of electronic learning systems.
This innovative learning portal has been able to attract many students to enrol at his centre to
learn Math in a fun way. His boredom made his mind wander to more fun things like Neopets and
eventually led him to integrate it into the conventional learning portals to add fun to learning,
resulting in an unconventional approach and platform to learn Math
Evaluation:
Sadly, in the context of Singapore, young people especially, turn to social media to relieve their
boredom and whatever creativity that could have been expanded on thinking of innovations are
often not tapped on. In fact, the social media and the internet has somewhat killed the creativity in
people, not just because they resort to it when bored but also because they do not bother with
thinking out of the box since the internet can provide instant answers to many of their queries and
being such a highly-connected society only makes it worse. Usually when students come up with
innovative ideas, they are not conceived from boredom but a need to do a project.

He also
highlights how a
psychologist

Response
It would have to depend on the circumstances and setting/support provided

trendyline

Points
Elaboration/
mentioned by
Explanation
the author
argues that
boredom is
good for the
creativity of kids.
(paragraph 9)

Evidence based on your society

Evidence 1
Many students who are bored in class distract others.
Evaluation:
Whether boredom can bring out creativity in a child will have to depend on other factors like
environment and support as well. In the case of students feeling bored in class, with class sizes of
up to 40 in Singapore, it is hard to imagine how a bored child can be helped to develop his
creativity. More likely, without attention, he will end up being a nuisance as he distracts others and
disrupts the lesson. However, if parents on their own can afford the time to allow their bored
children to discover new things, like providing a scribbling wall for self-expression, then perhaps
boredom will work to bring out the creativity in them. However, in land scarce Singapore, this is
quite out of the question. Creativity is also stifled by the lack of space to run around and also
because many children in Singapore have easy access to the TV, internet and smart phones and
often than not, when bored will play electronic games, than think of ways to do things differently or
think up new ideas. To bring out creativity in bored kids, there must also be the support of adults in
creating the environment and fostering that culture of creativity. Sadly, many Singaporeans used
to a very structured way of life, do not think of it this way and they think that by sending their
children to art classes and more enrichment classes, their children would become more creative,
which is really not the case.
Evidence 2
A research done by NIE and Straits Times report revealed that one of the causes of aggressive
behaviour among teens in Singapore is boredom. (The Straits Times, August 3, 1998) (Problem
behaviours of Singapore youth, https://repository.nie.edu.sg/bitstream/10497/15521/1/ERA-AME-

trendyline

Points
mentioned by
the author

Elaboration/
Explanation

Evidence based on your society


AMIC-2000-461_a.pdf

trendyline

You might also like